You are on page 1of 116

iNsuRanCe A2010 Dean Carale

pAgE 1♥

ABOITIZ SHIPPING CORPORATION V NEW INDIA ASSURANCE HELD


NO
COMPANY, LTD.
Ratio Where the shipowner fails to overcome the presumption of negligence, the
[CITATION] doctrine of limited liability cannot be applied.
QUISUMBING; May 2, 2006
Reasoning
NATURE - From the nature of their business and for reasons of public policy, common carriers
Petition for review on certiorari are bound to observe extraordinary diligence over the goods they transport according
to all the circumstances of each case. In the event of loss, destruction or deterioration
FACTS of the insured goods, common carriers are responsible, unless they can prove that the
- Societe Francaise Des Colloides loaded a cargo of textiles and auxiliary chemicals loss, destruction or deterioration was brought about by the causes specified in Article
from France on board a vessel owned by Franco-Belgian Services, Inc. 17341 of the Civil Code. In all other cases, common carriers are presumed to have
- The cargo was consigned to General Textile, Inc., in Manila and insured by been at fault or to have acted negligently, unless they prove that they observed
respondent New India Assurance Company, Ltd. extraordinary diligence. Moreover, where the vessel is found unseaworthy, the
- While in Hongkong, the cargo was transferred to M/V P. Aboitiz for transshipment to shipowner is also presumed to be negligent since it is tasked with the maintenance of
Manila. its vessel. Though this duty can be delegated, still, the shipowner must exercise close
- Before departing, the vessel was advised that it was safe to travel to its destination, supervision over its men.
but while at sea, the vessel received a report of a typhoon moving within its path. - In the present case, petitioner has the burden of showing that it exercised
- To avoid the typhoon, the vessel changed its course. However, it was still at the extraordinary diligence in the transport of the goods it had on board in order to invoke
fringe of the typhoon when its hull leaked. the limited liability doctrine. Differently put, to limit its liability to the amount of the
- On October 31, 1980, the vessel sank, but the captain and his crew were saved. insurance proceeds, petitioner has the burden of proving that the unseaworthiness of
- On November 3, 1980, the captain of M/V P. Aboitiz filed his “Marine Protest”, stating its vessel was not due to its fault or negligence.
that the wind force was at 10 to 15 knots at the time the ship foundered and - Considering the evidence presented and the circumstances obtaining in this case, we
described the weather as “moderate breeze, small waves, becoming longer, fairly find that petitioner failed to discharge this burden. Both the trial and the appellate
frequent white horse courts, in this case, found that the sinking was not due to the typhoon but to its
- Petitioner notified the consignee of the total loss of the vessel and all of its cargoes. unseaworthiness. Evidence on record showed that the weather was moderate when
- General Textile, lodged a claim with respondent for the amount of its loss. the vessel sank. These factual findings of the Court of Appeals, affirming those of the
- Respondent paid General Textile and was subrogated to the rights of the latter. trial court are not to be disturbed on appeal, but must be accorded great weight.
- Respondent hired a surveyor to investigate, and the same concluded that the cause These findings are conclusive not only on the parties but on this Court as well.
was the flooding of the holds brought about by the vessel’s questionable Disposition Petition is denied for lack of merit.
seaworthiness.
- Respondent filed a complaint for damages against petitioner Aboitiz, Franco-Belgian
PHILAMCARE HEALTH SYSTEMS, INC. V CA (TRINOS)
Services and the latter’s local agent, F.E. Zuellig, Inc. (Zuellig)
- On November 20, 1989, the trial court ruled in favor of respondent and held 379 SCRA 357
petitioner Aboitiz liable for the total value of the lost cargo plus legal interest YNARES-SANTIAGO; March 18, 2002
- The complaint with respect to Franco and Zuellig was dismissed
- Petitioner elevated the case to the Court of Appeals, which in turn, affirmed in toto NATURE
the trial court’s decision. Petition for review of CA decision
- Petitioner moved for reconsideration but the same was denied.
- Hence, this petition for review FACTS
Petitioner’s Claim - Ernani TRINOS, deceased husband of respondent Julita, applied for a health care
> Petitioner contends that respondent’s claim for damages should only be against the coverage with Philamcare Health Systems, Inc. In the standard application form,
insurance proceeds and limited to its pro-rata share in view of the doctrine of limited he answered no to the question: “Have you or any of your family members ever
liability consulted or been treated for high blood pressure, heart trouble, diabetes, cancer,
Respondent’s Comments liver disease, asthma or peptic ulcer? (If Yes, give details).”
> Respondent counters that the doctrine of real and hypothecary nature of maritime
law is not applicable in the present case because petitioner was found to have been
negligent. Hence, according to respondent, petitioner should be held liable for the 1
Art. 1734. Common carriers are responsible for the loss, destruction, or deterioration of the goods, unless the
total value of the lost cargo same is due to any of the following causes only:
(1) Flood, storm, earthquake, lightning, or other natural disaster or calamity;
(2) Act of the public enemy in war, whether international or civil;
ISSUE (3) Act of omission of the shipper or owner of the goods;
WON the limited liability doctrine applies in this case (4) The character of the goods or defects in the packing or in the containers;
(5) Order or act of competent public authority.
iNsuRanCe A2010 Dean Carale
pAgE 2♥

- The application was approved for period of one year; upon termination, it was - A contract of insurance3 is an agreement whereby one undertakes for a
extended for another 2 years. Amount of coverage was increased to a maximum consideration to indemnify another against loss, damage or liability arising from an
sum of P75T per disability. unknown or contingent event.
- During this period, Ernani suffered a HEART ATTACK and was confined at the Manila - An insurance contract exists where the following elements concur:
Medical Center (MMC) for one month. While her husband was in the hospital, Julita (a) The insured has an insurable interest;
tried to claim the hospitalization benefits. (b) The insured is subject to a risk of loss by the happening of the peril;
- Petitioner treated the Health Care Agreement (HCA) as void since there (c) The insurer assumes the risk;
was a concealment regarding Ernani’s medical history. Doctors at the MMC (d) Such assumption of risk is part of a general scheme to distribute actual losses
allegedly discovered at the time of his confinement, he was hypertensive, diabetic and among a large group of persons bearing a similar risk; and
asthmatic. Julita then paid the hospitalization expenses herself, amounting to about (e) In consideration of the insurer’s promise, the insured pays a premium.
P76T. 2. NO
- After her husband died, Julita instituted action for damages against Philamcare Ratio Where matters of opinion or judgment are called for, answers made in good
and its Pres. After trial, the lower court ruled in her favor and ordered Philamcare to faith and without intent to deceive will not avoid a policy even though they are untrue;
reimburse medical and hospital coverage amounting to P76T plus interest, until fully since in such case the insurer is not justified in relying upon such statement, but is
paid; pay moral damages of P10T; pay exemplary damages of P10T; atty’s fees of obligated to make further inquiry.
P20T. Reasoning
- CA affirmed the decision of the trial court but deleted all awards for damages and - The fraudulent intent on the part of the insured must be established to warrant
absolved petitioner Reverente. rescission of the insurance contract. The right to rescind should be exercised previous
Petitioner’s Claims to the commencement of an action on the contract. No rescission was made. Besides,
(1) Agreement grants “living benefits” such as medical check-ups and the cancellation of health care agreements as in insurance policies requires:
hospitalization which a member may immediately enjoy so long as he is alive upon (a) Prior notice of cancellation to insured;
effectivity of the agreement until its expiration. (b) Notice must be based on the occurrence after effective date of the policy of one or
(2) Only medical and hospitalization benefits are given under the agreement without more of the grounds mentioned;
any indemnification, unlike in an insurance contract where the insured is (c) Must be in writing, mailed or delivered to the insured at the address shown in the
indemnified for his loss. policy;
(3) HCAs are only for a period of one year; therefore, incontestability clause does (d) Must state the grounds relied upon provided in Section 64 of the Insurance Code
not apply, as it requires effectivity period of at least 2 yrs. and upon request of insured, to furnish facts on which cancellation is based.
(4) It is not an insurance company, governed by Insurance Commission, but a Health - These conditions have not been met. When the terms of insurance contract contain
Maintenance Organization under the authority of DOH. limitations on liability, courts should construe them in such a way as to preclude
(5) Trinos concealed a material fact in his application. insurer from non-compliance of obligation. Being a contract of adhesion, terms of an
(6) Julita was not the legal wife since at the time of their marriage, the deceased insurance contract are to be construed strictly against the party which prepared it –
was previously married to another woman who was still alive.* the insurer.
- Also, Philamcare had 12 months from the date of issuance of the Agreement within
ISSUES which to contest the membership of the patient if he had previous ailment of asthma,
1. WON a health care agreement is an insurance contract (If so, “incontestability and six months from the issuance of the agreement if the patient was sick of diabetes
clause” under the Insurance Code is applicable) or hypertension.
2. WON the HCA can be invalidated on the basis of alleged concealment * The health care agreement is in the nature of a contract of indemnity. Hence,
payment should be made to the party who incurred the expenses. It is clear that
HELD respondent paid all the hospital and medical bills; thus, she is entitled to
YES reimbursement.
Ratio Every person has an insurable interest in the life and health of himself 2. The Disposition Petition DENIED.
health care agreement was in the nature of non-life insurance, which is primarily a
contract of indemnity. Once the member incurs hospital, medical or any other
PINEDA V CA (INSULAR LIFE INSURANCE COMPANY)
expense arising from sickness, injury or other stipulated contingent, the health care
provider must pay for the same to the extent agreed upon under the contract. 226 SCRA 755
Reasoning DAVIDE; September 27, 1993

2 NATURE
Sec.10. Every person has an insurable interest in the life and health:
Appeal by certiorari for review and set aside the Decision of the public respondent
(1) of himself, of his spouse and of his children;
(2) of any person on whom he depends wholly or in part for education or support, or in whom he has a pecuniary Court of Appeals and its Resolution denying the petitioners' motion for reconsideration
interest;
(3) of any person under a legal obligation to him for the payment of money, respecting property or service, of
which death or illness might delay or prevent the performance; and 3
(4) of any person upon whose life any estate or interest vested in him depends. Section 2 (1) of the Insurance Code
iNsuRanCe A2010 Dean Carale
pAgE 3♥

FACTS eyes to the real state of the case, but should either refuse to deal with the agent at
- In 1983, Prime Marine Services, Inc. (PMSI) procured a group policy from Insular Life all, or should ascertain from the principal the true condition of affairs.
to provide life insurance coverage to its sea-based employees enrolled under the plan. Reasoning
During the effectivity of the policy, 6 covered employees perished at sea. They were - The execution by the principals of special powers of attorney, which clearly
survived by complainants-appellees, the beneficiaries under the policy. appeared to be in prepared forms and only had to be filled up with their names,
- complainants-appellees sought to claim death benefits due them and approached residences, dates of execution, dates of acknowledgement and others, excludes any
Capt. Roberto Nuval, President and GM of PMSI, then executed special powers of intent to grant a general power of attorney or to constitute a universal agency. Being
attorney authorizing Capt. Nuval to “follow up, ask, demand, collect and receive” for special powers of attorney, they must be strictly construed. Insular Life knew that a
their benefit indemnities of sums of money due them…” power of attorney in favor of Capt. Nuval for the collection and receipt of such
- Unknown to the complainants, PMSI filed with Insular Life claims for and in behalf of proceeds was a deviation from its practice with respect to group policies (that the
them through Capt. Nuval, even using the 5 special powers of attorney that they employer-policyholder is the agent of the insurer).
executed as documents. Insular Life then released 6 checks, payable to the order of - The employer acts as a functionary in the collection and payment of premiums and
the complainant-appellees, to the treasurer of PMSI (who happened to be Capt. in performing related duties. Likewise falling within the ambit of administration of a
Nuval’s son-in-law). Capt. Nuval then endorsed and deposited these checks (which group policy is the disbursement of insurance payments by the employer to the
were for the complainants) in his bank account. employees. Most policies, such as the one in this case, require an employee to pay a
- 3 years after, the complainants-appellees found out that they were entitled, as portion of the premium, which the employer deducts from wages while the remainder
beneficiaries, to life insular benefits under a group policy with respondent-appellant so is paid by the employer. This is known as a contributory plan as compared to a non-
they sought to recover these benefits from Insular Life. Insular Life denied the claim, contributory plan where the premiums are solely paid by the employer.
saying that their liability to complainants was already extinguished upon delivery to - the labor of the employees is the true source of the benefits, which are a
and receipt by PMSI of the 6 checks issued in the complainants’ names. Complainants form of additional compensation to them.
filed case with the Insurance Commission which decided in their favor. - the employer is the agent of the insurer in performing the duties of
- Insurance Commission held that the special powers of attorney executed by administering group insurance policies. It cannot be said that the employer acts
complainants in favor of the complainants do not contain in unequivocal and clear entirely for its own benefit or for the benefit of its employees in undertaking
terms authority to Capt. Nuval to obtain, receive, receipt from respondent company administrative functions. While a reduced premium may result if the employer relieves
insurance proceeds arising from the death of the seaman-insured; also, that Insular the insurer of these tasks, and this, of course, is advantageous to both the employer
Life did not convincingly refuted the claim of Mrs. Alarcon that neither she nor her and the employees, the insurer also enjoys significant advantages from the
husband executed a special power of authority in favor of Capt. Nuval (and therefore, arrangement. The reduction in the premium which results from employer-
the company should have not released the check to Capt. Nuval-PMSI); and that it did administration permits the insurer to realize a larger volume of sales, insurer to
not observe Sec 180(3), as repealed by Art. 225 of the Family Code, when it released realize a larger volume of sales, and at the same time the insurer's own administrative
the benefits due to the minor children of Ayo and Lontok, when the said complainants costs are markedly reduced.
did not post a bond as required - the employee has no knowledge of or control over the employer's actions in handling
- Insular Life appealed to the CA; CA modified the decision of the Insurance the policy or its administration. An agency relationship is based upon consent by one
Commission, eliminating the award to the Lontoks and Ayo person that another shall act in his behalf and be subject to his control. It is clear from
the evidence regarding procedural techniques here that the insurer-employer
ISSUES relationship meets this agency test with regard to the administration of the policy,
1. WON Insular Life should be liable to the complainants when they relied on the whereas that between the employer and its employees fails to reflect true agency.
special powers of attorney, which Capt. Nuval presented as documents, when they The insurer directs the performance of the employer's administrative acts, and if
released the checks to the latter these duties are not undertaken properly the insurer is in a position to exercise more
2. WON Insular Life should be liable to the complainants when they released the check constricted control over the employer's conduct.
in favor of Ayo and LOntok, even if no bond was posted as required - ON GROUP INSURANCE: Group insurance is essentially a single insurance
contract that provides coverage for many individuals. In its original and most
HELD common form, group insurance provides life or health insurance coverage for the
1. YES employees of one employer. The coverage terms for group insurance are
Ratio Third persons deal with agents at their peril and are bound to inquire as to the usually stated in a master agreement or policy that is issued by the insurer to a
extent of the power of the agent with whom they contract. The person dealing with representative of the group or to an administrator of the insurance program, such as
an agent must also act with ordinary prudence and reasonable diligence. an employer. Although the employer may be the titular or named insured, the
Obviously, if he knows or has good reason to believe that the agent is exceeding his insurance is actually related to the life and health of the employee. Indeed,
authority, he cannot claim protection. So if the suggestions of probable limitations be the employee is in the position of a real party to the master policy, and even in a non-
of such a clear and reasonable quality, or if the character assumed by the agent is of contributory plan, the payment by the employer of the entire premium is a part of the
such a suspicious or unreasonable nature, or if the authority which he seeks to total compensation paid for the services of the employee.
exercise is of such an unusual or improbable character, as would suffice to put an 2. YES
ordinarily prudent man upon his guard, the party dealing with him may not shut his Ratio Regardless of the value of the unemancipated common child's property, the
father and mother ipso jure become the legal guardian of the child's property.
iNsuRanCe A2010 Dean Carale
pAgE 4♥

However, if the market value of the property or the annual income of the child - On Feb. 5, 1991, William Lines, Inc. brought its vessel, M/V Manila City, to the Cebu
exceeds P50,000,00, a bond has to be posted by the parents concerned to guarantee Shipyard in Lapulapu City for annual dry-docking and repair.
the performance of the obligations of a general guardian. - On Feb. 6, 1991, an arrival conference was held between representatives of William
Reasoning Lines, Inc. and CSEW to discuss the work to be undertaken on the M/V Manila City. The
- Sec 180, Insurance Code: 'In the absence of a judicial guardian, the father, or in contracts, denominated as Work Orders, were signed thereafter., with the following
the latter's absence or incapacity, the mother of any minor, who is an insured or a stipulations:
beneficiary under a contract of life, health or accident insurance, may exercise, in “10. The Contractor shall replace at its own work and at its own cost any work or
behalf of said minor, any right under the policy, without necessity of court authority or material which can be shown to be defective and which is communicated in writing
the giving of a bond where the interest of the minor in the particular act involved does within one (1) month of redelivery of the vessel or if the vessel was not in the
not exceed twenty thousand pesos " Contractor’s Possession, the withdrawal of the Contractor’s workmen, or at its option
- …repealed by Art. 225, Family Code: "ART. 225. The father and the mother shall to pay a sum equal to the cost of such replacement at its own works. These conditions
jointly exercise legal guardianship over the property of their unemancipated common shall apply to any such replacements.
child without the necessity of a court appointment. In case of disagreement, the 11. Save as provided in Clause 10, the Contractor shall not be under any liability to
father's decision shall prevail, unless there is judicial order to the contrary. the Customer either in contract or for delict or quasi-delict or otherwise except for
Where the market value of the property or the annual income of the child exceeds negligence and such liability shall itself be subject to the following overriding
P50,000, the parent concerned shall be required to furnish a bond in such amount as limitations and exceptions, namely:
the court may determine, but not less than ten per centum (10%) of the value of the (a) The total liability of the Contractor to the Customer (over and above the liability to
property or annual income, to guarantee the performance of the obligations replace under Clause 10) or of any sub-contractor shall be limited in respect of any
prescribed for general guardians." defect or event (and a series of accidents arising out of the same defect or event
-"market value of the property or the annual income of the child": the shall constitute one defect or event) to the sum of Pesos Philippine Currency One
aggregate of the child's property or annual income; if this exceeds P50,000.00, a bond Million only.
is required - There is no evidence that the share of each of the minors in the proceeds x x x
of the group policy in question is the minor's only property. Without such evidence, it 20. The insurance on the vessel should be maintained by the customer and/or owner
would not be safe to conclude that, indeed, that is his only property. of the vessel during the period the contract is in effect.”
Disposition the instant petition is GRANTED. The Decision of 10 October 1991 and - While the M/V Manila City was undergoing dry-docking and repairs within the
the Resolution of 19 May 1992 of the public respondent in CA-G.R. SP No. 22950 are premises of CSEW, the master, officers and crew of M/V Manila City stayed in the
SET ASIDE and the Decision of the Insurance Commission in IC Case No. RD-058 is vessel, using their cabins as living quarters. Other employees hired by William Lines to
REINSTATED. Costs against the private respondent. SO ORDERED. do repairs and maintenance work on the vessel were also present during the dry-
docking.
- On February 16, 1991, after subject vessel was transferred to the docking quay, it
CEBU SHIPYARD ENGINEERING WORKS, INC. V WILLIAM
caught fire and sank, resulting to its eventual total loss.
LINES, INC. and PRUDENTIAL GUARANTEE and ASSURANCE - On February 21, 1991, William Lines, Inc. filed a complaint for damages against
COMPANY, INC. CSEW, alleging that the fire which broke out in M/V Manila City was caused by CSEW’s
[CITATION] negligence and lack of care.
- On July 15, 1991 was filed an Amended Complaint impleading Prudential as co-
PURISIMA; May 5, 1999 plaintiff, after the latter had paid William Lines, Inc. the value of the hull and
machinery insurance on the M/V Manila City. As a result of such payment Prudential
NATURE was subrogated to the claim of P45 million, representing the value of the said
Petition for review on certiorari insurance it paid.
On June 10, 1994, the trial court a quo came out with a judgment against CSEW:
FACTS 1. To pay unto plaintiff Prudential Guarantee and Assurance, Inc., the subrogee, the
- Cebu Shipyard and Engineering Works, Inc. (CSEW) is engaged in the business of amount of P45M, with interest at the legal rate until full payment is made; the amount
dry-docking and repairing of marine vessels while the Prudential Guarantee and of P56,715,000 representing loss of income of M/V MANILA CITY, with interest at the
Assurance, Inc. (Prudential) is in the non-life insurance business. legal rate until full payment is made;
- William Lines, Inc. is in the shipping business. It was the owner of M/V Manila City, a 2. To pay unto plaintiff, William Lines, Inc. the amount of P11M as payment, in
luxury passenger-cargo vessel, which caught fire and sank on Feb. 16, 1991. At the addition to what it received from the insurance company to fully cover the injury or
time of the unfortunate occurrence sued upon, subject vessel was insured with loss, in order to replace the M/V MANILA CITY, with interest at the legal rate until full
Prudential for P45M for hull and machinery. The Hull Policy included an “Additional payment is made; the sum of P927,039 for the loss of fuel and lub oil on board the
Perils (INCHMAREE)” Clause covering loss of or damage to the vessel through the vessel when she was completely gutted by fire at defendant, Cebu Shipyard’s quay,
negligence of, among others, ship repairmen with interest at the legal rate until full payment is made; the sum of P3,054,677.95 as
- Petitioner CSEW was also insured by Prudential for third party liability under a payment for the spare parts and materials used in the M/V MANILA CITY during dry-
Shiprepairer’s Legal Liability Insurance Policy. The policy was for P10 million only, docking with interest at the legal rate until full payment is made; P500,000 in moral
under the limited liability clause, to wit: damages;the amount of P10Min attorney’s fees; and to pay the costs of this suit.
iNsuRanCe A2010 Dean Carale
pAgE 5♥

- On September 3, 1997, the Court of Appeals affirmed the appealed decision of the 1. WON CSEW had “management and supervisory control“ of the m/v manila city at
trial court, ordering CSEW to pay Prudential, the subrogee, the sum of P45 Million, the time the fire broke out
with interest at the legal rate until full payment is made. 2. WON the doctrine of res ipsa loquitur applies against the crew
CSEW’s version: 3. WON CSEW’S expert evidence is admissible or of probative value
On Feb. 13, 1991, the CSEW completed the drydocking of M/V Manila City at its grave 4. WON Prudential has the right of subrogation against its own insured THE
dock. It was then transferred to the docking quay of CSEW where the remaining repair CONTRACTUAL 5. 5. 5. 5. WON the provisions limiting csew’s liability for negligence
to be done was the replating of the top of Water Ballast Tank No. 12 which was to a maximum of p1 million are valid
subcontracted by CSEW to JNB General Services. Tank Top No. 12 was at the rear
section of the vessel, on level with the flooring of the crew cabins located on the HELD
vessel’s second deck. 1. YES
At around 7AM of Feb. 16, 1991, the JNB workers trimmed and cleaned the tank top - The that factual findings by the CA are conclusive on the parties and are not
framing which involved minor hotworks (welding/cutting works). The said work was reviewable by this Court. They are entitled to great weight and respect, even finality,
completed at about 10AM. The JNB workers then proceeded to rig the steel plates, especially when, as in this case, the CA affirmed the factual findings arrived at by the
after which they had their lunch break. The rigging was resumed at 1PM trial court. When supported by sufficient evidence, findings of fact by the CA affirming
While in the process of rigging the second steel plate, the JNB workers noticed smoke those of the trial court, are not to be disturbed on appeal. The rationale behind this
coming from the passageway along the crew cabins. When one of the workers, Mr. doctrine is that review of the findings of fact of the CA is not a function that the
Casas, proceeded to the passageway to ascertain the origin of the smoke, he noticed Supreme Court normally undertakes.
that smoke was gathering on the ceiling of the passageway but did not see any fire as - The CA and the Cebu RTC are agreed that the fire which caused the total loss of
the crew cabins on either side of the passageway were locked. He immediately subject M/V Manila City was due to the negligence of the employees and workers of
sought out the proprietor of JNB, Mr. Buenavista, and the Safety Officer of CSEW, Mr. CSEW. Both courts found that the M/V Manila City was under the custody and control
Aves, who sounded the fire alarm. CSEW’s fire brigade immediately responded as of petitioner CSEW, when the ill-fated vessel caught fire. The decisions of both the
well as the other fire fighting units in Metro Cebu. However, there were no WLI lower court and the CA set forth clearly the evidence sustaining their finding of
representative, officer or crew to guide the firemen inside the vessel. actionable negligence on the part of CSEW. This factual finding is accorded great
- Despite the combined efforts of the firemen of the Lapulapu City Fire Dept., Mandaue weight and is conclusive on the parties. The court discerns no basis for disturbing such
Fire Dept., Cordova Fire Dept. Emergency Rescue Unit Foundation, and fire brigade of finding firmly anchored on enough evidence.
CSEW, the fire was not controlled until 2AM of the following day. - Furthermore, in petitions for review on certiorari, only questions of law may be put
- On the early morning of Feb. 17, 1991, gusty winds rekindled the flames on the into issue. Questions of fact cannot be entertained. The finding of negligence by the
vessel and fire again broke out. Then the huge amounts of water pumped into the CA is a question which this Court cannot look into as it would entail going into factual
vessel, coupled with the strong current, caused the vessel to tilt until it capsized and matters on which the finding of negligence was based. Such an approach cannot be
sank allowed by this Court in the absence of clear showing that the case falls under any of
- When M/V Manila City capsized, steel and angle bars were noticed to have been the exceptions to the well-established principle.
newly welded along the port side of the hull of the vessel, at the level of the crew The finding by the trial court and the Court of Appeals that M/V Manila City caught fire
cabins. William Lines did not previously apply for a permit to do hotworks on the said and sank by reason of the negligence of the workers of CSEW, when the said vessel
portion of the ship as it should have done pursuant to its work order with CSEW. was under the exclusive custody and control of CSEW is accordingly upheld.
Prudential’s version 2. YES
> At around 7AM of Feb. 16, 1991, the Chief Mate of M/V Manila City was inspecting - For the doctrine of res ipsa loquitur to apply to a given situation, the following
the various works being done by CSEW on the vessel, when he saw that some workers conditions must concur: (1) the accident was of a kind which does not ordinarily occur
of CSEW were cropping out steel plates on Tank Top No. 12 using acetylene, oxygen unless someone is negligent; and (2) that the instrumentality or agency which caused
and welding torch. He also observed that the rubber insulation wire coming out of the the injury was under the exclusive control of the person charged with negligence.
air-conditioning unit was already burning, prompting him to scold the workers. The facts and evidence on record reveal the concurrence of said conditions in the case
> At 2:45 PM of the same day, witnesses saw smoke coming from Tank No. 12. The under scrutiny. First, the fire that occurred and consumed M/V Manila City would not
vessel’s reeferman reported such occurence to the Chief Mate who immediately have happened in the ordinary course of things if reasonable care and diligence had
assembled the crew members to put out the fire. When it was too hot for them to been exercised. In other words, some negligence must have occurred. Second, the
stay on board and seeing that the fire cannot be controlled, the vessel’s crew were agency charged with negligence, as found by the trial court and the CA and as shown
forced to withdraw from CSEW’s docking quay. by the records, is the herein petitioner, CSEW, which had control over subject vessel
- In the morning of Feb. 17, 1991, M/V Manila City sank. As the vessel was insured when it was docked for annual repairs. So also, as found by the RTC, “other
with Prudential Guarantee, William Lines filed a claim for constructive total loss, and responsible causes, including the conduct of the plaintiff, and third persons, are
after a thorough investigation of the surrounding circumstances of the tragedy, sufficiently eliminated by the evidence.”
Prudential found the said insurance claim to be meritorious and issued a check in What is more, in the present case the trial court found direct evidence to prove that
favor of William Lines in the amount of P45 million pesos representing the total value the workers and/or employees of CSEW were remiss in their duty of exercising due
of M/V Manila City’s hull and machinery insurance. diligence in the care of subject vessel. The direct evidence substantiates the
conclusion that CSEW was really negligent. Thus, even without applying the doctrine
ISSUES of res ipsa loquitur, in light of the direct evidence on record, the ineluctable conclusion
iNsuRanCe A2010 Dean Carale
pAgE 6♥

is that CSEW was negligent and consequently liable for damages to the respondent, 20. The insurance on the vessel should be maintained by the customer and/or
William Lines, Inc. owner of the vessel during the period the contract is in effect.
3. NO - According to petitioner, under the aforecited clause, William Lines, Inc., agreed to
- Petitioner maintains that the CA erred in disregarding the testimonies of the fire assume the risk of loss of the vessel while under drydock or repair and to such extent,
experts, Messrs. David Grey and Gregory Michael Southeard, who testified on the it is benefited and effectively constituted as a co-assured under the policy.
probable origin of the fire in M/V Manila City. Petitioner avers that since the said fire - This theory of petitioner is devoid of sustainable merit. Clause 20 of the Work Order
experts were one in their opinion that the fire did not originate in the area of Tank Top in question is clear in the sense that it requires William Lines to maintain insurance on
No. 12 where the JNB workers were doing hotworks but on the crew accommodation the vessel during the period of dry-docking or repair. Concededly, such a stipulation
cabins on the portside No. 2 deck, the RTC and the CA should have given weight to works to the benefit of CSEW as the shiprepairer. However, the fact that CSEW
such finding based on the testimonies of fire experts; petitioner argues. benefits from the said stipulation does not automatically make it as a co-assured of
But courts are not bound by the testimonies of expert witnesses. Although they may William Lines. The intention of the parties to make each other a co-assured under an
have probative value, reception in evidence of expert testimonies is within the insurance policy is to be gleaned principally from the insurance contract or policy
discretion of the court, under Section 49, Rule 130 of the Revised Rules of Court. It is itself and not from any other contract or agreement because the insurance policy
never mandatory for judges to give substantial weight to expert testimonies. If from denominates the assured and the beneficiaries of the insurance. The hull and
the facts and evidence on record, a conclusion is readily ascertainable, there is no machinery insurance procured by William Lines, Inc. from Prudential named only
need for the judge to resort to expert opinion evidence. In the case under “William Lines, Inc.” as the assured. There was no manifestation of any intention of
consideration, the testimonies of the fire experts were not the only available evidence William Lines, Inc. to constitute CSEW as a co-assured under subject policy. It is
on the probable cause and origin of the fire. There were witnesses who were actually axiomatic that when the terms of a contract are clear its stipulations control.i] Thus,
on board the vessel when the fire occurred. Between the testimonies of the fire when the insurance policy involved named only William Lines, Inc. as the assured
experts who merely based their findings and opinions on interviews and the thereunder, the claim of CSEW that it is a co-assured is unfounded.
testimonies of those present during the fire, the latter are of more probative value. - Then too, in the Additional Perils Clause of the same Marine Insurance Policy, it is
provided that:
Subject to the conditions of this Policy, this insurance also covers loss of or damage to
vessel directly caused by the following:
4. YES xxx
- Petitioner contends that Prudential is not entitled to be subrogated to the rights of Negligence of Charterers and/or Repairers, provided such Charterers and/or Repairers
William Lines, Inc., theorizing that (1) the fire which gutted M/V Manila City was an are not an Assured hereunder.
excluded risk and (2) it is a co-assured under the Marine Hull Insurance Policy. - As correctly pointed out by respondent Prudential, if CSEW were deemed a co-
- It is petitioner’s submission that the loss of M/V Manila City or damage thereto is assured under the policy, it would nullify any claim of William Lines, Inc. from
expressly excluded from the coverage of the insurance because the same resulted Prudential for any loss or damage caused by the negligence of CSEW. Certainly, no
from “want of due diligence by the Assured, Owners or Managers” which is not shipowner would agree to make a shiprepairer a co-assured under such insurance
included in the risks insured against. Again, this theory of petitioner is bereft of any policy; otherwise, any claim for loss or damage under the policy would be invalidated.
factual or legal basis. It proceeds from a wrong premise that the fire which gutted Such result could not have been intended by William Lines, Inc.
subject vessel was caused by the negligence of the employees of William Lines, Inc. 5. NO
To repeat, the issue of who between the parties was negligent has already been - Although in this jurisdiction, contracts of adhesion have been consistently upheld as
resolved against CSEW. Upon proof of payment by Prudential to William Lines, Inc., valid per se; as binding as an ordinary contract, the Court recognizes instances when
the former was subrogated to the right of the latter to indemnification from CSEW. As reliance on such contracts cannot be favored especially where the facts and
aptly ruled by the Court of Appeals, the law on the matter is succinct and clear, to wit: circumstances warrant that subject stipulations be disregarded. Thus, in ruling on the
- Art. 2207. If the plaintiff’s property has been insured, and he has received indemnity validity and applicability of the stipulation limiting the liability of CSEW for negligence
from the insurance company for the injury or loss arising out of the wrong or breach of to P1M only, the facts and circumstances vis-a-vis the nature of the provision sought
contract complained of, the insurance company shall be subrogated to the rights of to be enforced should be considered, bearing in mind the principles of equity and fair
the insured against the wrongdoer or the person who has violated the contract. If the play.
amount paid by the insurance company does not fully cover the injury or loss, the - It is worthy to note that M/V Manila City was insured with Prudential for P45M. To
aggrieved party shall be entitled to recover the deficiency from the person causing determine the validity and sustainability of the claim of William Lines, Inc., for a total
the loss or injury. loss, Prudential conducted its own inquiry. Upon thorough investigation by its hull
- Thus, when Prudential, after due verification of the merit and validity of the surveyor, M/V Manila City was found to be beyond economical salvage and repair. The
insurance claim of William Lines, Inc., paid the latter the total amount covered by its evaluation of the average adjuster also reported a constructive total loss. The said
insurance policy, it was subrogated to the right of the latter to recover the insured loss claim of William Lines, Inc., was then found to be valid and compensable such that
from the liable party, CSEW. Prudential paid the latter the total value of its insurance claim. Furthermore, it was
- Petitioner theorizes further that there can be no right of subrogation as it is deemed ascertained that the replacement cost of the vessel (the price of a vessel similar to
a co-assured under the subject insurance policy. To buttress its stance that it is a co- M/V Manila City), amounts to P55M.
assured, petitioner placed reliance on Clause 20 of the Work Order which states: - Considering the aforestated circumstances, let alone the fact that negligence on the
part of petitioner has been sufficiently proven, it would indeed be unfair and
iNsuRanCe A2010 Dean Carale
pAgE 7♥

inequitable to limit the liability of petitioner to One Million Pesos only. As aptly held by "The insured shall give notice to the Company of any insurance or insurances already
the trial court, “it is rather unconscionable if not overstrained.” To allow CSEW to limit effected, or which may subsequently be effected, covering any of the property or
its liability to P1M notwithstanding the fact that the total loss suffered by the assured properties consisting of stocks in trade, goods in process and/or inventories only
and paid for by Prudential amounted to P45M would sanction the exercise of a degree hereby insured, and unless such notice be given and the particulars of such insurance
of diligence short of what is ordinarily required because, then, it would not be difficult or insurances be stated therein or endorsed on this policy pursuant to Section 50 of
for petitioner to escape liability by the simple expedient of paying an amount very the Insurance Code, by or on behalf of the Company before the occurrence of any loss
much lower than the actual damage or loss suffered by William Lines, Inc. or damage, all benefits under this policy shall be deemed forfeited, provided however,
Disposition Petition is DENIED. Resolution of the CA is AFFIRMED. that this condition shall not apply when the total insurance or insurances in force at
the time of loss or damage is not more than P200,000.00."
Petitioner’s comments
NEW LIFE ENTERPRISES V CA
> Petitioners contend that they are not to be blamed for the omissions, alleging that
207 SCRA 669 insurance agent Leon Alvarez (for Western) and Yap Kam Chuan (for Reliance and
REGALADO; March 31, 1992 Equitable) knew about the existence of the additional insurance coverage and that
they were not informed about the requirement that such other or additional insurance
NATURE should be stated in the policy, as they have not even read said policies.
Appeal by certiorari
ISSUE
FACTS WON New Life Enterprises’ claim for payment be denied
- The antecedents of this case show that Julian Sy and Jose Sy Bang have formed a
business partnership in the City of Lucena. Under the business name of New Life HELD
Enterprises, the partnership engaged in the sale of construction materials at its place YES
of business, a two storey building situated at Iyam, Lucena City. The facts show that Ratio Furthermore, when the words and language of documents are clear and plain or
Julian Sy insured the stocks in trade of New Life Enterprises with Western Guaranty readily understandable by an ordinary reader thereof, there is absolutely no room for
Corporation, Reliance Surety and Insurance Co. Inc., and Equitable Insurance interpretation or construction anymore. Courts are not allowed to make contracts for
Corporation. the parties; rather, they will intervene only when the terms of the policy are
- On May 15, 1981, Western Guaranty Corporation issued Fire Insurance Policy No. ambiguous, equivocal, or uncertain. The parties must abide by the terms of the
37201 in the amount of P350,000.00. This policy was renewed on May 13, 1982. contract because such terms constitute the measure of the insurer's liability and
- On July 30, 1981, Reliance Surety and Insurance Co., Inc. issued Fire Insurance compliance therewith is a condition precedent to the insured's right of recovery from
Policy No. 69135 in the amount of P300,000.00 (Renewed under Renewal Certificate the insurer.
No. 41997). An additional insurance was issued by the same company on November - While it is a cardinal principle of insurance law that a policy or contract of insurance
12, 1981 under Fire Insurance Policy No. 71547 in the amount of P700,000.00. is to be construed liberally in favor of the insured and strictly against the insurer
- On February 8, 1982, Equitable Insurance Corporation issued Fire Insurance Policy company, yet contracts of insurance, like other contracts, are to be construed
No. 39328 in the amount of P200,000.00. according to the sense and meaning of the terms which the parties themselves have
- Thus when the building occupied by the New Life Enterprises was gutted by fire at used. If such terms are clear and unambiguous, they must be taken and understood in
about 2:00 o'clock in the morning of October 19, 1982, the stocks in trade inside said their plain, ordinary and popular sense. Moreover, obligations arising from contracts
building were insured against fire in the total amount of P1,550,000.00. According to have the force of law between the contracting parties and should be complied with in
the certification issued by the Headquarters, Philippine Constabulary/Integrated good faith.
National Police, Camp Crame, the cause of fire was electrical in nature. According to Reasoning
the plaintiffs, the building and the stocks inside were burned. After the fire, Julian Sy a. The terms of the contract are clear and unambiguous. The insured is specifically
went to the agent of Reliance Insurance whom he asked to accompany him to the required to disclose to the insurer any other insurance and its particulars which he
office of the company so that he can file his claim. He averred that in support of his may have effected on the same subject matter. The knowledge of such insurance by
claim, he submitted the fire clearance, the insurance policies and inventory of stocks. the insurer's agents, even assuming the acquisition thereof by the former, is not the
He further testified that the three insurance companies are sister companies, and as a "notice" that would stop the insurers from denying the claim. Besides, the so-called
matter of fact when he was following-up his claim with Equitable Insurance, the Claims theory of imputed knowledge, that is, knowledge of the agent is knowledge of the
Manager told him to go first to Reliance Insurance and if said company agrees to pay, principal, aside from being of dubious applicability here has likewise been roundly
they would also pay. The same treatment was given him by the other insurance refuted by respondent court whose factual findings we find acceptable.
companies. Ultimately, the three insurance companies denied plaintiffs' claim for b. Petitioners should be aware of the fact that a party is not relieved of the duty to
payment. exercise the ordinary care and prudence that would be exacted in relation to other
Respondent’s comments contracts. The conformity of the insured to the terms of the policy is implied from his
> Western Guaranty Corporation through Claims Manager Bernard S. Razon told the failure to express any disagreement with what is provided for.
plaintiff that his claim 'is denied for breach of policy conditions.' Reliance Insurance
purveyed the same message as well as Equitable Insurance Corporation.
- The said policy in question follows:
iNsuRanCe A2010 Dean Carale
pAgE 8♥

FIRST QUEZON CITY INSURANCE CO. v. CA (DE DIOS


HELD
MARIKINA TRANSPORT CO)
YES
218 SCRA 526 - The insurance policy clearly placed the maximum limit of the petitioner's liability for
GRINO-AQUINO; February 28, 1993 damages arising from death or bodily injury at P12,000.00 per passenger and its
maximum liability per accident at (P50,000.00. Since only one passenger was injured
NATURE in the accident, the insurer's liability for the damages suffered by said passenger is
PETITION for review of the decision of the Court of Appeals. FQCIC seeks to limit to pegged to the amount of P12,000.00 only.
P12000, the amount specified in the insurance contract, it’s liability to indemnify the - The limit of P50,000.00 per accident means that the insurer's maximum liability for
respomdemt DMTC, for the damages suffered by a passenger, who accidentally fell off any single accident will not exceed P50,000.00 regardless of the number of
the bug. passengers killed or injured therein.
The bus company may not recover from the insurance company more than
FACTS P12,000.00 per passenger killed or injured, or (P50,000.00) per accident even if under
- After sending off certain seamen at the departure area of MIA, Jose V. del Rosario the judgment of the court, the erring bus operator will have to pay more than
proceeded to the public utility bus stop. While at the bus stop, the plaintiff saw a P12,000.00 to each injured passenger. The trial court's interpretation of the insurance
DMTC bus. While moving at a crawling pace, it was taking several passengers, all of contract was the correct interpretation.
whom managed to board the bus while it was already at the bus stop; plaintiff was the Disposition petition for review is GRANTED. The decision promulgated by the CA,
last one to board the bus. ordering the third party defendent, First Quezon City Insurance Co., Inc. to indemnify
While the plaintiff was still on the bus with his hand on the bus door, the slowly theI private respondent, (DMTC), the sum of P50,000.00 for the damages of the
moving bus sped forward at a high speed, as a result of which, the plaintiff lost passenger, Jose V. Del Rosario, is hereby modified by reducing the award to 12,000.00
balance and fell from the bus. As plaintiff clung instinctively to the handle bar, he was only. Costs against the private respondent De Dios Marikina Transportation Co., Inc.
dragged by the bus along the asphalted road. The bus driver, Gil Agpalo, abruptly
stopped the bus. Then fled from the scene, leaving the bus and the injured plaintiff TY V FIRST NATIONAL SURETY
behind.
1 SCRA 1324
- The plaintiff was brought to the Manila Sanitarium and Hospital where the doctors
performed 2 major surgical operations on plaintiffs right leg. LABRADOR; April 29, 1961
- Plaintiff was confined at the hospital for (40) days, from June 10, 1984 to August 26,
1984. Medical expenses totaled the amount of P69,444.41. Plaintiff’s medical FACTS
expenses were advanced by his employer Maglines but he was required to reimburse - At different times within a period of two months prior to 24 December 1953,
Maglines on a staggered basis by way of salary deductions. After his release from the Diosdado C. Ty, employed as operator mechanic foreman in the Broadway Cotton
hospital, he returned to the hospital for further treatment and checkup. The injuries Factory insured himself in 18 local insurance companies, among which being the 8
had left plaintiff with a huge scar on his right leg. Also, the plaintiff incurred lost above-named defendants, which issued to him personal accident policies. Plaintiff’s
earning by way of unearned salaries amounting to P7,500.00 due to said physical beneficiary was his employer, Broadway Cotton Factory, which paid the insurance
injuries and the consequent hospital confinement. premiums. On 24 December 1953, a fire broke out which totally destroyed the
- Plaintiff filed on June 26, 1985 the complaint against DMTC and its driver. Agpalo was Broadway Cotton Factory. Fighting his way out of the factory, plaintiff was injured on
later dropped as a party defendant because he could not be served with summons. the left hand by a heavy object. He was brought to the Manila Central University
Upon filing its answer, defendant DMTC filed a thirdparty complaint against First hospital, and after receiving first-aid, he went to the National Orthopedic Hospital for
Quezon City Insurance Co., Inc. September 17, 1985, third-party defendant filed its treatment of his injuries (fractures in index, middle, fourth, and fifth fingers of left
answer to the third-party complaint. hand). From 26 December 1953 to 8 February 1954, he underwent medical treatment
- TC held DMTC complaint dismissed for lack of merit and as regards the third-party in the hospital. The above-described physical injuries have caused temporary total
complaint First Quezon City Insurance Co., Inc. was to indemnify third-party plaintiff disability of plaintiff’s left hand. Plaintiff filed the corresponding notice of accident and
DMTC in the sum of P12,000.00 with interest. There being no satisfactory warrant the notice of claim with all of the above-named defendants to recover indemnity.
court dismissed the rest of the claims in the complaint and third-party complaint. Defendants rejected plaintiff’s claim for indemnity for the reason that there being no
- The bus company appealed to the CA, which modified the dispositive as regards the severance of amputation of the left hand, the disability suffered by him was not
third-party complaint, that the third-party defendant First Quezon City Insurance Co., covered by his policy.
Inc. be ordered to indemnify third-party plaintiff DMTC the SUM of P50,000.00 with - Plaintiff sued the defendants in the Municipality Court of this City, which dismissed
legal interest. Insurance company filed a MFR which was denied. his complaints. Thereafter, the plaintiff appealed to the Court of First Instance Manila,
Hence, this petition for review, assailing the appellate courts' interpretation of the presided by Judge Gregorio S. Narvasa, which absolved the defendants from the
provision of the insurance contract on the limit of the insurer's liability. complaints. Hence, the appeal.

ISSUE ISSUE
WON the CA erred in the interpretation of the insurance contract on the limit of the WON Diosdado Ty is entitled to indemnity under the insurance policy for the disability
insurer’s liability of his left hand
iNsuRanCe A2010 Dean Carale
pAgE 9♥

- When the repairs on the car had already been made, Misamis made a report of the
HELD accident to Capital Insurance.
- The agreement contained in the insurance policies is the law between the parties. As - Since Capital refused to pay for the total cost of to wage and repairs, suit was filed in
the terms of the policies are clear, express and specific that only amputation of the the municipal court originally.
left hand should be considered as a loss thereof, an interpretation that would include - The defendant-appellant admits liability in the amount of P150, but not for any
the mere fracture or other temporary disability not covered by the policies would excess thereof. The lower court did not exonerate the said appellant for the excess
certainly be unwarranted. In the case at bar, due to the clarity of the stipulation, because the company's absolution would render the insurance contract one-sided and
distinction between “temporary disability” and “total disability” need not be made in that the said insurer had not shown that the cost of repairs in the sum of P302.27 is
relation to one’s occupation means that the condition of the insurance is such that unreasonable, excessive or padded, nor had it shown that it could have undertaken
common prudence requires him to desist from transacting his business or renders him the repairs itself at less expense.
incapable of working. While the Court sympathizes with the plaintiff or his employer,
for whose benefit the policies were issued, it can not go beyond the clear and express ISSUE
conditions of the insurance policies, all of which define partial disability as loss of WON Capital Insurance can be made to pay more than P150
either hand by a amputation through the bones of the wrist.” There was no such
amputation in the case at bar. HELD
- The Supreme Court affirmed the appealed decision, with costs against the plaintiff- NO
appellant. - The insurance policy stipulated in paragraph 4 that if the insured authorizes the
repair the liability of the insurer, per its sub-paragraph (a), is limited to P150.00. The
literal meaning of this stipulation must control, it being the actual contract, expressly
MISAMIS LUMBER V CAPITAL INSURANCE
and plainly provided for in the policy.
17 SCRA 288 - Recourse to legal hermeneutics is not called for because paragraph 4 of the policy is
REYES; May 20, 1966 clear and specific and leaves no room for interpretation.
- The option to undertake the repairs is accorded to the insurance company per
NATURE paragraph 2. The said company was deprived of the option because the insured took
Direct appeal on a point of law from the judgment of the Court of First Instance of it upon itself to have the repairs made, and only notified the insurer when the repairs
Manila were done. As a consequence, paragraph 4, which limits the company's liability to
P150.00, applies.
FACTS - The insurance contract may be rather onerous ("one-sided", as the lower court put
- Misamis Lumber Corporation, under its former name, Lanao Timber Mills, Inc., it), but that in itself does not justify the abrogation of its express terms, terms which
insured its Ford Falcon motor car for the amount of P14,000 with Capital Insurance & the insured accepted or adhered to and which is the law between the contracting
Surety Company, Inc. The pertinent provisions of the policy provided, as follows: parties.
1. The Company will subject to the Limits of Liability indemnify the Insured against - To require the insurer to prove that the cost of the repairs ordered by the insured is
loss or damage to the Motor Vehicle and its accessories and spare parts whilst unreasonable, when the insurer was not given an opportunity to inspect and assess
thereon. the damage before the repairs were made, is contrary to elementary justice and
2. (a) by accidental collision or overturning or collision or overturning consequent equity.
when mechanical breakdown or consequent upon wear and tear.
3. At its option, the Company may pay in cash the amount of the loss or damage or SUN INSURANCE OFFICE LTD. V CA (TAN)
may repair, reinstate or replace the Motor Vehicle or any part thereof or its
195 SCRA 193
accessories or spare parts. The liability of the Company shall not exceed the value
of the parts lost or damaged and the reasonable cost of fitting such parts or the PARAS; March 13, 1991
value of the Motor Vehicle at the time of the loss or damage whichever is the loss.
The Insured's estimate of value stated in the schedule shall be the maximum NATURE
amount payable by the Company in respect of any claim for loss or damage. Petition for certiorari to review the decision of the CA
4. The Insured may authorize the repair of the Motor Vehicle necessitated by
damage for which the Company may be liable under this policy provided that: FACTS
(a) the estimated cost of such repair does not exceed the authorized Repair Limit. - Private respondent Emilio Tan took from the petitioner a Peso 300,000 property
(b) a detailed estimate of the cost is forwarded to the Company without delay and insurance policy to cover his interest in the electrical insurance store of his brother
providing also that the authorized repair limit is P150.00. housed in a building in Iloilo City on August 15, 1983. Four days after the issuance of
- One night, the insured car, while traveling along in Aurora Boulevard, passed over a the policy, the building including the insured store burned.
water hole which the driver did not see because an oncoming car did not dim its light. - On August 20, 1983, Tan filed his claim for fire loss. Sun Insurance, on February 29,
The crankcase and flywheel housing of the car broke when it hit a hollow block lying 1984, wrote the private respondent denying the claim. On April 3, 1984, private
alongside the water hole. The car was towed and repaired by Morosi Motors at a total respondent wrote another letter to the insurance company requesting reconsideration
cost of P302.27. of the denial. Tan’s lawyer wrote another letter to the insurance company inquiring
iNsuRanCe A2010 Dean Carale
pAgE 10♥

about the April 3 letter which sought for a reconsideration of the denial. In its reply to head office in Makati under the custody of its teller, Maribeth Alampay. The armored
the lawyer’s letter, Sun Insurance reiterated its denial of the claim and enclosed car was driven by Benjamin Magalong Y de Vera, escorted by Security Guard
therein copies of the two previous denials dated February 29, 1984 and May 17, 1985. Saturnino Atiga Y Rosete. Driver Magalong was assigned by PRC Management
- On November 20, 1985, Tan filed a civil case with the RTC. Petition filed a motion to Systems.
dismiss on the alleged ground that the action has already prescribed based on - After an investigation by the Pasay police, driver Magalong and guard Atiga were
Condition 27 of the Insurance Policy which stated that the window to file the charged, together with Batigue , Aquino and John Doe, with violation of P.D. 532 (Anti-
appropriate action with either the Insurance Commission or in any court of competent Highway Robbery Law)
jurisdiction is twelve months from the rejection of the claim. RTC denied the motion - Demands were made by the Producers upon the Fortune to pay the amount of the
and the subsequent motion for reconsideration. The CA likewise denied the petition of loss of P725,000.00 but the latter refused to pay as the loss is excluded from the
Sun Insurance. coverage of the insurance policy specifically under "General Exceptions"
> The company shall not be liable under this policy in respect of x x x (b) any loss
ISSUE caused by any dishonest, fraudulent or criminal act of the insured or any officer,
1. WON the court the filing of a motion for reconsideration interrupts the 12 months employee, partner, director, trustee or authorized representative of the Insured
prescription period to contest the denial of the insurance claim whether acting alone or in conjunction with others.
2. WON the rejection of the claim shall be deemed final only if it contains words to the - Fortune opposes the contention of Producers that Atiga and Magalong are not its
effect that the denial is final "officer, employee, x x x trustee or authorized representative x x x at the time of the
robbery
- Trial Court
HELD > On being “EMPLOYEES”
1. NO Magalong and Atiga were not employees or representatives of Producers as their
- The SC held that Condition 27 of the Insurance policy is very clear and free from any services as armored car driver and as security guard having been merely offered by
doubt or ambiguity. It has to be taken in its plain, ordinary, and popular sense. The PRC Management and by Unicorn Security and which latter firms assigned them to
rejection letter of February 29, 1984 was clear and plain. The Court noted that the one plaintiff. The wages and salaries of both Magalong and Atiga are presumably paid by
year period is likewise in accord with Section 23 of the Insurance Code which states their respective firms, which alone wields the power to dismiss them
that any condition which limits the time for commencing an action to a period of less > On being “AUTHORIZED REPRESENTATIVE”
than one year when the cause of action accrues is void. The right of action, according They were merely an assigned armored car driver and security guard for the money
to the SC, accrues at the time that the claim is rejected at the first instance. A request transfer. It was teller Maribeth Alampay who had "custody" of the P725,000.00 cash
for reconsideration of the denial cannot suspend the running of the prescriptive being transferred along a specified money route
period. The Court noted that the rationale for the one year period is to ensure that the - Court of Appeals
evidence as to the origin and cause of the destruction have not yet disappeared. > affirmed in toto
2. NO > A policy or contract of insurance is to be construed liberally in favor of the insured
- The Court clarified its ruling in Eagle Star Insurance Co. vs Chia Yu where it ruled that and strictly against the insurance company (New Life Enterprises vs. Court of Appeals;
“the cause of action in an insurance contract does not accrue until the Insured’s claim Sun Insurance Office, Ltd. vs. Court of Appeals). Contracts of insurance, like other
is finally rejected by the Insurer” by stating the use of the word “finally” cannot be contracts, are to be construed according to the sense and meaning of the terms which
construed to mean the rejection of a petition for reconsideration. What the court the parties themselves have used. If such terms are clear and unambiguous, they
referred to in effect is the rejection in the first instance as claimed by Sun Insurance must be taken and understood in their plain, ordinary and popular sense (New Life
Disposition The decision of the CA is reversed and set aside. The case is dismissed Enterprises Case; Sun Insurance Office).
> The language used by Fortune in the policy is plain, ordinary and simple. No other
interpretation is necessary. The word "employee" should be taken to mean in the
FORTUNE INSURANCE AND SURETY CO. INC.V CA
ordinary sense. The Labor Code is a special law specifically dealing with/and
(PRODUCERS BANK OF THE PHILIPPINES) specifically designed to protect labor and therefore its definition as to employer-
244 SCRA 308 employee relationships insofar as the application/enforcement of said Code is
DAVIDE; May 23, 1995 concerned must necessarily be inapplicable to an insurance contract. Had it intended
to apply the Labor Code in defining what the word "employee" refers to, it must/
should have so stated expressly in the insurance policy. Said driver and security
NATURE
guard cannot be considered as employees of Producers bank because it has no power
Petition for Review on certiorari of CA decision
to hire or to dismiss said driver and security guard under the contracts except only to
ask for their replacements from the contractors.
FACTS
- Fortune’s Contention
- Producers Bank of the Philippines filed a complaint against Fortune Insurance and
> when Producers commissioned a guard and a driver to transfer its funds from one
Surety Co., Inc. for recovery of P725,000.00 under the policy issued by Fortune. The
branch to another, they effectively and necessarily became its authorized
sum was allegedly lost on June 29, 1987 during a robbery of Producer's armored
representatives in the care and custody of the money. Assuming that they could not
vehicle while it was in transit to transfer the money from its Pasay City Branch to its
be considered authorized representatives, they were, nevertheless, employees of
iNsuRanCe A2010 Dean Carale
pAgE 11♥

Producers. It asserts that the existence of an employer-employee relationship "is


determined by law and being such, it cannot be the subject of agreement." Thus, if HELD
there was in reality an employer-employee relationship between Producers, on the NO
one hand, and Magalong and Atiga, on the other, the provisions in the contracts of Ratio A contract of insurance is a contract of adhesion, thus any ambiguity therein
Producers with PRC Management System for Magalong and with Unicorn Security should be resolved against the insurer, or it should be construed liberally in favor of
Services for Atiga which state that Producers is not their employer and that it is the insured and strictly against the insurer. Limitations of liability should be regarded
absolved from any liability as an employer, would not obliterate the relationship. with extreme jealousy and must be construed in such a way as to preclude the insurer
> an employer-employee relationship depends upon four standards: from non-compliance with its obligation. It goes without saying then that if the terms
(1) the manner of selection and engagement of the putative employee of the contract are clear and unambiguous, there is no room for construction and such
(2) the mode of payment of wages terms cannot be enlarged or diminished by judicial construction.
(3) the presence or absence of a power to dismiss and - An insurance contract is a contract of indemnity upon the terms and conditions
(4) the presence and absence of a power to control the putative employee's conduct. specified therein. It is settled that the terms of the policy constitute the measure of
> Of the four, the right-of-control test has been held to be the decisive factor. It the insurer's liability. In the absence of statutory prohibition to the contrary, insurance
asserts that the power of control over Magalong and Atiga was vested in and companies have the same rights as individuals to limit their liability and to impose
exercised by Producers. Fortune further insists that PRC Management System and whatever conditions they deem best upon their obligations not inconsistent with
Unicorn Security Services are but "labor-only" contractors under Article 106 of the public policy.
Labor Code which provides: Reasoning
Art. 106. Contractor or subcontractor. - There is "labor-only" contracting where the - It should be noted that the insurance policy entered into by the parties is a theft or
person supplying workers to an employer does not have substantial capital or robbery insurance policy which is a form of casualty insurance. Section 174 of the
investment in the form of tools, equipment, machineries, work premises, among Insurance Code provides:
others, and the workers recruited and placed by such persons are performing Sec. 174. Casualty insurance is insurance covering loss or liability arising from
activities which are directly related to the principal business of such employer. In accident or mishap, excluding certain types of loss which by law or custom are
such cases, the person or intermediary shall be considered merely as an agent of considered as failing exclusively within the scope of insurance such as fire or
the employer who shall be responsible to the workers in the same manner and marine. It includes, but is not limited to, employer's liability insurance, public
extent as if the latter were directly employed by him. liability insurance, motor vehicle liability insurance, plate glass insurance, burglary
> International Timber Corp. vs. NLRC - a "labor-only" contractor is equivalent to a and theft insurance, personal accident and health insurance as written by non-life
finding that there is an employer-employee relationship between the owner of the insurance companies, and other substantially similar kinds of insurance. (italics
project and the employee of the "labor-only" contractor supplied)
- Producer’s Contention - Except with respect to compulsory motor vehicle liability insurance, the Insurance
> Magalong and Atiga were not its employees since it had nothing to do with their Code contains no other provisions applicable to casualty insurance or to robbery
selection and engagement, the payment of their wages, their dismissal, and the insurance in particular. These contracts are, therefore, governed by the general
control of their conduct. provisions applicable to all types of insurance. Outside of these, the rights and
> International Timber Corp. is not applicable to all cases but only when it becomes obligations of the parties must be determined by the terms of their contract, taking
necessary to prevent any violation or circumvention of the Labor Code, a social into consideration its purpose and always in accordance with the general principles of
legislation whose provisions may set aside contracts entered into by parties in order insurance law.
to give protection to the working man. - With the foregoing principles in mind, it may now be asked whether Magalong and
> American President Lines vs. Clave should be applied which stated Atiga qualify as employees or authorized representatives has been aptly observed
In determining the existence of employer-employee relationship, the following that in burglary, robbery, and theft insurance, "the opportunity to defraud the insurer -
elements are generally considered, namely: (1) the selection and engagement of the moral hazard - is so great that insurers have found it necessary to fill up their
the employee; (2) the payment of wages; (3) the power of dismissal; and (4) the policies with countless restrictions, many designed to reduce this hazard. Seldom does
power to control the employee's conduct. the insurer assume the risk of all losses due to the hazards insured against." Persons
- Since under Producers' contract with PRC Management Systems it is the latter which frequently excluded under such provisions are those in the insured's service and
assigned Magalong as the driver of Producers' armored car and was responsible for his employment. The purpose of the exception is to guard against liability should the theft
faithful discharge of his duties and responsibilities, and since Producers paid the be committed by one having unrestricted access to the property. In such cases, the
monthly compensation of P1,400.00 per driver to PRC Management Systems and not terms specifying the excluded classes are to be given their meaning as understood in
to Magalong, it is clear that Magalong was not Producers' employee. As to Atiga, common speech. The terms "service" and "employment" are generally associated with
Producers relies on the provision of its contract with Unicorn Security Services which the idea of selection, control, and compensation.
provides that the guards of the latter "are in no sense employees of the CLIENT." - There is marked disagreement between the parties on the correct meaning of the
terms "employee" and "authorized representatives."
ISSUE It is clear to us that insofar as Fortune is concerned, it was its intention to exclude and
WON Fortune Insurance and Surety Co. Inc. is liable under the Money, Security, and exempt from protection and coverage losses arising from dishonest, fraudulent, or
Payroll Robbery policy it issued to Producers Bank of the Philippines or WON recovery criminal acts of persons granted or having unrestricted access to Producers' money or
is precluded under the general exceptions clause of the policy payroll. When it used then the term "employee," it must have had in mind any person
iNsuRanCe A2010 Dean Carale
pAgE 12♥

who qualifies as such as generally and universally understood, or jurisprudentially HELD


established in the light of the four standards in the determination of the employer- 1. NO
employee relationship or as statutorily declared even in a limited sense as in the case Ratio As the insurance contract is the law between the parties, Verendia is deemed
of Article 106 of the Labor Code which considers the employees under a "labor-only" to have forfeited his right to claim by the misrepresentation he made.
contract as employees of the party employing them and not of the party who Reasoning
supplied them to the employer. - the court reviewed the factual findings of the courts below, since it appears that
- But even granting for the sake of argument that these contracts were not "labor- there was a misapprehension of the facts by the CA.
only" contracts, and PRC Management Systems and Unicorn Security Services were - Verendia is found to have concocted the lease contract to deflect responsibility for
truly independent contractors, we are satisfied that Magalong and Atiga were, in the fire towards an alleged lessee, even making it appear that the alleged lessee had
respect of the transfer of Producer's money from its Pasay City branch to its head disappeared, inflated the value of the property, and insured same property with two
office in Makati, its "authorized representatives" who served as such with its teller other companies.
Maribeth Alampay. Howsoever viewed, Producers entrusted the three with the specific - An insurance contract is the law between the parties, its terms and conditions
duty to safely transfer the money to its head office, with Alampay to be responsible constitute the measure of the insurer’s liability and compliance therewith is a
for its custody in transit; Magalong to drive the armored vehicle which would carry the condition precedent to the insured’s right to recovery from the insurer.
money; and Atiga to provide the needed security for the money, the vehicle, and his - As it is also a contract of adhesion, an insurance contract should be liberally
two other companions. In short, for these particular tasks, the three acted as agents construed in favor of the insured and strictly against the insurer company which
of Producers. A "representative" is defined as one who represents or stands in the usually prepares it.
place of another; one who represents others or another in a special capacity, as an - Considering, however, the fact that Verendia used a false lease contract to support
agent, and is interchangeable with "agent." his claim, the terms of the policy should be strictly construed against the insured.
Disposition instant petition is hereby GRANTED. CA decision and RTC Makati Verendia failed to live by the terms of the policy, specifically Section 13 thereof which
decision are REVERSED and SET ASIDE. Civil Case is DISMISSED. is expressed in terms that are clear and unambiguous, that all benefits under the
policy shall be forfeited “If the claim be in any respect fraudulent, or if any false
declaration be made or used in support thereof, or if any fraudulent means or devises
VERENDIA V CA (FIDELITY & SURETY CO. OF THE PHILS)
are used by the Insured or anyone acting in his behalf to obtain any benefit under the
217 SCRA 417 policy”. Verendia, having presented a false declaration to support his claim for
MELO; January 22, 1993 benefits in the form of a fraudulent lease contract, he forfeited all benefits therein by
virtue of Section 13 of the policy in the absence of proof that Fidelity waived such
NATURE provision. Worse yet, by presenting a false lease contract, Verendia reprehensibly
Petition to review decision of the CA disregarded the principle that insurance contracts are uberrimae fidae and demand
the most abundant good faith.
FACTS Disposition Decision of CA reversed, and that of RTC is reinstated.
- Fidelity Co. issued a Fire Insurance Policy covering Verendia’s residential building in FIELDMEN'S INSURANCE CO. INC V VDA. DE SONGCO
the amount of P385k. Verendia also insured the same building with two other 25 SCRA 20
companies (Country Bankers Insurance for P56k, and Development Insurance for
FERNANDO; 1968
P400k).
- While all 3 policies were in force, the insured property was completely destroyed by
fire. Verendia filed a claim against Fidelity, but the latter refused payment, thus a FACTS
complaint was filed in the RTC. - An insurance firm, petitioner Fieldmen's Insurance Co., Inc., was not allowed to
Fidelity’s reason for refusal: the policy was avoided by reason of over-insurance, and escape liability under a common carrier insurance policy on the pretext that what was
that Verendia maliciously represented that the building was under lease to a Roberto insured, not once but twice, was a private vehicle and not a common carrier, the
Garcia, when it was actually a Marcelo Garcia who was the lessee. policy being issued upon the insistence of its agent who discounted fears of the
- RTC: policy was violated by Verendia when it failed to inform Fidelity of his other insured that his privately owned vehicle might not fall within its terms, the insured
insurance coverages, thus no need to pay. moreover being "a man of scant education," finishing only the first grade. So it was
- CA: reversed decision held in a decision of the lower court thereafter affirmed by respondent Court of
Appeals. Petitioner in seeking the review of the above decision of respondent Court of
ISSUE Appeals cannot be so sanguine as to entertain the belief that a different outcome
(There is a procedural issue involved here, but is irrelevant to our discussion. It could be expected. To be more explicit, we sustain the Court of Appeals.
concerns the filing of a motion for extension of time to file a motion for - The facts as found by respondent Court of Appeals, binding upon us, follow: "This is a
reconsideration, where the court said that although it now prohibits filing of such peculiar case. Federico Songco of Floridablanca, Pampanga, a man of scant education
motion, the instant motion was filed before the effectivity of this rule, thus allowing being only a first grader ..., owned a private jeepney with Plate No. 41-289 for the
the adjudication of the case) year 1960. On September 15, 1960, as such private vehicle owner, he was induced by
WON Fidelity was liable to pay Verendia considering the circumstances Fieldmen's Insurance Company Pampanga agent Benjamin Sambat to apply for a
Common Carrier's Liability Insurance Policy covering his motor vehicle ... Upon paying
iNsuRanCe A2010 Dean Carale
pAgE 13♥

an annual premium of P16.50, defendant Fieldmen's Insurance Company, Inc. issued the policy 5 was inescapable was set forth in the decision of respondent Court of
on September 19, 1960, Common Carriers Accident Insurance Policy No. 45-HO- Appeals. Thus: "Since some of the conditions contained in the policy issued by the
4254 ... the duration of which will be for one (1) year, effective September 15, 1960 to defendant-appellant were impossible to comply with under the existing conditions at
September 15, 1961. On September 22, 1961, the defendant company, upon payment the time and 'inconsistent with the known facts,' the insurer 'is estopped from
of the corresponding premium, renewed the policy by extending the coverage from asserting breach of such conditions.' From this jurisprudence, we find no valid reason
October 15, 1961 to October 15, 1962. This time Federico Songco's private jeepney to deviate and consequently hold that the decision appealed from should be affirmed.
carried Plate No. J-68136-Pampanga-1961. ... On October 29, 1961, during the The injured parties, to wit, Carlos Songco, Angelito Songco and Jose Manuel, for whose
effectivity of the renewed policy, the insured vehicle while being driven by Rodolfo hospital and medical expenses the defendant company was being made liable, were
Songco, a duly licensed driver and son of Federico (the vehicle owner) collided with a passengers of the jeepney at the time of the occurrence, and Rodolfo Songco, for
car in the municipality of Calumpit, province of Bulacan, as a result of which mishap whose burial expenses the defendant company was also being made liable was the
Federico Songco (father) and Rodolfo Songco (son) died, Carlos Songco (another son), driver of the vehicle in question. Except for the fact, that they were not fare paying
the latter's wife, Angelita Songco, and a family friend by the name of Jose Manuel passengers, their status as beneficiaries under the policy is recognized therein." 6
sustained physical injuries of varying degree." 1 - Even if it be assumed that there was an ambiguity, an excerpt from the Qua Chee
- It was further shown according to the decision of respondent Court of Appeals: "Amor Gan decision would reveal anew the weakness of petitioner's contention. Thus:
Songco, 42-year-old son of deceased Federico Songco, testifying as witness, declared "Moreover, taking into account the well known rule that ambiguities or obscurities
that when insurance agent Benjamin Sambat was inducing his father to insure his must be strictly interpreted against the party that caused them, the 'memo of
vehicle, he butted in saying: 'That cannot be, Mr. Sambat, because our vehicle is an warranty' invoked by appellant bars the latter from questioning the existence of the
"owner" private vehicle and not for passengers,' to which agent Sambat replied: appliances called for in the insured premises, since its initial expression, 'the
'whether our vehicle was an "owner" type or for passengers it could be insured undernoted appliances for the extinction of fire being kept on the premises insured
because their company is not owned by the Government and the Government has hereby, ... it is hereby warranted ...,' admits of interpretation as an admission of the
nothing to do with their company. So they could do what they please whenever they existence of such appliances which appellant cannot now contradict, should the parol
believe a vehicle is insurable' ... In spite of the fact that the present case was filed and evidence rule apply." 7
tried in the CFI of Pampanga, the defendant company did not even care to rebut Amor - To the same effect is the following citation from the same leading case: "This rigid
Songco's testimony by calling on the witness-stand agent Benjamin Sambat, its application of the rule on ambiguities has become necessary in view of current
Pampanga Field Representative." 2 business practices. The courts cannot ignore that nowadays monopolies, cartels and
- The plaintiffs in the lower court, likewise respondents here, were the surviving widow concentration of capital, endowed with overwhelming economic power, manage to
and children of the deceased Federico Songco as well as the injured passenger Jose impose upon parties dealing with them cunningly prepared 'agreements' that the
Manuel. On the above facts they prevailed, as had been mentioned, in the lower court weaker party may not change one whit, his participation in the 'agreement' being
and in the respondent Court of Appeals.1awphîl.nèt reduced to the alternative to 'take it or leave it' labelled since Raymond Saleilles
- The basis for the favorable judgment is the doctrine announced in Qua Chee Gan v. 'contracts by adherence' (contrats d'adhesion), in contrast to those entered into by
Law Union and Rock Insurance Co., Ltd., 3 with Justice J. B. L. Reyes speaking for the parties bargaining on an equal footing, such contracts (of which policies of insurance
Court. It is now beyond question that where inequitable conduct is shown by an and international bills of lading are prime examples) obviously call for greater
insurance firm, it is "estopped from enforcing forfeitures in its favor, in order to strictness and vigilance on the part of courts of justice with a view to protecting the
forestall fraud or imposition on the insured." 4 weaker party from abuses and imposition, and prevent their becoming traps for the
- As much, if not much more so than the Qua Chee Gan decision, this is a case where unwary (New Civil Code. Article 24; Sent. of Supreme Court of Spain, 13 Dec. 1934, 27
the doctrine of estoppel undeniably calls for application. After petitioner Fieldmen's February 1942)." 8
Insurance Co., Inc. had led the insured Federico Songco to believe that he could - The last error assigned which would find fault with the decision of respondent Court
qualify under the common carrier liability insurance policy, and to enter into contract of Appeals insofar as it affirmed the lower court award for exemplary damages as well
of insurance paying the premiums due, it could not, thereafter, in any litigation arising as attorney's fees is, on its face, of no persuasive force at all.
out of such representation, be permitted to change its stand to the detriment of the - The conclusion that inescapably emerges from the above is the correctness of the
heirs of the insured. As estoppel is primarily based on the doctrine of good faith and decision of respondent Court of Appeals sought to be reviewed. For, to borrow once
the avoidance of harm that will befall the innocent party due to its injurious reliance, again from the language of the Qua Chee Gan opinion: "The contract of insurance is
the failure to apply it in this case would result in a gross travesty of justice. one of perfect good faith (uberima fides) not for the insured alone,but equally so for
- That is all that needs be said insofar as the first alleged error of respondent Court of the insurer; in fact, it is more so for the latter, since its dominant bargaining position
Appeals is concerned, petitioner being adamant in its far-from-reasonable plea that carries with it stricter responsibility." 9
estoppel could not be invoked by the heirs of the insured as a bar to the alleged - This is merely to stress that while the morality of the business world is not the
breach of warranty and condition in the policy. lt would now rely on the fact that the morality of institutions of rectitude like the pulpit and the academe, it cannot descend
insured owned a private vehicle, not a common carrier, something which it knew all so low as to be another name for guile or deception. Moreover, should it happen thus,
along when not once but twice its agent, no doubt without any objection in its part, no court of justice should allow itself to lend its approval and support.1awphîl.nèt
exerted the utmost pressure on the insured, a man of scant education, to enter into - We have no choice but to recognize the monetary responsibility of petitioner
such a contract. Fieldmen's Insurance Co., Inc. It did not succeed in its persistent effort to avoid
- Nor is there any merit to the second alleged error of respondent Court that no legal complying with its obligation in the lower court and the Court of Appeals. Much less
liability was incurred under the policy by petitioner. Why liability under the terms of
iNsuRanCe A2010 Dean Carale
pAgE 14♥

should it find any receptivity from us for its unwarranted and unjustified plea to proferentum. Risk policies should be construed reasonably and in a manner as to
escape from its liability. make effective the intentions and expectations of the parties.
- the policies clearly stipulate that they cover the risks of non-delivery of an entire
MALAYAN INSURANCE CORP. V CA (TKC MARKETING CORP.) package and that it was petitioner itself that invited and granted the extensions and
collected premiums thereon.
270 SCRA 242
ROMERO; March 20, 1997 ISSUES
1. WON the arrest of the vessel was a risk covered under the subject insurance
NATURE policies
Petition for review on certiorari 2. WON insurance policies should be strictly construed against the insurer

FACTS HELD
- TKC Marketing Corp. was the owner/consignee of some 3,189.171 metric tons of 1.YES
soya bean meal which was loaded on board the ship MV Al Kaziemah for carriage from - With the incorporation of subsection 1.1 of Section 1 of the Institute War Clauses,
the port of Rio del Grande, Brazil, to the port of Manila. Said cargo was insured against "arrest" caused by ordinary judicial process is deemed included among the covered
the risk of loss by petitioner Malayan Insurance Corporation for which it issued two (2) risks. This interpretation becomes inevitable when subsection 1.1 of Section 1 of the
Marine Cargo Policies. Institute War Clauses provided that "this insurance covers the risks excluded from the
- While the vessel was docked in Durban, South Africa the civil authorities arrested Standard Form of English Marine Policy by the clause 'Warranted free of capture,
and detained it because of a lawsuit on a question of ownership and possession. TKC seizure, arrest, etc. x x x'" or the F.C. & S. Clause. Jurisprudentially, "arrests" caused
Marketing notified Malayan of the arrest of the vessel and made a formal claim for the by ordinary judicial process is also a risk excluded from the Standard Form of English
dollar equivalent on the policies (US$916,886.66) for non-delivery of the cargo. It Marine Policy by the F.C. & S. Clause.
likewise sought the assistance of Malayan on what to do with the cargo. - Petitioner cannot adopt the argument that the "arrest" caused by ordinary judicial
- Malayan replied that the arrest of the vessel by civil authority was not a peril process is not included in the covered risk simply because the F.C. & S. Clause under
covered by the policies. TKC advised Malayan that it might tranship the cargo and the Institute War Clauses can only be operative in case of hostilities or warlike
requested an extension of the insurance coverage until actual transhipment, which operations on account of its heading "Institute War Clauses."
extension was approved upon payment of additional premium. The insurance 2. YES
coverage was extended under the same terms and conditions embodied in the Ratio Insurance Policies should be construed liberally in favor of the insured and
original policies while in the process of making arrangements for the transhipment of strictly against the insurer.
the cargo from Durban to Manila. However the cargo was sold in Durban, South Africa, Reasoning
for US$154.40 per metric ton or a total of P10,304,231.75 due to its perishable nature - An insurance contract should be so interpreted as to carry out the purpose for which
which could no longer stand a voyage of twenty days to Manila and another twenty the parties entered into the contract which is, to insure against risks of loss or damage
days for the discharge thereof. It reduced its claim to US$448,806.09 (or its peso to the goods. Such interpretation should result from the natural and reasonable
equivalent of P9,879,928.89 at the exchange rate of P22.0138 per $1.00) representing meaning of language in the policy. Where restrictive provisions are open to two
its loss after the proceeds of the sale were deducted from the original claim.Malayan interpretations, that which is most favorable to the insured is adopted.
maintained its position that the arrest of the vessel by civil authorities on a question Indemnity and liability insurance policies are construed in accordance with the general
of ownership was an excepted risk under the marine insurance policies. rule of resolving any ambiguity therein in favor of the insured, where the contract or
Petitioners Claim policy is prepared by the insurer. A contract of insurance, being a contract of
- an arrest by civil authority is not compensable since the term "arrest" refers to adhesion, par excellence, any ambiguity therein should be resolved against the
"political or executive acts" and does not include a loss caused by riot or by ordinary insurer.Limitations of liability should be regarded with extreme jealousy and must be
judicial process as in this case construed in such a way as to preclude the insurer from noncompliance with its
- the deletion of the Free from Capture or Seizure Clause would leave the assured obligations
covered solely for the perils specified by the wording of the policy itself - It must be borne in mind that such contracts are invariably prepared by the
- the rationale for the exclusion of an arrest pursuant to judicial authorities is to companies and must be accepted by the insured in the form in which they are written.
eliminate collusion between unscrupulous assured and civil authorities. Any construction of a marine policy rendering it void should be avoided. Such policies
- any loss which private respondent may have incurred was in the nature and form of will, therefore, be construed strictly against the company in order to avoid a forfeiture,
unrecovered acquisition value brought about by a voluntary sacrifice sale and not by unless no other result is possible from the language used.
arrest, detention or seizure of the ship. - If a marine insurance company desires to limit or restrict the operation of the general
- its act of rejecting the claim was a result of its honest belief that the arrest of the provisions of its contract by special proviso, exception, or exemption, it should
vessel was not a compensable risk under the policies issued express such limitation in clear and unmistakable language.
Respondents Comments Be that as it may, exceptions to the general coverage are construed most strongly
- petitioner, being the sole author of the policies, "arrests" should be strictly against the company. Even an express exception in a policy is to be construed against
interpreted against it because the rule is that any ambiguity is to be taken contra the underwriters by whom the policy is framed, and for whose benefit the exception is
introduced.
iNsuRanCe A2010 Dean Carale
pAgE 15♥

Ratio An insurance policy being in the nature of an adhesion contract is to be strictly


construed against the insurer and liberally in favor of the insured.
WESTERN GUARANTY CORPORATION V CA (RODRIGUEZ, and
Reasoning
DE DIOS TRANSPORTATION CO) - Firstly, the Schedule of Indemnities does not purport to restrict the kinds of damages
187 SCRA 652 that may be awarded against Western once liability has arisen. Section 1, quoted
FELICIANO; July 20, 1990 above, does refer to certain "Limits of Liability" which in the case of the third party
liability section of the Master Policy, is apparently P50,000.00 per person per accident.
Within this over-all quantitative limit, all kinds of damages allowable by law "actual
FACTS
or compensatory damages"; "moral damages"; "nominal damages"; "temperate or
- At around 4:30 in the afternoon of 27 March 1982, while crossing Airport Road on a
moderate damages"; "liquidated damages"; and "exemplary damages" may be
pedestrian lane on her way to work, respondent Priscilla E. Rodriguez was struck by a
awarded by a competent court against the insurer once liability is shown to have
De Dios passenger bus owned by respondent De Dios Transportation Co., Inc., then
arisen, and the essential requisites or conditions for grant of each species of damages
driven by one Walter Saga y Aspero. The bus driver disregarded the stop signal given
are present. It appears to us self-evident that the Schedule of Indemnities was not
by a traffic policeman to allow pedestrians to cross the road. Priscilla was thrown to
intended to be an enumeration, much less a closed enumeration, of the specific kinds
the ground, hitting her forehead. She was treated at the Protacio Emergency Hospital
of damages which may be awarded under the Master Policy Western has issued.
and later on hospitalized at the San Juan De Dios Hospital. Her face was permanently
- Secondly, the reading urged by Western of the Schedule of Indemnities comes too
disfigured, causing her serious anxiety and moral distress.
close to working fraud upon both the insured and the third party beneficiary of Section
- Respondent bus company was insured with petitioner Western Guaranty Corporation
1, quoted above. For Western's reading would drastically and without warning limit
("Western") under its Master Policy which enumerated specific liabilities of the
the otherwise unlimited (save for the over-all quantitative limit of liability of
insurance company and ended with a clause to clarify the limitations of the amount
P50,000.00 per person per accident) and comprehensive scope of liability assumed by
which could be granted as indemnity.
the insurer Western under Section 1: "all sums necessary to discharge liability of the
- Respondent Priscilla Rodriguez filed a complaint for damages before the Regional
insured in respect of [bodily injury to a third party]". This result which is not
Trial Court of Makati against De Dios Transportation Co. and Walter A. Saga.
essentially different from taking away with the left hand what had been given with the
Respondent De Dios Transportation Co., in turn, filed a third-party complaint against
right hand we must avoid as obviously repugnant to public policy. If what Western
its insurance carrier, petitioner Western.
now urges is what Western intended to achieve by its Schedule of Indemnities, it was
- On 6 August 1985, the trial court rendered a decision in favor of respondent Priscilla
incumbent upon Western to use language far more specific and precise than that used
E. Rodriguez,
in fact by Western, so that the insured, and potential purchasers of its Master Policy,
- On appeal, the Court of Appeals affirmed in toto the decision of the trial court.
and the Office of the Insurance Commissioner, may be properly informed and act
Petitioner moved for the reconsideration of the appellate court's decision. In a
accordingly.
Resolution dated 10 January 1990, the Court of Appeals denied the motion for
- Petitioner Western would have us construe the Schedule of Indemnities as
reconsideration for lack of merit. Petitioner Western is now before us on a Petition for
comprising contractual limitations of liability which, as already noted, is
Review alleging that the Court of Appeals erred in holding petitioner liable to pay
comprehensively defined in Section 1 "Liability to the Public" of the Master Policy. It
beyond the limits set forth in the Schedule Indemnities and in finding Western liable
is well-settled, however, that contractual limitations of liability found in insurance
for loss of earnings, moral damages and attorney's fees. Succinctly stated, it is
contracts should be regarded by courts with a jaundiced eye and extreme care and
petitioner Western's position that it cannot be held liable for loss of earnings, moral
should be so construed as to preclude the insurer from evading compliance with its
damages and attorney's fees because these items are not among those included in
just obligations.
the Schedule Indemnities set forth in the insurance policy.
- Finally, an insurance contract is a contract of adhesion. The rule is well entrenched in
- Petitioner Western in effect contends before this Court, as it did before the Court of
our jurisprudence that the terms of such contract are to be construed strictly against
Appeals, that because the Schedule of Indemnities limits the amount payable for
the party which prepared the contract, which in this case happens to be petitioner
certain kinds of expenses "hospital room", "surgical expenses", "an aesthesiologists'
Western.
fee", "operating room" and "medical expenses" that Schedule should be read as
excluding liability for any other type of expense or damage or loss even though
actually sustained or incurred by the third party victim. We are not persuaded by QUA CHEE GAN V LAW UNION AND ROCK INSURANCE CO.,
Western's contention. LTD.
96 PHIL 85
ISSUE
WON the Schedule of indemnities as stated in the insurance policy should be REYES; December 17, 1955
construed strictly to exclude all others not explicitly stated therein
NATURE
An appeal by defendant insurance company from the decision of CFI in favor of the
plaintiff
HELD
NO FACTS
iNsuRanCe A2010 Dean Carale
pAgE 16♥

- before the last war, plaintiff-appellee owned 4 warehouses or bodegas in Tabaco, 3. WON the insured connived at the loss and fraudulently inflated the quantity of the
Albay, used for the storage of stocks of copra and of hemp, baled and loose, in which insured stock in the burnt bodegas
the appellee dealt extensively. They had been, with their contents, insured with the
defendant Company since 1937, and the loose made payable to the Philippine HELD
National Bank as mortgage of the hemp and crops, to the extent of its interest. 1. NO
- Fire of undetermined origin that broke out in the early morning of July 21, 1940, and Ratio It is usually held that where the insurer, at the time of the issuance of a policy
lasted almost one week, gutted and completely destroyed Bodegas Nos. 1, 2 and 4, of insurance, has knowledge of existing facts which, if insisted on, would invalidate
with the merchandise stored therein. Plaintiff-appellee informed the insurer by the contract from its very inception, such knowledge constitutes a waiver of conditions
telegram on the same date; and on the next day, the fire adjusters engaged by in the contract inconsistent with the facts, and the insurer is stopped thereafter from
appellant insurance company arrived and proceeded to examine and photograph the asserting the breach of such conditions. The law is charitable enough to assume, in
premises, pored over the books of the insured and conducted an extensive the absence of any showing to the contrary, that an insurance company intends to
investigation. The plaintiff having submitted the corresponding fire claims, totalling executed a valid contract in return for the premium received; and when the policy
P398,562.81 (but reduced to the full amount of the insurance, P370,000), the contains a condition which renders it voidable at its inception, and this result is known
Insurance Company resisted payment, claiming violation of warranties and conditions, to the insurer, it will be presumed to have intended to waive the conditions and to
filing of fraudulent claims, and that the fire had been deliberately caused by the execute a binding contract, rather than to have deceived the insured into thinking he
insured or by other persons in connivance with him. is insured when in fact he is not, and to have taken his money without consideration.
- Que Chee Gan, with his brother, Qua Chee Pao, and some employees of his, were Reasoning
indicted and tried in 1940 for the crime of arson, it being claimed that they had set - The appellant is barred estoppel to claim violation of the so-called fire hydrants
fire to the destroyed warehouses to collect the insurance. They were, however, warranty, for the reason that knowing fully all that the number of hydrants demanded
acquitted by the trial court. therein never existed from the very beginning, the appellant neverthless issued the
- the civil suit to collect the insurance money proceeded to its trial with the CFI holding policies in question subject to such warranty, and received the corresponding
that: judgment is rendered for the plaintiff and against the defendant condemning the premiums. The insurance company was aware, even before the policies were issued,
latter to pay the former — (a) Under the first cause of action, the sum of P146,394.48; that in the premises insured there were only two fire hydrants installed by Qua Chee
(b) Under the second cause of action, the sum of P150,000; (c) Under the third cause Gan and two others nearby, owned by the municipality of Tabaco, contrary to the
of action, the sum of P5,000; (d) Under the fourth cause of action, the sum of P15,000; requirements of the warranty in question
and (e) Under the fifth cause of action, the sum of P40,000; all of which shall bear - The plain, human justice of this doctrine is perfectly apparent. To allow a
interest at the rate of 8% per annum in accordance with Section 91 (b) of the company to accept one's money for a policy of insurance which it then
Insurance Act from September 26, 1940, until each is paid, with costs against the knows to be void and of no effect, though it knows as it must, that the
defendant. assured believes it to be valid and binding, is so contrary to the dictates of
- In its first assignment of error, the insurance company alleges that the trial Court honesty and fair dealing, and so closely related to positive fraud, as to the
should have held that the policies were avoided for breach of warranty, specifically abhorrent to fair-minded men. It would be to allow the company to treat the
the one appearing on a rider pasted (with other similar riders) on the face of the policy as valid long enough to get the premium on it, and leave it at liberty
policies.4 to repudiate it the next moment. This cannot be deemed to be the real
- It is argued that since the bodegas insured had an external wall perimeter of 500 intention of the parties. To hold that a literal construction of the policy
meters or 1,640 feet, the appellee should have 11 fire hydrants in the compound, and expressed the true intention of the company would be to indict it, for
that he actually had only 2, with a further pair nearby, belonging to the municipality of fraudulent purposes and designs which we cannot believe it to be guilty of.
Tabaco. - The appellant company so worded the policies that while exacting the greater
number of fire hydrants and appliances, it kept the premium discount at the minimum
ISSUES of 2 1/2%, thereby giving the insurance company a double benefit. Such abnormal
1. WON the defendant-appellant can claim the policies it had issued as void ab initio treatment of the insured strongly points at an abuse of the insurance company's
2. WON the insured violated the "Hemp Warranty" provisions of Policy No. 2637165 selection of the words and terms of the contract, over which it had absolute control.
against the storage of gasoline - Receipt of Premiums or Assessments after Cause for Forfeiture Other than
Nonpayment. — It is a well settled rule of law that an insurer which with knowledge of
facts entitling it to treat a policy as no longer in force, receives and accepts a
4 premium on the policy, estopped to take advantage of the forfeiture. It cannot treat
Memo. of Warranty. — The undernoted Appliances for the extinction of fire being kept on the premises insured
hereby, and it being declared and understood that there is an ample and constant water supply with sufficient the policy as void for the purpose of defense to an action to recover for a loss
pressure available at all seasons for the same, it is hereby warranted that the said appliances shall be maintained thereafter occurring and at the same time treat it as valid for the purpose of earning
in efficient working order during the currency of this policy, by reason whereof a discount of 2 1/2 per cent is and collecting further premiums.
allowed on the premium chargeable under this policy.
Hydrants in the compound, not less in number than one for each 150 feet of external wall measurement of - Moreover, taking into account the well known rule that ambiguities or obscurities
building, protected, with not less than 100 feet of hose piping and nozzles for every two hydrants kept under cover must be strictly interpreted against the party that caused them, the "memo of
in convenient places, the hydrants being supplied with water pressure by a pumping engine, or from some other warranty" invoked by appellant bars the latter from questioning the existence of the
source, capable of discharging at the rate of not less than 200 gallons of water per minute into the upper story of
the highest building protected, and a trained brigade of not less than 20 men to work the same.' appliances called for in the insured premises
iNsuRanCe A2010 Dean Carale
pAgE 17♥

On the alleged violations of the plaintiff The alleged violation of the warranty of date they needed is proved by the fact that the adjuster Alexander Stewart was able
100 feet of fire hose for every two hydrants, must be equally rejected, since the to prepare his own balance sheet that did not differ from that submitted by the
appellant's argument thereon is based on the assumption that the insured was bound insured except for the valuation of the merchandise, as expressly found by the Court
to maintain no less than eleven hydrants, which requirement appellant is estopped in the criminal case for arson.
from enforcing. 3. NO
- As to maintenance of a trained fire brigade of 20 men, the record is preponderant Ratio Both defenses are predicted on the assumption that the insured was in financial
that the same was organized, and drilled, from time to give, although not maintained difficulties and set the fire to defraud the insurance company, presumably in order to
as a permanently separate unit, which the warranty did not require. pay off the Philippine National Bank, to which most of the insured hemp and copra
2. NO was pledged. Both defenses are fatally undermined by the established fact that,
Ratio Here, again, by reason of the exclusive control of the insurance company over notwithstanding the insurer's refusal to pay the value of the policies the extensive
the terms and phraseology of the contract, the ambiguity must be held strictly against resources of the insured enabled him to pay off the National Bank in a short time; and
the insurer and liberally in favor of the insured, specially to avoid a forfeiture. if he was able to do so, no motive appears for attempt to defraud the insurer. While
Insurance is, in its nature, complex and difficult for the layman to the acquittal of the insured in the arson case is not res judicata on the present civil
understand. Policies are prepared by experts who know and can anticipate action, the insurer's evidence, to judge from the decision in the criminal case, is
the hearing and possible complications of every contingency. So long as practically identical in both cases and must lead to the same result, since the proof to
insurance companies insist upon the use of ambiguous, intricate and establish the defense of connivance at the fire in order to defraud the insurer "cannot
technical provisions, which conceal rather than frankly disclose, their own be materially less convincing than that required in order to convict the insured of the
intentions, the courts must, in fairness to those who purchase insurance, crime of arson."
construe every ambiguity in favor of the insured. An insurer should not be - As to the defense that the burned bodegas could not possibly have contained the
allowed, by the use of obscure phrases and exceptions, to defeat the very quantities of copra and hemp stated in the fire claims, the insurer's case rests almost
purpose for which the policy was procured. exclusively on the estimates, inferences and conclusions of its adjuster investigator
Reasoning who examined the premises during and after the fire. His testimony, however, was
- Appellee admitted that there were 36 cans of gasoline in the building designed. It based on inferences from the photographs and traces found after the fire, and must
However, gasoline is not specifically mentioned among the prohibited articles listed in yield to the contradictory testimony of those who actually saw the contents of the
the so-called "hemp warranty." The cause relied upon by the insurer speaks of "oils bodegas shortly before the fire, while inspecting them for the mortgagee Bank.
(animal and/or vegetable and/or mineral and/or their liquid products having a flash Disposition We find no reversible error in the judgment appealed from, wherefore
point below 300 Fahrenheit)", and is decidedly ambiguous and uncertain; for in the same is hereby affirmed.
ordinary parlance, "Oils" mean "lubricants" and not gasoline or kerosene. And how
many insured, it may well be wondered, are in a position to understand or determine
DEL ROSARIO V EQUITABLE INSURANCE & CASUALTY CO., INC
"flash point below 300 Fahrenheit.
- If the company intended to rely upon a condition of that character, it ought to have 8 SCRA 343
been plainly expressed in the policy. PAREDES; June 29, 1963
- The contract of insurance is one of perfect good faith not for the insured alone, but
equally so for the insurer; in fact, it is mere so for the latter, since its dominant NATURE
bargaining position carries with it stricter responsibility. Appeal from judgment of CFI Rizal
- Another point that is in favor of the insured is that the gasoline kept in Bodega No. 2
was only incidental to his business, being no more than a customary 2 day's supply for FACTS
the five or six motor vehicles used for transporting of the stored merchandise. "It is - Francisco del Rosario was insured by Equitable Insurance and Casualty Co. Inc under
well settled that the keeping of inflammable oils on the premises though prohibited by Personal Accident Policy no. 7136. The Company bound itself to pay P1000 to P3000
the policy does not void it if such keeping is incidental to the business." as indemnity for the death of the insured.
On the submission of books, voucbers, etc. The charge that the insured failed or - Under the policy:
refused to submit to the examiners of the insurer the books, vouchers, etc. demanded Part I. Indemnity for Death
by them was found unsubstantiated by the trial Court, and no reason has been shown If the insured sustains any bodily injury which is effected solely through
to alter this finding. The insured gave the insurance examiner all the date he asked violent, external, visible and accidental means, and which shall result,
for, and the examiner even kept and photographed some of the examined books in his independently of all other causes and within sixty days from the
possession. What does appear to have been rejected by the insured was the demand occurrence thereof, in the Death of the Insured, the Company shall pay the
that he should submit "a list of all books, vouchers, receipts and other records", but amount set opposite such injury:
the refusal of the insured in this instance was well justified, since the demand for a list
of all the vouchers (which were not in use by the insured) and receipts was positively Section 1. Injury sustained other than those specified below unless
unreasonable, considering that such listing was superfluous because the insurer was excepted hereinafter P1000
not denied access to the records, that the volume of Qua Chee Gan's business ran into Section 2. Injury sustained by the wrecking or disablement of a railroad
millions, and that the demand was made just after the fire when everything was in passenger car or street railway car in or on which the Insured is traveling
turmoil. That the representatives of the insurance company were able to secure all the as a farepaying passenger P1500
iNsuRanCe A2010 Dean Carale
pAgE 18♥

Section 3. Injury sustained by the burning of a church, theatre, public amounts that may be recovered for death for bodily injury, yet, there is not
library or municipal administration building while the Insured is therein at specific amount mentioned in the policy for death thru drowning although the
the commencement of the fire P2000 latter is, under Part VI of the policy, a ground for recovery thereunder. Since the
Section 4. Injury sustained by the wrecking or disablement of a regular defendant has bound itself to pay P1000 to P3000 as indemnity for the death of
passenger elevator car in which the Insured is being conveyed as a the insured but the policy does not positively state any definite amount that may
passenger (Elevator in mines exluded) P2500 be recovered in case of death by drowning, there is an ambiguity in this respect in
Section 5. Injury sustained by a stroke of lightning or by a cyclone P3000 the policy, which ambiguity must be interpreted in favor of the insured and strictly
xxxx xxxx xxxx against the insurer so as to allow a greater indemnity. x x x plaintiff is therefore
Part VI. Exceptions entitled to recover P3000.
This policy shall not cover disappearance of the Insured nor shall it cover Disposition Judgment appealed from is affirmed.
Death, Disability, Hospital fees, or Loss of time, caused to the insured:
x x x (h) By drowning except as a consequence of the wrecking or
GEAGONIA v. CA (COUNTRY BANKERS INSURANCE)
disablement in the Philippine waters of a passenger steam or motor vessel
in which the Insured is traveling as a farepaying passenger; x x x 8 SCRA 343
- A rider to the Policy contained the following; DAVIDE; February 6 1995
IV. DROWNING
It is hereby declared and agreed that exemption clause Letter (h) in PART FACTS
VI of the policy is hereby waived by the company, and to form a part of the -Geagonia is the owner of Norman's Mart located in the public market of San
provision covered by the policy. Francisco, Agusan del Sur. On 22 Dec 1989, he obtained from the private respondent
- Feb 24, 1957, Francisco del Rosario while on board the motor launch ISLAMA, with fire insurance policy for P100,000.00. The period of the policy was from 22 Dec 1989
his beneficiary to the policy, Remedios Jayme, were forced to jump off said launch on to 22 Dec 1990 and covered the ff: "Stock-in-trade consisting principally of dry goods
account of fire which broke out on said vessel, resulting in the death by drowning of such as RTW's for men and women wear and other usual to assured's business.
the insured and his beneficiary. -The policy contained the following condition:
- Simeon del Rosario, the insured’s father, filed a claim for payment with the "3. The insured shall give notice to the Company of any insurance or insurances
company. The company paid him P1000 pursuant to section 1 Part I of the policy. already effected, or which may subsequently be effected, covering any of the property
- On the same date, Atty. Francisco wrote to the company acknowledging receipt by or properties consisting of stocks in trade, goods in process and/or inventories only
his client of the P1000 but informing said company that said amount was not the hereby insured, and unless notice be given and the particulars of such insurance or
correct one. He claimed that the amount payable should be P1500 under the provision insurances be stated therein or endorsed in this policy pursuant to Section 50 of the
of Section 2 Part I, based on the rule of pari materia. Insurance Code, by or on behalf of the Company before the occurrence of any loss or
- The company referred the matter to the Insurance Coomissioner, who was of the damage, all benefits under this policy shall be deemed forfeited, provided however,
opinion that the liability of the company was only P1000. thus the company refused to that this condition shall not apply when the total insurance or insurances in force at
pay more that P1000. Atty. Francisco wrote a subsequent letter to company asking for the time of the loss or damage is not more than P200,000.00."
p3000, which the company refused to pay. -On 27 May 1990, fire of accidental origin broke out at around 7:30 p.m. at the public
- A complaint for recovery of the balance of P2000 was instituted with the CFI Rizal, market of San Francisco, Agusan del Sur. The petitioner's insured stocks-in-trade were
praying for a further sum of P10000 as attorney’s fees, expenses of litigation and completely destroyed prompting him to file w/ the private respondent a claim under
costs. the policy. On 28 Dec 1990, the private respondent denied the claim because it found
- CFI ruled in favor of petitioner, ordering the company to pay P2000 to del Rosario. that at the time of the loss the petitioner's stocks-in-trade were likewise covered by
two fire insurance policies for P100,000.00 each, issued by the Cebu Branch of the
ISSUE Philippines First Insurance Co., Inc. (PFIC).
How much should the indemnity be -The basis of the private respondent's denial was the petitioner's alleged violation of
Condition 3 of the policy.
HELD - Geagonia then filed a complaint against the private respondent w/ the Insurance
- All the parties agree that indemnity has to be paid, but the conflict centers on how Commission for the recovery of P100,000.00 under fire insurance policy, for attorney's
much it should be. fees, and costs of litigation. He claims that the time he obtained the private
- Where there is ambiguity with respect to the terms and conditions of the policy, the respondent's fire insurance policy he knew that the two policies issued by the PFIC
same will be resolved against the one responsible thereof. Generally, the insured has were already in existence; however, he had no knowledge of the provision in the
little, if any, participation in the preparation of the policy, together with the drafting of private respondent's policy requiring him to inform it of the prior policies; this
its terms and conditions. The interpretation of obscure stipulations in a contract requirement was not mentioned to him by the private respondent's agent; and had it
should not favor the party who caused the obscurity. been so mentioned, he would not have withheld such information. He further asserted
- SC agreed with the ruling of the lower court: that the total of the amounts claimed under the three policies was below the actual
x x x death by drowning is a ground for recovery apart from the bodily injury value of his stocks at the time of loss, w/c was P1M.
because death by bodily injury is covered by Part I of the policy while death by - The Insurance Commission found that the petitioner did not violate Condition 3 as he
drowning is covered by Part VI thereof. But while the policy mentions specific had no knowledge of the existence of the two fire insurance policies obtained from the
iNsuRanCe A2010 Dean Carale
pAgE 19♥

PFIC; that it was Cebu Tesing Textiles w/c procured the PFIC policies w/o informing him forfeiture of insurance policies should be construed most strictly against
or securing his consent; and that Cebu Tesing Textile, as his creditor, had insurable those for whose benefits they are inserted, and most favorably toward those
interest on the stocks. These findings were based on the petitioner's testimony that he against whom they are intended to operate. The reason for this is that, except
came to know of the PFIC policies only when he filed his claim with the private for riders which may later be inserted, the insured sees the contract already in its final
respondent and that Cebu Tesing Textile obtained them and paid for their premiums form and has had no voice in the selection or arrangement of the words employed
w/o informing him. The Insurance Commission then ordered the respondent company therein. On the other hand, the language of the contract was carefully chosen and
to pay complainant the sum of P100,000.00 with legal interest from the time the deliberated upon by experts and legal advisers who had acted exclusively in the
complaint was filed until fully satisfied plus the amount of P10,000.00 as attorney's interest of the insurers and the technical language employed therein is rarely
fees. understood by ordinary laymen.
-CA reversed the decision of the Insurance Commission because it found that the - With these principles in mind, we are of the opinion that Condition 3 of the
petitioner knew of the existence of the two other policies issued by the PFIC subject policy is not totally free from ambiguity and must be meticulously
analyzed. Such analysis leads us to conclude that (a) the prohibition applies
ISSUES only to double insurance, and (b) the nullity of the policy shall only be to the
1. WON the petitioner had prior knowledge of the two insurance policies issued by the extent exceeding P200,000.00 of the total policies obtained.
PFIC when he obtained the fire insurance policy from the private respondent, thereby, - Furthermore, by stating within Condition 3 itself that such condition shall not apply if
for not disclosing such fact, violating Condition 3 of the policy the total insurance in force at the time of loss does not exceed P200,000.00, the
2. if he had, WON he is precluded from recovering therefrom private respondent was amenable to assume a co-insurer's liability up to a loss not
exceeding P200,000.00. What it had in mind was to discourage over-insurance.
HELD Indeed, the rationale behind the incorporation of "other insurance" clause in fire
1. YES policies is to prevent over-insurance and thus avert the perpetration of fraud. When a
- We agree w/ the CA that the petitioner knew of the prior policies issued by the PFIC. property owner obtains insurance policies from two or more insurers in a total amount
His letter of 18 January 1991 to the private respondent conclusively proves this that exceeds the property's value, the insured may have an inducement to destroy
knowledge. His testimony to the contrary before the Insurance Commissioner and the property for the purpose of collecting the insurance. The public as well as the
which the latter relied upon cannot prevail over a written admission made ante litem insurer is interested in preventing a situation in which a fire would be profitable to the
motam. It was, indeed, incredible that he did not know about the prior policies since insured.
these policies were not new or original. Disposition Petition granted. The decision of the Court of Appeals in CA-G.R. SP No.
2. NO 31916 is SET ASIDE and the decision of the Insurance Commission in Case No. 3340 is
- It must, however, be underscored that unlike the "other insurance" clauses involved REINSTATED.
in General Insurance and Surety Corp. vs. Ng Hua or in Pioneer Insurance & Surety
Corp. vs. Yap, which read:
SUN INSURANCE OFFICE, LTD. V CA (LIM)
"The insured shall give notice to the company of any insurance or insurances already
effected, or which may subsequently be effected covering any of the property hereby 211 SCRA 554
insured, and unless such notice be given and the particulars of such insurance or CRUZ; July 17, 1992
insurances be stated in or endorsed on this Policy by or on behalf of the Company
before the occurrence of any loss or damage, all benefits under this Policy shall be NATURE
forfeited." or in the 1930 case of Santa Ana vs. Commercial Union Assurance Co. Petition for review from the decision of the Court of Appeals
which provided "that any outstanding insurance upon the whole or a portion of the
objects thereby assured must be declared by the insured in writing and he must cause FACTS
the company to add or insert it in the policy, without which such policy shall be null - Felix Lim was issued a Personal Accident Policy insurance with petitioner company
and void, and the insured will not be entitled to indemnity in case of loss," Condition with a face value of P200,000. His beneficiary was his wife Nerissa.
3 in the private respondent's policy No. F-14622 does not absolutely declare - October 6, 1982 – Felix accidentally shot himself in the head with his own gun. 
void any violation thereof. It expressly provides that the condition "shall not - He was playing with the handgun after he had removed the gun’s magazine (kasi
apply when the total insurance or insurances in force at the time of the loss naman…).
or damage is not more than P200,000.00." - He pointed the gun at his secretary and only witness Pilar Nalagon as a joke and
- Interpretation: It is a cardinal rule on insurance that a policy or insurance contract assured her that the gun was not loaded (are you sure…).
is to be interpreted liberally in favor of the insured and strictly against the company, - He then put the gun to his temple and fired it (haaay, sabi ko na nga ba).
the reason being, undoubtedly, to afford the greatest protection which the insured - Both parties are in agreement that there was no suicide.
was endeavoring to secure when he applied for insurance. It is also a cardinal - Nerissa claimed as Felix’s beneficiary but Sun Insurance would not grant her claim,
principle of law that forfeitures are not favored and that any construction which would saying that her husband’s death was not an accident.
result in the forfeiture of the policy benefits for the person claiming thereunder, will be - Nerissa sued Sun Insurance and won the case. Sun Insurance was ordered to pay
avoided, if it is possible to construe the policy in a manner which would permit her P200,000 representing the face value of the claim along with moral, exemplary
recovery, as, for example, by finding a waiver for such forfeiture. Stated differently, and compensatory damages and attorney’s fees. The decision was affirmed by the
provisions, conditions or exceptions in policies which tend to work a CA.
iNsuRanCe A2010 Dean Carale
pAgE 20♥

Petitioners’ Claim RIZAL SURETY & INSURANCE COMPANY V CA (TRANSWORLD


- Sun Insurance cites one of the four exceptions in the contract of insurance which
KNITTING MILLS, INC.)
includes bodily injury consequent upon the insured person attempting to commit
suicide or willfully exposing himself to needless peril in an attempt to save a human 336 SCRA 12
life. PURISIMA; July 18, 2000
- There mere act of pointing the gun to his temple showed that Felix willfully exposed
himself to danger because a gun should always be handled with caution. NATURE
Respondents’ Comments Petition for Review on Certiorari under Rule 45 of the Rules of Court
- Felix believed the gun to be safe because he had removed the magazine.
- He repeatedly assured his secretary that the gun was not loaded. FACTS
- Rizal Surety & Insurance Company (Rizal Insurance) issued Fire Insurance Policy No.
ISSUES 45727 in favor of Transworld Knitting Mills, Inc. (Transworld).
1. WON Felix Lim’s death was an accident, thus making his widow Nerissa liable to - Pertinent portions of subject policy on the buildings insured, and location thereof,
claim the accident insurance read:
2. WON the award of damages to Nerissa Lim was justified "‘On stocks of finished and/or unfinished products, raw materials and supplies
of every kind and description, the properties of the Insureds and/or held by
HELD them in trust, on commission or on joint account with others and/or for which
1. YES, Felix Lim’s death was an accident. they (sic) responsible in case of loss whilst contained and/or stored during the
Ratio There is no accident when a deliberate act is performed unless some additional, currency of this Policy in the premises occupied by them forming part of the
unexpected, independent and unforeseen happening occurs which produces or brings buildings situate (sic) within own Compound at MAGDALO STREET, BARRIO
bout their injury or death. UGONG, PASIG, METRO MANILA, PHILIPPINES, BLOCK NO. 601.’
Reasoning xxx...............xxx...............xxx
- An accident has been defined to be that which happens by chance or fortuitously ‘Said building of four-span lofty one storey in height with mezzanine portions is
without intention or design and which is unexpected, unusual and unforeseen. It an constructed of reinforced concrete and hollow blocks and/or concrete under
event that takes pace without one’s foresight or expectastion – an event that galvanized iron roof and occupied as hosiery mills, garment and lingerie
proceeds from an unknown cause or is an unusual effect of a known case and factory, transistor-stereo assembly plant, offices, warehouse and caretaker's
therefore not expected. It happens without any human agency, an event which, quarters.
under the circumstances, is unusual to and not expected by the person to whom it 'Bounds in front partly by one-storey concrete building under galvanized iron
happens. roof occupied as canteen and guardhouse, partly by building of two and partly
- The firing of the gun was deemed to be the unexpected and independent and one storey constructed of concrete below, timber above undergalvanized iron
unforeseen occurrence that led to the insured person’s death. roof occupied as garage and quarters and partly by open space and/or
- There was no willful exposure to needless peril for the part of Felix. Suicide and tracking/ packing, beyond which is the aforementioned Magdalo Street; on its
exposure to needless peril are similar in the sense that both signify disregard for one’s right and left by driveway, thence open spaces, and at the rear by open
life. Suicide imparts a positive act of ending one’s life whereas the latter indicates spaces.'"
recklessness that is almost suicidal in intent. - The same pieces of property insured with the petitioner were also insured with
- Accident insurance policies were never meant to reward the insured for his tendency New India Assurance Company, Ltd., (New India).
to show off or for his miscalculations. They were intended to provide for - Fire broke out in the compound of Transworld, razing the middle portion of its four-
contingencies. span building and partly gutting the left and right sections thereof. A two-storey
- Lim was unquestionably negligent but it should not prevent his widow from building (behind said four-span building) where fun and amusement machines and
recovering from the insurance policy he obtained precisely against accident. spare parts were stored, was also destroyed by the fire.
- Insurance contracts are, as a rule, supposed to be interpreted liberally in favor of the - Transworld filed its insurance claims with Rizal Surety & Insurance Company and
assured. New India Assurance Company but to no avail.
2. NO, the claim for damages should not be granted for being unjust. - Private respondent brought against the said insurance companies an action for
Ratio A person may be made liable to the payment of moral damages if his act is collection of sum of money and damages.
wrongful. The adverse result of an action does not per se make the act wrongful and - Petitioner Rizal Insurance countered that its fire insurance policy sued upon
subject the act or to the payment of moral damages. covered only the contents of the four-span building, which was partly burned, and
Reasoning not the damage caused by the fire on the two-storey annex building.
- Petitioner was acting in good faith when it resisted the private respondent’s claim on - The trial court dismissed the case as against The New India Assurance Co., Ltd. but
the ground that the death of the insured was covered by the exception. ordered defendant Rizal Surety And Insurance Company to pay Transwrold (sic)
- The issue was debatable and was clearly not raised only for the purpose of evading a Knitting Mills, Inc.
legitimate obligation. - Both the petitioner, Rizal Insurance Company, and private respondent, Transworld
Knitting Mills, Inc., went to the Court of Appeals, which required New India
iNsuRanCe A2010 Dean Carale
pAgE 21♥

Assurance Company to pay plaintiff-appellant the amount of P1,818,604.19 while supplies stored within the premises of respondent Transworld which was an integral
the Rizal Surety has to pay the plaintiff-appellant P470,328.67. part of the four-span building occupied by Transworld, knowing fully well the existence
- New India appealed to the Court theorizing inter alia that the private respondent of such building adjoining and intercommunicating with the right section of the four-
could not be compensated for the loss of the fun and amusement machines and span building.
spare parts stored at the two-storey building because it (Transworld) had no - Indeed, the stipulation as to the coverage of the fire insurance policy under
insurable interest in said goods or items. controversy has created a doubt regarding the portions of the building insured
- The Court denied the appeal with finality. thereby. Article 1377 of the New Civil Code provides:
- Petitioner Rizal Insurance and private respondent Transworld, interposed a Motion "Art.1377. The interpretation of obscure words or stipulations in a contract
for Reconsideration before the Court of Appeals, which reconsidered its decision of shall not favor the party who caused the obscurity"
July 15, 1993, as regards the imposition of interest. - Conformably, it stands to reason that the doubt should be resolved against the
- Undaunted, petitioner Rizal Surety & Insurance Company found its way to the petitioner, Rizal Surety Insurance Company, whose lawyer or managers drafted the
Court. fire insurance policy contract under scrutiny. Citing the aforecited provision of law in
point, the Court in Landicho vs. Government Service Insurance System, ruled:
ISSUE "This is particularly true as regards insurance policies, in respect of which it is
WON the fire insurance policy litigated upon protected only the contents of the main settled that the 'terms in an insurance policy, which are ambiguous, equivocal, or
building (four-span), and did not include those stored in the two-storey annex building uncertain x x x are to be construed strictly and most strongly against the insurer,
and liberally in favor of the insured so as to effect the dominant purpose of
HELD indemnity or payment to the insured, especially where forfeiture is involved' and
NO the reason for this is that the 'insured usually has no voice in the selection or
- Resolution of the issue posited hinges on the proper interpretation of the stipulation arrangement of the words employed and that the language of the contract is
in subject fire insurance policy regarding its coverage, which reads: selected with great care and deliberation by experts and legal advisers employed
"xxx contained and/or stored during the currency of this Policy in the premises by, and acting exclusively in the interest of, the insurance company.' "
occupied by them forming part of the buildings situate (sic) within own Compound - Equally relevant is the following disquisition of the Court in Fieldmen's Insurance
xxx" Company, Inc. vs. Vda. De Songco, to wit:
- It can be gleaned unerringly that the fire insurance policy in question did not limit its "'This rigid application of the rule on ambiguities has become necessary in view
coverage to what were stored in the four-span building. As opined by the trial court of of current business practices. The courts cannot ignore that nowadays
origin, two requirements must concur in order that the said fun and amusement monopolies, cartels and concentration of capital, endowed with overwhelming
machines and spare parts would be deemed protected by the fire insurance policy economic power, manage to impose upon parties dealing with them cunningly
under scrutiny, to wit: prepared 'agreements' that the weaker party may not change one whit, his
"First, said properties must be contained and/or stored in the areas occupied by participation in the 'agreement' being reduced to the alternative to 'take it or
Transworld and second, said areas must form part of the building described in the leave it' labelled since Raymond Saleilles 'contracts by adherence' (contrats [sic]
policy xxx" d'adhesion), in contrast to these entered into by parties bargaining on an equal
- Said building of four-span lofty one storey in height with mezzanine portions is footing, such contracts (of which policies of insurance and international bills of
constructed of reinforced concrete and hollow blocks and/or concrete under lading are prime example) obviously call for greater strictness and vigilance on
galvanized iron roof and occupied as hosiery mills, garment and lingerie factory, the part of courts of justice with a view to protecting the weaker party from
transistor-stereo assembly plant, offices, ware house and caretaker's quarter. abuses and imposition, and prevent their becoming traps for the unwary.'"
- The Court is mindful of the well-entrenched doctrine that factual findings by the - The issue of whether or not Transworld has an insurable interest in the fun and
Court of Appeals are conclusive on the parties and not reviewable by this Court, and amusement machines and spare parts, which entitles it to be indemnified for the loss
the same carry even more weight when the Court of Appeals has affirmed the findings thereof, had been settled in G.R. No. L-111118, entitled New India Assurance
of fact arrived at by the lower court. Company, Ltd., vs. Court of Appeals, where the appeal of New India from the decision
- In the case under consideration, both the trial court and the Court of Appeals found of the Court of Appeals under review, was denied with finality by this Court on
that the so called "annex " was not an annex building but an integral and inseparable February 2, 1994.
part of the four-span building described in the policy and consequently, the machines - The rule on conclusiveness of judgment, which obtains under the premises,
and spare parts stored therein were covered by the fire insurance in dispute. precludes the relitigation of a particular fact or issue in another action between the
- Verily, the two-storey building involved, a permanent structure which adjoins and same parties based on a different claim or cause of action. "xxx the judgment in the
intercommunicates with the "first right span of the lofty storey building", formed part prior action operates as estoppel only as to those matters in issue or points
thereof, and meets the requisites for compensability under the fire insurance policy controverted, upon the determination of which the finding or judgment was rendered.
sued upon. In fine, the previous judgment is conclusive in the second case, only as those matters
- So also, considering that the two-storey building aforementioned was already actually and directly controverted and determined and not as to matters merely
existing when subject fire insurance policy contract was entered into, petitioner involved therein."
should have specifically excluded the said two-storey building from the coverage of Disposition Decision, and the Resolution of the CA WERE AFFIRMED in toto. No
the fire insurance if minded to exclude the same but if did not, and instead, went on pronouncement as to costs.
to provide that such fire insurance policy covers the products, raw materials and
iNsuRanCe A2010 Dean Carale
pAgE 22♥

PAN MALAYAN INSURANCE CORPORATION vs. COURT OF CANLUBANG's claim under the policy, it simply meant that it had assumed to
reimburse the costs for repairing the damage to the insured vehicle.
APPEALS (ERLINDA FABIE & HER UNKNOWN DRIVER)
- It is a basic rule in the interpretation of contracts that the terms of a contract are to
184 SCRA 55; G.R. No. 81026 be construed according to the sense and meaning of the terms which the parties
CORTES; April 3, 1990 thereto have used. In the case of property insurance policies, the evident intention of
the contracting parties, i.e., the insurer and the assured, determine the import of the
FACTS various terms and provisions embodied in the policy. It is only when the terms of the
- December 10, 1985: PANMALAY filed a complaint for damages with the RTC of policy are ambiguous, equivocal or uncertain, such that the parties themselves
Makati against private respondents Erlinda Fabie and her driver. PANMALAY averred disagree about the meaning of particular provisions, that the courts will intervene. In
the following: that it insured a Mitsubishi Colt Lancer car registered in the name of such an event, the policy will be construed by the courts liberally in favor of the
Canlubang Automotive Resources Corporation [CANLUBANG]; that on May 26, 1985, assured and strictly against the insurer.
due to the "carelessness, recklessness, and imprudence" of the unknown driver of a - Considering that the very parties to the policy were not shown to be in disagreement
pick-up, the insured car was hit and suffered damages in the amount of P42,052.00; regarding the meaning and coverage of Section III-1, specifically sub-paragraph (a)
that PANMALAY defrayed the cost of repair of the insured car and, therefore, was thereof, it was improper for the appellate court to indulge in contract construction, to
subrogated to the rights of CANLUBANG against the driver of the pick-up and his apply the ejusdem generis rule, and to ascribe meaning contrary to the clear intention
employer, Erlinda Fabie; and that, despite repeated demands, defendants, failed and and understanding of these parties.
refused to pay the claim of PANMALAY. PANMALAY clarified that the damage caused to - Although the terms "accident" or "accidental" as used in insurance contracts have
the insured car was settled under the "own damage", coverage of the insurance not acquired a technical meaning, the Court has on several occasions defined these
policy. terms to mean that which takes place "without one's foresight or expectation, an
- Private respondents filed a Motion to Dismiss alleging that PANMALAY had no cause event that proceeds from an unknown cause, or is an unusual effect of a known cause
of action against them. RTC dismissed PANMALAY's complaint for no cause of action and, therefore, not expected." The concept "accident" is not necessarily synonymous
and denied PANMALAY's motion for reconsideration. CA affirmed. Hence, this petition with the concept of "no fault". It may be utilized simply to distinguish intentional or
for review. malicious acts from negligent or careless acts of man.
- Obiter Dicta: Even if under the above circumstances PANMALAY could not be deemed
ISSUE subrogated to the rights of its assured under Article 2207 of the Civil Code, PANMALAY
WON the insurer PANMALAY may institute an action to recover the amount it had paid would still have a cause of action against private respondents. The insurer who may
its assured in settlement of an insurance claim against private respondents as the have no rights of subrogation due to "voluntary" payment may nevertheless recover
parties allegedly responsible for the damage caused to the insured vehicle from the third party responsible for the damage to the insured property under Article
1236 of the Civil Code.
HELD Disposition Petition is GRANTED. Petitioner's complaint for damages against private
YES respondents is REINSTATED. Case remanded to the lower court for trial on the merits.
- Article 2207 of the Civil Code is founded on the well-settled principle of subrogation.
If the insured property is destroyed or damaged through the fault or negligence of a AMERICAN HOME ASSURANCE COMPANY V TANTUCO
party other than the assured, then the insurer, upon payment to the assured, will be
ENTERPRISES
subrogated to the rights of the assured to recover from the wrongdoer to the extent
that the insurer has been obligated to pay. 366 SCRA 740
- General Rule: Payment by the insurer to the assured operates as an equitable PUNO; October 8, 2001
assignment to the former of all remedies which the latter may have against the third
party whose negligence or wrongful act caused the loss. The right of subrogation is NATURE
not dependent upon, nor does it grow out of, any privity of contract or upon written Petition for Review on Certiorari assailing the Decision of the Court of Appeals.
assignment of claim. It accrues simply upon payment of the insurance claim by the
insurer. FACTS
- Exceptions: - Respondent Tantuco Enterprises, Inc. is engaged in the coconut oil milling and
a.) if the assured by his own act releases the wrongdoer or third party liable for the refining industry. It owns two oil mills. Both are located at its factory compound at
loss or damage, from liability Iyam, Lucena City. Respondent commenced its business operations with only one oil
b.) where the insurer pays the assured the value of the lost goods without notifying mill. In 1988, it started operating its second oil mill ( the new oil mill).
the carrier who has in good faith settled the assured's claim for loss - The two oil mills were separately covered by fire insurance policies issued by
c.) where the insurer pays the assured for a loss which is not a risk covered by the petitioner American Home Assurance Co. The first oil mill was insured Policy No. 306-
policy, thereby effecting "voluntary payment" 7432324-3 for the period March 1, 1991 to 1992.The new oil mill was insured under
- None of the exceptions are availing in the present case. Policy No. 306-7432321-9 for the same term. Official receipts indicating payment for
- When PANMALAY utilized the phrase "own damage" — a phrase which, incidentally, the full amount of the premium were issued by the petitioner's agent.
is not found in the insurance policy — to define the basis for its settlement of - Policy description:
iNsuRanCe A2010 Dean Carale
pAgE 23♥

Front: by a driveway thence at 18 meters distance by Bldg. No. 2. PERLA COMPANIA DE SEGUROS, INC. v, CA (MILAGROS
Right: by an open space thence by Bldg. No. 4.
CAYAS)
Left: Adjoining thence an imperfect wall by Bldg. No. 4.
Rear: by an open space thence at 8 meters distance.” 185 SCRA 741
- A fire that broke out in the early morning of September 30,1991 gutted and FERNAN; May 28, 1990
consumed the new oil mill. Petitioner rejected respondent’s claim for the insurance
proceeds on the ground that no policy was issued by it covering the burned oil mill. It NATURE
stated that the description of the insured establishment referred to another building. Petition for review on certiorari of a decision of the Court of Appeals
Petitioner’s Claim
The policies referred to the old mill, as stated in the description contained in the FACTS
policy. - Private respondent Milagros Cayas was the registered owner of a Mazda bus, insured
with Perla Compania de Seguros, Inc. (PCSI) under a policy issued on February 3,
ISSUE 1978.
WON new oil mill is insured by fire insurance policy - On December 17, 1978, the bus figured in an accident in Naic, Cavite injuring several
of its passengers.
HELD - One of them, 19-year old Edgardo Perea, sued Milagros Cayas for damages in the CFI
YES, new oil mill is insured. of Cavite, while three others agreed to a settlement of P4,000.00 each.
Ratio In construing the words used descriptive of a building insured, the greatest - After trial, the court rendered a decision in favor of Perea, ordering Cayas to
liberality is shown by the courts in giving effect to the insurance. In view of the compensate him, with an award of exemplary and moral damages, as well as
custom of insurance agents to examine buildings before writing policies upon them, attorney’s fees. ( P32,000 total)
and since a mistake as to the identity and character of the building is extremely - On November 11, 1981, Milagros Cayas filed a complaint for a sum of money and
unlikely, the courts are inclined to consider that the policy of insurance covers any damages against PCSI in the Court of First Instance of Cavite.
building which the parties manifestly intended to insure, however inaccurate the - In view of Milagros Cayas' failure to prosecute the case, the court motu propio
description may be. ordered its dismissal without prejudice.
Reasoning - Alleging that she had not received a copy of the answer to the complaint, and that
- The parties manifestly intended to insure the new oil mill. “On machineries and "out of sportsmanship", she did not file a motion to hold PCSI in default, Milagros
equipment with complete accessories usual to a coconut oil mill including stocks of Cayas moved for the reconsideration of the dismissal order. Said motion for
copra, copra cake and copra mills whilst contained in the new oil mill building, situate reconsideration was acted upon favorably by the court.
(sic) at UNNO. ALONG NATIONAL HIGH WAY, BO. IYAM, LUCENA CITY UNBLOCKED.” - About two months later, Milagros Cayas filed a motion to declare PCSI in default for
- If the parties really intended to protect the first oil mill, then there is no need to its failure to file an answer.
specify it as new. It would be absurd to assume that respondent would protect its first - The motion was granted and plaintiff was allowed to adduce evidence ex-parte.
oil mill for different amounts and leave uncovered its second one. - On July 13, 1982, the court rendered judgment by default ordering PCSI to pay
- As may be gleaned from the testimony of the petitioner’s employee, the source of Milagros Cayas P50,000 as compensation for the injured passengers, P5,000 as moral
the discrepancy happened during the preparation of the written contract. damages and P5,000 as attorney's fees.
- Respondent is not estopped from claiming that the policy description is wrong. - Said decision was set aside after the PCSI filed a motion therefor. Trial of the case
Evidence on record reveals that respondent’s operating manager, Mr. Edison Tantuco, ensued.
notified the petitioner’s agent with whom respondent negotiated for the contract - In due course, the court promulgated a decision ordering defendant Perla Compania
about the inaccurate description in the policy. However, Mr. Borja assured Mr. de Seguros, Inc. to pay plaintiff Milagros Cayas the sum of P50,000.00 under its
Tantuco that the use of the adjective new will distinguish the insured property. maximum liability as provided for in the insurance policy; and the sum of P5,000.00 as
- Regarding policy requirements that fire extinguishment appliances should be reasonable attorney's fee
available and in good working condition, warranty did not require respondent to - PCSI appealed to the Court of Appeals, which affirmed in toto the lower court's
provide for all the fire extinguishing appliances enumerated therein. Neither did it decision.
require that the appliances are restricted to those mentioned in the warranty. (Within - Its motion for reconsideration having been denied by said appellate court, PCSI filed
the vicinity of the new oil mill can be found the following devices: numerous portable this petition
fire extinguishers, two fire hoses, fire hydrant, and an emergency fire engine.)
- The object of the court in construing a contract is to ascertain the intent of the ISSUE
parties to the contract and to enforce the agreement which the parties have entered WON, as maintained by petitioner, its liability is limited only to the payment made by
into. In determining what the parties intended, the courts will read and construe the private respondent to Perea and only up to the amount of P12,000.00
policy as a whole and if possible, give effect to all the parts of the contract.
Disposition Petition is dismissed. HELD
YES
- The insurance policy involved explicitly limits petitioner's liability to P12,000.00 per
person and to P50,000.00 per accident.
iNsuRanCe A2010 Dean Carale
pAgE 24♥

- In Stokes vs. Malayan Insurance Co., Inc., the Court held that the terms of the CHICO-NAZARIO; March 5, 2007
contract constitute the measure of the insurer's liability and compliance therewith is a
condition precedent to the insured's right of recovery from the insurer. NATURE
- In the case at bar, the insurance policy clearly and categorically placed petitioner's Petition for review of CA decision
liability for all damages arising out of death or bodily injury sustained by one person
as a result of any one accident at P12,000.00. FACTS
- Said amount complied with the minimum fixed by the law then prevailing, Section - Petitioners POLTAN obtained a loan evidenced by a promissory note from the
377 of Presidential Decree No. 612, which provided that the liability of land MANTRADE Dev’t Corp. This was secured by a chattel mortgage over a 1-unit Nissan
transportation vehicle operators for bodily injuries sustained by a passenger arising Sentra vehicle.
out of the use of their vehicles shall not be less than P12,000. - With notice to petitioners, MANTRADE assigned to BPI, by way of a Deed of
- In other words, under the law, the minimum liability is P12,000 per passenger. Assignment, all its rights, title and interest to the promissory note and chattel
Petitioner's liability under the insurance contract not being less than P12,000.00, and mortgage.
therefore not contrary to law, morals, good customs, public order or public policy, said - Petitioners defaulted and so BPI demanded the whole balance of P286.5T including
stipulation must be upheld as effective, valid and binding as between the parties. accrued interest, or to return to BPI the possession of the motor vehicle for
- In like manner, we rule as valid and binding upon private respondent the condition foreclosure. It is specifically provided in the promissory note and chattel mortgage
requiring her to secure the written permission of petitioner before effecting any that failure to pay any installment when due shall make subsequent installments and
payment in settlement of any claim against her. the entire balance of the obligation due and payable.
- There is nothing unreasonable, arbitrary or objectionable in this stipulation as would - After they refused to do so, BPI then filed complaint.
warrant its nullification. The same was obviously designed to safeguard the insurer's - Petitioners claimed that BPI required them to obtain a motor vehicle insurance policy
interest against collusion between the insured and the claimants. from FGU Insurance Corporation (FGU Insurance). This is a sister company of BPI. They
- It being specifically required that petitioner's written consent be first secured before had been paying the monthly installments on the vehicle until it figured in an accident
any payment in settlement of any claim could be made, private respondent is where it became a total wreck. Under the terms of the insurance policy from FGU
precluded from seeking reimbursement of the payments made to the three other Insurance, the vehicle had to be replaced or its value paid to them. Due to the failure
passangers in view of her failure to comply with the condition contained in the and refusal of FGU Insurance to replace the vehicle or pay its value, they stopped
insurance policy. payment of the monthly installments.
- Clearly, the fundamental principle that contracts are respected as the law between - RTC ordered POLTANS to pay BPI the said amount.
the contracting parties finds application in the present case. - CA reversed and remanded case to RTC for trial on the merits.
- It was error on the part of the trial and appellate courts to have disregarded the - RTC again ruled in favor of BPI. CA affirmed.
stipulations of the parties and to have substituted their own interpretation of the
insurance policy. ISSUES
- In Phil. American General Insurance Co., Inc vs. Mutuc, we ruled that contracts which 1. WON contracts presented in evidence by BPI were unjust and unacceptable
are the private laws of the contracting parties should be fulfilled according to the contracts of adhesion
literal sense of their stipulations, if their terms are clear and leave no room for doubt 2. WON the terms and conditions of the comprehensive car insurance policy issued by
as to the intention of the contracting parties, for contracts are obligatory, no matter FGU should be deemed as having automatically operated in favor of BPI as the
what form they may be, whenever the essential requisites for their validity are assured mortgagee, and if so, it should be deemed as resulting in the extinguishment
present. of petitioner’s obligation
- In Pacific Oxygen & Acetylene Co. vs. Central Bank," it was stated that the first and
fundamental duty of the courts is the application of the law according to its express HELD
terms, interpretation being called for only when such literal application is impossible. 1. NO
- We observe that although Milagros Cayas was able to prove a total loss of only Ratio A contract of adhesion is one in which one of the parties imposes a ready-
P44,000.00, petitioner was made liable for the amount of P50,000.00, the maximum made form of contract, which the other party may accept or reject, but which the
liability per accident stipulated in the policy. This is patent error. An insurance latter cannot modify. It is just as binding as ordinary contracts.
indemnity, being merely an assistance or restitution insofar as can be fairly Reasoning
ascertained, cannot be availed of by any accident victim or claimant as an instrument - Petitioners failed to show that they were under duress or forced to sign the loan
of enrichment by reason of an accident. documents. The natural presumption is that one does not sign a document without
Disposition Petition granted. The decision of the Court of Appeals is modified in that first informing himself of its contents and consequences.
petitioner shall pay Milagros Cayas the amount of Twelve Thousand Pesos (P12,000. - Contracts of adhesion are not entirely prohibited even as the courts remain careful in
00) plus legal interest from the promulgation of the decision of the lower court until it scrutinizing the factual circumstances underlying each case to determine the
is fully paid and attorney's fees in the amount of P5,000.00. respective claims of contending parties on their efficacy.
2. NO
POLTAN v. BPI & JOHN DOE Reasoning
G.R. No. 164307 - Petitioners failed to show any provision in the insurance policy or mortgage contract
providing that the loss of the mortgaged vehicle extinguishes their principal obligation
iNsuRanCe A2010 Dean Carale
pAgE 25♥

to BPI. Phoenix Assurance Co., Ltd. the basic rule is that the insurance company has the
- While it is true that the proceeds from the insurance policy over the mortgaged burden of proving that the loss is caused by the risk excepted and for want of such
chattel is for the benefit of BPI, this will result in partial or full satisfaction of the proof, the company is liable. In the present case, there being no showing that the loss
obligation only if the insurer pays the mortgagee, BPI, or if the insurance proceeds was caused by any of the excepted perils, the insurer is liable under the policy.
were paid to BPI. In this case, upon the loss of the vehicle due to total wreck, the 2. YES
petitioners filed a claim under the insurance policy, collected and received the - Section 13 of the Insurance Code defines insurable interest in property as every
proceeds thereof, but did not settle their obligation with BPI which remained interest in property, whether real or personal, or any relation thereto, or liability in
outstanding despite the loss of the vehicle. respect thereof, of such nature that a contemplated peril might directly damnify the
Disposition CA decision AFFIRMED with the modification that the interest rate be insured. In principle, anyone has an insurable interest in property who derives a
reduced to 12% per annum from 24 May 1994 until fully paid, and the award of benefit from its existence or would suffer loss from its destruction whether he has or
attorney’s fees be reduced to P50T. has not any title in, or lien upon or possession of the property y. 16 Insurable interest
in property may consist in (a) an existing interest; (b) an inchoate interest founded on
FILIPINO MERCHANTS INS. v. CA (CHOA TIEK SENG) an existing interest; or (c) an expectancy, coupled with an existing interest in that out
of which the expectancy arises.
179 SCRA 638
- Respondent’s interest over the goods is based on the perfected contract of sale. The
REGALADO; November 28, 1989 perfected contract of sale between him and the shipper of the goods operates to vest
in him an equitable title even before delivery or before be performed the conditions of
NATURE the sale.
Review of the decision of the CA - Further, Article 1523 of the Civil Code provides that where, in pursuance of a
contract of sale, the seller is authorized or required to send the goods to the buyer,
FACTS delivery of the goods to a carrier, whether named by the buyer or not, for, the
- Plaintiff insured said shipment with defendant insurance company under said cargo purpose of transmission to the buyer is deemed to be a delivery of the goods to the
for the goods described as 600 metric tons of fishmeal in new gunny bags of 90 kilos buyer, the exceptions to said rule not obtaining in the present case. The Court has
each from Bangkok, Thailand to Manila against all risks under warehouse to heretofore ruled that the delivery of the goods on board the carrying vessels partake
warehouse terms. of the nature of actual delivery since, from that time, the foreign buyers assumed the
- Some of the goods arrived in bad condition. Plaintiff made a claim against Filipino risks of loss of the goods and paid the insurance premium covering them
Merchants Insurance Company. The latter refused to pay. Plaintiff brought an action - Moreover, the issue of lack of insurable interest was not raised in petitioner’s
against them. The defendant insurance company presented a third party complaint answer.
against the vessel and the arrastre contractor. Disposition Petition denied
- Judgment was rendered against the insurance company. On the third party
complaint, the third party defendants were ordered to pay the third party plaintiffs. GAISANO CAGAYAN v. INSURANCE Co. OF NORTH AMERICA
The CA affirmed, but modified the same with regard to the adjudication of the third-
490 SCRA 296
party complaint
Austria-Martinez; June 8, 2006
ISSUES
1. WON some fortuity, casualty or accidental cause is needed to be proved despite the NATURE
“all risks” policy (as asserted by the insurance company) Petition for review on certiorari of the Decision of the Court of Appeals
2. WON the respondent has an insurable interest
FACTS
HELD - Intercapitol Marketing Corporation (IMC) is the maker of Wrangler Blue Jeans. Levi
1. NO Strauss (Phils.) Inc. (LSPI) is the local distributor of products bearing trademarks
- The very nature of the term "all risks" must be given a broad and comprehensive owned by Levi Strauss & Co.. IMC and LSPI separately obtained from respondent fire
meaning as covering any loss other than a willful and fraudulent act of the insured. 7 insurance policies with book debt endorsements. The insurance policies provide for
This is pursuant to the very purpose of an "all risks" insurance to give protection to coverage on “book debts in connection with ready-made clothing materials which
the insured in those cases where difficulties of logical explanation or some mystery have been sold or delivered to various customers and dealers of the Insured anywhere
surround the loss or damage to property. in the Philippines.” The policies defined book debts as the “unpaid account still
- Generally, the burden of proof is upon the insured to show that a loss arose from a appearing in the Book of Account of the Insured 45 days after the time of the loss
covered peril, but under an "all risks" policy the burden is not on the insured to prove covered under this Policy.” The policies also provide for the following conditions:
the precise cause of loss or damage for which it seeks compensation. The insured 1. Warranted that the Company shall not be liable for any unpaid account
under an "all risks insurance policy" has the initial burden of proving that the cargo in respect of the merchandise sold and delivered by the Insured which
was in good condition when the policy attached and that the cargo was damaged are outstanding at the date of loss for a period in excess of six (6)
when unloaded from the vessel; thereafter, the burden then shifts to the insurer to months from the date of the covering invoice or actual delivery of the
show the exception to the coverage. As we held in Paris-Manila Perfumery Co. vs. merchandise whichever shall first occur.
iNsuRanCe A2010 Dean Carale
pAgE 26♥

2. Warranted that the Insured shall submit to the Company within twelve (12) days 3. WON the petitioner liable for the unpaid accounts
after the close of every calendar month all amount shown in their books of accounts
as unpaid and thus become receivable item from their customers and dealers. x x x HELD
- Petitioner is a customer and dealer of the products of IMC and LSPI. On February 25, 1. NO
1991, the Gaisano Superstore Complex in Cagayan de Oro City, owned by petitioner, - It is well-settled that when the words of a contract are plain and readily understood,
was consumed by fire. Included in the items lost or destroyed in the fire were stocks there is no room for construction. In this case, the questioned insurance policies
of ready-made clothing materials sold and delivered by IMC and LSPI. On February 4, provide coverage for “book debts in connection with ready-made clothing materials
1992, respondent filed a complaint for damages against petitioner. It alleges that IMC which have been sold or delivered to various customers and dealers of the Insured
and LSPI filed with respondent their claims under their respective fire insurance anywhere in the Philippines.”; and defined book debts as the “unpaid account still
policies with book debt endorsements; that as of February 25, 1991, the unpaid appearing in the Book of Account of the Insured 45 days after the time of the loss
accounts of petitioner on the sale and delivery of ready-made clothing materials with covered under this Policy.” Nowhere is it provided in the questioned insurance policies
IMC was P2,119,205.00 while with LSPI it was P535,613.00; that respondent paid the that the subject of the insurance is the goods sold and delivered to the customers and
claims of IMC and LSPI and, by virtue thereof, respondent was subrogated to their dealers of the insured.
rights against petitioner; that respondent made several demands for payment upon - Indeed, when the terms of the agreement are clear and explicit that they do not
petitioner but these went unheeded. In its Answer with Counter Claim dated July 4, justify an attempt to read into it any alleged intention of the parties, the terms are to
1995, petitioner contends that it could not be held liable because the property be understood literally just as they appear on the face of the contract. Thus, what
covered by the insurance policies were destroyed due to fortuities event or force were insured against were the accounts of IMC and LSPI with petitioner which
majeure; that respondent’s right of subrogation has no basis inasmuch as there was remained unpaid 45 days after the loss through fire, and not the loss or destruction of
no breach of contract committed by it since the loss was due to fire which it could not the goods delivered.
prevent or foresee; that IMC and LSPI never communicated to it that they insured their 2. NO
properties; that it never consented to paying the claim of the insured. - The present case clearly falls under paragraph (1), Article 1504 of the Civil Code:
- At the pre-trial conference the parties failed to arrive at an amicable settlement. ART. 1504. Unless otherwise agreed, the goods remain at the seller’s risk until the
Thus, trial on the merits ensued. On August 31, 1998, the RTC rendered its decision ownership therein is transferred to the buyer, but when the ownership therein is
dismissing respondent’s complaint. It held that the fire was purely accidental; that the transferred to the buyer the goods are at the buyer’s risk whether actual delivery
cause of the fire was not attributable to the negligence of the petitioner; that it has has been made or not, except that:
not been established that petitioner is the debtor of IMC and LSPI; that since the sales (1) Where delivery of the goods has been made to the buyer or to a bailee for the
invoices state that “it is further agreed that merely for purpose of securing the buyer, in pursuance of the contract and the ownership in the goods has been
payment of purchase price, the above-described merchandise remains the property of retained by the seller merely to secure performance by the buyer of his
the vendor until the purchase price is fully paid”, IMC and LSPI retained ownership of obligations under the contract, the goods are at the buyer’s risk from the
the delivered goods and must bear the loss. Dissatisfied, petitioner appealed to the time of such delivery; (Emphasis supplied)
CA. On October 11, 2000, the CA rendered its decision setting aside the decision of - Thus, when the seller retains ownership only to insure that the buyer will pay its
the RTC. The CA held that the sales invoices are proofs of sale, being detailed debt, the risk of loss is borne by the buyer. Accordingly, petitioner bears the risk of
statements of the nature, quantity and cost of the thing sold; that loss of the goods in loss of the goods delivered.
the fire must be borne by petitioner since the proviso contained in the sales invoices - IMC and LSPI did not lose complete interest over the goods. They have an insurable
is an exception under Article 1504 (1) of the Civil Code, to the general rule that if the interest until full payment of the value of the delivered goods. Unlike the civil law
thing is lost by a fortuitous event, the risk is borne by the owner of the thing at the concept of res perit domino, where ownership is the basis for consideration of who
time the loss under the principle of res perit domino; that petitioner’s obligation to IMC bears the risk of loss, in property insurance, one’s interest is not determined
and LSPI is not the delivery of the lost goods but the payment of its unpaid account by concept of title, but whether insured has substantial economic interest in
and as such the obligation to pay is not extinguished, even if the fire is considered a the property.
fortuitous event; that by subrogation, the insurer has the right to go against - Section 13 of our Insurance Code defines insurable interest as “every interest in
petitioner; that, being a fire insurance with book debt endorsements, what was property, whether real or personal, or any relation thereto, or liability in respect
insured was the vendor’s interest as a creditor. Petitioner filed a motion for thereof, of such nature that a contemplated peril might directly damnify the insured.”
reconsideration but it was denied by the CA in its Resolution dated April 11, 2001. Parenthetically, under Section 14 of the same Code, an insurable interest in property
may consist in: (a) an existing interest; (b) an inchoate interest founded on existing
ISSUES interest; or (c) an expectancy, coupled with an existing interest in that out of which
1. WON the CA erred in construing a fire insurance policy on book debts as one the expectancy arises.
covering the unpaid accounts of IMC and LSPI since such insurance applies to loss of - Therefore, an insurable interest in property does not necessarily imply a property
the ready-made clothing materials sold and delivered to petitioner. interest in, or a lien upon, or possession of, the subject matter of the insurance, and
2. WON IMC bears the risk of loss because it expressly reserved ownership of the neither the title nor a beneficial interest is requisite to the existence of such an
goods by stipulating in the sales invoices that “[i]t is further agreed that merely for interest, it is sufficient that the insured is so situated with reference to the property
purpose of securing the payment of the purchase price the above described that he would be liable to loss should it be injured or destroyed by the peril against
merchandise remains the property of the vendor until the purchase price thereof is which it is insured. Anyone has an insurable interest in property who derives a benefit
fully paid.” from its existence or would suffer loss from its destruction. Indeed, a vendor or seller
iNsuRanCe A2010 Dean Carale
pAgE 27♥

retains an insurable interest in the property sold so long as he has any interest Levi’s products in the amount of P535,613.00 in the fire that razed petitioner’s
therein, in other words, so long as he would suffer by its destruction, as where he has building on February 25, 1991.
a vendor’s lien. In this case, the insurable interest of IMC and LSPI pertain to the - Moreover, there is no proof of full settlement of the insurance claim of LSPI; no
unpaid accounts appearing in their Books of Account 45 days after the time of the loss subrogation receipt was offered in evidence. Thus, there is no evidence that
covered by the policies. respondent has been subrogated to any right which LSPI may have against petitioner.
3. YES Failure to substantiate the claim of subrogation is fatal to petitioner’s case for
- Petitioner’s argument that it is not liable because the fire is a fortuitous event under recovery of the amount of P535,613.00.
Article 1174 of the Civil Code is misplaced. As held earlier, petitioner bears the loss Disposition Petition is partly GRANTED. The assailed Decision dated October 11,
under Article 1504 (1) of the Civil Code. 2000 and Resolution dated April 11, 2001 of the Court of Appeals in CA-G.R. CV No.
- Moreover, it must be stressed that the insurance in this case is not for loss of goods 61848 are AFFIRMED with the MODIFICATION that the order to pay the amount of
by fire but for petitioner’s accounts with IMC and LSPI that remained unpaid 45 days P535,613.00 to respondent is DELETED for lack of factual basis.
after the fire. Accordingly, petitioner’s obligation is for the payment of money. Where
the obligation consists in the payment of money, the failure of the debtor to make the TAI TONG CHUACHE & CO v. INSURANCE COMMISSION and
payment even by reason of a fortuitous event shall not relieve him of his liability. The
TRAVELLERS MULTI-INDEMNITY CORPORATION
rationale for this is that the rule that an obligor should be held exempt from liability
when the loss occurs thru a fortuitous event only holds true when the obligation 158 SCRA 366
consists in the delivery of a determinate thing and there is no stipulation holding him GANCAYCO; February 29, 1988
liable even in case of fortuitous event. It does not apply when the obligation is
pecuniary in nature. NATURE
- Under Article 1263 of the Civil Code, “[i]n an obligation to deliver a generic thing, the Petition for review on certiorari of the decision of the Insurance Commission
loss or destruction of anything of the same kind does not extinguish the obligation.” If
the obligation is generic in the sense that the object thereof is designated merely by FACTS
its class or genus without any particular designation or physical segregation from all - Complainants Palomo acquired a parcel of land and a building located in Davao City.
others of the same class, the loss or destruction of anything of the same kind even They assumed the mortgage of the building in favor of SSS, which building was
without the debtor’s fault and before he has incurred in delay will not have the effect insured with respondent SSS Accredited Group of Insurers for P25K.
of extinguishing the obligation. This rule is based on the principle that the genus of a - On April 19, 1975, Azucena Palomo obtained a P100K loan from Tai Tong Chuache
thing can never perish. Genus nunquan perit. An obligation to pay money is Inc. (TTCC) and executed a mortgage over the land and the building in favor of Tai
generic; therefore, it is not excused by fortuitous loss of any specific Tong Chuache & Co. as security of payment .On April 25, 1975, Arsenio Chua,
property of the debtor. representative of TTCC insured the latter's interest with Travellers Multi-Indemnity
- Thus, whether fire is a fortuitous event or petitioner was negligent are matters Corporation (Travellers) for P100K (P70K for bldg and P30K for the contents thereof)
immaterial to this case. What is relevant here is whether it has been established that - On June 11, 1975, Pedro Palomo secured a Fire Insurance Policy, covering the
petitioner has outstanding accounts with IMC and LSPI. building for P50K with respondent Zenith Insurance Corporation (ZIC). Another Fire
- With respect to IMC, the respondent has adequately established its claim. Petitioner Insurance Policy was later procured from respondent Philippine British Assurance
has an outstanding account with IMC in the amount of P2,119,205.00, check voucher Company (PBAC), covering the same building for P50K and contents thereof for P70K.
evidencing payment to IMC, subrogation receipt executed by IMC in favor of On July 31, 1975, the building and the contents were totally razed by fire.
respondent upon receipt of the insurance proceeds. All these documents have been - Based on the computation of the loss, including the Travellers, respondents, ZIC,
properly identified, presented and marked as exhibits in court. The subrogation PBAC, and SSS paid their corresponding shares of the loss. Complainants were paid
receipt, by itself, is sufficient to establish not only the relationship of respondent as the following: P41,546.79 by PBAC, P11,877.14 by ZIC, and P5,936.57 by SSS.
insurer and IMC as the insured, but also the amount paid to settle the insurance claim. Demand was made from respondent Travellers for its share in the loss but was
The right of subrogation accrues simply upon payment by the insurance company of refused. Hence, complainants demanded from the other 3 respondents the balance of
the insurance claim. Respondent’s action against petitioner is squarely sanctioned by each share in the loss based on the computation excluding Travellers Multi-Indemnity
Article 2207 of the Civil Code which provides: in the amount of P30,894.31 (P5,732.79-ZIC: P22,294.62, PBAC: and P2,866.90, SSS)
Art. 2207. If the plaintiff’s property has been insured, and he has received but was refused, hence, this action.
indemnity from the insurance company for the injury or loss arising out of the wrong
or breach of contract complained of, the insurance company shall be subrogated to ISSUE
the rights of the insured against the wrongdoer or the person who has violated the WON petitioner Tai Tong has insurable interest in the said policy
contract. x x x
- Petitioner failed to refute respondent’s evidence. HELD
- As to LSPI, respondent failed to present sufficient evidence to prove its cause of YES
action. No evidentiary weight can be given to Exhibit “F Levi Strauss”, a letter dated - First, respondent insurance commission based its findings on mere inference.
April 23, 1991 from petitioner’s General Manager, Stephen S. Gaisano, Jr., since it is Respondent Insurance Commission absolved respondent insurance company from
not an admission of petitioner’s unpaid account with LSPI. It only confirms the loss of liability on the basis of the certification issued by the then CFI, that in a certain civil
action against the Palomos, Arsenio Lopez Chua stands as the complainant and not
iNsuRanCe A2010 Dean Carale
pAgE 28♥

Tai Tong Chuache. From said evidence respondent commission inferred that the credit - On September 21, 1990, BF filed a complaint against Mrs. Perez seeking recission
extended by herein petitioner to the Palomos secured by the insured property must and declaration of nullity of the insurance contract in question. Mrs. Perez filed a
have been paid. Such is a glaring error which this Court cannot sanction. conterclaim
- Second, it has been held in a long line of cases that when the creditor is in for the collection of Php150,000 plus damages.
possession of the document of credit, he need not prove non-payment for it is
presumed. The validity of the insurance policy taken b petitioner was not assailed by ISSUE
private respondent. Moreover, petitioner's claim that the loan extended to the WON there was a consummated contract of insurance between Perez and BF
Palomos has not yet been paid was corroborated by Azucena Palomo who testified
that they are still indebted to herein petitioner. So at the time of the fire, petitioner as HELD
mortgagee still had insurable interest therein. NO
- And third, petitioner's declaration that Arsenio Lopez Chua acts as the managing - An essential requisite of a valid contract is consent. Consent must be manifested by
partner of the partnership was corroborated by respondent insurance company. Thus the meeting of the offer and the acceptance upon the thing and the cause which are
Chua as the managing partner of the partnership may execute all acts of to constitute the contract.
administration including the right to sue debtors of the partnership in case of their - The offer must be certain and the acceptance absolute. When Perez filed the
failure to pay their obligations when it became due and demandable. Or at the least, application, it was subject to the acceptance of BF. The perfection was also further
Chua being a partner of petitioner Tai Tong Chuache & Company is an agent of the conditioned upon (1) the issuance of the policy,
partnership. Being an agent, it is understood that he acted for and in behalf of the (2) the payment of the premium, and (3) the delivery to and acceptance by the
firm. applicant in good health.
Disposition Appealed decision SET ASIDE and ANOTHER judgment is rendered order - The delivery and acceptance by the applicant was a suspensive condition which was
private respondent Travellers to pay petitioner the face value of Fire Insurance Policy not fulfilled inasmuch as the applicant was already dead at the time the policy was
in the amount of P100K. Costs against said private respondent. issued. The non-fulfillment of the condition resulted
in the non-perfection of the contract.
- An application for insurance is merely an offer which requires the overt act of the
PEREZ v. CA (BF LIFEMAN INSURANCE CORP.) insurer for it to ripen to a contract. Delay in acting on the application does not
constitute acceptance even though the insured has forwarded
323 SCRA 613
his first premium with his application. Delay, in this case, does not constitute gross
YNARES-SANTIAGO; January 28, 2000 negligence because the application was granted within the normal processing time.
Disposition Decision of CA affirmed in so far as it declared the insurance policy for
NATURE Php50,000 issued by BF null and void (no recission because it presupposes a valid
Petition for review on certiorari contract)

FACTS
VDA. DE SINDAYEN v. INSULAR
- Primitivo Perez has been insured with the BF Lifeman Insurance Corporation (BF
hereafter) since 1980 for Php20,000. 62 Phil 51
Sometime in 1987, Rodolfo Lalog (agent of BF) convinced him to apply for additional BUTTE; September 4, 1935
insurance coverage of Php50,000.
Perez accomplished the application form and passed the required medical FACTS
examination. He also paid Php2,075 premium) to Lalog. - Arturo Sindayen, up to the time of his death on January 19, 1933, was employed as a
- On November 25, 1987, Perez died while riding a banca which capsized during a linotype operator in the Bureau of Printing at Manila and had been such for eleven
storm. During this time his application papers for the additional insurance coverage years prior thereto. While there he made a written application on December 26, 1932,
was still with the Gumaca, Quezon to the defendant Insular Life Assurance Co., Ltd., through its agent, Cristobal
office of BF. Mendoza, for a policy of insurance on his life in the sum of P1,000 and he paid to the
- Without knowing that Perez died on November 25, 1987, BF approved Perez's agent P15 cash as part of the first premium. It was agreed with the agent that the
application and issued the corresponding policy for the Php50,000 on December 2, policy, when and if issued, should be delivered to his aunt. Felicidad Estrada, with
1987. whom Sindayen left the sum of P26.06 to complete the payment of the first annual
- Virginia Perez (wife of the deceased) claimed the benefits under the insurance premium of P40.06.
policies of the deceased, but she was only able to receive Php40,000 under the first - On January 1, 1933, Sindayen, who was then twenty-nine years of age, was
insurance policy. examined by the company's doctor who made a favorable report, to the company. On
BF refused to pay the proceeds amounting to Php150,000 under the additional policy January 11, 1933, The company accepted the risk and issued policy No. 47710 dated
coverage of Php50,000 because they maintain that such policy had not been back to December 1, 1932, and mailed the same to its agent, Cristobal Mendoza, in
perfected. Camiling, Tarlac, for delivery to the insured. -On January 11, 1933, Sindayen was at
work in the Bureau of Printing. On January 12, he complained of a severe headache
iNsuRanCe A2010 Dean Carale
pAgE 29♥

and remained at home. On January 15, he called a physician who found that he was a case of waiver or of estoppel, but a case where the local agents, in the exercise of
suffering from acute nephritis and uremia and on January 19, 1933, he died. the powers lodged in them, accepted the premium and delivered the policy. That act
- On January 18, 1933, the agent, in accordance with his agreement with the insured, binds their principal, the defendant.”
delivered the policy to Felicidad Estrada upon her payment of the balance of the first - Mendoza was duly licensed by the Insurance Commissioner to act as the agent of the
year's annual premium. The agent asked Felicidad Estrada if her nephew was in good defendant insurance company. The well known custom of the insurance business and
health and she replied that she believed so because she had no information that he the evidence in this case prove that Mendoza was not regarded by the company as a
was sick and he thereupon delivered to her the policy. mere conduit or automaton for the performance of the physical act of placing the
- On January 20, 1933, the agent learned of the death of Arturo Sindayen and called policy in the hands of the insured
on Felicidad Estrada and asked her to return the policy. But he did not return or offer - Granted that Mendoza's decision that the condition had been met by the insured and
to return the premium paid. Felicidad Estrada on his aforesaid statement gave him the that it was proper to make a delivery of the policy to him is just as binding on the
policy. company as if the decision had been made by its board of directors. Granted that
- On February 4, 1933 Insular Life obtained from the beneficiary, Sindayen’s wife, her Mendoza made a mistake of judgement because he acted on insufficient evidence as
signature to a legal document entitled "ACCORD, SATISFACTION AND RELEASE" to the state of health of the insured. But it is not charged that the mistake was
whereby in consideration of the sum of P40.06 paid to her by a check of the company, induced by any misconduct or omission of duty of the insured.
she "assigns, releases and forever discharges said Isular Life Assurance Co., Ltd., its - It is the interest not only the applicant but of all insurance companies as well that
successors and assigns, of all claims, obligation in or indebtedness. The said check for there should be some act which gives the applicant the definite assurance that the
P40.06 was never cashed but returned to the company and appears in the record of contract has been consummated. This sense of security and of peace of mind that
this case as Exhibit D. Thereupon this action was brought to enforce payment of the one's defendants are provided for without risk either of loss or of litigation is the
policy. bedrock of life insurance. When the policy is issued and delivered, in the absence of
By the terms of the policy, an annual premium of P40.06 is due on the first day of fraud or other grounds for rescission, it is plainly not within the intention of the parties
December of each year, the first premium already paid by the insured covering the that there should be any questions held in abeyance or reserved for future
period from December 1, 1932. It is to December 1, 1933. It is to be noted that the determination that leave the very existence of the contract in suspense and doubt.
policy was not issued and the company assumed no actual risk prior to January 11, - It is therefore in the public interest, for the public is profoundly and generally
1933.The application which the insured signed in Camiling, Tarlac, on December 26, interested in life insurance, as well as in the interest of the insurance companies
1932, contained among others the following provisions: themselves by giving certainly and security to their policies, that we are constrained
“3 That the said policy shall not take effect until the first premium has been paid and to hold, as we, do, that the delivery of the policy to the insured by an agent of the
the policy has been delivered to and accepted by me, while I am in good health.” company who is authorized to make delivery or without delivery is the final act which
-Main defense of the company in this case, namely, that the said policy never took binds the company (and the insured as well) in the absence of fraud or other legal
effect because of paragraph 3 of the application above quoted, for at the time of its ground for rescission
delivery by the agent as aforesaid the insured was not in good health - The company therefore having decided that all the conditions precedent to the
taking effect of the policy had been complied with and having accepted the premium
ISSUE and delivered the policy thereafter to the insured, the company is now estopped to
WON the insurance policy is valid assert that it never intended that the policy should take effect.

HELD
SEPARATE OPINION
YES
- There is one line of cases which holds that the stipulation contained in paragraph 3 is
in the nature of a condition precedent, that is to say, that there can be no valid IMPERIAL [dissent]
delivery to the insured unless he is in good health at the time; that this condition - "A local agent of an insurance company, whose only power is to solicit applications
precedent goes to the very essence of the contract and cannot be waived by the for insurance, and forward them to the company for approval, when, if approved to
agent making delivery of the policy the insured, has no power to waive any of the provision of the policy so delivered."
- On the other hand, a number of American decisions hold that an agent to whom a - It is clear, therefore, that the delivery of the policy by Mendoza does not bind the
life insurance policy similar to the one here involved was sent with instructions to defendant, nor is the defendant estopped from alleging its defense, for the simple
deliver it to the insured has authority to bind the company by making such delivery, reason that Mendoza was not an agent with authority to issue policies or to accept
although the insured was not in good health at the time of delivery, on the theory that risks in the name of his principle.
the delivery of the policy being the final act to the consummation of the contract, the -There is another ground upon which the majority opinion is based, namely, that the
condition as to the insurer's good health was waived by the company. defendant waived the defense it now invokes, by reason of the delivery of the policy
- we are inclined to the view that it is more consonant with the well known practice of by its invokes, by reason of the delivery of the policy by its agent. It is admitted that if
life insurance companies and the evidence in the present case to rest our decision on the delivery of the policy was due to fraud, legally there could have been no waiver. In
the proposition that Mendoza was authorized by the company to make the delivery of view of the facts established and admitted, there is no doubt, as to the existence of
the policy when he received the payment of the first premium and he was satisfied the fraud. -Estrada, as a representative of the insured was not only bound to give a
that the insured was in good health. As was well said in the case of MeLaurin vs. truthful information on the state of health of the insured, but it was her duty to find
Mutual Life Insurance Co. “It is plain, therefore, that upon the facts it is not necessarily out it his true state of health in order to give true and correct information. When she
iNsuRanCe A2010 Dean Carale
pAgE 30♥

gave Mendoza an incorrect information tending to create the impression that the Reasoning
insured was well when in fact he was seriously ill, there is no doubt that she - Until quite recently, all of the provisions concerning life insurance in the Philippines
committed fraud and imparted a deceitful information to the defendant agent were found in the Code of Commerce and the Civil Code. After July 1, 1915, there was,
however, in force the Insurance Act. No. 2427. Chapter IV of this Act concerns life and
ENRIQUEZ v. SUN LIFE OF CANADA health insurance. The Act expressly repealed Title VIII of Book II and Section III of Title
III of Book III of the code of Commerce. The law of insurance is consequently now
41 PHIL 269
found in the Insurance Act and the Civil Code.
MALCOLM; November 29, 1920 - While, as just noticed, the Insurance Act deals with life insurance, it is silent as to the
methods to be followed in order that there may be a contract of insurance. On the
NATURE other hand, the Civil Code, in article 1802, not only describes a contact of life annuity
Appeal from judgment of trial court denying plaintiff’s (administrator of the estate of markedly similar to the one we are considering, but in two other articles, gives strong
the late Joaquin Ma. Herrer) action to recover from the defendant life insurance clues as to the proper disposition of the case. For instance, article 16 of the Civil Code
company the sum of pesos 6,000 paid by the deceased for a life annuity. provides that "In matters which are governed by special laws, any deficiency of the
latter shall be supplied by the provisions of this Code." On the supposition, therefore,
FACTS which is incontestable, that the special law on the subject of insurance is deficient in
- On September 24, 1917, Joaquin Herrer made application to the Sun Life Assurance enunciating the principles governing acceptance, the subject-matter of the Civil code,
Company of Canada through its office in Manila for a life annuity. Two days later he if there be any, would be controlling. In the Civil Code is found article 1262 providing
paid the sum of P6,000 to the manager of the company's Manila office and was given that "Consent is shown by the concurrence of offer and acceptance with respect to the
a receipt. thing and the consideration which are to constitute the contract. An acceptance made
- The application was immediately forwarded to the head office of the company at by letter shall not bind the person making the offer except from the time it came to
Montreal, Canada. On November 26, 1917, the head office gave notice of acceptance his knowledge.
by cable to Manila. (Whether on the same day the cable was received, notice was sent - According to the provisional receipt, three things had to be accomplished by the
by the Manila office of Herrera that the application had been accepted, is a disputed insurance company before there was a contract: (1) There had to be a medical
point, which will be discussed later.) On December 4, 1917, the policy was issued at examination of the applicant; (2) there had to be approval of the application by the
Montreal. On December 18, 1917, attorney Aurelio A. Torres wrote to the Manila office head office of the company; and (3) this approval had in some way to be
of the company stating that Herrer desired to withdraw his application. The following communicated by the company to the applicant. The further admitted facts are that
day the local office replied to Mr. Torres, stating that the policy had been issued, and the head office in Montreal did accept the application, did cable the Manila office to
called attention to the notification of November 26, 1917. This letter was received by that effect, did actually issue the policy and did, through its agent in Manila, actually
Mr. Torres on the morning of December 21, 1917. Mr. Herrer died on December 20, write the letter of notification and place it in the usual channels for transmission to the
1917. addressee.
- The chief clerk of the Manila office of Sun Life testified that he prepared the letter - The contract for a life annuity in the case at bar was not perfected because it has not
and handed it to the local manager, Mr. E. E. White, for signature. The local manager, been proved satisfactorily that the acceptance of the application ever came to the
Mr. White, testified to having received the cablegram accepting the application of Mr. knowledge of the applicant.
Herrer from the home office on November 26, 1917. He said that on the same day he Disposition Judgment is reversed, and the plaintiff shall have and recover from the
signed a letter notifying Mr. Herrer of this acceptance. The witness further said that defendant the sum of P6,000 with legal interest from November 20, 1918, until paid,
letters, after being signed, were sent to the chief clerk and placed on the mailing desk without special finding as to costs in either instance.
for transmission. Mr. Tuason, who was the chief clerk on November 26, 1917, was not
called as a witness.
- For the defense, attorney Manuel Torres testified to having prepared the will of
Joaquin Ma. Herrer. That on this occasion, Mr. Herrer mentioned his application for a
life annuity, and that he said that the only document relating to the transaction in his VELASCO and ACOSTA v. APOSTOL and MAHARLIKA
possession was the provisional receipt. Rafael Enriquez, the administrator of the
INSURANCE CO., INC.
estate, testified that he had gone through the effects of the deceased and had found
no letter of notification from the insurance company to Mr. Herrer. 173 SCRA 228
REGALADO.; May 9, 1989
ISSUE
WON there exists a contract for life annuity between Herrer and defendant NATURE
Petition for review on certiorari
HELD
NO FACTS
Ratio The law applicable to the case is found to be the second paragraph of article - Petitioners were plaintiffs in a civil case of which public respondent Hon. Apostol was
1262 of the Civil Code providing that an acceptance made by letter shall not bind the the judge.
person making the offer except from the time it came to his knowledge. - The case was an offshoot of an incident: plaintiffs were riding in their car, when a
iNsuRanCe A2010 Dean Carale
pAgE 31♥

taxicab crossed a center island in the road and collided with their car. Private
respondent Maharlika was eventually impleaded as a defendant in this case, with an
allegation that the taxicab involved was insured against third party liability for TIBAY v. CA (FORTUNE LIFE & GENERAL INSURANCE)
P20,000.00 with private respondent at the time of the accident
257 SCRA 126
- Maharlika claimed there was no cause of action against it because at the time of the
accident, the alleged insurance policy was not in force due to the non-payment of the BELLOSILLO; May 24, 1996
premium thereon. Also, even if the cab had been insured, the complaint would be
premature since the policy provides that the insurer would be liable only when the FACTS
insured becomes legally liable. - On 22 January 1987 Fortune Life and General Insurance Co., Inc. (FORTUNE) issued
- Trial court ruled in favor of the plaintiff, holding the defendants liable for repair of Fire Insurance Policy No. 136171 in favor of Violeta R. Tibay and/or Nicolas Roraldo on
the car, medical expenses, etc. BUT Maharlike was exonerated on the gnd that the their two-storey residential building located at 5855 Zobel Street, Makati City,
policy was not in force. together with all their personal effects therein. The insurance was for P600,000
- Petitioners elevated this case to this court, faulting the respondent judge for covering the period from 23 January 1987 to 23 January 1988. On 23 January 1987, of
considering the defense of late payment of premium when “the same was waived at the total premium of P2,983.50, Violeta Tibay only paid P600 thus leaving a
the pre-trial”, hence the evidence of late payment should be disregarded supposedly considerable balance unpaid.
because the private respondent had admitted that such fact was not in issue. - On 8 March 1987 the insured building was completely destroyed by fire. Two days
- (More pertinent to this class: ) petitioners assert that the private respondent had later, Violeta Tibay paid the balance of the premium. On the same day, she filed with
agreed to grant the then prospective insured a credit extension of the premium due. FORTUNE a claim on the fire insurance policy. Her claim was accordingly referred to
- This controversy arose under the old insurance law, Act No. 2427. its adjuster, Goodwill Adjustment Services, Inc. (GASI), which immediately wrote
- The accident occurred in 1973. The complaint was filed on July 20, 1974.  Violeta requesting her to furnish it with the necessary documents for the investigation
both before the effectivity of Presidential Decree no. 612, the subsequent insurance and processing of her claim. Petitioner forthwith complied. On 28 March 1987 she
law which repealed its predecessor signed a nonwaiver agreement with GASI to the effect that any action taken by the
- The former insurance law, which applies to the case here, provided: An insurer is companies shall not be, or be claimed to be, an admission of liability.
entitled to the payment of premium as soon as the thing insured is exposed to the - FORTUNE denied the claim of Violeta for violation of Policy Condition No. 2♪ and of
peril insured against, unless there is clear agreement to grant the insured credit Sec. 77 of the Insurance Code. Efforts to settle the case before the Insurance
extension of the premium due. No policy issued by an insurance company is valid and Commission proved futile. On 3 March 1988 Violeta and the other petitioners sued
binding unless and until the premium thereof has been paid. FORTUNE for damages in the amount of P600,000 representing the total coverage of
the fire insurance policy plus 12% interest per annum, P100,000 moral damages, and
ISSUE attorney's fees equivalent to 20% of the total claim. The trial court ruled for
WON the insurance policy would be valid and binding notwithstanding the non- petitioners. CA reversed.
payment of the premium
ISSUE
HELD WON a fire insurance policy is valid, binding and enforceable upon mere partial
NO payment of premium
Ratio Act No. 2427: an insurance policy would be valid and binding notwithstanding
the non-payment of the premium if there was a clear agreement to grant to the HELD
insured credit extension. Such agreement may be express or implied. NO
Reasoning Ratio Where the insurer and the insured expressly stipulated that the policy is not in
- Petitioners maintain that in spite of their late payment, the policy is binding because force until the premium has been fully paid the payment of partial premium by the
there was an implied agreement to grant a credit extension so as to make the policy assured in this particular instance should not be considered the payment required by
effective. To them, the subsequent acceptance of the premium and delivery of the the law and the stipulation of the parties. Rather, it must be taken in the concept of a
policy estops the respondent company from asserting that the policy is ineffective. deposit to be held in trust by the insurer until such time that the full amount has been
The court however sees no proof of any such implied agreement. The purported nexus tendered and duly receipted for.
between the delivery of the policy and the grant of credit extension is too tenuous to Reasoning
support the conclusion for which petitioners contend. - As expressly agreed upon in the contract, full payment must be made before the risk
 Parenthetically mention: in the present law, Section 77 of the Insurance Code of occurs for the policy to be considered effective and in force. Thus, no vinculum juris
1978 has deleted the clause "unless there is clear agreement to grant the insured whereby the insurer bound itself to indemnify the assured according to law ever
credit extension of the premium due" which was then involved in this controversy. resulted from the fractional payment of premium. The insurance contract itself
Disposition Fnding no reversible error, the judgment appealed from is hereby expressly provided that the policy would be effective only when the premium was paid
AFFIRMED. in full. It would have been altogether different were it not so stipulated. Ergo,


This policy including any renewal thereof and/or any endorsement thereon is not in force until the premium has
been fully paid to and duly receipted by the Company in the manner provided herein.
iNsuRanCe A2010 Dean Carale
pAgE 32♥

petitioners had absolute freedom of choice whether or not to be insured by FORTUNE - A third policy was again issued for the period 1 March 1984 to 1 March 1985. For
under the terms of its policy and they freely opted to adhere thereto. this, petitioner made two installment payments, both accepted by AHAC. Thereafter,
- Indeed, and far more importantly, the cardinal polestar in the construction of an petitioner refused to pay the balance of the premium. AHAC filed an action to recover
insurance contract is the intention of the parties as expressed in the policy. Courts the unpaid balance of P314,103.05.
have no other function but to enforce the same. The rule that contracts of insurance - Petitioner explained that it discontinued the payment of premiums because the
will be construed in favor of the insured and most strongly against the insurer should policy did not contain a credit clause in its favor and the receipts for the installment
not be permitted to have the effect of making a plain agreement ambiguous and then payments covering the policy for 1984-85, as well as the two (2) previous policies,
construe it in favor of the insured. Verily, it is elemental law that the payment of stated the following reservations:
premium is requisite to keep the policy of insurance in force. If the premium is not 2. Acceptance of this payment shall not waive any of the company rights to deny
paid in the manner prescribed in the policy as intended by the parties the policy is liability on any claim under the policy arising before such payments or after the
ineffective. Partial payment even when accepted as a partial payment will not keep expiration of the credit clause of the policy; and
the policy alive even for such fractional part of the year as the part payment bears to 3. Subject to no loss prior to premium payment. If there be any loss such is not
the whole payment. covered.
Disposition Petition is DENIED. Decision of the CA is AFFIRMED. - Petitioner further claimed that the policy was never binding and valid, and no risk
attached to the policy. It then pleaded a counterclaim for P152k for the premiums
already paid for 1984-85, and in its answer with amended counterclaim, sought the
refund of P924,206.10 representing the premium payments for 1982-85.
- Trial court dismissed the complaint and the counterclaim upon the following findings:
SEPARATE OPINION (1) payment of the premiums of the three policies were made during the term of said
policies, hence, it could not be said, inspite of the reservations, that no risk attached
under the policies; (2) as regards the unpaid premiums, in view of the reservation in
VITUG [dissent] the receipts ordinarily issued by AHAC on premium payments the only plausible
- The law neither requires, nor measures the strength of the vinculum juris by, any
conclusion is that AHAC has no right to demand their payment after the lapse of the
specific amount of premium payment. It should thus be enough that payment on the
term of said policy on March 1, 1985. Therefore, Tuscany was justified in refusing to
premium, partly or in full, is made by the insured which the insurer accepts. In fine, it
pay the same.
is either that a juridical tie exists (by such payment) or that it is not extant at all (by
- CA modified the decision by ordering Tuscany to pay the balance of the premiums
an absence thereof). Once the juridical relation comes into being, the full efficacy, not
due on the third policy plus legal interest until fully paid, and affirming the denial of
merely pro tanto, of the insurance contract naturally follows. Verily, not only is there
the counterclaim.
an insurance perfected but also a partially performed contract. In case of loss,
Petitioner’s Claims
recovery on the basis of the full contract value, less the unpaid premium can
Petitioner argues that where the premiums is not actually paid in full, the policy would
accordingly be had; conversely, if no loss occurs, the insurer can demand the
only be effective if there is an acknowledgment in the policy of the receipt of premium
payment of the unpaid balance of the premium. The insured, on the one hand, cannot
pursuant to Sec. 78 of the Insurance Code. The absence of an express
avoid the obligation of paying the balance of the premium while the insurer, upon the
acknowledgment in the policies of such receipt of the corresponding premium
other hand, cannot treat the contract as valid only for the purpose of collecting
payments, and petitioner's failure to pay said premiums on or before the effective
premiums and as invalid for the purpose of indemnity.
dates of said policies rendered them invalid. Petitioner thus concludes that there
cannot be a perfected contract of insurance upon mere partial payment of the
MAKATI TUSCANY v. CA ( AMERICAN HOME ASSURANCE CO.) premiums because under Sec. 77 of the Insurance Code, no contract of insurance is
215 SCRA 462 valid and binding unless the premium thereof has been paid, notwithstanding any
agreement to the contrary.
BELLOSILLO; November 6, 1992
ISSUE
NATURE WON payment by installment of the premiums due on an insurance policy invalidates
Appeal from decision of the CA the contract of insurance

FACTS HELD
- American Home Assurance Co. (AHAC), represented by American International Ratio Where the risk is entire and the contract is indivisible, the insured is not entitled
Underwriters (Phils.), Inc., issued in favor of petitioner Makati Tuscany Condominium to a refund of the premiums paid if the insurer was exposed to the risk insured for any
Corporation an insurance policy on the latter's building and premises, for the period 1 period, however brief or momentary.
March 1982 to1 March 1983. The premium was paid on installments all of which were Reasoning
accepted by AHAC. - The obligation to pay premiums when due is ordinarily as indivisible obligation to pay
- A second policy was issued to renew the first one, this time covering the period 1 the entire premium. Here, the parties herein agreed to make the premiums payable in
March 1983 to 1 March 1984. This was also pain in installment basis. installments, and there is no pretense that the parties never envisioned to make the
iNsuRanCe A2010 Dean Carale
pAgE 33♥

insurance contract binding between them. And the insured never informed the insurer ROMERO; September 22, 1994
that it was terminating the policy because the terms were unacceptable.
- There is nothing in Section 77 which suggests that the parties may not agree to NATURE CERTIORARI
allow payment of the premiums in installment, or to consider the contract as valid and
binding upon payment of the first premium. FACTS
- The records clearly show that petitioner and private respondent intended subject - June 29, 1985- 7 months after the issuance of Santos Areola's Personal Accident
insurance policies to be binding and effective notwithstanding the staggered payment Insurance Policy No. PA-20015 (covering a period of one year), Prudential unilaterally
of the premiums. Acceptance of payments speaks loudly of the insurer's intention to cancelled the same since company records revealed that Areola failed to pay his
honor the policies it issued to petitioner. premiums.
- Section 78 of the Insurance Code in effect allows waiver by the insurer of the o Under the terms of the statement of account issued by Prudential, Areola was
condition of prepayment by making an acknowledgment in the insurance policy of supposed to pay the total amount of P1,609.65 which included the premium of
receipt of premium as conclusive evidence of payment so far as to make the policy P1,470.00, documentary stamp of P110.25 and 2% premium tax of P29.40.
binding despite the fact that premium is actually unpaid. Section 77 merely precludes o The statement of account stated that it must not be considered a receipt as an
the parties from stipulating that the policy is valid even if premiums are not paid, but official receipt will be issued upon payment of the account. And if payment was
does not expressly prohibit an agreement granting credit extension, and such an made to a representative, the client must demand for a Provisional Receipt and if
agreement is not contrary to morals, good customs, public order or public policy. Official Receipts aren’t received within 7 days, Prudential should be notified. If
- At the very least, both parties should be deemed in estoppel to question the payment is made to their office, clients should demand for an OR.
arrangement they have voluntarily accepted. - August 3, 1985- Prudential offered to reinstate same policy it had previously
Disposition Judgment affirmed. Costs against petitioner. cancelled and even proposed to extend its lifetime to December 17, 1985, upon a
finding that the cancellation was erroneous and that the premiums were paid in full by
SOUTH SEA SURETY AND INSURANCE v. CA (VALENZUELA Areola but were not remitted by Teofilo M. Malapit, Prudential's branch manager.
HARDWOOD) Petitioners’ Claims
- The fraudulent act of in misappropriating Areola’s premium payments is the
244 SCRA 744 proximate cause of the cancellation of the insurance policy.
VITUG; June 2, 1995 - Areola theorized that Malapit's act of signing and even sending the notice of
cancellation himself, notwithstanding his personal knowledge of petitioner-insured's
NATURE full payment of premiums, further reinforces the allegation of bad faith.
Petition for review on certiorari - Such fraudulent act committed by Malapit is attributable to Prudential.
- Malapit's actuations are therefore not separate and distinct from that of Prudential’s.
FACTS It must, therefore, bear the consequences of the erroneous cancellation of subject
- Hardwood entered into agreement with Seven Bros Shipping, where latter undertook insurance policy caused by the non-remittance by its own employee of the premiums
to load the former’s logs on vessel. Hardwood insured the logs with South Sea Surety paid.
which issued Marine Cargo Insurance Policy. The vessel sank Jan 25, 1984. - Subsequent reinstatement could not possibly absolve respondent insurance
- Hardwood filed claim with South Sea and Seven Bros. Trial Court favored Hardwood. company from liability, there being an obvious breach of contract. After all damage
CA decided against South Sea, but absolved Seven Bros. South Sea filed this instant had already been inflicted on him and no amount of rectification could remedy the
petition. same.
Respondent’s Argument
ISSUES - Prudential argues that where reinstatement, the equitable relief sought by Areola
WON the insurance contract was already in effect when the vessel sank was granted at an opportune moment, i.e. prior to the filing of the complaint, Areola is
left without a cause of action on which to predicate his claim for damages.
HELD - Reinstatement effectively restored Areola to all his rights under the policy.
YES
- It is already in effect because Hardwood has already paid the insurance premium. ISSUES
It delivered the check to Victorio Chua before the vessel sank, but Victorio Chua was 1. WON the erroneous act of canceling subject insurance policy entitle petitioner-
only to deliver the check to South Sea five days after the vessel sank. insured to payment of damages
Appellant argues that Chua was not its broker, but it was found that Chua was 2. WON the subsequent act of reinstating the wrongfully cancelled insurance policy
authorized by South Sea to receive the premium on its behalf. obliterate whatever liability for damages Prudential has

HELD
AREOLA v. CA (PRUDENTIAL GUARANTEE AND ASSURANCE, 1. YES
2. NO
INC.) Reasoning
236 SCRA 643
iNsuRanCe A2010 Dean Carale
pAgE 34♥

- Malapit's fraudulent act of misappropriating the premiums paid by petitioner-insured Disposition Petition for review on certiorari is hereby GRANTED. RTC’ s DECISION is
is beyond doubt directly imputable to Prudential. REINSTATED.
- A corporation, such as respondent insurance company, acts solely thru its
employees. The latter’s acts are considered as its own for which it can be held to UCPB GENERAL INSURANCE CO., INC. v. MASAGANA
account.
- The facts are clear as to the relationship between private respondent insurance TELAMART, INC.
company and Malapit. His act of receiving the premiums collected is well within the 308 SCRA 259
province of his authority as manager. Thus, his receipt of said premiums is receipt by PARDO; June 15, 1999
private respondent insurance company who, by provision of law, particularly under
Article 1910 of the Civil Code, is bound by the acts of its agent. NATURE
- Article 1910 thus reads: Petition for review on certiorari of a decision of the Court of Appeals.
Art. 1910. The principal must comply with all the obligations which the agent may
have contracted within the scope of his authority. FACTS
As for any obligation wherein the agent has exceeded his power, the principal is not - On April 15, 1991, petitioner issued five (5) insurance policies covering respondent's
bound except when he ratifies it expressly or tacitly. various property described therein against fire, for the period from May 22, 1991 to
- Malapit's failure to remit the premiums he received cannot constitute a defense for May 22, 1992.
private respondent insurance company; no exoneration from liability could result - In March 1992, petitioner evaluated the policies and decided not to renew them upon
therefrom. expiration of their terms on May 22, 1992. Petitioner advised respondent's broker,
- Prudential’s earlier act of reinstating the insurance policy can not obliterate the Zuellig Insurance Brokers, Inc. of its intention not to renew the policies.
injury inflicted on petitioner-insured. - On April 6, 1992, petitioner gave written notice to respondent of the non-renewal of
- Respondent company should be reminded that a contract of insurance creates the policies at the address stated in the policies.
reciprocal obligations for both insurer and insured. - On June 13, 1992, fire razed respondent's property covered by three of the insurance
- Reciprocal obligations are those which arise from the same cause and in which each policies petitioner issued.
party is both a debtor and a creditor of the other, such that the obligation of one is - On July 13, 1992, respondent presented to petitioner's cashier at its head office five
dependent upon the obligation of the other. (5) manager's checks in the total amount of P225,753.95, representing premium for
- Under the circumstances of instant case, the relationship as creditor and debtor the renewal of the policies from May 22, 1992 to May 22, 1993. No notice of loss was
between the parties arose from a common cause: i.e., by reason of their agreement to filed by respondent under the policies prior to July 14, 1992.
enter into a contract of insurance under whose terms, Prudential promised to extend - On July 14, 1992, respondent filed with petitioner its formal claim for indemnification
protection to Areola against the risk insured for a consideration in the form of of the insured property razed by fire. On the same day, petitioner returned to
premiums to be paid by the latter. respondent the five manager's checks that it tendered, and at the same time rejected
- Under the law governing reciprocal obligations, particularly the second paragraph of respondent's claim for the reasons (a) that the policies had expired and were not
Article 1191, the injured party, Areola in this case, is given a choice between renewed, and (b) that the fire occurred on June 13, 1992, before respondent's tender
fulfillment or rescission of the obligation in case one of the obligors, such as of premium payment.
respondent insurance company, fails to comply with what is incumbent upon him. - On July, 21, 1992, respondent filed with the Regional Trial Court, Branch 58, Makati
- However, said article entitles the injured party to payment of damages, regardless of City, a civil complaint against petitioner for recovery, of P18.645,000.00, representing
whether he demands fulfillment or rescission of the obligation. the face value of the policies covering respondent's insured property razed by fire,
- Untenable then is reinstatement insurance company's argument, namely, that and for attorney's fees.
reinstatement being equivalent to fulfillment of its obligation, divests petitioner- - On October 23, 1992, after its motion to dismiss had been denied, petitioner filed an
insured of a rightful claim for payment of damages. Such a claim finds no support in answer to the complaint. It alleged that the complaint "fails to state a cause of
our laws on obligations and contracts. action"; that petitioner was not liable to -respondent for insurance proceeds under the
DAMAGES: policies because at the time of the loss of respondent's property due to fire, the
- The nature of damages to be awarded, however, would be in the form of nominal policies had long expired and were not renewed.
damages After due trial, on March 10, 1993, the Regional Trial Court, Branch 58, Makati,
- Although the erroneous cancellation of the insurance policy constituted a breach of rendered decision, the dispositive portion of which reads:
contract, Prudential within a reasonable time took steps to rectify the wrong "WHEREFORE, premises considered, judgment is hereby rendered in favor of the
committed by reinstating the insurance policy of petitioner. plaintiff and against the defendant, as follows.
- Moreover, no actual or substantial damage or injury was inflicted on petitioner Areola "(1) Authorizing and allowing the plaintiff to consign/deposit with this Court the sum of
at the time the insurance policy was cancelled. P225,753.95 (refused by the defendant) as full payment of the corresponding
- Nominal damages are "recoverable where a legal right is technically violated and premiums for the replacement-renewal policies for Exhibits A, B, C, D and E; "(2)
must be vindicated against an invasion that has produced no actual present loss of Declaring plaintiff to have fully complied with its obligation to pay the premium
any kind, or where there has been a breach of contract and no substantial injury or thereby rendering the replacement-renewal policy of Exhibits A, B, C, D and E
actual damages whatsoever have been or can be shown. effective and binding for the duration May 22, 1992 until May 22, 1993; and, ordering
defendant to deliver forthwith to plaintiff the said replacement-renewal policies; "(3)
iNsuRanCe A2010 Dean Carale
pAgE 35♥

Declaring Exhibits A & B, in force from August 22, 1991 up to August 23, 1992 and arrangement though actual payment of premium was tendered on a later date and
August 9, 1991 to August 9, 1992, respectively; and "(4) Ordering the defendant to after the occurrence of the (fire) risk insured against.” The Court resolved this issue in
pay plaintiff the sums of. (a) P18,645,000.00 representing the latter's claim for the negative in view of Section 77 of the Insurance Code and its decisions in
indemnity under Exhibits A, B & C and/or its replacement-renewal policies; (b) 25% of Valenzuela v. Court of Appeals; South Sea Surety and Insurance Co., Inc. v. Court of
the total amount due as and for attorney's fees; (c) P25,000.00 as necessary litigation Appeals; and Tibay v. Court of Appeals. Accordingly, it reversed and set aside the
expenses; and, (d) the costs of suit. decision of the Court of Appeals.
“xxx ” - Respondent seasonably filed a motion for the reconsideration of the adverse verdict.
- In due time, petitioner appealed to the Court of Appeals (CA). The CA promulgated It alleges in the motion that the SC had made in the decision its own findings of facts,
its decision affirming that of the Regional Trial Court with the modification that item which are not in accord with those of the trial court and the Court of Appeals. The
No. 3 of the dispositive portion was deleted, and the award of attorney's fees was courts below correctly found that no notice of non-renewal was made within 45 days
reduced to 10% of the total amount due. before 22 May 1992, or before the expiration date of the fire insurance policies. Thus,
It held that following previous practice, respondent was allowed a 60- to 90-day credit the policies in question were renewed by operation of law and were effective and valid
term for the renewal of its policies, and that the acceptance of the late premium on 30 June 1992 when the fire occurred, since the premiums were paid within the 60-
payment suggested an understanding that payment could be made later. Hence, this to 90-day credit term.
appeal. - Respondent likewise disagrees with its ruling that parties may neither agree
expressly or impliedly on the extension of credit or time to pay the premium nor
ISSUE consider a policy binding before actual payment. It urges the Court to take judicial
WON the fire insurance policies issued by petitioner to the respondent covering the notice of the fact that despite the express provision of Section 77 of the Insurance
period May 22, 1991 to May 22, 1992, had expired on the latter date or had been Code, extension of credit terms in premium payment has been the prevalent practice
extended or renewed by an implied credit arrangement though actual payment of in the insurance industry. Most insurance companies, including Petitioner, extend
premium was tendered on a later date after the occurrence of the risk (fire) insured credit terms because Section 77 of the Insurance Code is not a prohibitive injunction
against but is merely designed for the protection of the parties to an insurance contract. The
Code itself, in Section 78, authorizes the validity of a policy notwithstanding non-
HELD payment of premiums.
NO - Respondent also asserts that the principle of estoppel applies to Petitioner. Despite
- An insurance policy, other than life, issued originally or on renewal, is not valid and its awareness of Section 77 Petitioner persuaded and induced Respondent to believe
binding until actual payment of the premium. Any agreement to the contrary is void. that payment of premium on the 60- to 90-day credit term was perfectly alright; in
The parties may not agree expressly or impliedly on the extension of credit or time to fact it accepted payments within 60 to 90 days after the due dates. By extending
pay the premium and consider the policy binding before actual payment. credit and habitually accepting payments 60 to 90 days from the effective dates of
Disposition Judgment reversed and set aside the policies, it has implicitly agreed to modify the tenor of the insurance policy and in
effect waived the provision therein that it would pay only for the loss or damage in
case the same occurred after payment of the premium.
- Petitioner filed an opposition to the Respondent’s motion for reconsideration. It
argues that both the trial court and the Court of Appeals overlooked the fact that on 6
April 1992 Petitioner sent by ordinary mail to Respondent a notice of non-renewal and
sent by personal delivery a copy thereof to Respondent’s broker, Zuellig. Both courts
likewise ignored the fact that Respondent was fully aware of the notice of non-
renewal. A reading of Section 66 of the Insurance Code readily shows that in order for
an insured to be entitled to a renewal of a non-life policy, payment of the premium
due on the effective date of renewal should first be made. Respondent’s argument
UCPB GENERAL INSURANCE CO., INC. v. MASAGANA that Section 77 is not a prohibitive provision finds no authoritative support.
- The following facts, as found by the trial court and the Court of Appeals, are indeed
TELAMART, INC. (EN BANC)
duly established:
356 SCRA 307 1. For years, Petitioner had been issuing fire policies to the Respondent, and
DAVIDE; April 4, 2001 these policies were annually renewed.
2. Petitioner had been granting Respondent a 60- to 90-day credit term within
NATURE which to pay the premiums on the renewed policies.
Motion for reconsideration of the decision of the Supreme Court. 3. There was no valid notice of non-renewal of the policies in question, as there
is no proof at all that the notice sent by ordinary mail was received by
FACTS Respondent, and the copy thereof allegedly sent to Zuellig was ever transmitted
- In its decision of 15 June 1999, the SC defined the main issue to be “whether the fire to Respondent.
insurance policies issued by petitioner to the respondent covering the period from 4. The premiums for the policies in question in the aggregate amount of
May 22, 1991 to May 22, 1992 had been extended or renewed by an implied credit P225,753.95 were paid by Respondent within the 60- to 90-day credit term and
iNsuRanCe A2010 Dean Carale
pAgE 36♥

were duly accepted and received by Petitioner’s cashier. such practice. Estoppel then is the fifth exception to Section 77.
Disposition Judgment reconsidered and set aside, that of the Court of Appeals
ISSUE affirmed in toto.
WON Sec. 77 of the Insurance Code of 1978 must be strictly applied to Petitioner’s
advantage despite its practice of granting a 60- to 90-day credit term for the payment SEPARATE OPINION
of premiums
HELD
VITUG
NO
- An essential characteristic of an insurance is its being synallagmatic, a highly
- Section 77 of the Insurance Code of 1978 provides:
reciprocal contract where the rights and obligations of the parties correlate and
SEC. 77. An insurer is entitled to payment of the premium as soon as the thing
mutually correspond.
insured is exposed to the peril insured against. Notwithstanding any agreement to
- By weight of authority, estoppel cannot create a contract of insurance, neither can it
the contrary, no policy or contract of insurance issued by an insurance company is
be successfully invoked to create a primary liability, nor can it give validity to what
valid and binding unless and until the premium thereof has been paid, except in the
the law so procribes as a matter of public policy.
case of a life or an industrial life policy whenever the grace period provision applies.
- This Section is a reproduction of Section 77 of P.D. No. 612 (The Insurance Code)
promulgated on 18 December 1974. In turn, this Section has its source in Section 72 PARDO [dissent]
of Act No. 2427 otherwise known as the Insurance Act as amended by R.A. No. 3540, - An assured’s failure to give notice of the fire immediately upon its occurrence
approved on 21 June 1963, which read: blatantly showed the fraudulent character of its claims. Respondent is required by law
SEC. 72. An insurer is entitled to payment of premium as soon as the thing insured and by express terms of the policy to give immediate written notice of loss. This must
is exposed to the peril insured against, unless there is clear agreement to grant the be complied with in the utmost good faith.
insured credit extension of the premium due. No policy issued by an insurance - Assuming arguendo that the 60- to 90-day credit has been agreed between the
company is valid and binding unless and until the premium thereof has been paid. parties, respondent could not still invoke estoppel to back up its claim. Estoppel
(Underscoring supplied) cannot give validity to an act that is prohibited by law or against public policy. The
- It can be seen at once that Section 77 does not restate the portion of Section 72 actual payment of premiums is a condition precedent to the validity of an insurance
expressly permitting an agreement to extend the period to pay the premium. But contract other than life insurance policy. Any agreement to the contrary is void as
there are exceptions to Section 77. against law and public policy.
The first exception is provided by Section 77 itself, and that is, in case of a life or
industrial life policy whenever the grace period provision applies. ACME SHOE RUBBER & PLASTIC CORP. v. CA (DOMESTIC
The second is that covered by Section 78 of the Insurance Code, which provides: INSURANCE COMPANY OF THE PHILS.)
SEC. 78. Any acknowledgment in a policy or contract of insurance of the receipt of
134 SCRA 155
premium is conclusive evidence of its payment, so far as to make the policy
binding, notwithstanding any stipulation therein that it shall not be binding until MELENCIO-HERRERA; January 17, 1985.
premium is actually paid.
- A third exception was laid down in Makati Tuscany Condominium Corporation vs. NATURE
Court of Appeals, wherein we ruled that Section 77 may not apply if the parties have Petition for Review on Certiorari of the Decision of the then Court of Appeals (CA-G. R.
agreed to the payment in installments of the premium and partial payment has been No. 58917-R), denying recovery on an insurance policy, thereby reversing the
made at the time of loss. Tuscany has provided a fourth exception to Section 77, judgment of the Court of First Instance of Rizal, Branch XII, at Caloocan City, which
namely, that the insurer may grant credit extension for the payment of the premium. had allowed such recovery.
This simply means that if the insurer has granted the insured a credit term for the
payment of the premium and loss occurs before the expiration of the term, recovery FACTS
on the policy should be allowed even though the premium is paid after the loss but - ACME Shoe Rubber and Plastic Corporation (ACME) had been insuring yearly against
within the credit term. fire its building, machines and general merchandise with Domestic Insurance
Moreover, there is nothing in Section 77 which prohibits the parties in an insurance Company (INSURER) since 1946. On May 14, 1962, ACME continued to insure its
contract to provide a credit term within which to pay the premiums. That agreement properties with INSURER in the amount of P200,000 for the period May 15, 1962 up to
is not against the law, morals, good customs, public order or public policy. The May 15, 1963.
agreement binds the parties. Article 1306 of the Civil Code provides: - On May 14, 1963, INSURER issued Renewal Receipt to cover the period May 15, 1963
ART. 1306. The contracting parties may establish such stipulations clauses, terms to May 15, 1964.
and conditions as they may deem convenient, provided they are not contrary to - On January 8, 1964, ACME paid P3,331.26 as premium. The INSURER applied the
law, morals, good customs, public order, or public policy. payment as renewal premium for the period of May 15, 1963 to May 15, 1964.
- Finally, it would be unjust and inequitable if recovery on the policy would not be - On May 15, 1964, INSURER issued a Renewal Receipt for the period of May 15, 1964
permitted against Petitioner, which had consistently granted a 60- to 90-day credit to May 15, 1965 (for renewal premium of P3,331.26 yet to be paid) with a stamped
term for the payment of premiums despite its full awareness of Section 77. Estoppel note that says that the insurance will be deemed valid and binding only when the
bars it from taking refuge under said Section since Respondent relied in good faith on premium and documentary stamps have actually been paid in full and duly
iNsuRanCe A2010 Dean Carale
pAgE 37♥

acknowledged in an official receipt. ACME was given 90 days to pay otherwise the Disposition The judgment under review is hereby affirmed. Without pronouncement
policy would automatically become void and ineffective. (ACME should pay short as to costs.
period premium for 90 days before the period expires. If they are able to pay the
whole amount before the 90-day period, the automatic termination won’t apply PEDRO ARCE v. THE CAPITAL INSURANCE & SURETY CO., INC.
anymore).
11 SCRA 63
- On May 26, 1964, ACME, through its President, signed a promissory note saying that
they promise to pay the premium and documentary stamps and agreed to the ABAD SANTOS; September 30, 1982.
automatic cancellation penalty for not complying.
- On October 13, 1964, ACME’s properties were completely destroyed by fire. ACME NATURE
filed insurance claim but the INSURER disclaimed liability on the ground that as of the Appeal from CFI decision on question of law.
date of loss, the properties burned were not covered by insurance.
- ACME claims that the January 8, 1964 payment was for the period 1964-1965 and FACTS
that INSURER had no right to apply it to the period 1963-1964 because under RA - Arce (INSURED) owned a residential house which was insured with the appellant
3540, the policy was void and INSURER could have validly disclaimed liability for loss COMPANY since 1961. In November 1965, the COMPANY sent to the INSURED a
had one occurred then. Renewal Certificate to cover the period from December 5, 1965 to December 5,1966,
- TC found INSURER liable for P200k and opined that there was a clear intention on the and requested payment of the corresponding premium. Anticipating that the premium
INSURER's part to grant ACME a credit extension for the payment of the premium due; could not be paid on time, the INSURED asked for an extension which was granted by
and that to allow the INSURER to apply the premium ACME paid on January 8, 1964. the COMPANY. After the lapse of the requested extension, INSURED still failed to pay
CA reversed TC and dismissed the suit on the ground that, as of the moment of loss, the premium. Thereafter, the house of the INSURED was totally destroyed by fire.
ACME's properties were not insured and the INSURER could not be held liable for any Upon INSURED's presentation of claim for indemnity, he was told that no indemnity
indemnity as a result of the loss. was due because the premium was not paid. The INSURED sued the COMPANY for
indemnity.
ISSUE - The trial court held the COMPANY liable to indemnify the INSURED on the ground
WON the premium payment for 1964-1965 was paid that since the COMPANY could have demanded payment of the premium, mutuality of
obligation required that it should be liable on the policy.
HELD
NO ISSUE
- Not having paid the 1964-1965 premium within the extension granted, and pursuant WON the COMPANY can be held liable on its policy
to R.A. No. 3540, the policy was automatically cancelled and there was no insurance
coverage to speak of as of the date of the fire on October 13, 1964. HELD
- The pertinent provision of Republic Act No. 3540 reads: NO.
"Sec. 72. An insurer is entitled to payment of the premium as soon as the thing - The Court commiserates with the INSURED. They are well aware that many insurance
insured is exposed to the peril insured against, unless there is clear agreement to companies have fallen into the condemnable practice of collecting premiums promptly
grant the insured credit extension of the premium due. No policy issued by an but resort to all kinds of excuses to deny or delay payment of just claims. Unhappily
insurance company is valid and binding unless and until the premium thereof has the instant case is one where the insurer has the law on its side.
been paid." - Sec. 72 of the Insurance Act, as amended by R.A. No. 3540 reads:
- RA 3540 was approved on June 20, 2963 and was put into effect on Oct 1, 1963. It "SEC. 72. An insurer is entitled to payment of premium as soon as the thing insured
could not be applied retroactively to the renewal of the policy for the 1963-1964 is exposed to the perils insured against, unless there is clear agreement to grant
period because said policy was renewed on May 14, 1963. (Laws have no retroactive credit extension for the premium due. No policy issued by an insurance company is
effect unless the contrary is provided.) Therefore, the Jan 8, 1964 payment was valid and binding unless and until the premium thereof has been paid."
properly applied to the 1963-1964 premium. The Trial Court's opinion that there was a - It is obvious from both the Insurance Act, as amended, and the stipulation of the
clear agreement to grant ACME credit extension for 1964-1965 is negated by ACME's parties that time is of the essence in respect of the payment of the insurance
Promissory Note binding itself to pay within ninety days from the effective date of this premium so that if it is not paid the contract does not take effect unless there is still
policy, 15th May, 1964. The credit extension was granted for 90 days only. (So wala another stipulation to the contrary. In the instant case, the INSURED was given a
na by August 16, 1964.) grace period to pay the premium but the period having expired with no payment
- If ACME was granted credit extensions in the past, the promissory note it signed did made, he cannot insist that the COMPANY is nonetheless obligated to him.
away with such credit arrangement. Also, before RA 3540, the Renewal Receipts - Prior to the amendment (italicized portion above), an insurance contract was
issued by INSURER did not contain the auto-cancellation after 90 days note. By 1964, effective even if the premium had not been paid so that an insurer was obligated to
however, the situation had changed by the passage of the RA: no policy could be pay indemnity in case of loss and correlatively he had also the right to sue for
valid and binding unless and until the premium thereof had been paid. payment of the premium. But the amendment to Sec. 72 has radically changed the
- What became automatically cancelled by R.A. No. 3540 was the 1964-1965 policy for legal regime in that unless the premium is paid there is no insurance.
ACME's failure to pay the premium within the 90-day extension granted, and in Disposition The decision of the court a quo is reversed; the appellee's complaint is
accordance with the express terms of the Promissory Note that it had signed. dismissed. No special pronouncement as to costs.
iNsuRanCe A2010 Dean Carale
pAgE 38♥

- Irrelevant facts: The premium costs P38.10. After the fire, the COMPANY issued a - Although the check was due for payment on January 16, 1961 and Plastic Era had
check for P300 to Arce as donation. Arce accepted the check, but still sued the sufficient funds to cover it as of January 19, 1961, Capital Insurance decided to hold
company. the same for thirty-five (35) days before presenting it for payment. Having held the
CAPITAL INC. v. PLASTIC ERA CO. check for such an unreasonable period of time, Capital Insurance was estopped from
claiming a forfeiture of its policy for non-payment even if the check had been
65 SCRA 134
dishonored later. Where the check is held for an unreasonable time before presenting
MARTIN; July 18, 1975 it for payment, the insurer may be held estopped from claiming a forfeiture if the
check is dishonored.
NATURE Disposition The decision of the CA is AFFIRMED in toto.
Petition for review of a decision of the CA affirming the decision of the CFI of Manila
MALAYAN INSURANCE CO., INC. v. ARNALDO and PINCA
FACTS
- On December 17, 1960, petitioner Capital Insurance & Surety Co., Inc. delivered to 154 SCRA 672
the respondent Plastic Era Manufacturing Co., Inc., its open Fire Policy No. 22760 CRUZ; October 12, 1987
wherein the former undertook to insure the latter's building, equipments, raw
materials, products and accessories located at Sheridan Street, Mandaluyong, Rizal. FACTS
The policy expressly provides that if the property insured would be destroyed or - On June 7, 1981, Malayan Insurance Co. (MICO), issued fire insurance for the amount
damaged by fire after the payment of the premiums, at anytime between the 15th of P14,000 on the property of private respondent, Pinca, effective July 1981-1982.
day of December 1960 and one o'clock in the afternoon of the 15th day of December MICO later allegedly cancelled the policy for non-payment of the premium and sent a
1961, the insurance company shall make good all such loss or damage in an amount notice to Pinca. On Dec. 24 Adora, an agent of MICO, received Pinca’s payment, which
not exceeding P100,000.00. When the policy was delivered, Plastic Era failed to pay was remitted to MICO. On Jan. 18, 1982, Pinca’s property was completely burned. On
the corresponding insurance premium. On January 8, 1961, in partial payment of the Feb. 5, MICO returned Pinca’s payment to Adora on the ground that her policy had
insurance premium, Plastic Era delivered to Capital Insurance, a check for the amount been cancelled; the latter refused to accept it. Her demand for payment having been
of P1,000.00 postdated January 16, 1961. However, Capital Insurance tried to deposit rejected by MICO, Pinca went to the Insurance Commission. Public respondent
the check only on February 20, 1961 and the same was dishonored by the bank for Arnaldo, the Insurance Commissioner, sustained Pinca, hence this petition from MICO.
lack of funds. Records show MICO received Arnaldo’s decision on April 10; MICO filed a MFR on April
- Two days after the insurance premium became due, at about 4:00 to 5:00 o'clock in 25 which was denied on June 4; MICO received notice of this denial on June 14; instant
the morning, the property insured by Plastic Era was destroyed by fire. In less than a petition was filed on July 2.
month Plastic Era demanded from Capital Insurance the payment of the sum of
P100,000.00 as indemnity for the loss of the insured property under Policy No. 22760 ISSUES
but the latter refused for the reason that, among others, Plastic Era failed to pay the Procedural
insurance premium. 1. WON the petition should be dismissed for late filing
Substantive
ISSUES 2. WON there was a valid insurance contract at the time of the loss
1. WON a contract of insurance has been duly perfected between petitioner and 3. WON Adora was authorized to receive such payment
respondent 4. WON an adjuster is indispensable in the valuation of the loss
2. WON the dishonored check constituted payment
HELD
HELD Procedural
1. YES 1. YES
- Tender of draft or check in order to effect payment that would extinguish the - Petitioner invokes Sec 416 of the Insurance Code which grants it 30 days from notice
debtor's liability should be actually cashed. If the delivery of the check of Plastic Era to of the Insurance Commission within which to appeal by certiorari with the Court. MICO
Capital Insurance were to be viewed in the light of the foregoing, no payment of the filed its MFR on April 25, 15 days after the notice; the reglementary period began to
premium had been effected. Significantly, Capital Insurance accepted the promise of run again after June 13. Since the petition was filed only on July 2, it was tardy by 4
Plastic Era to pay the insurance premium within 30 days from the effective date of days. Alternatively it invokes Rule 45 of the Rules of Court for certiorari but the
policy. By so doing, it has implicitly agreed to modify the tenor of the insurance policy petition still exceeds the 15 day limit from the June 13 notice.
and in effect, waived the provision therein that it would only pay for the loss or -Respondents, on the other hand, invoke Sec. 39 of B.P. 129 which pegs the period for
damage in case the same occurs after the payment of the premium. Considering that appeal from decisions of any court in all cases at 15 days from the notice of the
the insurance policy is silent as to the mode of payment, Capital Insurance is deemed decision appealed from. Since the MFR was filed only 15 days after receiving notice of
to have accepted the promissory note in payment of the premium. This rendered the the decision, it was already 18 days late by July 2. So whichever is applied, the
policy immediately operative on the date it was delivered. petition is still late.
2. YES Substantive
2. YES
iNsuRanCe A2010 Dean Carale
pAgE 39♥

- A valid cancellation requires the following conditions based on Sections 64-65 of the 1941. But due to the exigencies of the war It closed the branch office at Manila during
Code: prior notice which must be based on the occurrence of one or more of the 1942 up to September 1945.
grounds mentioned in Sec 64 (in this case, non-payment of premium), after the - Plaintiff issued a number of life insurance policies in the Philippines containing
effective date of the policy; the notice must be written and mailed to the address on stipulations referred to as NONFORFEITURE CLAUSES5
the policy; it must state the ground(s) for cancellation and the insurer must furnish - From January 1, 1942 to December 31, 1946, Plaintiff head office at Toronto applied
details upon the request of the insured. the provisions of the automatic premium loan clauses upon the nonpayment of the
- It is undisputed that payment of premium was made. Petitioner relies heavily on Sec corresponding premiums by the people who subscribed to the insurance. The net
77 of the Insurance Code to contest this, the said provision requiring payment of amount of premiums advanced (by the company) or loaned (to the insured) as
premium as soon as the thing is exposed to the peril insured against and that the payment for the premium due totaled P1,069,254.98.
policy is invalid without it. However, this is not applicable in the instant case as - Meer, the Collector of the National Internal Revenue assessed the net amount of
payment was eventually made. It is to be noted that the premium invoice was premium at P17,917.12 pursuant to SEC.255, National Internal Revenue Code6
stamped “Payment Received”, indicating an understanding between the parties that - Company protested the assessment, but paid the taxes anyway. Then they filed a
payment could be made later. This is furthered by the fact that Adora had earlier told complaint to recover money paid under protest for taxes
her to call him anytime she was ready with her payment. The Court also finds it - CFI: Dissmiss complaint
strange that MICO only sought to return Pinca’s Jan. 15 payment only on Feb. 5, long - PLAINTIFF’s MAIN CONTENTION: when it made premium loans or premium advances
after her house had burned down—this makes petitioner’s motives highly suspect. by virtue of the non-forfeiture clauses, it did not collect premiums within the meaning
- MICO claims to have sent a notice to Pinca, who flatly denied receiving one. Pinca did of the above sections of the law, and therefore it is not amenable to the tax therein
not have to prove this since the strict language of Sec 64 requires that MICO ensure provided.
the cancellation was actually sent to and received by the insured.
- MICO also suggests that Pinca knew the policy had been cancelled and was paying ISSUES
the premium in order to renew the policy. A close study of the transcripts show, 1. WON premium advances made by plaintiff-appellant under the automatic premium
however, that Pinca only meant to renew the policy had it been cancelled but not if it loan clause of its policies are premiums collected' by the Company subject to tax
was still in effect—it was conditional. Payment was thus legally made on the original 2. WON, in the application of the automatic premium loan clause of plaintiff-
transaction and validly received by Adora, who was not informed of the alleged appellant's policies, there is 'payment in money, notes, credits, or any substitutes for
cancellation and thus saw no reason to reject the payment. money
3. YES 3. WON the collection of the alleged deficiency premium taxes constitutes double
- Sec. 306 of the Insurance Code provides that any insurance company that delivers a taxation
policy to its agent is deemed to have authorized such agent to receive payment of 4. WON the making of premium advances, granting for the sake of argument that it
premium on its behalf. It is a well-known principle under the law of agency that amounted to collection of premiums, were done in Toronto, Canada
payment to an authorized agent is equivalent to payment to the principal himself. 5. WON the fact that plaintiff-appellant was not doing business in the Philippines
MICO’s acknowledgement of Adora as its agent thus defeats its contention that he was during the period from January 1, 1942 to September 30, 1945, inclusive, exempts it
not authorized to receive payments on its behalf. from payment of premium taxes corresponding to said period
4. NO
- In absence of fraud, the amount of the loss may be determined on the basis of such
proof offered by the insured. Here. The certification of the Integrated National Police HELD
as the extent of the loss should suffice. NOTE (example given by the plaintiff): "Suppose that 'A', 30 years of age,
Disposition petition is DENIED secures a 20-year endowment policy for P5,000 from plaintiff-appellant Company and
pays an annual premium of P250. 'A' pays the first ten yearly premiums amounting to
MANUFACTURERS LIFE INSURANCE CO. v. MEER P2,500 and on this amount plaintiff-appellant pays the corresponding taxes under
89 PHIL 351 5"'8. Automatic Premium Loan.-This Policy shall not lapse for non-payment of any premium after it has been three full years in force, it, at the due date of such premium, the Cash Value of this Policy
BENGZON, June 29, 1951 and of any bonus additions and dividends left on accumulation (after deducting any indebtedness to the company and the interest accrued thereon) shall exceed the amount of said premium. In which

event the company will, without further request, treat the premium then due as paid, and the amount of such premium, with interest from its actual due date at six per cent per annum, compounded

NATURE yearly, and one per cent, compounded yearly, for expenses, shall be a first lien on this Policy in the Company's favour in priority to the claim of any assignee or any other person. The accumulated lien

may at any time, while the Policy is in force, be paid in whole or in part.
APPEAL from a judgment of the Court of First Instance of Manila 'When the premium falls due and is not paid in cash within the month's grace, if the Cash Value of this policy and of any bonus additions and dividends left on accumulation (after deducting any

accumulated indebtedness) be less than the premium then due, the Company will, without further requests, continue this insurance in force for a period * * *.

FACTS '10. Cash and Paid-Up Insurance Values.-At the end of the third policy year or thereafter, upon the legal surrender of this Policy to the Company while there is no default in premium payments or within

two months after the due date of the premium in default, the Company will (1) grant a cash value as specified in Column (A) increased by the cash value of any bonus additions and dividends left on
(this is a tax case. What’s really important here is the definition of CASH SURRENDER accumulation, which have been alloted to this Policy, less all indebtedness to the Company on this Polley an the date of ouch surrender, or (2) endorse this Policy as a Non-Participating Paid-up Polley

VALUE). for the amount as specified In Column (B) of the Table of Guaranteed Values * * *.

- Manufacturers Life Insurance Company is a duly organized corporation which has its '11. Extended Insurance-After the premiums for three or more full years have been paid hereunder in cash, if any subsequent premium is not paid when due, and there is no indebtedness to the

Company on the written request of the insured * * *."


head office at Toronto. It is duly registered and licensed to engage in life insurance
6"SEC. 255. Taxes on insurance premiums.-There shall be collected from every person, company, or corporation (except purely cooperative companies or associations) doing insurance business of any
business in the Philippines, and, maintains a branch office in Manila. It was engaged in
such business in the Philippines for more than five years before and including the year sort in the Philippines a tax of one per centum of the total premiums collected * * * whether such permiums are paid in money, notes, credits, or any substitute for money but premiums refunded within

six months after payment on account of rejection of risk or returned for other reason to person insured shall not be included in the taxable receipts * * *."
iNsuRanCe A2010 Dean Carale
pAgE 40♥

section 255 of the National Internal Revenue Code. Suppose also that the cash value Disposition finding no prejudicial error in the appealed decision, we hereby affirm it
of said policy after the payment of the 10th annual premium amounts to P1,000." with costs.
When on the eleventh year the annual premium fell due and the insured remitted no
money within the mouth grace, the insurer treated the premium then over due as
paid from the cash value, the amount being a loan to the policyholder1 who
could discharge it at any time with interest at 6 per cent. The insurance contract,
therefore, continued in force for the eleventh year.
1. YES
- Based on the example given by the plaintiff, the insurer collected the amount of
P250 as the annual premium for the eleventh year on the said policy when it loaned to
“A” the sum of P250. The insurer “became a creditor” of the loan, but not of the ANDRES v. CROWN LIFE INSURANCE
premium that had already been paid (advanced by the insurer). The insurer is
102 Phil. 919
entitled to collect interest on the loan, not on the premium. "A" paid the premium for
the eleventh year; but in turn he became a debtor of the company for the sum of REYES, J.B.L., Jan.28, 1958
P250. This debt he could repay either by later remitting the money to the insurer or by
letting the cash value compensate for it. The debt may also be deducted from the NATURE
amount of the policy should "A" die thereafter during the continuance of the policy. Appeal from judgment of CFI
- ON ARGUMENT THAT THE ASSETS OF THE INSURER REMAINED THE SAME AFTER THE APPLICATION OF THE AUTOMATIC
PREMIUM LOAN CLAUSE: there was an increase in assets in the form of CREDIT for the FACTS
advances made (in the example, the P250 for the 11th year). - Feb. 13, 1950: For the sum of P5,000, defendant-appellee Crown Life issued an
- ON ARGUMENT THAT IF THE CREDIT IS PAID OUT OF THE CASH SURRENDER VALUE, THERE WERE NO NEW FUNDS ADDED insurance policy in the name of plaintiff-appellant Rufino and his wife, with the
TO THE COMPANY'S ASSETS”: Cash surrender value "as applied to a life insurance policy, is stipulation that the premiums are to be paid semi-annually.
the amount of money the company agrees to pay to the holder of the policy if he - The premiums for the 1st and 2nd semester of the 1st year, in the amount of P165.15
surrenders it and releases his claims upon it. The more premiums the insured has paid were paid by Rufino but the premium for the third semester, in the same amount,
the greater will be the surrender value; but the surrender value is always a lesser sum was not paid.
than the total amount of premiums paid." (Cyclopedia Law Dictionary 3d. ed. 1077.) - Jan. 6, 1951, Crown Life, through its branch secretary, wrote to Mr. and Mrs. Andres
The cash value or cash surrender value is therefore an amount which the insurance advising them that their insurance policy lapsed on Dec. 26, 1950 and the amount of
company holds In trust for the insured to be delivered to him upon demand. It is P165.15 was overdue, giving them 60 days from the date of lapse to file an
therefore a liability of the company to the insured. Now then, when the company's application for reinstatement. Crown Life later sent another letter telling the spouses
credit for advances is paid out of the cash value or cash surrender value, that value Andres that their insurance policy was no longer in force.
and the company's liability is thereby diminished pro tanto. - Feb. 1951: Plaintiff and his wife executed a Statement of Health and application for
2. YES reinstatement of the aforesaid policy.
- the insurer agreed to consider the premium paid on the strength of the automatic - Feb. 20, 1951: Plaintiff wrote a letter to the defendant, enclosed with a money order
loan. The premium was therefore paid by means of a "note" or "credit" or "other for P100. Upon acceptance, defendant advised Rufino that its main office had
substitute for money" and the tax is due because section 255 above quoted levies approved the application and that the reinstatement of the lapsed policy was subject
taxes according to the total premiums collected by the insurer "whether such to the payment of the remaining premium balance of P65.15.
premiums are paid in money, notes, credits or any substitute for money. - May 3, 1951: Severa Andres died of dystocia, contracted pelvis.
3. NO - May 5, 1951: Plaintiff sent a letter enclosed with a money order in the amount of
- No constitutional prohibition against double taxation. P65, for the remaining balance due.
4. NO - May 15, 1951: Defendant sent a letter with official receipt of the P165.15 paid by
- The loans are made to policyholders in the Philippines, who in turn pay therewith the Rufino as well as a Certificate of Reinstatement.
premium to the insurer thru the Manila branch. Approval of appellant's position will - June 7, 1951: Rufino presented a death claim as survivor-beneficiary of his deceased
enable foreign insurers to evade the tax by contriving to require that premium wife. Payment was denied by the defendant.
payments shall be made at their head offices. What is important, the law does not - April 1952: Rufino filed a complaint in CFI against Crown Life for the recovery of the
contemplate premiums collected in the Philippines. It is enough that the insurer is amount of P5,000 as the face value of a joint 20-year endowment insurance policy
doing insurance business in the Philippines, irrespective of the place of its issued by defendant in favor of plaintiff and his wife, on Feb. 13, 1950. In its answer,
organization or establishment. Crown Life disclaimed liability and set forth the special defense that the
5. NO aforementioned policy had already lapsed.
- Although during those years the appellant was not open for new business because its - Aug. 5, 1954: CFI rendered a decision absolving the defendant company from any
branch office was closed, still it was practically and legally, operating in this country liability on the ground that the policy had lapsed and it was not reinstated at the time
by collecting premiums on its outstanding policies, incurring the risks and/or enjoying of the plaintiff’s wife’s death. Plaintiff later appealed to the CA but the same was
the benefits consequent thereto, without having previously taken any steps indicating certified by the CA to the SC for having no question of fact.
withdrawal in good faith from this field of economic activity.
iNsuRanCe A2010 Dean Carale
pAgE 41♥

ISSUE 191 SCRA 1


WON the insurance policy, which has been in a state of lapse before May 3, 1951, has
GUTIERREZ; October 19, 1990
been validly and completely reinstated after said date (Was there a perfected
contract of reinstatement after the policy lapsed due to non-payment of premiums?)
NATURE
Petition for review of the decision of theca.
HELD
NO
FACTS
Ratio The stipulation in a life insurance policy giving the insured the privilege to
- Petitioner Arturo P. Valenzuela is a General Agent of private respondent Philippine
reinstate it upon written application does not give the insured absolute right to such
American General Insurance Company, Inc. (Philamgen for short) since 1965. As such,
reinstatement by the mere filing of an application. The Company has the right to deny
he was authorized to solicit and sell in behalf of Philamgen all kinds of non-life
the reinstatement if it is not satisfied as to the insurability of the insured and if the
insurance, and in consideration of services rendered was entitled to receive the full
latter does not pay all overdue premiums and all other indebtedness to the Company.
agent's commission of 32.5% from Philamgen under the scheduled commission rates.
After the death of the insured the insurance Company cannot be compelled to
- From 1973 to 1975, Valenzuela solicited marine insurance from one of his clients,
entertain an application for reinstatement of the policy because the conditions
the Delta Motors, Inc. (Division of Electronics Airconditioning and Refrigeration) in the
precedent to reinstatement can no longer be determined and satisfied.
amount of P4.4 Million from which he was entitled to a commission of 32%. However,
Reasoning
Valenzuela did not receive his full commission which amounted to P1.6 Million from
- The stipulations of facts render it undisputable that the original policy lapsed for
the P4.4 Million insurance coverage of the Delta Motors. During the period 1976 to
non-payment of premiums on Dec. 26, 1950, upon expiration of the 31-day grace
1978, premium payments amounting to P1,946,886.00 were paid directly to
period.
Philamgen and Valenzuela's commission to which he is entitled amounted to
- As found by the lower court, the conditions set forth in the policy for reinstatement
P632,737.00.
as provided in the contract itself are the following: (A) application shall be made
- In 1977, Philamgen started to become interested in and expressed its intent to share
within 3 years from the date of lapse; (B) there should be a production of evidence of
in the commission due Valenzuela on a fifty-fifty basis. Valenzuela refused.
the good health of the insured; (C) if the rate of premium depends upon the age of the
- Because of the refusal of Valenzuela, Philamgen and its officers took drastic action
Beneficiary, there should likewise be a production of evidence of his or her good
against Valenzuela. They: (a) reversed the commission due him by not crediting in his
health; (D) there should be presented such other evidence of insurability at the date
account the commission earned from the Delta Motors, Inc. insurance ; (b) placed
of application for reinstatement; (E) there should be no change which has taken place
agency transactions on a cash-and-carry basis; (c) threatened the cancellation of
in such good health and insurability subsequent to the date of such application and
policies issued by his agency; and (d) started to leak out news that Valenzuela has a
before the policy is reinstated; and (F) all overdue premiums and other indebtedness
substantial account with Philamgen. All of these acts resulted in the decline of his
in respect of the policy, together with interest at 6%, compounded annually, should
business as insurance agent.
first be paid.
- Then on December 27, 1978, Philamgen terminated the General Agency Agreement
- The plaintiff did not comply with the last condition; for he only paid P100 before his
of Valenzuela.
wife’s death; and despite the Company’s reminders, he only remitted the balance of
- Lower court: the termination of Valenzuela as General Agent was improper because
P65.15 two days after his wife died. On the face of such facts, the Company had the
the record will show the principal cause of the termination of the plaintiff as General
right to treat the contract as lapsed and refuse payment of the policy.
Agent of defendant Philamgen was his refusal to share his Delta commission.
- Rufino contends that the condition regarding payment of the premium was waived
- CA: In any event the principal's power to revoke an agency at will is so pervasive,
by the insurance Company through its letters, wherein it made statements such as: “If
that the Supreme Court has consistently held that termination may be effected even if
you are unable to pay the full amount immediately, send as large amount as possible
the principal acts in bad faith, subject only to the principal's liability for damages. (CA
and advise us how soon you expect to be able to pay the balance; we will work out an
ordered Valenzuela to pay Philamgen the amount of One Million Nine Hundred Thirty-
adjustment most beneficial to you.” The Court found the statements to be too vague
Two Thousand Five Hundred Thirty-Two and 17/100 Pesos (P1,932,532.17) with legal
and indefinite to indicate an intention on the insurer’s part to waive the full payment
interest)
as prerequisite to the reinstatement of the lapsed policy. The Court reiterated the rule
that a waiver must be clear and positive, the intent to waive shown clearly and
ISSUES
convincingly. On the other hand, It found subsequent letters sent by defendant
1. WON whether or not Philamgen and/or its officers can be held liable for damages
indicating that they insisted on full payment of the premium before the policy was
due to the termination of the General Agency Agreement it entered into with the
reinstated and that defendant did not consider partial payment as sufficient
petitioners
consideration for the reinstatement. Plaintiff-Appellant’s failure to remit the balance
2. WON petitioners are liable to Philamgen for the unpaid and uncollected premiums
before the death of his wife operated to deprive him of any right to waive the policy
and recover the face value thereof.
HELD
Disposition Judgment appealed from is affirmed.
1. YES
- If a principal acts in bad faith and with abuse of right in terminating the agency, then
VALENZUELA v. CA (PHILIPPINE AMERICAN GENERAL he is liable in damages.
INSURANCE COMPANY, INC.)
iNsuRanCe A2010 Dean Carale
pAgE 42♥

- There is an exception to the principle that an agency is revocable at will and that is
when the agency has been given not only for the interest of the principal but for the
CHAPTER V – THE POLICY, PARTIES THERETO, &
interest of third persons or for the mutual interest of the principal and the agent. In RIGHTS THEREON
these cases, it is evident that the agency ceases to be freely revocable by the sole will
of the principal
DE LIM v. SUN LIFE ASSURANCE COMPANY OF CANADA
(PROCEDURAL: Where the findings of the Court of Appeals and the trial court are
contrary to each other, this Court may scrutinize the evidence on record 41 PHIL 263
- After a painstaking review of the entire records of the case and the findings of facts MALCOLM; November 29, 1920
of both the court a quo and respondent appellate court, the Court affirmed the trial
court’s findings.) NATURE
- The principal cause of the termination of Valenzuela as General Agent of Philamgen Appeal from an order of the CFI of Zamboanga sustaining a demurrer to plaintiff's
arose from his refusal to share his Delta commission. The records sustain the complaint upon the ground that it fails to state a cause of action.
conclusions of the trial court on the apparent bad faith of the private respondents in
terminating the General Agency Agreement of petitioners. FACTS
- It is also evident from the records that the agency involving petitioner and private - On July 6, 1917, Luis Lim of Zamboanga made application to the Sun Life Assurance
respondent is one "coupled with an interest," and, therefore, should not be freely Company of Canada for a policy of insurance on his life in the sum of P5,000. In his
revocable at the unilateral will of the latter. application Lim designated his wife, Pilar de Lim, the plaintiff herein, as the
- The private respondents by the simple expedient of terminating the General Agency beneficiary. The first premium of P433 was paid by Lim, and upon such payment the
Agreement appropriated the entire insurance business of Valenzuela. With the company issued what was called a ''provisional policy." Luis Lim died on August 23,
termination of the General Agency Agreement, Valenzuela would no longer be entitled 1917, after the issuance of the provisional policy but before approval of the
to commission on the renewal of insurance policies of clients sourced from his agency. application by the home office of the insurance company. Pilar de Lim brought an
Worse, despite the termination of the agency, Philamgen continued to hold Valenzuela action to recover from the Sun Life sum of P5,000, the amount named in the
jointly and severally liable with the insured for unpaid premiums. Under these provisional policy.
circumstances, it is clear that Valenzuela had an interest in the continuation of the - The "provisional policy" reads: "Received (subject to the following stipulations and
agency when it was unceremoniously terminated not only because of the commissions agreements) the sum of P433, being the amount of the first year's premium for a Life
he should continue to receive from the insurance business he has solicited and Assurance Policy on the life of Mr. Luis D. Lim of Zamboanga for P5,000, for which an
procured but also for the fact that by the very acts of the respondents, he was made application dated the 6th day of July, 1917, has been made to the Sun Life Assurance
liable to Philamgen in the event the insured fail to pay the premiums due. They are Company of Canada.
estopped by their own positive averments and claims for damages. - The above-mentioned life is to be assured in accordance with the terms and
- "The principal may not defeat the agent's right to indemnification by a termination of conditions contained or inserted by the Company in the policy which may be granted
the contract of agency (Erskine v. Chevrolet Motors Co. 185 NC 479, 117 SE 706, 32 by it in this particular case for four months only from the date of the application,
ALR 196). provided that the Company shall confirm this agreement by issuing a policy on said
- For the pivotal factor rendering Philamgen and the other private respondents liable application when the same shall be submitted to the Head Office in Montreal. Should
in damages is that the termination by them of the General Agency Agreement the Company not issue such a policy, then this agreement shall be null and void ab
was tainted with bad faith. This is in accordance with the precepts in Human initio, and the Company shall be held not to have been on the risk at all, but in such
Relations enshrined in our Civil Code. case the amount herein acknowledged shall be returned.
2. NO. The respondent court erred in holding Valenzuela liable. There was no factual
and legal basis for the award. Under Section 77 of the Insurance Code, the ISSUE
remedy for the non-payment of premiums is to put an end to and render the insurance WON the contract of insurance between Luis Lim and Sun Life Assurance Company of
policy not binding - "Sec. 77 . . . [N]otwithstanding any agreement to the contrary, no Canada was perfected
policy or contract of insurance is valid and binding unless and until the premiums
thereof have been paid except in the case of a life or industrial life policy whenever HELD
the grace period provision applies (P.D. 612, as amended otherwise known as the NO.
Insurance Code of 1974) - The document it is to be a provisional policy "for four months only from the date of
- This is buttressed by Section 776 of the Insurance Code (Presidential Decree No. this application." Immediately following the words fixing the four months period comes
612, promulgated on December 18, 1974), which now provides that no contract of the word "provided" which has the meaning of "if." Otherwise stated, the policy for
Insurance by an insurance company is valid and binding unless and until the premium four months is expressly made subject to the affirmative condition that the company
thereof has been paid, notwithstanding any agreement to the contrary." shall confirm this agreement by issuing a policy on said application when the same
Disposition Petition is GRANTED. CA decision SET ASIDE. The decision of the TC shall be submitted to the head office in Montreal. To re-enforce the same there follows
REINSTATED with the MODIFICATIONS. And that the contractual relationship between the negative condition - "Should the company not issue such a policy, then this
Arturo P. Valenzuela and Philippine American General Insurance Company shall be agreement shall be null and void ab initio, and the company shall be held not to have
deemed terminated upon the satisfaction of the judgment as modified. been on the risks." Certainly language could hardly be used which would more clearly
stipulate that the agreement should not go into effect until the home office of the
iNsuRanCe A2010 Dean Carale
pAgE 43♥

company should confirm it by issuing a policy. As we read and understand the so-
called provisional policy, it amounts to nothing but an acknowledgment on behalf of
the company, that it has received from the person named therein the sum of money ISSUES
agreed upon as the first year's premium upon a policy to be issued upon the 1. WON CA erred in holding petitioner liable to DBP as beneficiary in a group life
application, if the application is accepted by the company. insurance contract from a complaint filed by the widow of the decedent/mortgagor
- It is of course a primary rule that a contract of insurance, like other contracts, must 2. WON CA erred in not finding that Dr. Leuterio concealed that he had hypertension,
be assented to by both parties either in person or by their agents. So long as an which would vitiate the insurance contract
application for insurance has not been either accepted or rejected, it is merely an 3. WON CA erred in holding Grepalife liable for P86,200.00 without proof of the actual
offer or proposal to make a contract. The contract, to be binding from the date of the outstanding mortgage payable by the mortgagor to DBP
application must have been a completed contract, one that leaves nothing to be done,
nothing to be completed, nothing to be passed upon, or determined, before it shall HELD
take effect. There can be no contract of insurance unless the minds of the parties 1. NO
have met in agreement. Our view is, that a contract of insurance was not here Ratio Insured, being the person with whom the contract was made, is primarily the
consummated by the parties. proper person to bring suit. Subject to some exceptions, insured may thus sue,
- The trial court committed no error in sustaining the demurrer and dismissing the although the policy is taken wholly or in part for the benefit of another person named
case. It is to be noted, however that counsel for appellee admits the liability of the or unnamed, and although it is expressly made payable to another as his interest may
company for the return of the first premium to the estate of the deceased. appear or otherwise. Although a policy issued to a mortgagor is taken out for the
benefit of the mortgagee and is made payable to him, yet the mortgagor may sue
GREAT PACIFIC LIFE v. CA (LEUTERIO) thereon in his own name, especially where the mortgagee's interest is less than the
full amount recoverable under the policy. (See Sec. 8, Insurance Code)
316 SCRA 677
Reasoning
QUISUMBING; October 13, 1999 [a] The insured private respondent did not cede to the mortgagee all his rights or
interests in the insurance, the policy stating that: “In the event of the debtor's death
NATURE before his indebtedness with the Creditor (DBP) shall have been fully paid, an amount
Petition for Review of CA decision to pay the outstanding indebtedness shall first be paid to the creditor and the balance
of sum assured, if there is any, shall then be paid to the beneficiary/ies designated by
FACTS the debtor.” When DBP submitted the insurance claim against Grepalife, the latter
- A contract of group life insurance was executed between petitioner Great Pacific Life denied payment thereof, interposing the defense of concealment committed by the
Assurance Corporation (hereinafter Grepalife) and Development Bank of the insured. Thereafter, DBP collected the debt from the mortgagor and took the
Philippines (hereinafter DBP). Grepalife agreed to insure the lives of eligible housing necessary action of foreclosure on the residential lot of private respondent.
loan mortgagors of DBP. [b] Since a policy of insurance upon life or health may pass by transfer, will or
- In Nov. 1983, Dr. Wilfredo Leuterio, a physician and a housing debtor of DBP applied succession to any person, whether he has an insurable interest or not, and such
for membership in the group life insurance plan. In an application form, Dr. Leuterio person may recover it whatever the insured might have recovered, the widow of the
answered Qs concerning his health condition as follows: decedent Dr. Leuterio may file the suit against the insurer, Grepalife.
Q: Have you ever had, or consulted, a physician for a heart condition, high blood 2. NO
pressure, cancer, diabetes, lung, kidney or stomach disorder or any other physical Ratio The fraudulent intent on the part of the insured must be established to entitle
impairment? No. the insurer to rescind the contract. Misrepresentation as a defense of the insurer to
Q: Are you now, to the best of your knowledge, in good health? Yes. avoid liability is an affirmative defense and the duty to establish such defense by
- Grepalife issued an insurance coverage of Dr. Leuterio, to the extent of his DBP satisfactory and convincing evidence rests upon the insurer. In the case at bar, the
mortgage indebtedness of P86,200.00. In Aug. 1984, Dr. Leuterio died due to petitioner failed to clearly and satisfactorily establish its defense, and is therefore
"massive cerebral hemorrhage." DBP submitted a death claim to Grepalife. Grepalife liable to pay the proceeds of the insurance.
denied the claim because Dr. Leuterio was not physically healthy when he applied for Reasoning
an insurance. Grepalife insisted that Dr. Leuterio did not disclose he had been [a] The insured, Dr. Leuterio, had answered in his insurance application that he was in
suffering from hypertension, which caused his death. Allegedly, such non-disclosure good health and that he had not consulted a doctor or any of the enumerated
constituted concealment that justified the denial of the claim. ailments, including hypertension; when he died the attending physician had certified
- Herein respondent Medarda Leuterio, widow, filed a complaint with RTC against in the death certificate that the former died of cerebral hemorrhage, probably
Grepalife for "Specific Performance with Damages." Dr. Mejia, who issued the death secondary to hypertension. From this report, petitioner Grepalife refused to pay the
certificate, testified that Dr. Leuterio complained of headaches presumably due to insurance claim. It alleged that the insured had concealed the fact that he had
high blood pressure. The inference was not conclusive because Dr. Leuterio was not hypertension.
autopsied, hence, other causes were not ruled out. [b] Contrary to Grepalife’s allegations, there was no sufficient proof that the insured
- RTC ruled in favor of respondent widow and against Grepalife. CA sustained the RTC had suffered from hypertension. Aside from the statement of the insured's widow who
decision. Hence, the present petition. was not even sure if the medicines taken by Dr. Leuterio were for hypertension, the
iNsuRanCe A2010 Dean Carale
pAgE 44♥

appellant had not proven nor produced any witness who could attest to Dr. Leuterio's - On January 13, 1964 - the defendant wrote the plaintiff denying the latter's claim, on
medical history. the ground that defendant's investigation revealed that the entire shipment of logs
[c] Grepalife had failed to establish that there was concealment made by the insured, covered by the two marines policies were received in good order at their point of
hence, it cannot refuse payment of the claim. destination. It was further stated that the said loss may not be considered as covered
3. NO under cover note because the said note had become 'null and void by virtue of the
- Considering the supervening event that DBP foreclosed in 1995 their residential lot, issuance of two marine policies.
in satisfaction of mortgagor's outstanding loan, the insurance proceeds shall inure to - The CFI of Manila ruled in favour of the petitioner.
the benefit of the heirs of the deceased person or his beneficiaries. Equity dictates - The Court of Appeals reversed the decision of the CFI.
that DBP should not unjustly enrich itself at the expense of another. Hence, it cannot
collect the insurance proceeds, after it already foreclosed on the mortgage. The ISSUES
proceeds now rightly belong to Dr. Leuterio's heirs represented by his widow, herein 1. WON the cover note is null and void for lack of valuable consideration because no
private respondent. separate premiums are collected by private respondent on all its cover notes
- The Court ruled this issue based on the clear provisions of the policy. The mortgagor 2. WON the court of appeals erred in holding that private respondent was released
paid the premium according to the coverage of his insurance, which states that: "The from liability under the cover note due to unreasonable delay in giving notice of loss
policy states that upon receipt of due proof of the Debtor's death during the terms of because the court disregarded the proven fact that private respondent did not
this insurance, a death benefit in the amount of P86,200.00 shall be paid… In the promptly and specifically object to the claim on the ground of delay in giving notice of
event of the debtor's death before his indebtedness with the creditor shall have been loss and, consequently, objections on that ground are waived under section 84 of the
fully paid, an amount to pay the outstanding indebtedness shall first be paid to the insurance act
Creditor and the balance of the Sum Assured, if there is any shall then be paid to the
beneficiary/ies designated by the debtor." From this, it is clear that Grepalife is liable HELD
and that Dr. Leuterio’s heirs must get the proceeds. 1. NO
Disposition Petition DENIED. CA Decision AFFIRMED with modification. Ratio Cover note is issued with a consideration when, by express stipulation, the
cover note is made subject to the terms and conditions of the marine policies, and the
PACIFIC TIMBER EXPORT CORPORATION v. CA (WORKMEN’S payment of premiums is one of the terms of the policies.
Reasoning
INSURANCE CO)
a. the cover note in question is subject to the terms and conditions of the marine
112 SCRA 199 policies
DE CASTRO; February 25, 1982 b. Nature of the Cover Note: The fact that no separate premium was paid on the
Cover Note before the loss insured against occurred, does not militate against the
FACTS validity of petitioner's contention, for no such premium could have been paid, since by
- March 19, 1963 - the plaintiff secured temporary insurance from the defendant for the nature of the Cover Note, it did not contain, as all Cover Notes do not contain
its exportation of 1,250,000 board feet of Philippine Lauan and Apitong logs to be particulars of the shipment that would serve as basis for the computation of the
shipped from the Diapitan Bay, Quezon to Okinawa and Tokyo, Japan. The defendant premiums. As a logical consequence, no separate premiums are intended or required
issued on said date Cover Note No. 1010, insuring the said cargo of the plaintiff to be paid on a Cover Note.
"Subject to the Terms and Conditions of the WORKMEN'S INSURANCE COMPANY, INC. c. The petitioner paid in full all the premiums as called for by the statement issued by
printed Marine Policy form as filed with and approved by the Office of the Insurance private respondent after the issuance of the two regular marine insurance policies,
Commissioner. thereby leaving no account unpaid by petitioner due on the insurance coverage, which
- April 2, 1963 - The two (2) regular marine cargo policies were issued by the must be deemed to include the Cover Note. If the Note is to be treated as a separate
defendant in favor of the plaintiff. The total cargo insured under the two marine policy instead of integrating it to the regular policies subsequently issued, the purpose
policies accordingly consisted of 1,395 logs, or the equivalent of 1,195,498 bd. ft. and function of the Cover Note would be set at naught or rendered meaningless, for it
- After the issuance of cover note but before the issuance of the two marine policies is in a real sense a contract, not a mere application for insurance which is a mere
some of the logs intended to be exported were lost during loading operations in the offer. Had all the logs been lost during the loading operations, but after the issuance
Diapitan Bay due to bad weather. of the Cover Note, liability on the note would have already arisen even before
- April 4, 1963 - The plaintiff informed the defendant about the loss of 'approximately payment of premium. This is how the cover note as a "binder" should legally operate;
32 pieces of logs' during loading through a letter. otherwise, it would serve no practical purpose in the realm of commerce, and is
- The plaintiff subsequently submitted a 'Claim Statement' demanding payment of the supported by the doctrine that where a policy is delivered without requiring payment
loss under the second marine cargo policy. of the premium, the presumption is that a credit was intended and policy is valid.
- July 17, 1963 - the defendant requested the First Philippine Adjustment Corporation 2. NO
to inspect the loss and assess the damage. - The private respondent company never raised this ground in the proceedings. It
- August 23, 1963 - the adjuster reported that 'the loss of 30 pieces of logs is not must be because it did not find any delay, as this Court fails to find a real and
covered by the two policies inasmuch as said policies covered the actual number of substantial sign thereof. But even on the assumption that there was delay, this Court
logs loaded on board. But it is covered by Cover Note. is satisfied and convinced that as expressly provided by law, waiver can successfully
be raised against private respondent. Thus Section 84 of the Insurance Act provides:
iNsuRanCe A2010 Dean Carale
pAgE 45♥

"Section 84. - Delay in the presentation to an insurer of notice or proof of loss is - However, there is no evidence on record that the building was worth P5,800,000.00
waived if caused by any act of his or if he omits to take objection promptly and at the time of the loss. On the contrary, the building was insured at P2,500,000.00,
specifically upon that ground." and this must be considered, by agreement, the actual value of the property insured
- From what has been said, We find duly substantiated petitioner's assignments of on the day the fire occurred. This valuation becomes even more believable if it is
error. remembered that at the time the building was burned it was still under construction
Disposition The appealed decision is set aside and the decision of the Court of First and not yet completed.
Instance is reinstated in toto with the affirmance of this Court. - The Court notes that the policy in this case is an open policy and is subject to the
express condition that:
DEVELOPMENT INSURANCE v. IAC (PHIL UNION REALTY "Open Policy.
This is an open policy as defined in Sec57 of the Insurance Act. In the event of loss,
DEVELOPMENT CORP)
whether total or partial, it is understood that the amount of the loss shall be subject
143 SCRA 62 to appraisal and the liability of the company, if established, shall be limited to the
CRUZ; July 16, 1986 actual loss, subject to the applicable terms, conditions, warranties and clauses of
this Policy, and in no case shall exceed the amount of the policy."
FACTS - As defined in the aforestated provision, which is now Sec60 of the Insurance Code,
- A fire occurred in the building of the private respondent and it sued for recovery of "an open policy is one in which the value of the thing insured is not agreed upon but is
damages from the petitioner on the basis of an insurance contract between them. The left to be ascertained in case of loss.".
petitioner allegedly failed to answer on time and was declared in default by TC. A - The actual loss has been ascertained in this case and the Court will respect such
judgment of default was rendered on the strength of the evidence submitted ex parte factual determination in the absence of proof that it was arrived at arbitrarily. There is
by the private respondent, which was allowed full recovery of its claimed damages. no such showing. Hence, applying the open policy clause as expressly agreed upon by
- On learning of this decision, the petitioner moved to lift the order of default, invoking the parties in their contract, we hold that the private respondent is entitled to the
excusable neglect, and to vacate the judgment by default. Its motion was denied. payment of indemnity under the said contract in the total amount of P508,867.00.
- On appeal, IAC affirmed the TC decision in toto. - The refusal of its vice-president to receive the private respondent's complaint, as
reported in the sheriff's return, was the first indication of the petitioner's intention to
ISSUE prolong this case and postpone the discharge of its obligation to the private
1. WON default of petitioner is based on excusable neglect respondent under this agreement. That intention was revealed further in its
2. What is the amount of indemnity due to the private respondent under its insurance subsequent acts ---- or inaction ---- which indeed enabled it to avoid payment for more
contract? WON CFI was correct in interpreting the contract than five years from the filing of the claim against it in 1980.
Disposition The appealed decision is affirmed in full, with costs against the
HELD petitioner.
1. NO
- Summons was served through its vice-president. There were even several extensions HARDING v. COMMERCIAL UNION ASSURANCE
to the original period to answer. As a consequence, the TC, on motion of the private
38 PHIL 464
respondent filed declared the petitioner in default. This was done almost one month
later. Even so, the petitioner made no move at all for two months thereafter. It was FISHER; August 10, 1918
only more than one month after the judgment of default was rendered by the TC that
it filed a motion to lift the order of default and vacate the judgment by default. FACTS
- There is a pattern of inexcusable neglect. - Mrs. Harding was the owner of a Studebaker automobile; in consideration of the
2. The policy is an open policy which means that the actual loss, as determined, will payment to the defendant of the premium of P150, by said plaintiff, Mrs. Henry E.
represent the total indemnity due the insured from the insurer except only that the Harding, with the consent of her husband, the defendant by its duly authorized agent,
total indemnity shall not exceed the face value of the policy. Smith, Bell & Company (limited), made its policy of insurance in writing upon said
- The petitioner argues that since at the time of the fire the building insured was worth automobile was set forth in said policy to be P3,000 that the value of said automobile
P5,800,000.00, the private respondent should be considered its own insurer for the was set forth in said policy to be P3,000; that on March 24, 1916, said automobile was
difference between that amount and the face value of the policy and should share pro totally destroyed by fire; that the loss thereby to plaintiffs was the sum of P3,000.
rata in the loss sustained. Accordingly, the private respondent is entitled to an - The defendant’s version is that by request of Mrs. Harding, it issued the policy of
indemnity of only P67,629.31, the rest of the loss to be shouldered by it alone. The insurance on an automobile alleged by the said plaintiff to be her property. It was
petitioner cites Condition 17 of the policy, which provides: made by means of a proposal in writing signed and delivered by said plaintiff to the
"If the property hereby insured shall, at the breaking out of any fire, be collectively defendant, guaranteeing the truth of the statements contained therein which said
of greater value than the sum insured thereon then the insured shall be considered proposal is referred to in the said policy of insurance made a part thereof; that certain
as being his own insurer for the difference, and shall bear a ratable proportion of of the statements and representations contained in said proposal and warranted by
the loss accordingly. Every item, if more than one, of the policy shall be separately said plaintiff to be true, to wit: (a) the price paid by the proposer for the said
subject to this condition." automobile; (b) the value of said automobile at the time of the execution and delivery
of the said proposal and (c) the ownership of said automobile, were false and known
iNsuRanCe A2010 Dean Carale
pAgE 46♥

to be false by the said plaintiff at the time of signing and delivering the said proposal 1. WON Mrs. Harding was not the owner of the automobile at the time of the issuance
and were made for the purpose of misleading and deceiving the defendant, and of the policy, and, therefore, had no insurable interest in it
inducing the defendant, relying upon the warranties, statements, and representations 2. WON the statement regarding the cost of the automobile was a warranty, that the
contained in the said proposal and believing the same to be true, issued the said statement was false, and that, therefore, the policy never attached to the risk
policy of insurance.
- The evidence shows that Hermanos, the Manila agents for the Studebaker HELD
automobile, sold the automobile to Canson for P3,200 (testimony of Mr. Diehl); who 1. NO
sold the said automobile to Henry Harding for the sum of P1,500. Harding sold the - Article 1334 of the Civil Code which provides that "All gifts between spouses during
said automobile to J. Brannigan for the sum of P2,000 who sold the said automobile the marriage shall be void. Moderate gifts which the spouses bestow on each other on
Henry Harding for the sum of P2,800; Henry Harding gave the said automobile to his festive days of the family are not included in this rule."
wife as a present; that said automobile was repaired and repainted at the Luneta - Even assuming that defendant might have invoked article 1334 as a defense, the
Garage at a cost of some P900; that while the said automobile was at the Luneta burden would be upon it to show that the gift in question does not fall within the
Garage; the latter solicited of Mrs. Harding the insurance of said automobile by the exception therein established. We cannot say, as a matter of law, that the gift of an
Company; that a proposal was filled out by the said agent and signed by the plaintiff automobile by a husband to his wife is not a moderate one. Whether it is or is not
Mrs. Henry E. Harding, and in said proposal under the heading "Price paid by would depend upon the circumstances of the parties, as to which nothing is disclosed
proposer," is the amount of "3,500" and under another heading "Present value" is the by the record.
amount of "3,000". - We are of the opinion that it would be unfair to hold the policy void simply because
- After the said proposal was made a representative of the Manila agent of defendant the outlay represented by the automobile was made by the plaintiff's husband and not
went to the Luneta Garage and examined said automobile and Mr. Server, the General by his wife, to whom he had given the automobile. It cannot be assumed that
Manager of the Luneta Garage, an experienced automobile mechanic, testified that at defendant should not have issued the policy unless it were strictly true that the price
the time this automobile was insured it was worth about P3,000, and the defendant, representing the cost of the machine had been paid by the insured and by no other
by and through its said agent Smith, Bell & Company (limited), thereafter issued a person ? that it would no event insure an automobile acquired by gift, inheritance,
policy of insurance upon proposal in which policy the said automobile was described exchange, or any other title not requiring the owner to make a specific cash outlay for
as of the "present value" of P3,000 and the said defendant charged the said plaintiff its acquisition.
Mrs. Henry E. Harding as premium on said policy the sum of P150, or 5 per cent of the 2. NO
then estimated value of P3,000. - It has not been shown by the evidence that the statement was false; on the contrary
- The "Schedule" in said policy of insurance describes the automobile here in question, we believe that it shows that the automobile had in fact cost more than the amount
and provides in part of follows: mentioned. The court below found, and the evidence shows, that the automobile was
"That during the period above set forth and during any period for which the company bought by plaintiff's husband a few weeks before the issuance of the policy in
may agree to renew this policy the company will subject to the exception and question for the sum of P2,800, and that between that time and the issuance of the
conditions contained herein or endorsed hereon indemnify the insured against loss of policy some P900 was spent upon it in repairs and repainting.
or damage to any motor car described in the schedule hereto (including accessories) - The witness Server, an expert automobile mechanic, testified that the automobile
by whatever cause such loss or damage may be occasioned and will further indemnify was practically as good as new at the time the insurance was effected. The form of
the insured up to the value of the car or P3,000 whichever is the greater against any proposal upon which the policy was issued does not call for a statement regarding the
claim at common law made by any person (not being a person in the said motor car value of the automobile at the time of its acquisition by the applicant for the
nor in the insured's service) for loss of life or for accidental bodily injury or damage to insurance, but merely a statement of its cost. The amount stated was less than the
property caused by the said motor car including law costs payable in connection with actual outlay which the automobile represented to Mr. Harding, including repairs,
such claim when incurred with the consent of the company." when the insurance policy was issued.
- On March 24, 1916, the said automobile was totally destroyed by fire, and that the - The court below found and the evidence shows, without dispute, that the proposal
iron and steel portions of said automobile which did not burn were taken into the upon which the policy in question was issued was made out by defendant's agent by
possession of the defendant by and through its agent Smith, Bell & Company (limited), whom the insurance was solicited, and that appellee simply signed the same. It also
and sold by it for a small sum, which had never been tendered to the plaintiff prior to appears that an examiner employed by the defendant made an inspection of the
the trial of this case, but in open court during the trial the sum of P10 as the proceeds automobile before the acceptance of the risk, and that the sum after this examination.
of such sale was tendered to plaintiff and refused. The trial court found that Mrs. Harding, in fixing the value of the automobile at P3,000,
- Trial judge decided that there was no proof of fraud on the part of plaintiff in her acted upon information given her by her husband and by Mr. Server, the manager of
statement of the value of the automobile, or with respect to its ownership; that she the Luneta Garage. She merely repeated the information which had been given her by
had an insurable interest therein; and that defendant, having agreed to the estimated her husband, and at the same time disclosed to defendant's agent the source of her
value, P3,000, and having insured the automobile for that amount, upon the basis of information. There is no evidence to sustain the contention that this communication
which the premium was paid, is bound by it and must pay the loss in accordance with was made in bad faith. We do not think that the facts stated in the proposal can be
the stipulated insured value. held as a warranty of the insured, even if it should have been shown that they were
incorrect in the absence of proof of willful misstatement. Under such circumstance,
ISSUE the proposal is to be regarded as the act of the insurer and not of the insured.
iNsuRanCe A2010 Dean Carale
pAgE 47♥

Disposition Plaintiff was the owner of the automobile in question and had an to indemnify another against loss, damage or liability arising from an unknown or
insurable interest therein; that there was no fraud on her part in procuring the contingent event.
insurance; that the valuation of the automobile, for the purposes of the insurance, is - In particular, a marine insurance undertakes to indemnify the assured against
binding upon the defendant corporation, and that the judgment of the court below is, marine losses, such as the losses incident to a marine adventure. Section 99 of the
therefore, correct and must be affirmed, with interest, the costs of this appeal to be Insurance Code enumerates the coverage of marine insurance.
paid by the appellant. - Relatedly, a mutual insurance company is a cooperative enterprise where the
members are both the insurer and insured. In it, the members all contribute, by a
WHITE GOLD MARINE SERVICES v. PIONEER INSURANCE system of premiums or assessments, to the creation of a fund from which all losses
and liabilities are paid, and where the profits are divided among themselves, in
464 SCRA 448
proportion to their interest. Additionally, mutual insurance associations, or clubs,
QUISUMBING; July 28, 2005 provide three types of coverage, namely, protection and indemnity, war risks, and
defense costs.
NATURE - A P & I Club is “a form of insurance against third party liability, where the third party
This petition for review assails the Decision of the Court of Appeals, affirming the is anyone other than the P & I Club and the members.” By definition then, Steamship
Decision of the Insurance Commission. Both decisions held that there was no violation Mutual as a P & I Club is a mutual insurance association engaged in the marine
of the Insurance Code and the respondents do not need license as insurer and insurance business.
insurance agent/broker. - The records reveal Steamship Mutual is doing business in the country albeit without
the requisite certificate of authority mandated by Section 187 of the Insurance Code.
FACTS It maintains a resident agent in the Philippines to solicit insurance and to collect
- White Gold procured a protection and indemnity coverage for its vessels from payments in its behalf. We note that Steamship Mutual even renewed its P & I Club
Steamship Mutual through Pioneer Insurance. Subsequently, White Gold was issued a cover until it was cancelled due to non-payment of the calls. Thus, to continue doing
Certificate of Entry and Acceptance. Pioneer also issued receipts evidencing payments business here, Steamship Mutual or through its agent Pioneer, must secure a license
for the coverage. When White Gold failed to fully pay its accounts, Steamship Mutual from the Insurance Commission.
refused to renew the coverage. - Since a contract of insurance involves public interest, regulation by the State is
- Steamship Mutual thereafter filed a case against White Gold for collection of sum of necessary. Thus, no insurer or insurance company is allowed to engage in the
money to recover the latter’s unpaid balance. White Gold on the other hand, filed a insurance business without a license or a certificate of authority from the Insurance
complaint before the Insurance Commission claiming that Steamship Mutual violated Commission.
Sections 186 and 187 of the Insurance Code, while Pioneer violated Sections 299, 2. YES
300 and 301 in relation to Sections 302 and 303, thereof. - SEC. 299 . . .
- The Insurance Commission dismissed the complaint. It said that there was no need - No person shall act as an insurance agent or as an insurance broker in the
for Steamship Mutual to secure a license because it was not engaged in the insurance solicitation or procurement of applications for insurance, or receive for services in
business. It explained that Steamship Mutual was a Protection and Indemnity Club (P obtaining insurance, any commission or other compensation from any insurance
& I Club). Likewise, Pioneer need not obtain another license as insurance agent company doing business in the Philippines or any agent thereof, without first
and/or a broker for Steamship Mutual because Steamship Mutual was not engaged in procuring a license so to act from the Commissioner, which must be renewed annually
the insurance business. Moreover, Pioneer was already licensed, hence, a separate on the first day of January, or within six months thereafter.
license solely as agent/broker of Steamship Mutual was already superfluous. Disposition The petition is PARTIALLY GRANTED. The Decision dated July 30, 2002
- The Court of Appeals affirmed the decision of the Insurance Commissioner. In its of the Court of Appeals affirming the Decision dated May 3, 2000 of the Insurance
decision, the appellate court distinguished between P & I Clubs vis-à-vis conventional Commission is hereby REVERSED AND SET ASIDE. The Steamship Mutual Underwriting
insurance. The appellate court also held that Pioneer merely acted as a collection Association (Bermuda) Ltd., and Pioneer Insurance and Surety Corporation are
agent of Steamship Mutual. ORDERED to obtain licenses and to secure proper authorizations to do business as
insurer and insurance agent, respectively. The petitioner’s prayer for the revocation
ISSUES of Pioneer’s Certificate of Authority and removal of its directors and officers, is
1. WON Steamship Mutual, a P & I Club, is engaged in the insurance business in the DENIED.
Philippines
2. WON Pioneer needs a license as an insurance agent/broker for Steamship Mutual
PANDIMAN v. MARINE MANNING MNGT CORP.
HELD 460 SCRA 418
1. YES GARCIA; June 21, 2005
- The test to determine if a contract is an insurance contract or not, depends on the
nature of the promise, the act required to be performed, and the exact nature of the NATURE
agreement in the light of the occurrence, contingency, or circumstances under which Petition for certiorari to review CA decision
the performance becomes requisite. It is not by what it is called. Basically, an
insurance contract is a contract of indemnity. In it, one undertakes for a consideration FACTS
iNsuRanCe A2010 Dean Carale
pAgE 48♥

- Benito Singhid was hired as chief cook on board the vessel MV Sun Richie Five for a FILIPINAS COMPANIA DE SEGUROS V CHRISTERN, HUENEFELD
term of one year by Fullwin Maritime Limited through its Philippine agent, Marine
AND CO INC
Manning and Management Corporation. While the said vessel was on its way to
Shanghai from Ho Chih Minh City, Benito suffered a heart attack and subsequently 89 PHIL 54
died on June 24, 1997. PARAS; May 25, 1951
- Apparently, the vessel and the crew were insured with Ocean Marine Mutual
Insurance Association Limited (OMMIAL), a Protective and Indemnity Club of which Sun FACTS
Richie Five Bulkers S.A. is a member. Pandiman Philippines, the petitioner, is the local - October 1, 1941 - Christern Huenefeld, & Co., Inc., after payment of corresponding
correspondent of OMMIAL. premium, obtained from the Filipinas Cia. de Seguros a fire policy in the sum of
- Benito’s widow, Rosita, filed a claim for death benefits with Marine which referred P1000,000, covering merchandise contained in No. 711 Roman Street, Binondo
her to Pandiman. After her submission of the required documentation, Pandiman Manila.
recommended payment of the death benefits amounting to $79,000. However, - February 27, 1942 or during the Japanese military occupation - building and insured
payment has not been made. merchandise were burned. In due time the Huenefeld Co submitted to the Filipinas Cia
- Rosita filed a complaint with the Labor Arbiter naming Marine, Pandiman, OMMIAL, its claim under the policy. The salvage goods were sold at public auction and, after
and Fullwin as respondents. The Arbiter ordered all the respondents, except deducting their value, the total loss suffered by the respondent was fixed at P92,650.
Pandiman, to jointly and severally pay the widow the death benefits plus legal fees. - Filipinas Cia refused to pay the claim on the ground that the policy in favor of the
The NLRC, on appeal by Marine, limited the liable parties to Pandiman and OMMIAL respondent had ceased to be in force on the date the United States declared war
but maintained the money award. The CA sustained the decision of the NLRC. Hence against Germany, the respondent Corporation (though organized under and by virtue
this appeal. of the laws of the Philippines) being controlled by the German subjects and the
Filipinas Cia being a company under American jurisdiction when said policy was issued
on October 1, 1941. Filipinas Cia, however, in pursuance of the order of the Director of
ISSUE Bureau of Financing, Philippine Executive Commission, dated April 9, 1943, paid to the
1. WON Pandiman may be held liable for the death benefits Huenefeld Co the sum of P92,650 on April 19, 1943.
2. WON Marine and its foreign principal, Fullwin, should be absolved from the death - August 6, 1946 – action filed in CFI Manila to recover from the Huenefeld Co the sum
claim liabilities of P92,650 above mentioned. The theory of the Filipinas Cia is that the insured
merchandise were burned up after the policy issued in 1941 in favor of Huenefeld Co
HELD has ceased to be effective because of the outbreak of the war between the United
1. NO States and Germany on December 10, 1941, and that the payment made by the
- Pandiman is not an insurance agent as defined by Section 300 7 of the Insurance Filipinas Cia to Huenefeld Co during the Japanese military occupation was under
Code. In this case, there was no showing that Pndiman in fact negotiated the pressure.
insurance contract between Sun Richie Five and the insurer OMMIAL. Even, if - CFI: dismissed the action without pronouncement as to costs.
Pandiman were an agent, payment for claims arising from peril insured against, to - CA: CFI judgment affirmed, with costs. The case is now before us on appeal by
which the insurer is liable, is definitely not one of the liabilities of an insurance agent. certiorari from the decision of the Court of Appeals.
Thus, there is no legal basis whatsoever for holding petitioner solidarily liable with
insurer OMMIAL for the widow’s claim for death benefits. Also, Pandiman is not a party ISSUE
to the insurance contract and hence under Article 1311 of the Civil Code, it is not WON the policy in question became null and void upon the declaration of war between
liable for the obligation arising out of the insurance contract. United States and Germany
2. NO
- Fullwin, as Benito’s principal employer is liable under the employment contract. HELD
Marine is also bound by its undertaking pursuant to the Rules and Regulations YES
Governing Overseas Employment that “it shall assume joint and solidary liability with Ratio The Philippine Insurance Law (Act No. 2427, as amended,) in section 8,
the employer for all the claims and liabilities which may arise in connection with the provides that "anyone except a public enemy may be insured." It stands to reason
implementation of the contract, including but not limited to the payment of wages, that an insurance policy ceases to be allowable as soon as an insured becomes a
heath and disability compensation and repatriation”. In other words, both Fullwin and public enemy.
Marine should be held liable for whatever death benefits the widow of Benito may be > Effect of war, generally. - All intercourse between citizens of belligerent powers
entitled to. which is inconsistent with a state of war is prohibited by the law of nations. Such
Disposition The petition is granted and the CA decision is reversed and set aside. prohibition includes all negotiations, commerce, or trading with the enemy; all acts
which will increase, or tend to increase, its income or resources; all acts of
voluntary submission to it; or receiving its protection; also all acts concerning the
7
Section 300. Any person who for compensation solicits or obtains on behalf of any insurance company transmits
transmission of money or goods; and all contracts relating thereto are thereby
for a person other than himself an application for a policy or contract of insurance to or from such company or nullified. It further prohibits insurance upon trade with or by the enemy, upon the
offers or assumes to act in the negotiating of such insurance shall be an insurance agent within the intent of this life or lives of aliens engaged in service with the enemy; this for the reason that the
section and shall thereby become liable to all the duties, requirements, liabilities, and penalties to which an subjects of one country cannot be permitted to lend their assistance to protect by
insurance agent is subject.
iNsuRanCe A2010 Dean Carale
pAgE 49♥

insurance the commerce or property of belligerent, alien subjects, or to do anything


detrimental too their country's interest. The purpose of war is to cripple the power
and exhaust the resources of the enemy, and it is inconsistent that one country
should destroy its enemy's property and repay in insurance the value of what has
been so destroyed, or that it should in such manner increase the resources of the INSULAR LIFE ASSURANCE CO. v. EBRADO
enemy, or render it aid, and the commencement of war determines, for like
80 SCRA 181
reasons, all trading intercourse with the enemy, which prior thereto may have been
lawful. All individuals therefore, who compose the belligerent powers, exist, as to MARTIN; October 28, 1977
each other, in a state of utter exclusion, and are public enemies. (6 Couch, Cyc. of
Ins. Law, pp. 5352-5353.) NATURE
> In the case of an ordinary fire policy, which grants insurance only from year, or Appeal from judgment of RTC.
for some other specified term it is plain that when the parties become alien
enemies, the contractual tie is broken and the contractual rights of the parties, so FACTS
far as not vested. lost. (Vance, the Law on Insurance, Sec. 44, p. 112.) - Buenaventura Ebrado obtained a whole-life insurance policy from Insular, for
Reasoning P5,882.00 with a rider for accidental death benefits for the same amount. He
- The Court of Appeals overruled the contention of the petitioner that the respondent designated Carponia Ebrado as the revocable beneficiary, referring to her as the wife.
corporation became an enemy when the United States declared war against Germany, - Afterwards, he died as a result of an accident when he was hit by a falling branch of
relying on English and American cases which held that a corporation is a citizen of the a tree. Carponia filed a claim for the proceeds as the designated beneficiary in the
country or state by and under the laws of which it was created or organized. It policy, although she admits that she and Buenaventura were merely living as husband
rejected the theory that nationality of private corporation is determined by the and wife without the benefit of marriage. The legal wife, Pascuala Vda De Ebrado, also
character or citizenship of its controlling stockholders. filed her claim as the widow of the deceased.
- There is no question that majority of the stockholders of the respondent corporation - Insular then filed an interpleader in court (CFI Rizal) to determine to whom the
were German subjects. Therefore, Huenefeld Co became an enemy corporation upon proceeds should be paid. CFI declared that Carponia was disqualified from becoming
the outbreak of the war between the United States and Germany. The English and beneficiary of the insured and directing the Insular to pay the proceeds to the estate
American cases relied upon by the Court of Appeals have lost their force in view of the of Buenaventura.
latest decision of the Supreme Court of the United States in Clark vs. Uebersee Finanz
Korporation, decided on December 8, 1947, in which the controls test has been ISSUE
adopted. In "Enemy Corporation" by Martin Domke, a paper presented to the Second 1. WON a common-law wife named as beneficiary in the insurance policy of a legally
International Conference of the Legal Profession held at the Hague (Netherlands) in married man claim the proceeds of the same
August. 1948 also discussed this dilemma
> In Clark vs. Uebersee Finanz Korporation, A. G., dealing with a Swiss corporation HELD
allegedly controlled by German interest, the Court: "The property of all foreign 1. NO
interest was placed within the reach of the vesting power (of the Alien Property Ratio The prohibition that husband and wife cannot donate to each other applies to
Custodian) not to appropriate friendly or neutral assets but to reach enemy interest common-law relationships. As the appointment of a beneficiary in insurance may be
which masqueraded under those innocent fronts. . . . The power of seizure and considered a donation, one cannot name as beneficiary his common-law wife.
vesting was extended to all property of any foreign country or national so that no Reasoning
innocent appearing device could become a Trojan horse." - It is quite unfortunate that the Insurance Code does not contain any specific
- The respondent having become an enemy corporation on December 10, 1941, the provision grossly resolutory of the prime question at hand.
insurance policy issued in its favor on October 1, 1941, by the petitioner (a Philippine - Rather, general rules of civil law should be applied to resolve the issue. Art.2011, CC
corporation) had ceased to be valid and enforcible, and since the insured goods were states: “The contract of insurance is governed by special laws. Matters not expressly
burned after December 10, 1941, and during the war, the respondent was not entitled provided for in such special laws shall be regulated by this Code.” Thus, when not
to any indemnity under said policy from the petitioner. However, elementary rules of otherwise specifically provided for by the Insurance Law, the contract of life insurance
justice (in the absence of specific provision in the Insurance Law) require that the is governed by the general rules of the civil law regulating contracts.
premium paid by the respondent for the period covered by its policy from December - Also, Art.2012 “any person who is forbidden from receiving any donation under
11, 1941, should be returned by the petitioner. Article 739 cannot be named beneficiary of a life insurance policy by the person who
Disposition the appealed decision is hereby reversed and the respondent cannot make a donation to him.” Common-law spouses are, definitely, barred from
corporation is ordered to pay to the petitioner the sum of P77,208.33, Philippine receiving donations from each other.
currency, less the amount of the premium, in Philippine currency, that should be - Art.739, CC: The following donations shall be void:
returned by the petitioner for the unexpired term of the policy in question, beginning 1. Those made between persons who were guilty of adultery or concubinage at the
December 11, 1941. time of donation;
- In essence, a life insurance policy is no different from a civil donation insofar as the
beneficiary is concerned. Both are founded upon the same consideration: liberality. A
beneficiary is like a donee, because from the premiums of the policy which the
iNsuRanCe A2010 Dean Carale
pAgE 50♥

insured pays out of liberality, the beneficiary will receive the proceeds or profits of the one hand; and Basilia Berdin, his widow by the second marriage and their seven
said insurance. As a consequence, the proscription in Art.739 CC should equally children, on the other hand, who are entitled to the remaining one-half, or 8/16, each
operate in life insurance contracts. The mandate of Art.2012 cannot be laid aside: any of them to receive an equal share of 1/16.
person who cannot receive a donation cannot be named as beneficiary in the life - Dissatisfied with the foregoing ruling and apportionment made by the GSIS, Basilia
insurance policy of the person who cannot make the donation. Berdin and her children filed on October 10, 1966 a petition for mandamus with
- Policy considerations and dictates of morality rightly justify the institution of a barrier preliminary injunction in the Court of First Instance of Surigao.
between common-law spouses in regard to property relations since such relationship - The CFI of Surigao ruled in favor of respondent Rosario Diaz and upheld the ruling of
ultimately encroaches upon the nuptial and filial rights of the legitimate family. There GSIS in all aspect. Thus, Basilia Berdin and her children appealed said decision to the
is every reason to hold that the bar in donations between legitimate spouses and Supreme Court.
those between illegitimate ones should be enforced in life insurance policies since the
same are based on similar consideration. ISSUE
- So long as marriage remains the threshold of family laws, reason and morality WON GSIS was correct in awarding half of the retirement benefit of the deceased to
dictate that the impediments imposed upon married couple should likewise be Rosario Diaz, the first wife, notwithstanding the fact that the petitioners were named
imposed upon extra-marital relationship. If legitimate relationship is circumscribed by as beneficiaries of the life insurance
these legal disabilities, with more reason should an illicit relationship be restricted by
these disabilities. HELD
Disposition Decision AFFIRMED. YES
CONSUEGRA v. GSIS - The GSIS offers two separate and distinct systems of benefits to its members, one is
the life insurance and the other is the retirement insurance. These two distinct
37 SCRA 315
systems of benefits are paid out from two distinct and separate funds that are
ZALDIVAR; January 30, 1971 maintained by the GSIS. Thus, it doesn’t necessarily mean that the beneficiaries in the
life insurance are also the beneficiaries in the retirement insurance.
NATURE - Consuegra started in the government service sometime during the early part of
Appeal from the decision of the Court of First Instance of Surigao del Norte awarding 1943, or before 1943. In 1943 Com. Act 186 was not yet amended, and the only
the 8/16 part of the proceeds of the deceased Consuegra’s retirement benefits to benefits then provided for in said Com. Act 186 were those that proceed from a life
Rosario Diaz. insurance. Upon entering the government service Consuegra became a compulsory
member of the GSIS, being automatically insured on his life, pursuant to the
FACTS provisions of Com. Act 186 which was in force at the time. During 1943 the operation
- The late Jose Consuegra, at the time of his death, was employed as a shop foreman of the Government Service Insurance System was suspended because of the war, and
of the office of the District Engineer in the province of Surigao del Norte. In his the operation was resumed sometime in 1946. When Consuegra designated his
lifetime, Consuegra contracted two marriages, the first with herein respondent Rosario beneficiaries in his life insurance he could not have intended those beneficiaries of his
Diaz, solemnized in the parish church of San Nicolas de Tolentino, Surigao, Surigao, on life insurance as also the beneficiaries of his retirement insurance because the
July 15, 1937, out of which marriage were born two children, namely, Jose Consuegra, provisions on retirement insurance under the GSIS came about only when Com. Act
Jr. and Pedro Consuegra, but both predeceased their father; and the second, which 186 was amended by Rep. Act 660 on June 16, 1951. Hence, it cannot be said that
was contracted in good faith while the first marriage was subsisting, with herein because herein appellants were designated beneficiaries in Consuegra's life insurance
petitioner Basilia Berdin, on May 1, 1957 in the same parish and municipality, out of they automatically became the beneficiaries also of his retirement insurance.
which marriage were born seven children, namely, Juliana, Pacita, Maria Lourdes, Jose, - The provisions of subsection (b) of Section 11 of Commonwealth Act 186, as
Rodrigo, Lenida and Luz, all surnamed Consuegra. amended by Rep. Act 660, clearly indicate that there is need for the employee to file
- Being a member of the Government Service Insurance System (GSIS, for short) when an application for retirement insurance benefits when he becomes a member of the
Consuegra died on September 26, 1965, the proceeds of his life insurance under GSIS, and he should state in his application the beneficiary of his retirement
policy No. 601801 were paid by the GSIS to petitioner Basilia Berdin and her children insurance. Hence, the beneficiary named in the life insurance does not automatically
who were the beneficiaries named in the policy. become the beneficiary in the retirement insurance unless the same beneficiary in the
- However, Consuegra did not designate any beneficiary who would receive the life insurance is so designated in the application for retirement insurance.
retirement insurance benefits due to him. Respondent Rosario Diaz, the widow by the - In the case of the proceeds of a life insurance, the same are paid to whoever is
first marriage, filed a claim with the GSIS asking that the retirement insurance named the beneficiary in the life insurance policy. As in the case of a life insurance
benefits be paid to her as the only legal heir of Consuegra, considering that the provided for in the Insurance Act, the beneficiary in a life insurance under the GSIS
deceased did not designate any beneficiary with respect to his retirement insurance may not necessarily be an heir of the insured. The insured in a life insurance may
benefits. Petitioner Basilia Berdin and her children, likewise, filed a similar claim with designate any person as beneficiary unless disqualified to be so under the
the GSIS, asserting that being the beneficiaries named in the life insurance policy of provisions of the Civil Code. And in the absence of any beneficiary named in the
Consuegra, they are the only ones entitled to receive the retirement insurance life insurance policy, the proceeds of the insurance will go to the estate of the insured.
benefits due the deceased Consuegra. Resolving the conflicting claims, the GSIS ruled - On the other hand, the beneficiary of the retirement insurance can only claim the
that the legal heirs of the late Jose Consuegra were Rosario Diaz, his widow by his first proceeds of the retirement insurance if the employee dies before retirement. If the
marriage who is entitled to one-half, or 8/16, of the retirement insurance benefits, on employee failed or overlooked to state the beneficiary of his retirement insurance, the
iNsuRanCe A2010 Dean Carale
pAgE 51♥

retirement benefits will accrue to his estate and will be given to his legal heirs in enforced to the extent of contradicting the very provisions of said law contained in
accordance with law, as in the case of a life insurance if no beneficiary is named in the Section 13, thereof”
insurance policy. - When the provisions of a law are clear and explicit, the courts can do nothing but
Disposition Petition Denied. It is Our view, therefore, that the respondent GSIS had apply its clear and explicit provisions (Velasco vs. Lopez)
correctly acted when it ruled that the proceeds of the retirement insurance of the late 2. NO
Jose Consuegra should be divided equally between his first living wife Rosario Diaz, on - The disqualification mentioned in Article 739 is not applicable to herein appellee
the one hand, and his second wife Basilia Berdin and his children by her. Candelaria Davac because she was not guilty of concubinage, there being no proof
that she had knowledge of the previous marriage of her husband Petronilo.
ART. 2012. Any person who is forbidden from receiving any donation under Article
SSS v. DAVAC
739 cannot be named beneficiary of a life insurance policy by the person who
17 SCRA 863 cannot make any donation to him according to said article.
BARRERA: July 30, 1966 ART. 739. The following donations shall be void:
(1) Those made between persons who were guilty of adultery or concubinage at the
NATURE time of the donation; (the court did not decide whether this partakes the nature of a
APPEAL from a resolution Of the Social Security Commission. life insurance policy)
3. NO
FACTS - The benefit receivable under the Act is in the nature of a special privilege or an
- Petronilo Davac, became a member of the Social Security System (SSS for short) on arrangement secured by the law pursuant to the policy of the State to provide social
September 1, 1957. In the Member's Record he designated respondent, Candelaria security to the workingmen. The amounts that may thus be received cannot be
Davac as his beneficiary and indicated his relationship to her as that of "wife". considered as property earned by the member during his lifetime. His contribution to
- He died on April 5, 1959. It appears that the deceased contracted two marriages, the the fund constitutes only an insignificant portion thereof. Then, the benefits are
first, with Lourdes Tuplano on August 29, 1946, who bore him a child, Romeo Davac, specifically declared not transferable, and exempted from tax, legal processes, and
and the second, with Candelaria Davac on January 18, 1949, with whom he had a lien. Furthermore, in the settlement of claims thereunder, the procedure to be
minor daughter, Elizabeth Davac. Both filed their claims for death benefit with the observed is governed not by the general provisions of law, but by rules and
SSS. regulations promulgated by the Commission. Thus, if the money is payable to the
- Social Security Commission issued the resolution declaring respondent Candelaria estate of a deceased member, it is the Commission, not the probate or regular court
Davac as the person entitled to receive the death benefits payable for the death of that determines the person or persons to whom it is payable.
Petronilo Davac. - They are disbursed from a public special fund created by Congress.The sources of
this special fund are the covered employee's contribution (equal to 2-1/2 per cent of
ISSUES the employee's monthly compensation) ; the employer's 'Contribution (equivalent to
1. WON the Social Security Commission Candelaria Davac is entitled to receive the 3-1/2 per cent of the monthly compensation of the covered employee) ;and the
death benefits Government contribution which consists in yearly appropriation of public funds to
2. WON a beneficiary under the Social Security System partakes of the nature of a assure the maintenance of an adequate working balance of the funds of the System.
beneficiary in a life insurance policy and, therefore the designation made in the Additionally, Section 21 of the Social Security Actprovides that the benefits prescribed
person DAVAC as bigamous wife is null and void, because it contravenes the in this Act shall not be diminished and the Government of the Republic of the
provisions of the Civil Code Philippines accepts general responsibility for the solvency of the System.
3. WON the benefits accruing from membership with SSS forms part of the conjugal - The benefits under the Social Security Act are not intended by the lawmaking body
property thus the resolution deprives the lawful wife of her share in the conjugal to form part of the estate of the covered –members.
property as well as of her own and her child's legitime in the inheritance - Social Security Act is not a law of succession.
Disposition Resolution of the Social Security Commission appealed is affirmed
HELD
1. YES FRANCISCO DEL VAL v. ANDRES DEL VAL
- Section 13, RA1161 provides that the beneficiary "as recorded" by the employee's
29 PHIL 534
employer is the one entitled to the death benefits.
- Section 13, Republic Act No. 1161, as amended by Republic Act No. 1792, in force at MORELAND; February 16, 1915
the time of Petronilo Davac's death provides: Upon the covered employee's death or
total and permanent disability under such conditions as the Commission may define, NATURE
before becoming eligible for retirement and if either such death or disability is not Appeal from a judgment of the Court of First Instance of the city of Manila dismissing
compensable under the Workmen's Compensation Act, he or. in case of his death, his the complaint with costs.
beneficiaries, as recorded by his employer shall be entitled to the following benefit:
- In Tecson vs. Social Security System. Section 13 was construed:"it may be true that FACTS
the purpose of the coverage under the Social Security System is protection of the
employee as well as of his family, but this purpose or intention of the law cannot be
iNsuRanCe A2010 Dean Carale
pAgE 52♥

- Plaintiffs and defendant are brothers and sisters; that they are the only heirs at law - The SC agreed with the finding of the trial court that the proceeds of the life-
and next of kin of Gregorio Nacianceno del Val, who died in Manila on August 4, 1910, insurance policy belong exclusively to the defendant as his individual and separate
intestate property, we agree. That the proceeds of an insurance policy belong exclusively to the
- During the lifetime of the deceased he took out insurance on his life for the sum of beneficiary and not to the estate of the person whose life was insured, and that such
P40,000 and made it payable to the defendant ANDRES DEL VAL as sole beneficiary. proceeds are the separate and individual property of the beneficiary, and not of the
After his death the defendant collected the face of the policy. From said policy he heirs of the person whose life was insured, is the doctrine in America. We believe that
paid the sum of P18,365.20 to redeem certain real estate which the decedent had sold the same doctrine obtains in these Islands by virtue of section 428 of the Code of
to third persons with a right to repurchase. Commerce, which reads:
- The redemption of said premises was made by the attorney of defendant ANDRES in "The amounts which the underwriter must deliver to the person insured, in
the name of the plaintiffs and the defendant as heirs of the deceased vendor. It fulfillment of the contract, shall be the property of the latter, even against the
further appears from the pleadings that the defendant, on the death of the deceased, claims of the legitimate heirs or creditors of any kind whatsoever of the person who
took possession of most of his personal property, which he still has in his possession, effected the insurance in favor of the former."
and that he has also the balance on said insurance policy amounting to P21,634.80. 2. NO
- Plaintiffs contend that the amount of the insurance policy belonged to the estate of - The contract of life insurance is a special contract and the destination of the
the deceased and not to the defendant personally; that, therefore, they are entitled to proceeds thereof is determined by special laws which deal exclusively with that
a partition not only of the real and personal property, but also of the P40,000 life subject. The Civil Code has no provisions which relate directly and specifically to life-
insurance. The complaint prays a partition of all the property, both real and personal, insurance contracts or to the destination of life insurance proceeds. That subject is
left by the deceased; that the defendant account for P21,634.80, and that the sum be regulated exclusively by the Code of Commerce which provides for the terms of the
divided equally among the plaintiffs and defendant along with the other property of contract, the relations of the parties and the destination of the proceeds of the policy.
deceased. - Assuming that the proceeds of the life-insurance policy being the exclusive property
- The defendant denies the material allegations of the complaint and sets up as of the defendant and he having used a portion thereof in the repurchase of the real
special defense and counterclaim that the redemption of the real estate sold by his estate sold by the decedent prior to his death with right to repurchase, and such
father was made in the name of the plaintiffs and himself instead of in his name alone repurchase having been made and the conveyance taken in the names of all of the
without his knowledge or consent. Andres contends that it was not his intention to use heirs instead of the defendant alone, plaintiffs claim that the property belongs to the
the proceeds of the insurance policy for the benefit of any person but himself, he heirs in common and not to the defendant alone.
alleging that he was and is the sole owner thereof and that it is his individual property. - The Court rejected this contention unless the fact appear or be shown that the
He, therefore, asks that he be declared the owner of the real estate redeemed by the defendant acted as he did with the intention that the other heirs should enjoy with
payment of the P18,365.20, the owner of the remaining P21,634.80, the balance of him the ownership of the estate ---- in other words, that he proposed, in effect, to
the insurance policy, and that the plaintiffs account for the use and occupation of the make a gift of the real estate to the other heirs. If it is established by the evidence
premises so redeemed since the date of the redemption. that was his intention and that the real estate was delivered to the plaintiffs with that
- The trial court refused to give relief to either party and dismissed the action. In this understanding, then it is probable that their contention is correct and that they are
appeal, it is claimed by the attorney for the plaintiffs that insurance provisions in the entitled to share equally with the defendant therein. If, however, it appears from the
Code of Commerce are subordinated to the provisions of the Civil Code as found in evidence in the case that the conveyances were taken in the name of the plaintiffs
article 1035. This article reads: without his knowledge or consent, or that it was not his intention to make a gift to
"An heir by force of law surviving with others of the same character to a succession them of the real estate, then it belongs to him. If the facts are as stated, he has two
must bring into the hereditary estate the property or securities he may have received remedies. The one is to compel the plaintiffs to reconvey to him and the other is to let
from the deceased during the life of the same, by way of dowry, gift, or for any good the title stand with them and to recover from them the sum he paid on their behalf.
consideration, in order to compute it in fixing the legal portions and in the account of - For the complete and proper determination of the questions at issue in this case, the
the division." Court was of the opinion that the cause should be returned to the trial court with
- Counsel also claims that the proceeds of the insurance policy were a donation or gift instructions to permit the parties to frame such issues as will permit the settlement of
made by the father during his lifetime to the defendant and that, as such, its ultimate all the questions involved and to introduce such evidence as may be necessary for the
destination is determined by those provisions of the Civil Code which relate to full determination of the issues framed. Upon such issues and evidence taken
donations, especially article 819. This article provides that "gifts made to children thereunder the court will decide the questions involved according to the evidence,
which are not betterments shall be considered as part of their legal portion." subordinating his conclusions of law to the rules laid down in this opinion.
REMANDED.
ISSUES
1. WON the insurance belongs to the defendant and not to the decedent’s estate
GERCIO v. SUN LIFE ASSURANCE OF CANADA
2. WON the Civil code provisions on succession prevail over any other law with respect
to the insurance 48 PHIL 53
MALCOLM; September 28, 1925
HELD
1. YES NATURE
iNsuRanCe A2010 Dean Carale
pAgE 53♥

Mandamus to compel Sun Life Assurance Co. of Canada to change the beneficiary in 2. NO
the policy issued by the defendant company on the life of the plaintiff Hilario Gercio Ratio The wife has an insurable interest in the life of her husband. The beneficiary
has an absolute vested interest in the policy from the date of its issuance and
FACTS delivery. So when a policy of life insurance is taken out by the husband in which the
- On January 29, 1910, the Sun Life Assurance Co. of Canada issued an insurance wife is named as beneficiary, she has a subsisting interest in the policy. And this
policy on the life of Hilario Gercio. The policy was what is known as a 20-year applies to a policy to which there are attached the incidents of a loan value, cash
endowment policy. By its terms, the insurance company agreed to insure the life of surrender value, an automatic extension by premiums paid, and to an endowment
Hilario Gercio for the sum of P2,000, to be paid him on February 1, 1930, or if the policy, as well as to an ordinary life insurance policy. If the husband wishes to retain to
insured should die before said date, then to his wife, Mrs. Andrea Zialcita, should she himself the control and ownership of the policy he may so provide in the policy. But if
survive him; otherwise to the executors, administrators, or assigns of the insured. The the policy contains no provision authorizing a change of beneficiary without the
policy did not include any provision reserving to the insured the right to change the beneficiary's consent, the insured cannot make such change. Accordingly, it is held
beneficiary. that a life insurance policy of a husband made payable to the wife as beneficiary, is
- On the date the policy was issued, Andrea Zialcita was the lawful wife of Hilario the separate property of the beneficiary and beyond the control of the husband.
Gercio. Towards the end of the year 1919, she was convicted of the crime of adultery. - Unlike the statutes of a few jurisdictions, there is no provision in the Philippine Law
On September 4, 1920, a decree of divorce was issued in civil case no. 17955, which permitting the beneficiary in a policy for the benefit of the wife of the husband to be
had the effect of completely dissolving their bonds of matrimony changed after a divorce. It must follow, therefore, in the absence of a statute to the
- On March 4, 1922, Hilario Gercio formally notified the Sun Life that he had revoked contrary, that if a policy is taken out upon a husband's life the wife is named as
his donation in favor of Andrea Zialcita, and that he had designated in her stead his beneficiary therein, a subsequent divorce does not destroy her rights under the policy.
present wife, Adela Garcia de Gercio, as the beneficiary of the policy. Gercio Reasoning
requested the insurance company to eliminate Andrea Zialcita as beneficiary. This, - Yore vs. Booth
the insurance company has refused and still refuses to do. “. . . It seems to be the settled doctrine, with but slight dissent in the courts of this
country, that a person who procures a policy upon his own life, payable to a
ISSUES designated beneficiary, although he pays the premiums himself, and keeps the
1. (Preliminary) WON the provisions of the Code of Commerce and the Civil Code shall policy in his exclusive possession, has no power to change the beneficiary, unless
be in force in 1910, or the provisions of the Insurance Act now in force, or the general the policy itself, or the charter of the insurance company, so provides. In policy,
principles of law, guide the court in its decision although he has parted with nothing, and is simply the object of another's bounty,
2. WON the insured, the husband, has the power to change the beneficiary, the former has acquired a vested and irrevocable interest in the policy, which he may keep
wife, and to name instead his actual wife, where the insured and the beneficiary have alive for his own benefit by paying the premiums or assessments if the person who
been divorced and where the policy of insurance does not expressly reserve to the effected the insurance fails or refuses to do so.”
insured the right to change the beneficiary - Connecticut Mutual Life Insurance Company vs Schaefer
“We do not hesitate to say, however, that a policy taken out in good faith and valid
HELD at its inception, is not avoided by the cessation of the insurable interest, unless
1. Whether the case be considered in the light of the Code of Commerce, the Civil such be the necessary effect of the provisions of the policy itself.. . . .In our
Code, or the Insurance Act, the deficiencies in the law will have to be supplemented judgment of life policy, originally valid, does not cease to be so by the cessation of
by the general principles prevailing on the subject. To that end, we have gathered the the assured party's interest in the life insured.”
rules which follow from the best considered American authorities. In adopting these - Central National Bank of Washington City vs. Hume
rules, we do so with the purpose of having the Philippine Law of Insurance conform as “It is indeed the general rule that a policy, and the money to become due under it,
nearly as possible to the modern Law of Insurance as found in the United States belong, the moment it is issued, to the person or persons named in it as the
proper. beneficiary or beneficiaries, and that there is no power in the person procuring the
- Court’s first duty is to determine what law should be applied to the facts. The insurance, by any act of his, by deed or by will, to transfer to any other person the
insurance policy was taken out in 1910, that the Insurance Act. No. 2427, became interest of the person named.”
effective in 1914, and that the effort to change the beneficiary was made in 1922. - In re Dreuil & Co.
- Code of Commerce- there can be found in it no provision either permitting or “In so far as the law of Louisiana is concerned, it may also be considered settled
prohibiting the insured to change the beneficiary. that where a policy is of the semitontine variety, as in this case, the beneficiary has
- Civil Code- it would be most difficult, if indeed it is practicable, to test a life a vested right in the policy, of which she cannot be deprived without her consent”
insurance policy by its provisions. In the case of Del Val vs. Del Val, it declined to - Wallace vs Mutual Benefit Life Insurance Co.
consider the proceeds of the insurance policy as a donation or gift, saying "the “As soon as the policy was issued Mrs. Wallace acquired a vested interest therein,
contract of life insurance is a special contract and the destination of the proceeds of which she could not be deprived without her consent, except under the terms of
thereof is determined by special laws which deal exclusively with that subject. The the contract with the insurance company. No right to change the beneficiary was
Civil Code has no provisions which relate directly and specifically to life-insurance reserved. Her interest in the policy was her individual property, subject to be
contracts or to the destination of life-insurance proceeds. . . ." divested only by her death, the lapse of time, or by the failure of the insured to pay
- Insurance Act- there is likewise no provision either permitting or prohibiting the the premiums. She could keep the policy alive by paying the premiums, if the
insured to change the beneficiary. insured did not do so. It was contingent upon these events, but it was free from the
iNsuRanCe A2010 Dean Carale
pAgE 54♥

control of her husband. He had no interest in her property in this policy, contingent - Respondent Judge denied petitioner’s Urgent Motion, thus allowing private
or otherwise. Her interest was free from any claim on the part of the insured or his respondent to adduce evidence, the consequence of which was the issuance of the
creditors. He could deprive her of her interest absolutely in but one way, by living questioned Order granting the petition. Petitioner then filed a MFR which was also
more than twenty years.” denied hence this petition.
- Filley vs. Illinois Life Insurance Company
“The benefit accruing from a policy of life insurance upon the life of a married man, ISSUE
payable upon his death to his wife, naming her, is payable to the surviving 1. WON the designation of the irrevocable beneficiaries could be changed or amended
beneficiary named, although she may have years thereafter secured a divorce from without the consent of all the irrevocable beneficiaries
her husband, and he was thereafter again married to one who sustained the 2. WON the irrevocable beneficiaries herein, one of whom is already deceased while
relation of wife to him at the time of his death. the others are all minors could validly give consent to the change or amendment in
The rights of a beneficiary in an ordinary life insurance policy become vested upon the designation of the irrevocable beneficiaries
the issuance of the policy, and can thereafter, during the life of the beneficiary, be
defeated only as provided by the terms of the policy.” HELD
- On the admitted facts and the authorities supporting the nearly universally accepted 1. NO
principles of insurance, we are irresistibly led to the conclusion that the question at - Based on the provision of their contract and the law applicable, it is only with the
issue must be answered in the negative consent of all the beneficiaries that any change or amendment in the policy
Disposition The judgment appealed from will be reversed and the complaint ordered concerning the irrevocable beneficiaries may be legally and validly effected. Both the
dismissed as to the appellant. law and the Policy do not provide for any other exception.
Reasoning
- Since the policy was procured in 1968, the applicable law in this case is the
SEPARATE OPINION
Insurance Act and under that law, the beneficiary designated in a life insurance
contract cannot be changed without the consent of the beneficiary because he has a
JOHNSON [concur] vested interest in the policy.
- I agree with the majority of the court, that the judgment of the lower court should be - The Beneficiary Designation Indorsement in the policy in the name of Dimayuga
revoked, but for a different reason. The purpose of the petition is to have declared the states that the designation of the beneficiaries is irrevocable: “no right or privilege
rights of certain persons in an insurance policy which is not yet due and payable. It under the Policy may be exercised, or agreement made with the Company to any
may never become due and payable. The premiums may not be paid, thereby change in or amendment to the Policy, without the consent of the said
rendering the contract of insurance of non effect, and many other things may occur, beneficiary/beneficiaries.”
before the policy becomes due, which would render it non effective. The plaintiff and - Contracts which are the private laws of the contracting parties should be fulfilled
the other parties who are claiming an interest in said policy should wait until there is according to the literal sense of their stipulations, if their terms are clear and leave no
something due them under the same. For the courts to declare now who are the room for doubt as to the intention of the contracting parties, for contracts are
persons entitled to receive the amounts due, if they ever become due and payable, is obligatory, no matter in what form they may be, whenever the essential requisites for
impossible, for the reason that nothing may ever become payable under the contract their validity are present.
of insurance, and for many reasons such persons may never have a right to receive - Finally, the fact that the contract of insurance does not contain a contingency when
anything when the policy does become due and payable. In my judgment, the action the change in the designation of beneficiaries could be validly effected means that it
is premature and should have been dismissed. was never within the contemplation of the parties.
2. NO
PHIL. AMERICAN LIFE INSURANCE v. PINEDA - The parent-insured cannot exercise rights and/or privileges pertaining to the
insurance contract, for otherwise, the vested rights of the irrevocable beneficiaries
175 SCRA 416 would be rendered inconsequential. The alleged acquiescence of the 6 children
PARAS; July 19, 1989 beneficiaries cannot be considered an effective ratification to the change of the
beneficiaries from irrevocable to revocable. They were minors at the time, and could
NATURE not validly give consent. Neither could they act through their father-insured since their
Petition for review on certiorari the orders of CFI Judge Pineda interests are quite divergent from one another.
Disposition questioned Orders of respondent judge are nullified and set aside.
FACTS
- In 1968, Private Respondent Rodolfo Dimayuga procured an ordinary life insurance SUN LIFE ASSURANCE v. INGERSOLL
policy from the petitioner company and designated his wife and children as
irrevocable beneficiaries. On Feb. 22, 1980, Dimayuga filed with the CFI a petition to 41 PHIL 331
amend the designation of the beneficiaries in his life policy from irrevocable to STREET; November 8, 1921
revocable. Petitioner filed an Urgent Motion to reset hearing as well as its comment
and/or Opposition to the respondent’s petition. NATURE
Action of interpleader
iNsuRanCe A2010 Dean Carale
pAgE 55♥

which the insolvent might have exercised for his own benefit, and under subsection 5
FACTS the trustee acquires any property of the insolvent which the latter could by any means
- April 16, 1918, Sun Life Assurance Company of Canada (Sun Life), in consideration of have assigned to another. The Insolvency Law here in force, in common with the
the payment of a stipulated annual premium during the period of the policy, or until predecessor laws above-mentioned, contains nothing similar to these provisions.
the premiums had been completely paid for twenty years, issued a policy of insurance On the applicability of the Insolvency Law
on the life of Dy Poco for US$12,500, payable to the said assured or his assigns on the - Sec 32 of the Insolvency Law among other things, declares that the assignment to be
21st day of February, 1938, and if he should die before that date then to his legal made by the clerk of the court "shall operate to vest in the assignee all of the estate
representatives. of the insolvent debtor not exempt by law from execution." Moreover, by section 24,
- June 23, 1919, the assured, Dy Poco, was adjudged an involuntary insolvent by the the court is required, upon making an order adjudicating any person insolvent, to stay
CFI Manila, and Frank B. Ingersoll was appointed assignee of his estate. any civil proceedings pending against him; and it is declared in section 60 that no
- July 10, 1919, Dy Poco died, and on August 21, 1919, Tan Sit, was duly appointed as creditor whose debt is provable under the Act shall be allowed, after the
the administratrix of his intestate estate. commencement of proceedings in insolvency, to prosecute to final judgment any
- By the terms of the policy it was provided that after the payment of three full action therefor against the debtor. In connection with the foregoing may be
premiums, the assured could surrender the policy to the company for a "cash mentioned subsections 1 and 2 of section 36, as well as the opening words of section
surrender value," indicated in an annexed table; but inasmuch as no more than two 33, to the effect that the assignee shall have the right and power to recover and to
premiums had been paid upon the policy now in question up to the time of the death take into his possession, all of the estate, assets, and claims belonging to the
of the assured, this provision had not become effective; and it does not appear that insolvent, except such as are exempt by law from execution.
the company would in accordance with its own usage or otherwise have made any - These provisions clearly evince an intention to vest in the assignee, for the benefit of
concession to the assured in the event he had desired, before his death, to surrender all the creditors of the insolvent, such elements of property and property right as
the policy. It must therefore be accepted that this policy had no cash surrender value, could be reached and subjected by process of law by any single creditor suing alone.
at the time of the assured's death, either by contract or by convention practice of the And this is exactly as it should be: for it cannot be supposed that the Legislature
company in such cases. would suppress the right of action of every individual creditor upon the adjudication of
- Both Ingersoll, as assignee, and Tan Sit, as administratix of Dy Poco's estate, insolvency, and at the same time allow the insolvent debtor to retain anything subject
asserted claims to the proceeds of the policy. The lower court found that Ingersoll had to the payment of his debts in a normal state of solvency.
a better right and ordered Sun Life to pay the insurance proceeds to him. - "leviable assets" and "assets in insolvency" are practically coextensive terms. Hence,
in determining what elements of value constitute assets in insolvency, SC is at liberty
ISSUE to consider what elements of value are subject to be taken upon execution, and vice
WON Ingersoll, as assignee, has a right to the proceeds of the insurance versa.
On whether a policy of insurance having no cash surrender value, but payable to
HELD insured or his legal representative, is property that may be taken upon execution
NO against him.
On the Philippine Insolvency Law (Act No. 1956) - Philippine laws declare no exemption with respect to insurance policies; and this
- The property and interests of the insolvent which become vested in the assignee of species of property is not enumerated, in section 48 of the Insolvency Law, among
the insolvent are specified in section 32 of the Insolvency Law which reads as follows: items from the ownership of which the assignee is excluded. Moreover, all life
"SEC. 32. As soon as an assignee is elected or appointed and qualified, the insurance policies are declared by law to be assignable, regardless of whether the
clerk of the court shall, by an instrument under his hand and seal of the court, assign assignee has an insurable interest in the life of the insured or not (Insurance Act No.
and convey to the assignee all the real and personal property, estate, and effects of 2427, sec. 166).
the debtor with all his deeds, books, and papers relating thereto, and such assignment - SC has held that insurance policies having a present cash surrender value are
shall relate back to the commencement of the proceedings in insolvency, and shall subject to be taken upon execution. (Misut Garcia vs. West Coast San Francisco Life
relate back to the acts upon which the adjudication was founded, and by operation of Ins. Co.)
law shall vest the title to all such property, estate, and effects in the assignee, - a policy devoid of a cash surrender value cannot be either "leviable assets" or
although the same is then attached on mesne process, as the property of the debtor. "assets in insolvency."
Such assignment shall operate to vest in the assignee all of the estate of the insolvent - the assignee in insolvency acquired no beneficial interest in the policy of insurance
debtor not exempt by law from execution." in question; that its proceeds are not liable for any of the debts provable against the
- the Insolvency Law is in great part a copy of the Insolvency Act of California, enacted insolvent in the pending proceedings, and that said proceeds should therefore be
in 1895, though it contains a few provisions from the American Bankruptcy Law of delivered to his administratrix.
1898 On applicable US case
- Under each of said laws the assignee acquires all the real and personal property, - In re McKinney: no beneficial interest in the policy had ever passed to the assignee
estate, and effects of the debtor, not exempt by law from execution, with all deeds, over and beyond what constituted the surrender value, and that the legal title to the
books and papers relating thereto; and while this language is broad, it nevertheless policy was vested in the assignee merely in order to make the surrender value-
lacks the comprehensiveness of section 70 (a) of the American Bankruptcy Law of available to him. The assignee should surrender the policy upon the payment to him
1898 in at least two particulars; for under subsection 3 of section 70 (a) of the last of said value, as he was in fact directed to do. The assignee in bankruptcy had no
mentioned law, the trustee in bankruptcy acquires the right to exercise any powers right to keep the estate unsettled for an indefinite period, for the mere purpose of
iNsuRanCe A2010 Dean Carale
pAgE 56♥

speculating upon the chances of the bankrupt's death. As regards everything beyond commencement of proceedings in insolvency, to prosecute to final judgment any
the surrender value, the assignee in bankruptcy would, after the discharge of the action therefor against the debtor. In connection with the foregoing may be
bankrupt, have no insurable interest in the life of the bankrupt. mentioned subsections 1 and 2 of section 36, as well as the opening words of section
- surrender value of a policy "arises from the fact that the fixed annual premiums is 33, to the effect that the assignee shall have the right and power to recover and to
much in excess of the annual risk during the earlier years of the policy, an excess take into his possession, all of the estate, assets, and claims belonging to the
made necessary in order to balance the deficiency of the same premium to meet the insolvent, except such as are exempt by law from execution.
annual risk during the latter years of the policy. This excess in the premium paid over - These provisions clearly evince an intention to vest in the assignee, for the benefit of
the annual cost of insurance, with accumulations of interest, constitutes the surrender all the creditors of the insolvent, such elements of property and property right as
value. Though this excess of premiums paid is legally the sole property of the could be reached and subjected by process of law by any single creditor suing alone.
company, still in practical effect, though not in law, it is moneys of the assured And this is exactly as it should be: for it cannot be supposed that the Legislature
deposited with the company in advance to make up the deficiency in later premiums would suppress the right of action of every individual creditor upon the adjudication of
to cover the annual cost of insurance, instead of being retained by the assured and insolvency, and at the same time allow the insolvent debtor to retain anything subject
paid by him to the company in the shape of greatly-increased premiums, when the to the payment of his debts in a normal state of solvency.
risk is greatest. It is the 'net reserve' required by law to be kept by the company for - "leviable assets" and "assets in insolvency" are practically coextensive terms. Hence,
the benefit of the assured, and to be maintained to the credit of the policy. So long as in determining what elements of value constitute assets in insolvency, SC is at liberty
the policy remains in force the company has not practically any beneficial interest in to consider what elements of value are subject to be taken upon execution, and vice
it, except as its custodian, with the obligation to maintain it unimpaired and suitably versa.
invested for the benefit of the insured. This is the practical, though not the legal, On whether a policy of insurance having no cash surrender value, but payable to the
relation of the company to this fund. "Upon the surrender of the policy before the insured or his legal representative, is property that may be taken upon execution
death of the assured, the company, to be relieved from all responsibility for the against him.
increased risk, which is represented by this accumulating reserve, could well afford to - Philippine laws declare no exemption with respect to insurance policies; and this
surrender a considerable part of it to the assured, or his representative. A return of a species of property is not enumerated, in section 48 of the Insolvency Law, among
part in some form or other is now Usually made." (In re McKinney) items from the ownership of which the assignee is excluded. Moreover, all life
- the stipulation providing for a cash surrender value is a comparatively recent insurance policies are declared by law to be assignable, regardless of whether the
innovation in life insurance. Formerly the contracts provided — as they still commonly assignee has an insurable interest in the life of the insured or not (Insurance Act No.
do in the policies issued by fraternal organizations and benefit societies — for the 2427, sec. 166).
payment of a premium sufficient to keep the estimated risk covered; and in case of a - SC has held that insurance policies having a present cash surrender value are
lapse the policy-holder received nothing. Furthermore, the practice is common among subject to be taken upon execution. (Misut Garcia vs. West Coast San Francisco Life
insurance companies even now to concede nothing in the character of cash surrender Ins. Co., 41 Phil., 258.)
value, until three full premiums have been paid, as in this case. - a policy devoid of a cash surrender value cannot be either "leviable assets" or
- CONLUSION (from this case and other English and American cases cited following the "assets in insolvency."
same opinion): the assignee acquires no beneficial interest in insurance effected on - the assignee in insolvency acquired no beneficial interest in the policy of insurance
the life of the insolvent, except to the extent that such insurance contains assets in question; that its proceeds are not liable for any of the debts provable against the
which can be realized upon as of the date when the petition of insolvency is filed. The insolvent in the pending proceedings, and that said proceeds should therefore be
explanation is to be found in the consideration that the destruction of a contract of life delivered to his administratrix.
insurance is not only highly prejudicial to the insured and those dependent upon him, Disposition Judgment reversed. Sun Life is directed to pay the proceeds of the policy
but is inimical to the interests of society. Insurance is a species of property that should to Tan Sit.
be conserved and not dissipated. As is well known, life insurance is increasingly
difficult to obtain with advancing years, and even when procurable after the age of
fifty, the cost is then so great as to be practically prohibitive to many. Insolvency is a
CHAPTER VI – RESCISSION OF INSURANCE
disaster likely to overtake men in mature life; and one who has gone through the CONTRACTS: CONCEALMENT,
process of bankruptcy usually finds himself in his declining years with the MISREPRESENTATION, & BREACH OF WARRANTIES
accumulated savings of years swept away and earning power diminished. The courts
are therefore practically unanimous in refusing to permit the assignee in insolvency to
wrest from the insolvent a policy of insurance which contains in it no present NG v. ASIAN CRUSADER LIFE ASSURANCE CORP
realizable assets. 122 SCRA 461
On the applicability of the Insolvency Law ESCOLIN; May 30, 1983
- Sec 32 of the Insolvency Law among other things, declares that the assignment to be
made by the clerk of the court "shall operate to vest in the assignee all of the estate
FACTS
of the insolvent debtor not exempt by law from execution." Moreover, by section 24,
- On May 12, 1962, Kwong Nam applied for a 20-year endowment insurance on his life
the court is required, upon making an order adjudicating any person insolvent, to stay
for the sum of P20,000, with his wife, Ng Gan Zee, as beneficiary.
any civil proceedings pending against him; and it is declared in section 60 that no
creditor whose debt is provable under the Act shall be allowed, after the
iNsuRanCe A2010 Dean Carale
pAgE 57♥

- He died on Dec 1963 of cancer of the liver with metastasis. All premiums had been good faith of his belief as to the nature of his ailment and operation. Indeed, such
paid at the time of his death. statement must be presumed to have been made by him without knowledge of its
- Ng presented a claim for payment of the face value of the policy. Appellant (Asian incorrectness and without any deliberate intent on his part to mislead the appellant.
Crusader) denied the claim on the ground that the answers given by the insured to 3) Waiver:
the questions appearing in his application for life insurance were untrue. While it may be conceded that, from the viewpoint of a medical expert, the
-Appellant: the insured was guilty of misrepresentation when information communicated was imperfect, the same was nevertheless sufficient to
1) he answered "No" to the question (in the application) of "Has any life insurance have induced appellant to make further inquiries about the ailment and operation of
company ever refused your application for insurance or for reinstatement of a lapsed the insured.
policy or offered you a policy different from that applied for?" when in fact, Insular Life Section 32 of Insurance Law [Act No. 2427] provides:
denied his application for reinstatement of his lapsed life insurance policy “The right to information of material facts may be waived either by the terms of
2) he gave the appellant's medical examiner false and misleading information as to insurance or by neglect to make inquiries as to such facts where they are distinctly
his ailment and previous operation when he said he was implied in other facts of which information is communicated.”
“operated on for a Tumor [mayoma] of the stomach… associated with ulcer of It has been held that where, "upon the face of the application, a question appears to
stomach. Tumor taken out was hard and of a hen's egg size. Operation was two years be not answered at all or to be imperfectly answered, and the insurers issue a policy
ago in Chinese General Hospital by Dr. Yap. Claims he is completely recovered.” without any further inquiry, they waive the imperfection of the answer and render the
Medical report show that insured was operated on for "peptic ulcer", involving the omission to answer more fully immaterial.
excision of a portion of the stomach, not tumor. Disposition the judgment appealed from is hereby affirmed, with costs against
appellant
ISSUE
WON there was concealment (Was appellant, because of insured's aforesaid
CANILANG v. CA (GREAT PACIFIC LIFE ASSURANCE CORP.)
representation, misled or deceived into entering the contract or in accepting the risk
at the rate of premium agreed upon?) 223 SCRA 443
FELICIANO; June 17, 1993
HELD
NO NATURE
-"concealment exists where the assured had knowledge of a fact material to the risk, Petition for review on certiorari of the decision of the Court of Appeals
and honesty, good faith, and fair dealing requires that he should communicate it to
the assurer, but he designedly and intentionally withholds the same." FACTS
- It has also been held "that the concealment must, in the absence of inquiries, be not - June 18, 1982 – Jaime Canilang was diagnosed by Dr. Claudio to have sinus
only material, but fraudulent, or the fact must have been intentionally withheld." tachycardia. He was directed by the doctor to take a tranquilizer (Trazepam) and a
Reasoning beta-blocker drug (Aptin).
1) The evidence shows that the Insular Life Assurance Co., Ltd. approved Kwong - August 3, 1982 – Jaime consulted Dr. Claudio again and was diagnosed to have acute
Nam's request for reinstatement and amendment of his lapsed insurance policy on bronchitis.
April 24, 1962…. It results, therefore, that when on May 12, 1962 Kwong Nam - August 4, 1982 – Jaime applied for a nonmedical insurance policy with Great Pacific
answered `No' to the question whether any life insurance company ever refused his Life Assurance Company. He named his wife Thelma as his beneficiary. He was issue
application for reinstatement of a lapsed policy he did not misrepresent any fact. the policy with a face value of P19,700 effective August 9, 1982.
8 - August 5, 1983 – Jaime died of congestive heart failure, anemia and chronic anemia.
2) Assuming that the aforesaid answer given by the insured is false, Sec. 27 of the
Insurance Law nevertheless requires that fraudulent intent on the part of the insured Thelma filed her claim but the insurance company refused to grant it on the ground
be established to entitle the insurer to rescind the contract. And as correctly observed that Jaime had concealed information.
by the lower court, "misrepresentation as a defense of the insurer to avoid liability is - Thelma filed a complaint against Great Pacific to recover the insurance proceeds.
an `affirmative’ defense. The duty to establish such a defense by satisfactory and She testified that she was not aware of her husband’s ailments and that she thought
convincing evidence rests upon the defendant. The evidence before the Court does he had died from a kidney disorder.
not clearly and satisfactorily establish that defense." - Great Pacific presented as witness Dr. Quismorio who testified that Jaime’s insurance
-Kwong Nam had informed the appellant's medical examiner that the tumor for which application was the basis of his medical declaration and she explained that an
he was operated on was ''associated with ulcer of the stomach." In the absence of applicant was required to undergo medical examination only if the applicant had
evidence that the insured had sufficient medical knowledge as to enable him to disclosed that he had previously been consulted with a doctor and had been
distinguish between "peptic ulcer" and "a tumor", his statement that said tumor was hospitalized.
"associated with ulcer of the stomach" should be construed as an expression made in - The Insurance Commissioner ordered Great Pacific to pay Thelma the insurance
proceeds, including attorney’s fees, holding that Jaime’s illness was not that serious as
8 to Great Pacific’s decision to insure him and that there was no concealment on the
"Sec. 27. Such party to a contract of insurance must communicate to the other, in good faith, all facts within his part of Jaime with regard to his illness.
knowledge which are material to the contract, and which the other has not the means of ascertaining, and as to Petitioners’ Claim:
which he makes no warranty."
> Thelma argues that the non-disclosure of Jaime did not amount to fraud.
iNsuRanCe A2010 Dean Carale
pAgE 58♥

> She also argues that the CA erred in not holding that the issue in the case agreed - Had Canilang disclosed his visits to his doctor, the diagnosis made and the medicines
upon between the parties before the Insurance Commission is whether or not Jaime prescribed by such doctor, in the insurance application, it may be reasonably assumed
'intentionally' made material concealment in stating his state of health; that Great Pacific would have made further inquiries and would have probably refused
Respondents’ Comments: to issue a non-medical insurance policy or, at the very least, required a higher
> The CA reversed the Insurance Commissioner’s decision, holding that the use of the premium for the same coverage.
word 'intentionally" by the Insurance Commissioner in defining and resolving the issue - As held in the case of Saturnino vs. Philippine-American Life Insurance, “the waiver
agreed upon by the parties at pre-trial before the Insurance Commissioner was not of medical examination in a non-medical insurance contract renders even more
supported by the evidence and that the issue agreed upon by the parties had been material the information inquired of the applicant concerning previous condition of
whether Jaime made a material concealment as to the state of his health at the time health and diseases suffered, for such information necessarily constitutes an
of the filing of insurance application, justifying the denial of the claim. important factor which the insurer takes into consideration in deciding whether to
> It also found that the failure of Jaime to disclose previous medical consultation and issue the policy or not.”
treatment constituted material information which should have been communicated to Disposition the Petition for Review is DENIED for lack of merit and the Decision of the
Great Pacific to enable the latter to make proper inquiries. Court of Appeals dated 16 October 1989 in C.A.-G.R. SP No. 08696 is hereby
AFFIRMED.
ISSUES
1. WON Jaime intentionally withheld information from Great Pacific YU PANG CHENG v. CA
2. WON the information withheld would have been material to Great Pacific’s decision
105 PHIL 930
to grant Jaime the insurance policy
BAUTISTA ANGELO; May 29, 1959
HELD
1. YES FACTS
Ratio Section 27 of the Insurance Code of 1978 is properly read as referring to "any - September 5, 1950: Yu Pang Eng submitted parts II and III of his application for
concealment without regard to whether such concealment is intentional or insurance consisting of the medical declaration made by him to the medical examiner
unintentional. The restoration in 1985 by B.P. Blg. 874 of the phrase "whether of defendant and the medical examiner's report
intentional or unintentional" merely underscored the fact that all throughout (from - September 7: he submitted part I of his application which is the declaration made by
1914 to 1985), the statute did not require proof that concealment must be him to an agent of defendant
"intentional" in order to authorize rescission by the injured party. - September 8: defendant issued to the insured Policy No. 812858
Reasoning - December 27, 1950: the insured entered St. Luke's Hospital for medical treatment
- Art. 27 of the 1978 Insurance Code reads that “a concealment entitles the injured but he died on February 27, 1951.
party to rescind a contract of insurance,” which does not include the words “whether - According to the death certificate, he died of "infiltrating medullary carcinoma,
intentional or unintentional” from the previous statutes. The Insurance Commissioner Grade 4, advanced cardiac and of lesser curvature, stomach metastases spleen."
relied on this deletion in arguing that the statute intended to limit the kinds of - Plaintiff, brother and beneficiary of the insured, demanded from defendant the
concealment which generate a right to rescind on the part of the injured party to payment of the proceeds of the insurance policy and when the demand was refused,
"intentional concealments." he brought the present action.
- In the case at bar, the nature of the facts not conveyed to the insurer was such that - The insured, in his application for insurance, particularly in his declarations to the
the failure to communicate must have been intentional rather than merely examining physician, stated the following in answering the questions propounded to
inadvertent. him:
> Jaime could not have been unaware that his heart beat would at times rise to high 14. Have you ever had any of the following diseases or symtoms? Each question must
and alarming levels and that he had consulted a doctor twice two months before be read and answered "Yes" or "No.".
applying for non-medical insurance. "Gastritis, Ulcer of the Stomach or any disease of that organ? No.
> The last medical consultation took place just the day before the insurance "Vertigo, Dizziness, Fainting-spells or Unconsciouness? No.
application was filed. "Cancer, Tumors or Ulcers of any kind? No.
2. YES - 15. Have you ever consulted any physician riot included in any of the above
Ratio Materiality relates rather to the "probable and reasonable influence of the answers? Give names and address or physicians list ailments or accidents and date.
facts" upon the party to whom the communication should have been made, in No."
assessing the risk involved in making or omitting to make further inquiries and in - It appears that the insured entered the Chinese General Hospital for medical
accepting the application for insurance; that "probable and reasonable influence of treatment on January 29, 1950 having stayed there up to February 11, 1950.
the farts" concealed must, of course, be determined objectively, by the judge - An X-ray picture of his stomach was taken and the diagnosis made of him by his
ultimately. doctors showed that his illness was "peptic ulcer, bleeding."
Reasoning
- The information which Jaime failed to disclose was material to the ability of Great ISSUE
Pacific to estimate the probable risk he presented as a subject of life insurance. WON the insured is guilty of concealment of some facts material to the risk insured
against which has the effect of avoiding the policy as found by respondent court.
iNsuRanCe A2010 Dean Carale
pAgE 59♥

PACIFIC BANKING CORP v. CA (ORIENTAL ASSURANCE


HELD
CORPORATION)
- It should be noted that the insured's confinement in the Chinese General Hospital
took place from January 29, 1950 to February 11, 1950, whereas his application for 168 SCRA 1
insurance wherein he stated his answers to the questions propounded to him by the PARAS; November 28, 1988
examining physician of defendant was submitted to defendant on September 5, 1950.
- It is apparent that when the insured gave his answers regarding his previous NATURE
ailment, particularly with regard to "Gastritis, Ulcer of the Stomach or any disease of Petition for review on certiorari of the CA decision, which set aside the decision of CFI
that organ" and "Vertigo, Dizziness, Fainting-spells or Unconsciousness", he concealed Manila, which had in turn granted the complaint for a sum of money in civil case filed
the ailment of which he was treated in the Chinese General Hospital which precisely by Pacific Banking against Oriental Assurance.
has direct connection with the subject of the questions propounded.
- The negative answers given by the insured regarding his previous ailment, or his FACTS
concealment of the fact that he was hospitalized and treated for sometime of peptic - October 21,1963: an open Fire Policy was issued to the Paramount Shirt
ulcer and had suffered from "dizziness, anemia, abdominal pains and tarry stools", Manufacturing Co. (insured), by which Oriental Assurance Corporation bound itself to
deprived defendant of the opportunity to make the necessary inquiry as to the nature indemnify the insured for any loss or damage, not exceeding P61,000.00, caused by
of his past illness so that it may form its estimate relative to the approval of his fire to its property consisting of stocks, materials and supplies usual to a shirt factory,
application. including furniture, fixtures, machinery and equipment while contained in the ground,
- Had defendant been given such opportunity, considering the previous illness of the second and third floors of the building situated at number 256 Jaboneros St., San
insured as disclosed by the records of the Chinese General Hospital, defendant would Nicolas, Manila, for a period of one year commencing from that date to October 21,
probably had never consented to the issuance of the policy in question. In fact, 1964.
according to the death certificate, the insured died of "infiltrating medullary - Insured was at the time of the issuance of the policy and is up to this time, a debtor
carcinoma, Grade, 4, advanced cardiac and of lesser curvature, stomach metastases of Pacific Banking in the amount of not less P800,000.00 and the goods described in
spleen", which may have a direct connection with his previous illness. the policy were held in trust by the insured for the Pacific Banking under thrust
- Our Insurance Law provides that "A neglect to communicate that which a party receipts.
knows and ought to communicate, is called concealment" (Section 25, Act No. 2427). - Said policy was duly endorsed to Pacific Banking as mortgagee/trustor of the
Whether intentional or unintentional, the concealment entitles the insurer to rescind properties insured, with the knowledge and consent of Oriental Assurance to the
the contract of insurance (Section 26). effect that "loss if any under this policy is payable to the Pacific Banking Corporation".
- Our law even requires the insured to communicate to the insurer all facts within his - While the aforesaid policy was in full force and effect, a fire broke out on the subject
knowledge which are material to the contract and which the other party has not the premises destroying the goods contained in its ground and second floors. Counsel for
means of ascertaining (Section 27), and the materiality is to be determined not by the the Pacific Banking sent a letter of demand to Oriental Assurance for indemnity due to
event but solely by the probable and reasonable influence of the facts upon the party the loss of property by fire. Oriental Assurance informed counsel that it was not yet
to whom the communication is due (Section 30). ready to accede to the latter's demand as the former is awaiting the final report of the
- Argente vs. West Coast Life Insurance Co.: "One ground for the rescission of a insurance adjuster, H.H. Bayne Adjustment Company.
contract of insurance under the Insurance Act is 'a concealment', which in section 25 - Said insurance adjuster notified counsel for the Pacific Banking that the insured
is defined 'A neglect to communicate that which a party knows and ought to under the policy had not filed any claim with it, nor submitted proof of loss which is a
communicate.' Appellant argues that the concealment was immaterial and insufficient clear violation of Policy Condition No.11, and for which reason, determination of the
to avoid the policy. We cannot agree. In an action on a life insurance policy where the liability of Oriental Assurance could not be had. Pacific Banking's counsel replied
evidence conclusively shows that the answers to questions concerning diseases were asking the insurance adjuster to verify from the records of the Bureau of Customs the
untrue, the truth or falsity of the answers become the determining factor. If the policy entries of merchandise taken into the customs bonded warehouse razed by fire as a
was procured by fraudulent representations, the contract of insurance apparently set reliable proof of loss.
forth therein was never legally existent. It can fairly be assumed that had the true - For failure of the insurance company to pay the loss as demanded, Pacific Banking
facts been disclosed by the assured, the insurance would never have been granted." field before CFI an action for a sum of money against the Oriental Assurance, in the
Disposition Decision affirmed. principal sum of P61,000.00 issued in favor of Paramount Shirt Manufacturing Co.
Oriental Assurance defenses
(a) lack of formal claim by insured over the loss and
(b) premature filing of the suit as neither plaintiff nor insured had submitted any proof
of loss on the basis of which defendant would determine its liability and the amount
thereof, either to the Oriental Assurance or its adjuster H.H. Bayne Adjustment Co.
Pacific Banking
> presented evidence that insured has undeclared co-insurances with the following:
P30,000.00 with Wellington Insurance; P25,000. 00 with Empire Surety and
GREAT PACIFIC LIFE v. CA (supra p.34)
P250,000.00 with Asian Surety; undertaken by insured Paramount on the same
property covered by its policy with Oriental Assurance whereas the only co-insurances
iNsuRanCe A2010 Dean Carale
pAgE 60♥

declared in the subject policy are those of P30,000.00 with Malayan, P50,000.00 with 2. NO
South Sea, and P25.000.00 with Victory - Subject mortgage clause pecifically provides: “Loss, if any, under this policy, shall be
- NOTE: the defense of fraud and/or violation of non-declaration of co-insurances was payable to the PACIFIC BANKING CORPORATION Manila mortgagee/trustor as its
not pleaded in the answer, also not pleaded in the Motion to Dismiss. interest may appear, it being hereby understood and agreed that this insurance as to
- CFI denied Oriental Assurance's motion on the ground that since the defense was the interest of the mortgagee/trustor only herein, shall not be invalidated by any act
raised for the first time, it must be deemed to have waived the requirement of proof of or neglect except fraud or misrepresentation, or arson of the mortgagor or
loss. Case was submitted for decision. But upon MR, Oriental Asurance was allowed to owner/trustee of the property insured; provided, that in case the mortgagor or owner/
present additional evidence, "in order to prove that 'insured has committed a violation trustee neglects or refuses to pay any premium, the mortgagee/ trustor shall, on
of condition No. 3 of the policy in relation to the other Insurance Clause.' " CFI demand pay the same.”
eventually adjudged Oriental Assurance liable to the Pacific Banking under the said - The paragraph clearly states the exceptions to the general rule that insurance as to
contract of insurance. the interest of the mortgagee, cannot be invalidated; namely: fraud, or
- Court of Appeals reversed. Pacific Banking's MR denied. misrepresentation or arson.
- Concealment of the aforecited co-insurances can easily be fraud, or in the very least,
ISSUES misrepresentation. It is but fair and just that where the insured who is primarily
1. WON insured is guilty of fraud entitled to receive the proceeds of the policy has by its fraud and/or
2. WON mortgagee/assignee can still claim from the insurance misrepresentation, forfeited said right, with more reason Pacific Banking which is
merely claiming as indorsee of said insured, cannot be entitled to such proceeds.
HELD - The fact of fraud was tried by express or at least implied consent of the parties.
1. YES Pacific Banking did not only object to the introduction of evidence but on the contrary,
- The crux of the controversy centers on two points: (a) unrevealed co-insurances presented the very evidence that proved its existence.
which violated policy conditions No. 3; and (b) failure of the insured to file the required - Be that as it may, SC has ample authority to give beyond the pleadings where in the
proof of loss prior to court action. interest of justice and the promotion of public policy, there is a need to make its own
- Policy Condition No. 3 explicitly provides: “The Insured shall give notice to the finding to support its conclusion. Otherwise stated, the Court can consider a fact which
Company of any insurance already effected, or which may subsequently be effected, surfaced only after trial proper.
covering any of the property hereby insured, and unless such notice be given and the - Generally, the cause of action on the policy accrues when the loss occurs, but when
particulars of such insurance or insurances be stated in or endorsed on this Policy by the policy provides that no action shall be brought unless the claim is first presented
or on behalf of the Company before the occurrence of any loss or damage, all benefit extrajudicially in the manner provided in the policy, the cause of action will accrue
under this policy shall be forfeited.” from the time the insurer finally rejects the claim for payment.
- It is not disputed that the insured failed to reveal before the loss three other - In the case at bar, policy condition No. 11 specifically provides that the insured shall
insurances. By reason of said unrevealed insurances, the insured had been guilty of a on the happening of any loss or damage give notice to the company and shall within
false declaration; a clear misrepresentation and a vital one because where the insured fifteen (15) days after such loss or damage deliver to the Oriental Assurance (a) a
had been asked to reveal but did not, that was deception. Otherwise stated, had the claim in writing giving particular account as to the articles or goods destroyed and the
insurer known that there were many co-insurances, it could have hesitated or plainly amount of the loss or damage and (b) particulars of all other insurances, if any.
desisted from entering into such contract. Hence, the insured was guilty of clear fraud. Likewise, insured was required "at his own expense to produce, procure and give to
- Pacific Banking's contention that the allegation of fraud is but a mere inference or the company all such further particulars, plans, specifications, books, vouchers,
suspicion is untenable. Concrete evidence of fraud or false declaration by the insured invoices, duplicates or copies thereof, documents, proofs and information with respect
was furnished by the Pacific Banking itself when the facts alleged in the policy under to the claim".
clauses "Co-Insurances Declared" and "Other Insurance Clause" are materially - Evidence adduced shows that 24 days after the fire, Pacific Banking merely wrote
different from the actual number of co-insurances taken over the subject property. letters to Oriental Assurance to serve as a notice of loss, thereafter, the former did not
Consequently, the whole foundation of the contract fails, the risk does not attach and furnish the latter whatever pertinent documents were necessary to prove and
the policy never becomes a contract between the parties. Representations of facts are estimate its loss. Instead, Pacific Banking shifted upon Oriental Assurance the burden
the foundation of the contract and if the foundation does not exist, the superstructure of fishing out the necessary information to ascertain the particular account of the
does not arise. Falsehood in such representations is not shown to vary or add to the articles destroyed by fire as well as the amount of loss.
contract, or to terminate a contract which has once been made, but to show that no - Oriental Assurance and its adjuster notified Pacific Banking that insured had not yet
contract has ever existed (Tolentino). A void or inexistent contract is one which has no filed a written claim nor submitted the supporting documents in compliance with the
force and effect from the very beginning, as if it had never been entered into, and requirements set forth in the policy. Despite the notice, the latter remained unheedful.
which cannot be validated either by time or by ratification. Since the required claim by insured, together with the preliminary submittal of
- As the insurance policy against fire expressly required that notice should be given by relevant documents had not been complied with, it follows that Oriental Assurance
the insured of other insurance upon the same property, the total absence of such could not be deemed to have finally rejected Pacific Banking's claim and therefore the
notice nullifies the policy. latter's cause of action had not yet arisen. Compliance with condition No. 11 is a
- Argument that notice of co-insurances may be made orally is preposterous and requirement sine qua non to the right to maintain an action as prior thereto no
negates policy condition No. 20 which requires every notice and other violation of Pacific Banking's right can be attributable to Oriental Assurance. As before
communications to the insurer to be written or printed. such final rejection, there was no real necessity for bringing suit. Pacific Banking
iNsuRanCe A2010 Dean Carale
pAgE 61♥

should have endeavored to file the formal claim and procure all the documents, Petitioner’s Claim
papers, inventory needed by Oriental Assurance or its adjuster to ascertain the > The insured did not disclose facts relevant to the issuance of the policy, thus
amount of loss and after compliance await the final rejection of its claim. Indeed, the rescission of the contract may be invoked by the insurance company.
law does not encourage unnecessary litigation. Respondents’ Comments
- Pacific Banking prematurely filed the civil case and dismissal thereof was warranted > The actual cause of death was not relevant to the concealed information, and the
under the circumstances. While it is a cardinal principle of insurance law that a policy policy was entered into by the insured in good faith.
or contract of insurance is to be construed liberally in favor of the insured and strictly
as against the insurer company yet, contracts of insurance, like other contracts, are to ISSUE
be construed according to the sense and meaning of the terms which the parties WON the concealment renders the insurance policy rescissible
themselves have used. If such terms are clear and unambiguous, they must be taken
and understood in their plain, ordinary and popular sense. HELD
- Contracts of insurance are contracts of indemnity upon the terms and conditions YES
specified in the policy. The parties have a right to impose such reasonable conditions Ratio The terms of the contract are clear. The insured is specifically required to
at the time of the making of the contract as they may deem wise and necessary. The disclose to the insurer matters relating to his health.
agreement has the force of law between the parties. The terms of the policy Reasoning
constitute the measure of the insurer's liability, and in order to recover, the insured SEC. 26 (IC)
must show himself within those terms. The compliance of the insured with the terms A neglect to communicate that which a party knows and ought to communicate,
of the policy is a condition precedent to the light of recovery. is called a concealment.
- It appearing that insured has violated or failed to perform the conditions under No. 3 SEC. 31 (IC)
and 11 of the contract, and such violation or want of performance has not been Materiality is to be determined not by the event, but solely by the probable and
waived by the insurer, the insured cannot recover, much less the herein Pacific reasonable influence of the facts upon the party to whom communication is due, in
Banking. Courts are not permitted to make contracts for the parties; the function and forming his estimate of the disadvantages of the proposed contract or in making his
duty of the courts is simply to enforce and carry out the contracts actually made. inquiries
Disposition Petition dismissed. CA affirmed. - The information which the insured failed to disclose was material and relevant to the
approval and the issuance of the insurance policy. The matters concealed would have
SUNLIFE ASSURANCE COMPANY v. CA (SPS. BACANI) definitely affected petitioner's action on his application, either by approving it with the
corresponding adjustment for a higher premium or rejecting the same.
245 SCRA 268
- Good faith is no defense in concealment. It appears that such concealment was
QUIASON; June 22, 1995 deliberate on the part of the insured.
- The waiver of a medical examination [in a non-medical insurance contract] renders
NATURE even more material the information required of the applicant concerning previous
A petition for review on certiorari. condition of health and diseases suffered, for such information necessarily constitutes
an important factor which the insurer takes into consideration in deciding whether to
FACTS issue the policy or not.
- April 15, 1986: Robert John B. Bacani procured a life insurance contract for himself - Anent the finding that the facts concealed had no bearing to the cause of death of
from SUNLIFE (petitioner) valued at P100K. The designated beneficiary was his the insured, it is well settled that the insured need not die of the disease he had failed
mother, Bernarda Bacani (respondent). to disclose to the insurer. It is sufficient that his non-disclosure misled the insurer in
- June 26, 1987: the insured died in a plane crash. Bernarda Bacani filed a claim with forming his estimates of the risks of the proposed insurance policy or in making
Sunlife, seeking the benefits of the insurance policy taken by her son. Petitioner inquiries
conducted an investigation and its findings prompted it to reject the claim on the Disposition Petition is granted and the decision of CA is reversed and set aside.
ground that the insured did not disclose facts material to the issuance of the policy.
The insured gave false statements in the application when he answered in the
EGUARAS v. GREAT EASTERN
negative to the question “have you ever had or sought advice for urine, kidney,
bladder disorder?” 33 PHIL. 263
- Sunlife discovered that two weeks prior to the issuance, insured was diagnosed with TORRES.; January 24, 1916
renal failure, was confined, and underwent tests.
- November 17, 1988: Bacani and her husband filed for specific performance against NATURE
Sunlife. RTC granted the plea on the ground that that the facts concealed by the Appeal filed through bill of exceptions from the judgment of the CFI
insured were made in good faith and under the belief that they need not be disclosed,
and that the disclosure was not material since the policy was non-medical. FACTS
- Sunlife appealed to the CA, but the latter denied the appeal on the ground that the - Francisca Eguaras filed a written complaint in court, alleging as a cause of action
cause of death was unrelated to the facts concealed by the insured. that her son-in-law Dominador Albay had applied in writing to the defendant insurance
company to insure his life for the sum of P5,000, naming as the beneficiary in case of
iNsuRanCe A2010 Dean Carale
pAgE 62♥

his death the plaintiff Francisca Eguaras; that after compliance with the requisites and - Qua Chee Gan informed the insurance company of the fire. Fire adjusters of the
the investigation carried on by the defendant company, it accepted the application for company conducted an extensive investigation. Qua Chee Gan submitted the
insurance and issued the policy; that, said policy being in force, the insured died, and corresponding fire claims, totaling P398,562.81 (but reduced to the full amount of the
despite the fact that the beneficiary submitted satisfactory proofs of his death and insurance, P370,000), the Insurance Company resisted payment, claiming violation of
that the defendant company investigated the event, still it refused and continues to warranties and conditions, filing of fraudulent claims, and that the fire had been
refuse to pay to the plaintiff the value of the policy. deliberately caused by the insured or by other persons in connivance with him.
- Defendant set forth in special defense that the insurance policy issued in the name - Qua Chee Gan, his brother and his employees were tried for arson, where counsel of
of Dominador Albay had been obtained through fraud and deceit known and the insurance company acted as a private prosecutor. They were acquitted.
consented to by the interested parties and is therefore completely illegal, void, and - This civil suit was then instituted to claim against the insurance company. The CFI
ineffective. ruled in favor of Qua Chee Gan and ordered Law Union Rock Co. to pay.
- A criminal case for frustrated estafa was filed by defendant against Ponciano
Remigio, Castor Garcia and Francisca Eguaras. They were acquitted, and claim that ISSUES
the judgment produces the effect of res judicata in the present suit. 1. WON there was a breach of the fire hydrant warranty
2. WON the insured violated the Hemp warranty
ISSUE 3. WON Qua Chee Gan is guilty of overvaluation
WON the life insurance obtained by Dominador Albay was issued through fraud and 4. WON Qua Chee Gan caused the fire
deceit 5. WON there was an error in the amount of copra and hemp lost
6. WON the claims contained false and fraudulent statements
HELD
YES HELD
Ratio In a contract where one of the contracting parties may have given his consent 1. NO
through error, violence, intimidation, or deceit, and in any of such cases the contract - It is argued that he should have 11 fire hydrants in the compound, but he only had 2.
is void, even though, despite this nullity, no crime was committed. There may not We are in agreement with the trial Court that the appellant is barred by waiver (or
have been estafa in the case at bar, but it was conclusively demonstrated by the trial rather estoppel) to claim violation of the so-called fire hydrants warranty, for the
that deceit entered into the insurance contract, fulfillment whereof is claimed, and reason that knowing fully all that the number of hydrants demanded therein never
therefore the conclusions reached by the court in the judgment it rendered in the existed from the very beginning, the appellant nevertheless issued the policies in
criminal proceedings for estafa do not affect this suit, nor can they produce in the question subject to such warranty, and received the corresponding premiums.
present suit the force of res adjudicata. 2. NO
Reasoning - The insurance company avers that the insured violated the hemp warranty when it
- It is proven that the signatures on the insurance applications reading "Dominado admitted that it had 36 cans of gasoline in the building. It is well to note that gasoline
Albay" are false and forged; that the person who presented himself to Dr. Vidal to be is not specifically mentioned among the prohibited articles listed in the so-called
examined was not the real Dominador Albay, but Castor Garcia who was positively "hemp warranty." The cause relied upon by the insurer speaks of "oils (animal and/or
identified by Dr. Vidal; that at the time of the application for insurance and the vegetable and/or mineral and/or their liquid products having a flash point below 300o
issuance of the policy which is the subject matter of this suit the real Dominador Albay Fahrenheit", and is decidedly ambiguous and uncertain; for in ordinary parlance, "Oils"
was informed of all those machinations, wherefore it is plain that the insurance mean "lubricants" and not gasoline or kerosene. And how many insured, it may well
contract between the defendant and Dominador Albay is null and void because it is be wondered, are in a position to understand or determine "flash point below 003o
false, fraudulent and illegal. Fahrenheit. Here, again, by reason of the exclusive control of the insurance company
Disposition The judgment appealed from is reversed and the defendant absolved over the terms and phraseology of the contract, the ambiguity must be held strictly
from the complaint without special finding as to the costs. against the insurer and liberally in favor of the insured, especially to avoid a forfeiture
- Another point that is in favor of the insured is that the gasoline kept in Bodega No. 2
QUA CHEE GAN v. LAW UNION AND ROCK was only incidental to his business, being no more than a customary 2 day's supply for
the five or six motor vehicles used for transporting of the stored merchandise). "It is
98 PHIL 85
well settled that the keeping of inflammable oils on the premises though prohibited by
REYES; December 17, 1955 the policy does not void it if such keeping is incidental to the business." (Bachrach vs.
British American Ass. Co., 17 Phil. 555, 560)
FACTS 3. NO
- Qua Chee Gan insured 4 of his bodegas with Law Union & Rock Insurance Co in 1937. - The charge that the insured failed or refused to submit to the examiners of the
These bodegas were used for the storage of stocks of copra and of hemp, baled and insurer the books, vouchers, etc. demanded by them was found unsubstantiated by
loose. the trial Court, and no reason has been shown to alter this finding.
- Fire of undetermined origin that broke out in the early morning of July 21, 1940, and - In view of the discrepancy in the valuations between the insured and the adjuster
lasted almost one week, gutted and completely destroyed Bodegas Nos. 1, 2 and 4, Stewart for the insurer, the Court referred the controversy to a government auditor,
with the merchandise stored therein. Apolonio Ramos; but the latter reached a different result from the other two. Not only
that, but Ramos reported two different valuations that could be reached according to
iNsuRanCe A2010 Dean Carale
pAgE 63♥

the methods employed. Clearly then, the charge of fraudulent overvaluation cannot cent) of the hemp stock; had the insured acted with fraudulent intent, nothing
be seriously entertained. prevented him from increasing the value of all of his copra, hemp and buildings in the
4. NO same proportion. This also applies to the alleged fraudulent claim for burned empty
- This defense is predicted on the assumption that the insured was in financial sacks, that was likewise explained to our satisfaction and that of the trial Court. The
difficulties and set the fire to defraud the insurance company, presumably in order to rule is that to avoid a policy, the false swearing must be willful and with intent to
pay off the Philippine National Bank, to which most of the insured hemp and copra defraud which was not the cause. Of course, the lack of fraudulent intent would not
was pledged. This defense is fatally undermined by the established fact that, authorize the collection of the expected profit under the terms of the polices, and the
notwithstanding the insurer's refusal to pay the value of the policies the extensive trial Court correctly deducted the same from its award.
resources of the insured enabled him to pay off the National Bank in a short time; and Disposition Decision affirmed
if he was able to do so, no motive appears for attempt to defraud the insurer. While
the acquittal of the insured in the arson case is not res judicata on the present civil ARGENTE v. WEST COAST LIFE
action, the insurer's evidence, to judge from the decision in the criminal case, is
51 PHIL 725
practically identical in both cases and must lead to the same result, since the proof to
establish the defense of connivance at the fire in order to defraud the insurer "cannot MALCOLM; March 19, 1928
be materially less convincing than that required in order to convict the insured of the
crime of arson. FACTS
5. NO - This is an action upon a joint life insurance policy for P15,000 issued by the West
- As to the defense that the burned bodegas could not possibly have contained the Coast Life Insurance Co., on May 15, 1925, in favor of Bernardo Argente, and his wife,
quantities of copra and hemp stated in the fire claims, the insurer's case rests almost Vicenta de Ocampo, the latter having died on November 18, 1925. Fraud in obtaining
exclusively on the estimates, inferences and conclusions of its adjuster investigator, the policy was pleaded by way of special defense. On the issue thus suggested, the
Alexander D. Stewart, who examined the premises during and after the fire. His court adopted the theory of the defendant, and held the insurance policy null and
testimony, however, was based on inferences from the photographs and traces found void, with the result that the complaint was dismissed, with costs.
after the fire, and must yield to the contradictory testimony of engineer Andres -Bernardo Argente signed an application for joint insurance with his wife in the sum of
Bolinas, and specially of the then Chief of the Loan Department of the National Bank's P2,000. The wife, Vicenta de Ocampo, signed a like application for the same policy.
Legaspi branch, Porfirio Barrios, and of Bank Appraiser Loreto Samson, who actually - Bernardo Argente and his wife was examined by Dr. Cesareo Sta. Ana, a medical
saw the contents of the bodegas shortly before the fire, while inspecting them for the examiner for the West Coast Life Insurance Co. which did not show previous and
mortgagee Bank existing health problems.
6. NO - A temporary policy for P15,000 was issued to Bernardo Argente and his wife as of
- Appellant insurance company also contends that the claims filed by the insured May 15, 1925. In view of the fact that more than thirty days had elapsed since the
contained false and fraudulent statements that avoided the insurance policy. But the applicants were examined by the company's physician, each of them was required to
trial Court found that the discrepancies were a result of the insured's erroneous file a certificate of health before the policy was delivered to them.
interpretation of the provisions of the insurance policies and claim forms, caused by - On November 18, 1925, Vicenta de Ocampo died of cerebral apoplexy. Thereafter
his imperfect knowledge of English, and that the misstatements were innocently made Bernardo Argente presented a claim. Following investigation conducted by the
and without intent to defraud. The trial court’s ruling must be upheld. Manager of the Manila office of the insurance company, it was apparently disclosed
- For example, the occurrence of previous fires in the premises insured in 1939, altho that the answers given by the insured in their medical examinations with regard to
omitted in the claims, Exhibits EE and FF, were nevertheless revealed by the insured their health and previous illnesses and medical attendance were untrue. West Coast
in his claims Exhibits Q (filed simultaneously with them), KK and WW. Considering that Life Insurance Co. refused to pay the claim of Bernardo Argente, and wrote him to the
all these claims were submitted to the smae agent, and that this same agent had paid effect that the claim was rejected because the insurance was obtained through fraud
the loss caused by the 1939 fire, we find no error in the trial Court's acceptance of the and misrepresentation.
insured's explanation that the omission in Exhibits EE and FF was due to inadvertance, - It is admitted that it appears in the Medical Examiner's Report that Bernardo Argente
for the insured could hardly expect under such circumstances, that the 1939 would gave false responses. As well as with the Medical Examiner's Report that Vicenta de
pass unnoticed by the insurance agents. Similarly, the 20 per cent overclaim on 70 Ocampo. It is, however, not disputed that Vicenta de Ocampo was taken by a
per cent of the hemo stock, was explained by the insured as caused by his belief that patrolman, at the request of her husband, Bernardo Argente, on May 19, 1924, to the
he was entitled to include in the claim his expected profit on the 70 per cent of the Meisic police station, and from there was transferred to the San Lazaro Hospital. In
hemp, because the same was already contracted for and sold to other parties before San Lazaro Hospital, her case was diagnosed by the admitting physician as
the fire occurred. Compared with other cases of over-valuation recorded in our judicial "alcoholism," but later Doctor Domingo made a diagnosis of probable "manic-
annals, the 20 per cent excess in the case of the insured is not by itself sufficient to depressive psychosis," and still, later in Mary Chiles Hospital, made a final diagnosis of
establish fraudulent intent. Certainly, the insured's overclaim of 20 per cent in the "phycho-neurosis."
case at bar, duly explained by him to the Court a quo, appears puny by comparison - Bernardo Argente, while readily conceding most of the facts herein narrated, yet
(compared to other cases cited by the court), and can not be regarded as "more than alleges that both he and his wife revealed to the company's physician, Doctor Sta.
misstatement, more than inadvertence of mistake, more than a mere error in opinion, Ana, all the facts concerning their previous illnesses and medical attendance, but that
more than a slight exaggeration" that would entitle the insurer to avoid the policy. It is Doctor Sta. Ana, presumably acting in collusion with the insurance agent, Jose
well to note that the overcharge of 20 per cent was claimed only on a part (70 per Geronimo del Rosario, failed to record them in the medical reports. The evidence on
iNsuRanCe A2010 Dean Carale
pAgE 64♥

these points consists of the testimony of the plaintiff and his subordinate clerk, "If the assured has exclusive knowledge of material facts, he should fully and fairly
Apolonio Espiritu, on the one hand, and of the testimony of Doctor Sta. Ana and Jose disclose the same, whether he believes them material or not. But notwithstanding
Geronimo del Rosario on the other. This was rejected by the Trial Court. Trial judge this general rule it will not infrequently happen, especially in life risks, that the
found with the insurance company with regard to the question of fact. SC agrees. assured may have a knowledge actual or presumed of material facts, and yet
There appears no motive whatever on the part of Doctor Sta. Ana to falsify the entertain an honest belief that they are not material. . . . The determination of the
Medical Examiner's Reports and thereby not only jeopardize his career as a physician, point whether there has or has not been a material concealment must rest largely in
but also gravely implicate himself criminally. all cases upon the form of the questions propounded and the exact terms of the
contract. Thus, where in addition to specifically named diseases the insured was
ISSUE asked whether he had had any sickness within ten years, to which he answered
WON the contract of insurance may be rescinded 'No,' and it was proven that within that period he had had a slight attack of
pharyngitis, it was held a question properly for the jury whether such an
inflammation of the throat was a 'sickness' within the intent of the inquiry, and the
court remarked on the appeal decision that if it could be held as a matter of law
HELD that the policy was thereby avoided, then it was a mere device on the part of
YES insurance companies to obtain money without rendering themselves liable under
- Bernardo Argente and his wife applications’ were false with respect to their state of the policy. . . .
health during the period of five years preceding the date of such applications and that " . . . The question should be left to the jury whether the assured truly represented
they knew the representations made by them in their applications were false. The the state of his health so as not to mislead or deceive the insurer; and if he did not
question arises as to the state of the law in relation thereto. deal in good faith with the insurer in that matter, then the inquiry should be made,
- One ground for the rescission of a contract of insurance under the Insurance Act is "a Did he know the state of his health so as to be able to furnish a proper answer to
concealment," which in section 25 is defined as "A neglect to communicate that which such questions as are propounded? A Massachusetts case, if construed as it is
a party knows and ought to communicate." In an action on a life insurance policy frequently cited, would be opposed to the above conclusion; but, on the contrary, it
where the evidence conclusively shows that the answers to questions concerning sustains it, for the reason that symptoms of consumption had so far developed
diseases were untrue, the truth or falsity of the answers become the determining themselves within a few months prior to effecting the insurance as to induce a
factor. If the policy was procured by fraudulent representations, the contract of reasonable belief that the applicant had that fatal disease, and we should further
insurance apparently set forth therein was never legally existent. It can fairly be construe this case as establishing the rule that such a matter cannot rest alone
assumed that had the true facts been disclosed by the assured, the insurance would upon the assured's belief irrespective of what is a reasonable belief, but that it
never have been granted. ought to be judged by the criterion whether the belief is one fairly warranted by the
- In Joyce, The Law of Insurance, second edition, volume 3, Chapter LV, is found the circumstances. A case in Indiana, however, holds that if the assured has some
following: affection or ailment of one or more of the organs inquired about so well defined and
"The basis of the rule vitiating the contract in cases of concealment is that it marked as to materially derange for a time the functions of such organ, as in the
misleads or deceives the insurer into accepting the risk, or accepting it at the rate case of Bright's disease, the policy will be avoided by a nondisclosure, irrespective
of premium agreed upon; The insurer, relying upon the belief that the assured will of the fact whether the assured knew of such ailment or not. . . ."
disclose every material fact within his actual or presumed knowledge, is misled into - Lastly, appellant contends that even if the insurance company had a right to rescind
a belief that the circumstance withheld does not exist, and he is thereby induced to the contract, such right cannot now be enforced in view of the provisions of section 47
estimate the risk upon a false basis that it does not exist. The principal question, of the Insurance Act providing "Whenever a right to rescind a contract of insurance is
therefore, must be, Was the assurer misled or deceived into entering a contract given to the insurer by any provision of this chapter, such right must be exercised
obligation or in fixing the premium of insurance by a withholding of material previous to the commencement of an action on the contract." This section was
information or facts within the assured's knowledge or presumed knowledge? derived from section 2583 of the California Civil Code, but in contrast thereto, makes
"It therefore follows that the assurer in assuming a risk is entitled to know every use of the imperative "must" instead of the permissive "may." Nevertheless, there are
material fact of which the assured has exclusive or peculiar knowledge, as well as two answers to the problem as propounded. The first is that the California law as
all material facts which directly tend to increase the hazard or risk which are known construed by the code examiners, at whose recommendation it was adopted,
by the assured, or which ought to be or are presumed to be known by him. And a conceded that "A failure to exercise the right (of rescission), cannot, of course,
concealment of such facts vitiates the policy. 'It does not seem to be necessary . . . prejudice any defense to the action which the concealment may furnish." (Codes of
that the . . . suppression of the truth should have been willful.' If it were but an California Annotated; Tan Chay Heng vs. West Coast Life Insurance Company [1927],
inadvertent omission, yet if it were material to the risk and such as the plaintiff p. 80, ante.) The second answer is that the insurance company more than one month
should have known to be so, it would render the policy void. But it is held that if previous to the commencement of the present action wrote the plaintiff and informed
untrue or false answers are given in response to inquiries and they relate to him that the insurance contract was void because it had been procured through
material facts the policy is avoided without regard to the knowledge or fraud of fraudulent representations, and offered to refund to the plaintiff the premium which
assured, although under the statute statements are representations which must be the latter had paid upon the return of the policy for cancellation. As held in California
fraudulent to avoid the policy. So under certain codes the important inquiries are as to a fire insurance policy, where any of the material representations are false, the
whether the concealment was willful and related to a matter material to the risk. insurer's tender of the premium and notice that the policy is canceled, before the
xxx xxx xxx
iNsuRanCe A2010 Dean Carale
pAgE 65♥

commencement of suit thereon, operate to rescind the contract of insurance. (Rankin the principal approves the risk and a receipt is given by the agent; because private
vs. Amazon Insurance Co. [1891], 89 Cal., 203.) respondent failed to accept Pacific Life’s offer for the Juvenile Triple Action plan, there
Disposition Judgment affirmed, with the costs of this instance against the appellant. was no meeting of the minds and thus no contract. Also, being an authorized agent of
Pacific Life, Ngo Hing must have known the company did not offer the insurance
GREAT PACIFIC LIFE v. CA (NGO HING) applied for and merely took a chance on Mondragon’s recommendation.
Disposition the decision appealed from is set aside, absolving Pacific Life from their
89 SCRA 543
civil liabilities
DE CASTRO, J; April 30, 1979
EDILLON v. MANILA BANKERS LIFE
NATURE
Petition for certiorari 117 SCRA 187
VASQUEZ; September 30, 1982
FACTS
- On March 14, 1957, private respondent Ngo Hing filed an application with the Great NATURE
Pacific Life Assurance Co. (Pacific Life) for a 20 year endowment policy of P50k on the Appeal from a decision of the CFI
life of his 1 year old daughter, Helen. Ngo Hing supplied the essetntial data which
petitioner Mondragon, branch manager of the Pacific Life in Cebu, wrote on the FACTS
corresponding form in his own handwriting, later typing the data on an application - Sometime in April 1969, Carmen O, Lapuz applied with respondent insurance
form signed by Ngo Hing. The latter paid the P1077.75 annual premium but retained corporation for insurance coverage against accident and injuries. In the application
P1,317 as commission as he was also a duly authorized agent of Pacific Life. The form which was dated April 15, 1969, she gave the date of her birth as July 11, 1904.
binding deposit receipt was then issued to Ngo Hing; Mondragon handwrote his strong On the same date, she paid the sum of P20.00 representing the premium for which
recommendation for the approval of the application on the back of the form. she was issued the corresponding receipt signed by an authorized agent of the
- On April 30, Mondragon received a letter from Pacific Life which stated that the 20 respondent insurance corporation. Upon the filing of said application and the payment
year endowment plan was not available for minors below 7, but that Pacific Life could of the premium on the policy applied for, the respondent insurance corporation issued
consider the same under the Juvenile Triple Action Plan, advising that if the offer was to Carmen O. Lapuz its Certificate of Insurance. The policy was to be effective for a
acceptable, the Juvenile Non-Medical Declaration be sent to the company. period of 90 days.
-Mondragon allegedly failed to inform Ngo Hing of the non-acceptance of the - On May 31, 1969 or during the effectivity of the Insurance, Carmen O. Lapuz died in
insurance plan, instead writing Pacific Life again, recommending the approval of the a vehicular accident.
endowment plan to children since customers had been asking for such coverage since - On June 7, 1969, petitioner Regina L. Edillon, a sister of the insured and who was the
1954. named beneficiary in the policy, filed her claim for the proceeds of the insurance,
-On May 28, 1957, Helen died of influenza. Ngo Hing sought the payment of the submitting all the necessary papers and other requisites with the private respondent.
proceeds of the insurance, but having failed to do so, filed an action for recovery with Her claim having been denied, Regina L. Edillon instituted this action in the Court of
the CFI of Cebu. The Court ordered Pacific Life to pay P50k with 6% interest, hence First Instance of Rizal.
this petition. - In resisting the claim of the petitioner, the respondent insurance corporation relies
on a provision contained in the Certificate of Insurance, excluding its liability to pay
ISSUE claims under the policy in behalf of "persons who are under the age of sixteen (16)
WON the binding deposit receipt constituted a temporary contract of the life insurance years of age or over the age of sixty (60) years ..." It is pointed out that the insured
in question being over sixty (60) years of age when she applied for the insurance coverage, the
policy was null and void, and no risk on the part of the respondent insurance
HELD corporation had arisen therefrom.
NO - RTC dismissed the complaint.
- The binding deposit receipt is merely a provisional contract and only upon
compliance with the ff conditions: (1) that the company be satisfied that the applicant ISSUE
was insurable on standard rates (2) that if the company does not accept the WON the acceptance by the private respondent insurance corporation of the premium
application and offers a different policy, the insurance contract shall not be binding and the issuance of the corresponding certificate of insurance should be deemed a
until the applicant accepts the new policy (3) that if the applicant is not found to be waiver of the exclusionary condition of overage stated in the said certificate of
insurable on standard rates and the application is disapproved, the insurance shall not insurance
be in force at any time and the premium be returned to the applicant.
-This implies the receipt is merely an acknowledgement, on behalf of the company, HELD
that the Cebu branch of Pacific Life had received the premium and had accepted the YES
application subject to processing by the insurance company, which will approve or - The age of the insured Carmen 0. Lapuz was not concealed to the insurance
reject it depending on whether the applicant is insurable on standard rates. As such, company. Her application for insurance coverage which was on a printed form
the receipt was never in force—it does not insure outright. No liability attaches until furnished by private respondent and which contained very few items of information
iNsuRanCe A2010 Dean Carale
pAgE 66♥

clearly indicated her age of the time of filing the same to be almost 65 years of age. After a policy of life insurance made payable on the death of the insured shall have
Despite such information which could hardly be overlooked in the application form, been in force during the lifetime of the insured for a period of two years from the date
considering its prominence thereon and its materiality to the coverage applied for, the of its issue or of its last reinstatement, the insurer cannot prove that the policy is void
respondent insurance corporation received her payment of premium and issued the ab initio or is rescindable by reason of the fraudulent concealment or
corresponding certificate of insurance without question. The accident which resulted misrepresentation of the insured or his agent.
in the death of the insured, a risk covered by the policy, occurred on May 31, 1969 or - According to the petitioners, the Insurance Law was amended and the second
FORTY-FIVE (45) DAYS after the insurance coverage was applied for. There was paragraph of Section 48 added to prevent the insurance company from exercising a
sufficient time for the private respondent to process the application and to notice that right to rescind after the death of the insured
the applicant was over 60 years of age and thereby cancel the policy on that ground if - The so-called "incontestability clause" precludes the insurer from raising the
it was minded to do so. If the private respondent failed to act, it is either because it defenses of false representations or concealment of material facts insofar as health
was willing to waive such disqualification; or, through the negligence or incompetence and previous diseases are concerned if the insurance has been in force for at least
of its employees for which it has only itself to blame, it simply overlooked such fact. two years during the insured's lifetime. The phrase "during the lifetime" found in
Under the circumstances, the insurance corporation is already deemed in estoppel. Its Section 48 simply means that the policy is no longer considered in force after the
inaction to revoke the policy despite a departure from the exclusionary condition insured has died. The key phrase in the second paragraph of Section 48 is "for a
contained in the said policy constituted a waiver of such condition. period of two years."
Disposition Judgment appealed from is REVERSED and SET ASIDE and respondent - The policy was issued on November 6,1973 and the insured died on April 26,1975.
insurance corporation is ordered to pay to the petitioner the proceeds of Insurance The policy was thus in force for a period of only one year and five months. Considering
that the insured died before the two-year period had lapsed, respondent company is
HARDING v. COMMERCIAL UNION (supra p.36) not, therefore, barred from proving that the policy is void ab initio by reason of the
insured's fraudulent concealment or misrepresentation.
- The petitioners contend that there could have been no concealment or
TAN v. CA (PHILIPPINE AMERICAN LIFE INSURANCE
misrepresentation by their late father because Tan Lee Siong did not have to buy
COMPANY) insurance. He was only pressured by insistent salesmen to do so
174 SCRA 403 -The legislative answer to the arguments posed by the petitioners is the
GUTIERREZ; June 29, 1989 "incontestability clause" added by the second paragraph of Section 48. The insurer
has two years from the date of issuance of the insurance contract or of its last
NATURE reinstatement within which to contest the policy, whether or not, the insured still lives
Review on certiorari of the decision of the Court of Appeals affirming the decision of within such period. After two years, the defenses of concealment or
the Insurance Commissioner misrepresentation, no matter how patent or well founded, no longer lie

FACTS TAN CHAY HENG v. WEST COAST LIFE INSURANCE


- On September 23,1973, Tan Lee Siong, father of herein petitioners, applied for life 51 PHIL 80
insurance in the amount of P 80,000.00 with respondent company. Said application JOHNS; November 21, 1927
was approved and was issued effective November 6, 1973
- On April 26,1975, Tan Lee Siong died of hepatoma (Exhibit B). Petitioners then filed FACTS
with respondent company their claim for the proceeds of the life insurance policy - Plaintiff alleges that defendant accepted and approved a life insurance policy of for
-respondent company denied petitioners' claim and rescinded the policy by reason of the sum of P10,000 in which the plaintiff was the sole beneficiary; that the policy was
the alleged misrepresentation and concealment of material facts made by the issued upon the payment by the said Tan Ceang of the first year's premium
deceased Tan Lee Siong in his application for insurance. The premiums paid on the amounting to P936; that in and by its terms, the defendant agreed to pay the plaintiff
policy were thereupon refunded as beneficiary the amount of the policy upon the receipt of the proofs of the death of
- Petitioners filed on November 27, 1975, a complaint against the former with the the insured while the policy was in force; that without any premium being due or
Office of the Insurance Commissioner. Commissioner denied petition. CA affirmed unpaid, Tan Ceang died on May 10, 1925; that in June, 1925, plaintiff submitted the
Commissioners decision proofs of the death of Tan Ceang with a claim for the payment of the policy which the
defendant refused to pay, for which he prays for a corresponding judgment, with legal
ISSUE interest from the date of the policy, and costs.
WON according to Sec. 48 of the Insurance Code, insurance company is barred from - Defendant alleges that the insurance policy on the life of Tan Ceang, upon which
rescinding contract plaintiff's action is based, was obtained by the plaintiff in confabulation with one Go
Chulian, of Bacolod, Negros Occidental; Francisco Sanchez of the same place; and Dr.
HELD V. S. Locsin, of La Carlota, Negros Occidental, thru fraud and deceit perpetrated
- Section 48. Whenever a right to rescind a contract of insurance is given to the against this defendant in the following manner, to wit:
insurer by any provision of this chapter, such right must be exercised previous to the 1. Go, Sanchez and Locsin, caused Tan Caeng to sign an application for
commencement of an action on the contract. insurance with the defendant in the sum of P10,000, in which it was said that Tan
iNsuRanCe A2010 Dean Carale
pAgE 67♥

Ceang was single and was a merchant, and that the plaintiff Tan Chai Heng, the for some years previous to the dates above mentioned, were engaged in the illicit
beneficiary, was his nephew, whereas in truth and in fact and as the plaintiff and enterprise of procuring fraudulent life insurances from the present defendant, similar
his said coconspirators well knew, the said Tan Ceang was not single but was to the one in question, and which enterprise was capitalized by him by furnishing the
married and had several children; and was not a merchant but a mere employee funds with which to pay the premium on said fraudulent insurance; that the said Go
of Tan Quina from whom he received only a meager salary, and that plaintiff was Chulian was the one who furnished the money with which to pay the first and only
not a nephew of the said Tan Ceang. annual premium on the insurance here in question, amounting to P936.50; that the
2. Tan Ceang was seriously ill, suffering from pulmonary tuberculosis of said Go Chulian, on August 28, 1926, was convicted by the Court of First Instance of
about three years' duration, which illness was incurable and was well known to the City of Manila, in criminal case No. 31425 of that court, of the crime of falsification
the plaintiff and his said coconspirators. of private documents in connection with an fraudulent insurance, similar to the
3. Locsin, in his capacity as medical examiner for the defendant, prepared present, committed against this defendant in the month of September, 1924; that in
and falsified the necessary medical certificate, in which it was made to appear, the same case the said Francisco Sanchez was one of the coaccused of the said Go
among other things, that Tan Ceang had never used morphine, cocaine or any Chulian but was discharged from the complaint, because he offered himself and was
other drug; that he was then in good health and had never consulted any utilized as a state's witness; that there is another civil action now pending against Go
physician; that he had never spit blood; and that there was no sign of either Chulian and Sanchez in the Court of First Instance of Manila (civil case No. 28680), in
present or past disease of his lungs; whereas in truth and in fact, plaintiff and which the present defendant is the plaintiff, for the recovery of the amounts of two
coconspirators well knew, Tan Ceang was addicted to morphine, cocaine, and insurance policies aggregating P19,000, fraudulently obtained by the said Go Chulian
opium and had been convicted and imprisoned therefor, and for about three year and Sanchez.
prior thereto had been suffering from pulmonary tuberculosis. - To this, plaintiff filed a demurrer which was granted.
4. Plaintiff caused a confidential report to the defendant insurance
company to be signed by one V. Sy Yock Kian, who was an employee of Go ISSUE
Chulian, in which it was falsely represented that Tan Ceang was worth about WON defense is barred by Art. 47
P40,000, had an annual income of from eight to ten thousand pesos net, had the
appearance of good health, and never had tuberculosis. HELD
5. After said application for insurance, medical certificate and confidential NO
report had been prepared and falsified, plaintiff and coconspirators caused the Ratio The word "rescind" has a well defined legal meaning, and as applied to
same to be forwarded to the defendant at its office in Manila, the medical contracts, it presupposes the existence of a contract to rescind.
certificate thru the said Dr. V. S. Locsin as medical examiner, and said application Reasoning
for insurance and confidential report thru the said Francisco Sanchez in his - Plaintiff vigorously contends that section 47 of the Insurance Act should be applied,
capacity as one of the agents of the defendant insurance company in the and that when so applied, defendant is barred and estopped to plead and set forth the
Province of Occidental Negros; that the defendant, believing that the matters alleged in its special defense. That section is as follows:
representations made in said document were true, and relying thereon, Whenever a right to rescind a contract of insurance is given to the insurer by any
provisionally accepted the said application for insurance on the life of Tan Ceang provision of this chapter, such right must be exercised previous to the
in the sum of P10,000 and issued a temporary policy pending the final approval or commencement of an action on the contract.
disapproval of said application by defendant's home-office in San Francisco, - It will be noted that defendant does not seek to have the alleged insurance contract
California, where in case of approval a permanent policy was to be issued; that rescinded. It denies that it ever made any contract of insurance on the life of Tan
such permanent policy was never delivered to the plaintiff because defendant Ceang or that any such a contract ever existed, and that is the question which it seeks
discovered the fraud before its delivery. to have litigated by its special defense. In the very nature of things, if the defendant
6. That the first agreed annual premium on the insurance in question of never made or entered into the contract in question, there is no contract to rescind,
P936.50 not having been paid within 60 days after medical examination of the and, hence, section 47 upon which the lower based its decision in sustaining the
applicant as required by the regulations of the defendant insurance company, demurrer does not apply. As stated, an action to rescind a contract is founded upon
plaintiff and coconspirators caused Tan Ceang to sign a health certificate for and presupposes the existence of the contract which is sought to be rescinded. If all of
reinstatement; that the said temporary policy was delivered by defendant to the the material matters set forth and alleged in the defendant's special plea are true,
insured on April 10, 1925, in the belief that said statements and representations there was no valid contract of insurance, for the simple reason that the minds of the
were true and in reliance thereon. parties never met and never agreed upon the terms and conditions of the contract.
7. 2 ½ months after the supposed medical examination above referred to, We are clearly of the opinion that, if such matters are known to exist by a
and exactly 1 month after the date of the health certificate for reinstatement preponderance of the evidence, they would constitute a valid defense to plaintiff's
above set forth, Tan Ceang died in Valladolid, Occidental Negros, of pulmonary cause of action. Upon the question as to whether or not they or are not true, we do
tuberculosis, the same illness from which suffering at the time it is supposed he not at this time have or express any opinion, but we are clear that section 47 does not
was examined by Dr. V. S. Locsin, but that the plaintiff coconspirators, pursuant apply to the allegations made in the answer, and that the trial court erred in
to their conspiracy, caused the said Dr. V. S. Locsin to state falsely in the sustaining the demurrer.
certificate of death that the said Tan Ceang had died of cerebral hemorrhage. Disposition The judgment of the lower court is reversed and the case is remanded
- Defendant also alleges that plaintiff was, like V. Sy Yock Kian, an employee of Go for such other and further proceedings as are not inconsistent with this opinion, with
Chulian; that the latter was the ringleader of a gang of malefactors, who, during, and costs against the plaintiff.
iNsuRanCe A2010 Dean Carale
pAgE 68♥

- By the plain terms of the policy, other insurance without the consent of petitioner
PIONEER INSURANCE AND SURETY CORPORATION v. YAP would ipso facto avoid the contract. It required no affirmative act of election on the
part of the company to make operative the clause avoiding the contract, wherever the
61 SCRA 426
specified conditions should occur. Its obligations ceased, unless, being informed of the
FERNANDEZ; December 19, 1974 fact, it consented to the additional insurance.
- The obvious purpose of the aforesaid requirement in the policy is to prevent over-
NATURE insurance and thus avert the perpetration of fraud. The public, as well as the insurer,
Appeal by certiorari from CA decision affirming a CFI decision which declared plaintiff is interested in preventing the situation in which a fire would be profitable to the
Yap entitled to recover from defendant Pioneer Insurance and Surety Corp, the full insured. According to Justice Story: "The insured has no right to complain, for he
amount of the damage inquired in Policy No. 4219 assents to comply with all the stipulation on his side, in order to entitle himself to the
benefit of the contract, which, upon reason or principle, he has no right to ask the
FACTS court to dispense with the performance of his own part of the agreement, and yet to
- Yap owned a store in a 2 storey building, where she sold shopping bags and bind the other party to obligations, which, but for those stipulation would not have
footwear. Her son-in-law was in charge of the store been entered into."
- April 19, 1962- Yap took out Fire Insurance Policy No. 4216 from Pioneer with a face Disposition the appealed judgment of the Court of Appeals is reversed and set aside,
value of P25,000 covering her stocks, office furniture, fixtures, etc. and the petitioner absolved from all liability under the policy.
- among the conditions set forth:
The Insured shall give notice to the Company of any insurance or insurances
NEW LIFE ENTERPRISES v. CA
already effected, or which may subsequently be effected, covering any of the
property hereby insured, and unless such notice be given and the particulars of 207 SCRA 609
such insurance or insurances be stated in, or endorsed on this Policy by or on REGALADO; March 31, 1992
behalf of the Company before the occurrence of any loss or damage, all benefits
under this Policy shall be forfeited. (emphasis supplied) FACTS
It is understood that, except as may be stated on the face of this policy there - Julian Sy and Jose Sy Bang are partners engaged in the business of selling
is no other insurance on the property hereby covered and no other insurance is construction materials under the business name “New Life Enterprises.” Julian Sy
allowed except by the consent of the Company endorsed hereon. Any false insured against fire the stocks in trade of New Life Enterprises with Western Guaranty
declaration or breach or this condition will render this policy null and void. Corporation, Reliance Surety and Insurance Co. Inc., and Equitable Insurance
- At the time of insurance of Policy 4219(April 19, 1962), an insurance policy for Corporation in the aggregate amount of P1,550,000.00. When the building where New
P20,000 issued by the Great American Insurance Company covering the same Life Enterprises was located, along with the stocks in trade therein, were gutted by
properties was noted on said policy as co-insurance. fire, petitioners filed an insurance claim against the three companies. The insurance
- August 29, 1962 : parties executed an endorsement on Policy 4219 stating: companies all denied Julian Sy’s claim on the ground of “breach of policy condition,”
It is hereby declared and agreed that the co-insurance existing at present under this (i.e., the “other insurance” clause which required New Life Enterprises to inform each
policy is as follows: P20,000.00 � Northwest Ins., and not as originally stated. of the insurance companies in case the former insures with another company the
(emphasis supplied) same property already insured by each of the insurance companies).
Except as varied by this endorsement, all other terms and conditions remain - Because of the denial of their claims for payment by the 3 insurance companies,
unchanged. petitioners filed separate civil actions against the former before the Regional Trial
- September 26, 1962: Yap took out another fire insurance policy for P20,000 covering Court of Lucena City, which cases were consolidated for trial. The trial court ruled in
the same properties, from Federal Insurance Company. This policy was procured favor of petitioner. However, the Court of Appeals reversed the trial court’s decision,
without notice to and the written consent of Pioneer, and was therefore not noted as a found petitioner to have violated Clauses 3 and 27 of the separate insurance policies
co-insurance in Policy 4219. issued by the 3 companies, and exonerated the insurance companies from liability.
- December 19, 1962: Fire burned Yap’s store
ISSUE
ISSUE WON petitioners violated the “Other Insurance Clause” of the insurance policies
WON petitioner should be absolved from liability on Fire insurance Policy No. 4219 on
account of any violation by respondent Yap of the co-insurance clause therein HELD
YES
HELD - Petitioners admit that the respective insurance policies issued by private
YES respondents did not state or endorse thereon the other insurance coverage obtained
- The petitioner should be absolved. or subsequently effected on the same stocks in trade for the loss of which
Reasoning compensation is claimed by petitioners. It is further admitted by petitioners that
- There was a violation by Yap of the co-insurance clause contained in Policy No. 4219 Equitable's policy stated "nil" in the space thereon requiring indication of any co-
which resulted in the avoidance of the petitioner’s liability. insurance although there were 3 policies subsisting on the same stocks in trade at the
iNsuRanCe A2010 Dean Carale
pAgE 69♥

time of the loss, namely, that of Western in the amount of P350,000.00 and two 2
policies of Reliance in the total amount of P1,000,000.00.
- The coverage by other insurance or co-insurance effected or subsequently arranged
by petitioners were neither stated nor endorsed in the policies of the 3 private
respondents, warranting forfeiture of all benefits thereunder if we are to follow the
express stipulation in Policy Condition No. 3.
- The terms of the contract are clear and unambiguous. The insured is specifically YOUNG v. MIDLAND TEXTILE INSURANCE CO.
required to disclose to the insurer any other insurance and its particulars which he
30 PHIL 617
may have effected on the same subject matter. The knowledge of such insurance by
the insurer's agents, even assuming the acquisition thereof by the former, is not the JOHNSON; March 31, 1915
"notice" that would stop the insurers from denying the claim. Besides, the so-called
theory of imputed knowledge, that is, knowledge of the agent is knowledge of the FACTS
principal, aside from being of dubious applicability here has likewise been roundly - K.S. Young had a candy and fruit store on the Escolta, Manila, and occupied a
refuted by respondent court whose factual findings we find acceptable. The mere fact building at 321 Calle Claveria, as a residence and bodega. The Midland Textile
that Yap Kam Chuan was an agent for both Reliance and Equitable does not justify the Insurance Co. in consideration of the payment of a premium of P60, entered into a
allegation that the two are sister companies. Availment of the services of the same contract of insurance with Young by the terms of which the company, upon certain
agents and adjusters by different companies is a common practice in the insurance conditions, promised to pay Young the sum of P3,000 in case said residence and
business and such facts do not warrant the speculative conclusion of the trial court. bodega and contents should be destroyed by fire.
- Considering the terms of the policy which required the insured to declare other - One of the conditions of the contract is: "Warranty B – It is hereby declared and
insurances, the statement in question must be deemed to be a statement (warranty) agreed that during the pendency of this policy no hazardous goods be stored or kept
binding on both insurer and insured, that there were no other insurance on the for sale, and no hazardous trade or process be carried on, in the building to which this
property. The annotation then, must be deemed to be a warranty that the property insurance applies, or in any building connected therewith."
was not insured by any other policy. Violation thereof entitled the insurer to rescind. - Young placed in the residence and bodega three boxes filled with fireworks. Said
- The obvious purpose of the aforesaid requirement in the policy is to prevent over- residence and bodega and the contents thereof were partially destroyed by fire.
insurance and thus avert the perpetration of fraud. The public, as well as the insurer, - The fireworks had been given to Young by the former owner of the Luneta Candy
is interested in preventing the situation in which a fire would be profitable to the Store. He intended to use them in the celebration of the Chinese New Year. However,
insured. The insured has no right to complain, for he assents to comply with all the the authorities of the city of Manila had prohibited the use of fireworks on said
stipulations on his side, in order to entitle himself to the benefit of the contract, which, occasion, so Young then placed them in the bodega where they remained from the 4th
upon reason or principle, he has no right to ask the court to dispense with the or 5th of February, 1913 until after the fire of March 18, 1913.
performance of his own part of the agreement, and yet to bind the other party to - Both parties agree that the fireworks come within the phrase "hazardous goods,"
obligations, which, but for those stipulations, would not nave been entered into. mentioned in "Warranty B" of the policy; that the fireworks were found in a part of the
- It is not disputed that the insured failed to reveal before the loss three other building not destroyed by the fire and that they in no way contributed to the fire, or to
insurances. By reason of said unrevealed insurances, the insured had been guilty of a the loss that resulted.
false declaration; a clear misrepresentation and a vital one because where the insured - The lower court rendered a judgment in favor of Young for the sum of P2,708.78,
had been asked to reveal but did not, that was deception. Otherwise stated, had the and costs.
insurer known that there were many co-insurances, it could have hesitated or plainly
desisted from entering into such contract. Hence, the insured was guilty of clear fraud. ISSUE
- As the insurance policy against fire expressly required that notice should be given by 1. WON the placing of the fireworks in the building insured, they being "hazardous
the insured of other insurance upon the same property, the total absence of such goods," was a violation of the terms of the contract of insurance and especially of
notice nullifies the policy. "Warranty B."
- Additionally, insofar as the liability of respondent Reliance is concerned, it is not
denied that the complaint for recovery was filed in court by petitioners only on January HELD
31, 1984, or after more than one (1) year had elapsed from petitioners' receipt of the 1. YES.
insurers' letter of denial on November 29, 1982. Reasoning It is admitted by both parties that the fireworks are hazardous goods. The
- The condition contained in an insurance policy that claims must be presented within defendant alleged that they were "stored." The plaintiff contends that under all the
one year after rejection is not merely a procedural requirement but an important facts and circumstances of the case, they were not “stored” in said building, and that
matter essential to a prompt settlement of claims against insurance companies as it the placing of them in the building was not a violation of the terms of the contract.
demands that insurance suits be brought by the insured while the evidence as to the - Whether a particular article is "stored" or not must, in some degree, depend upon
origin and cause of destruction have not yet disappeared. the intention of the parties. Nearly all of the cases cited by the lower court are cases
where the article was being put to some reasonable and actual use, which might
easily have been permitted by the terms of the policy, and within the intention of the
QUA CHEE GAN v. LAW UNION (supra p.48) parties, and excepted from the operation of the warranty, like the present.
iNsuRanCe A2010 Dean Carale
pAgE 70♥

- (1) Where merchants have had or kept the "hazardous" articles in small quantities,
and for actual daily use, for sale, such as gasoline, gunpowder, etc.; (2) Where such
FILIPINAS LIFE ASSURANCE v. NAVA
articles have been brought on the premises for actual use thereon, and in small
quantities, such as oil, paints, etc; and (3) Where such articles or goods were used for 17 SCRA 210
lighting purposes, and in small quantities. BAUTISTA ANGELO; May 20, 1966
- In the present case no claim is made that the "hazardous goods" were placed in the
bodega for present or daily use. It is admitted that they were placed in the bodega NATURE
"for future use," or for future consumption, or for safe keeping. It seems clear to us Petition for review of a decision of the Court of Appeals
that the "hazardous goods" in question were "stored" in the bodega, as that word is
generally defined. That being true, suppose the defendant had made an examination FACTS
of the premises, even in the absence of a fire, and had found the "hazardous goods" - Before the war, Nava entered into a contract of insurance with Insular Life Assurance
there, would it not have been justified in declaring the policy null and of no effect by Co., Ltd. (face value of P5k), and 17 separate contracts of life insurance with Filipinas
reason of a violation of its terms? If it might, then may it not repudiate its liability, Life Assurance Co. (total face value of P90k). Each and everyone of the 18 policies
even after the fire? If the "warranty" is a term of the contract, will not its violation issued by defendants to plaintiff contains a loan clause of the following tenor:
cause a breach and justify noncompliance or repudiation? Policy loans. After three full years' premiums have been paid upon this
- Contracts of insurance are contracts of indemnity, upon the terms and conditions Policy, if no premium payment is in default, the Company, subject to its then
specified therein. Parties have a right to impose such reasonable conditions at the existing rules, will advance on proper assignment and delivery of this Policy and on
time of the making of the contract as they deem wise and necessary. The rate of the sole security thereof a sum equal to, or at the option of the owner less than, the
premium is measured by the character of the risk assumed. The insurer, for a cash value specified in the Schedule of Policy Values, less any existing indebtedness
comparatively small consideration, undertakes to guarantee the insured against loss on or secured by this Policy and any unpaid balance of the premium for the current
or damage, upon the terms and conditions agreed upon, and upon no other. When the policy-year; provided interest at six per centum per annum on the whole amount of
insurer is called upon to pay, in case of loss, he may justly insist upon a fulfillment of the loan is paid in advance to the end of the current policy-year. At the end of the
the terms of the contract. If the insured, cannot bring himself within the terms and current policy-year interest at the same rate for one year in advance will be due
conditions of the contract, he is not entitled to recover for any loss suffered. The and payable, and annually thereafter, and if not so paid will be added to the
terms of the contract constitute the measure of the insurer's liability. If the contract principal and bear the same rate of interest. Failure to repay any such loan or
has been terminated, by a violation of its terms on the part of the insured, there can interest shall not avoid this Policy unless the total indebtedness shall equal or
be no recovery. Compliance with the terms of the contract is a condition precedent to exceed the full amount of the loan value available hereunder.
the right of recovery. Any indebtedness on this Policy shall first be deducted from any money
- Young argues that since the "storing" of the fireworks on the premises did not payable or in any settlement under this Policy.
contribute in any way to the damage occasioned by the fire, he should be permitted to - Nava had so far paid to Insular a total of P2,574; and to Filipinas Life, a total of
recover. That argument, however, is beside the question, if the "storing" was a P32,072.60.
violation of the terms of the contract. The violation of the terms of the contract, by - April 28, 1948: Nava applied to the companies for a P5k loan in line with the loan
virtue of the provisions of the policy itself, terminated, at the election of either party, clause, but they refused to grant it because certain regulations issued by the
the contractual relations. Insurance Commissioner required the insurance companies to withhold the payments
- Young paid a premium based upon the risk at the time the policy was issued. on premiums made during the Japanese occupation because the same shall be subject
Certainly, the placing of the firecrackers in the building insured increased the risk. to future adjustments " as soon as debtor-creditor relationship is established" and
Young had not paid a premium based upon the increased risk, neither had the because of such process of "withholding" plaintiff was not entitled to borrow any
defendant issued a policy upon the theory of a different risk. He was enjoying, if his amount until such adjustment has been made.
contention may be allowed, the benefits of an insurance policy upon one risk, - Sept 30, 1948: Nava called the attention of the insurance companies to the SC
whereas, as a matter of fact, it was issued upon an entirely different risk. The decision (Haw Pia v. China Banking Corporation) establishing and recognizing the
defendant had neither been paid nor had issued a policy to cover the increased risk. relationship of debtor and creditor with respect to payments in fiat currency made
An increase of risk which is substantial and which is continued for a considerable during the Japanese occupation on pre-war obligations.
period of time, is a direct and certain injury to the insurer, and changes the basis upon - Companies still refused saying that the SC decision was not applicable to
which the contract of insurance rests. transactions undertaken during Japanese occupation when they relate to life insurance
Disposition Decision of the lower court is REVERSED. policies.
- Feb 4, 1949: Nava was again refused even if the total amount of the cash surrender
TAN v. CA (supra p.51) values of the 18 policies reached the sum of P9,468.29.
- Feb 10, 1949: Nava brought case to the CFI Manila praying for the rescission of the
abovementioned 18 policies and for the refund to him of all the premiums so far paid
AREOLA v. CA (supra p.26)
by him to defendants in the amount of P31,633.80, plus 6% interest thereon as
damages
TAN CHAY v. WEST COAST (supra p.51) - Nov 28, 1951: companies passed a resolution which was approved by the Insurance
Commissioner, giving full credit to all premium payments made by their policyholders
iNsuRanCe A2010 Dean Carale
pAgE 71♥

in fiat currency during the Japanese occupation on account of pre-war policies for material warranty, or other material provision of a policy, on the part of either party
which reason they filed an amended answer offering to pay plaintiff the amount of thereto, entitles the other to rescind."
P9,468.29 which represents the aggregate cash surrender values of all the policies in - "The general rule is that a breach of the agreement to make the loan does not entitle
question as of February 10, 1949, but apparently this offer was refused. the insured to rescind the contract," is not controlling in this jurisdiction. Firstly, it was
- CFI: (1) rescinded the insurance contracts; (2) ordered defendant Filipinas Life not shown that the insurance laws in the states where said ruling prevails contain a
Assurance Co. to pay plaintiff the amount of P32,072.60; and (3) ordered defendant provision identical to Section 69 of our Insurance Law we quoted above, and secondly,
Insular Life Assurance Co., Ltd. to pay plaintiff the amount of P2,574.00 the rule cited by Vance is not a rule uniformly followed by all states in the US, for on
- CA affirmed. this matter there is a marked divergence of opinion.
2. NO
ISSUES - Considering that our Insurance Law does not contain an express provision as to what
1. WON CA erred in ruling that as a consequence of the decision in the Haw Pia case the court should do in cases of rescission of an insurance policy under Section 69, the
petitioners violated the loan clause contained in the insurance policies thereby provision that should apply is that embodied in Article 1225 of the old Civil Code, as
entitling respondent to their rescission postulated in Article 16 of the same Code, which provides that on matters which are
2. WON CA erred in ruling that by virtue of Article 1295 of the old Civil Code not governed by special laws the provisions of said Code shall supplement its
petitioners should refund to defendant all the premiums paid on his insurance policies deficiency. And said Article 1295 provides:
as a consequence of their rescission ART. 1295. Rescission makes necessary the return of the things which were the
3. WON CA erred in not ruling that, even if respondent is entitled to the rescission of subject-matter of the contract, with their fruits, and of the price paid, with interest
said insurance policies, he can only recover their cash surrender value at the time the thereon. ...xxx
complaint was filed - Said the petitioners: "Recovery of the full amount of the premium after the insurer
has sustained for sometime the risk of the insurance and the insured has enjoyed the
HELD benefit of protection is obviously unjust and is so recognized by the better
1. NO. authorities." The ruling above quoted merely represents the minority rule in the US,
- Even assuming the validity of the Insurance Commissioner’s regulations, the fact the majority rule being that the insured can recover all premiums paid, in some cases
however is that such requirement has already lost its legal effect and value when our with interest in case of wrongful cancellation, repudiation, termination or rescission of
Supreme Court rendered its decision in the Haw Pia case wherein it was declared, the contract of life insurance.
among others, that all payments made in fiat currency during the Japanese occupation - Contention that because respondent cannot restore to petitioners the "value of the
in relation with any contractual obligation executed before the war were valid to all benefit of protection" which he might have received under the 18 life insurance
intents and purposes, and yet petitioners apparently did not give any importance to policies in question he is not entitled to rescind them under the provision of Article
such decision for in their opinion it does not have any application to transactions 1295 of the old Civil Code, is untenable because said article only contemplates a
which have any relation to payment of premiums on life insurance policies. transaction whether material things are involved, and do not refer to intangible ones
- It cannot be denied that a life insurance policy involves a contractual obligation which cannot be the subject of restoration, for to interpret it otherwise would be to
wherein the insured becomes duty bound to pay the premiums agreed upon, lest he defeat the law itself with the result that rescission can never be had under Section 69
runs the risk of having his insurance policy lapse if he fails to pay such premiums. of our Insurance Law.
- The fact that if the insured had paid in full the premiums corresponding to the first 3 - It cannot be denied that petitioners had in turn already derived material benefits
years of the life of his policy he cannot be considered delinquent that would cause the from the use of premiums paid to them by respondent before, during and after the
lapse of his policy if the same contains an automatic premium payment clause cannot last war from which they must have realized huge profits, and in this light alone
divest such policy of its contractual nature, for the result of such failure would only be petitioners cannot claim prejudice or unfairness if they are ordered to refund the
for him to pay later the premium plus the corresponding interest depending upon the premiums paid by respondents.
condition of the policy. But certainly it does not cease to be a contractual liability 3. NO.
insofar as the payment of that premium is concerned for whether he likes it or not - Issue is corollary to preceding issue. No need to refute.
that premium has to be paid lest he allows the lapse of his policy. Consequently, the Disposition Decision appealed from is AFFIRMED. Costs against petitioners
payment of premiums on the life insurance policies made by Nava before and during
the war up to the time he applied for the loan in question with petitioners should be
considered likewise as valid payments upon the theory that such insurance policies
CHAPTER VII. RISKS AND COVERAGES
are in the nature of a contractual obligation within the meaning of the civil law. In
effect, therefore, those payments were made by a debtor to a creditor within the VDA. DE BATACLAN v. MEDINA
meaning of the requirement of the regulations of the Insurance Commissioner and as 102 PHIL 181
such they can offer no excuse to petitioners for refusing to grant the loan as MONTEMAYOR; October 22, 1957
contemplated in the loan clause embodied in the policies in question.
- It is clear from the foregoing that the petitioners violated the loan clause embodied
FACTS
in each of the 18 life insurance policies issued to respondent to rescind all said
- Juan Bataclan rode Bus 30 of Medina Transport, driven by Saylon, shortly after
policies under Section 69 of the Insurance Act, which provides: "The violation of a
midnight. The bus was running very fast. One of the front tires burst. Bus fell into
iNsuRanCe A2010 Dean Carale
pAgE 72♥

canal and turned turtle. 4 passengers couldn’t get out, including Bataclan. Gasoline - While said insurance policy was in full force and effect, the insured, Carlie Surposa,
began to leak from the overturned bus. 10 men came to help. 1 carried a torch and died on October 18, 1988 as a result of a stab wound inflicted by one of the 3
when he approached the bus, fire started, killing the trapped passengers. unidentified men without provocation and warning on the part of the former as he and
- TC opined that proximate cause of Bataclan’s death was not the overturning of bus his cousin, Winston Surposa, were waiting for a ride on their way home after attending
but the fire. At the time fire started, Bataclan, though injured, was still alive and the celebration of the "Maskarra Annual Festival."
damages were awarded, not for his death, but for physical injuries suffered. - Thereafter, Julia Surposa and the other beneficiaries of said insurance policy filed a
written notice of claim with the FINMAN Corp which denied said claim contending that
ISSUE/S murder and assault are not within the scope of the coverage of the insurance policy.
WON the proximate cause is the overturning of the bus or the fire - Feb. 24, 1989: Surposa filed a complaint with the Insurance Commission which
subsequently ordered FINMAN to pay Surposa the proceeds of the policy with interest.
HELD - CA affirmed said decision.
- The proximate cause is the overturning of the bus.
- Ordinarily, when a bus overturns and pins down passenger, merely causing him ISSUE
injuries. If through some event, unexpected and extraordinary, the bus is set on fire, WON CA committed GAD in applying the principle of "expresso unius exclusio alterius"
and passenger is burned to death, one might contend that the proximate cause was in a personal accident insurance policy (since death resulting from murder and/or
the fire and not the overturning of the vehicle. assault are impliedly excluded in said insurance policy considering that the cause of
- But here, the proximate cause of Bataclan’s death is the overturning of the bus, this death of the insured was not accidental but rather a deliberate and intentional act of
for the reason that when the vehicle turned not only on its side but completely on its the assailant in killing the former as indicated by the location of the lone stab wound
back, leaking of gasoline from the tank was not unnatural or unexpected. on the insured) [TF they cannot be made to indemnify the Surposa heirs]
- The coming of the men with the torch was in response to the call for help, made
only not by the passengers but even the driver and conductor, and because it was HELD
very dark, about 2:30 am, rescuers had to carry a light with them. Coming as they did NO
from rural area where lanterns and flashlights were not available, they had to use a - The record is barren of any circumstance showing how the stab wound was inflicted.
torch. What was more natural than that said rescuers should innocently approach the While the act may not exempt the unknown perpetrator from criminal liability, the fact
overturned vehicle to extend aid. remains that the happening was a pure accident on the part of the victim. The insured
- The coming of the men with the torch was to be expected, and was a natural died from an event that took place without his foresight or expectation, an event that
sequence of the overturning of the bus, the trapping of some of its passengers and proceeded from an unusual effect of a known cause and, therefore, not expected.
the call for outside help. Reasoning
- The burning of bus can also in part be attributed to negligence of carrier, through its - De la Cruz vs. Capital Insurance & Surety Co., Inc (1966)~ The terms "accident" and
driver and conductor. They, or at least the driver, should have known that in the "accidental" as used in insurance contracts have not acquired any technical meaning,
position in which the overturned bus was, gasoline could and must have leaked from and are construed by the courts in their ordinary and common acceptation. Thus, the
the gasoline tank and soaked the area in and around the bus. Gasoline can be smelt terms have been taken to mean that which happen by chance or fortuitously, without
and detected even from a distance, and yet neither the driver nor the conductor intention and design, and which is unexpected, unusual, and unforeseen. An accident
would appear to have cautioned or taken steps to warn rescuers not to bring the is an event that takes place without one's foresight or expectation an event that
lighted torch too near the bus. proceeds from an unknown cause, or is an unusual effect of a known cause and,
therefore, not expected.
Ratio The generally accepted rule is that, death or injury does not result from
FINMAN GENERAL ASSURANCE CORPORATION v. CA
accident or accidental means within the terms of an accident-policy if it is the natural
(SURPOSA) result of the insured's voluntary act, unaccompanied by anything unforeseen except
213 SCRA 493 the death or injury. There is no accident when a deliberate act is performed unless
NOCON; September 2, 1992 some additional, unexpected, independent, and unforeseen happening occurs which
produces or brings about the result of injury or death. In other words, where the
death or injury is not the natural or probable result of the insured's voluntary act, or if
NATURE
something unforeseen occurs in the doing of the act which produces the injury, the
Certiorari
resulting death is within the protection of the policies insuring against death or injury
from accident.
FACTS
- The personal accident insurance policy involved herein specifically enumerated only
- Oct. 22, 1986: Carlie Surposa was insured with Finman General Assurance
10 circumstances wherein no liability attaches to FINMAN for any injury, disability or
Corporation under Finman General Teachers Protection Plan Master Policy No. 2005
loss suffered by the insured as a result of any of the stimulated causes.
and Individual Policy No. 08924 with his parents, spouses Julia and Carlos Surposa,
-The principle of " expresso unius exclusio alterius" the mention of one thing implies
and brothers Christopher, Charles, Chester and Clifton, all surnamed, Surposa, as
the exclusion of another thing is therefore applicable in the instant case since murder
beneficiaries.
and assault, not having been expressly included in the enumeration of the
circumstances that would negate liability in said insurance policy: the failure of the
iNsuRanCe A2010 Dean Carale
pAgE 73♥

FINMAN to include death resulting from murder or assault among the prohibited risks some risk coming to him in pursuing that errand, but that risk always existed it being
leads inevitably to the conclusion that it did not intend to limit or exempt itself from inherent in the position he was holding. He cannot therefore be blamed solely for
liability for such death. doing what he believed was in keeping with his duty as a watchman and as a citizen.
- A1377 NCC: The interpretation of obscure words or stipulations in a contract shall And he cannot be considered as making an arrest as an officer of the law, as
not favor the party who caused the obscurity. contended, simply because he went with the traffic policeman, for certainly he did not
- NPC vs. CA [1986]~ It is well settled that contracts of insurance are to be go there for that purpose nor was he asked to do so by the policeman.
construed liberally in favor of the insured and strictly against the insurer. Thus - Much less can it be pretended that Basilio died in the course of an assault or murder
ambiguity in the words of an insurance contract should be interpreted in favor of its considering the very nature of these crimes. In the first place, there is no proof that
beneficiary. the death of Basilio is the result of either crime for the record is barren of any
Disposition DENIED for lack of merit. circumstance showing how the fatal shot was fired. Perhaps this may be clarified in
the criminal case now pending in court as regards the incident but before that is done
anything that might be said on the point would be a mere conjecture. Nor can it be
CALANOC v. CA
said that the killing was intentional for there is the possibility that the malefactor had
98 PHIL 79 fired the shot merely to scare away the people around for his own protection and not
BAUTISTA; December 16, 1955 necessarily to kill or hit the victim. In any event, while the act may not exempt the
triggerman from liability for the damage done, the fact remains that the happening
FACTS was a pure accident on the part of the victim. The victim could have been either the
- Basilio was a watchman of the Manila Auto Supply located at the corner of Avenida policeman or Atty. Ojeda for it cannot be pretended that the malefactor aimed at the
Rizal and Zurbaran. He secured a life insurance policy from the Philippine American deceased precisely because he wanted to take his life.
Life Insurance Company in the amount of P2,000 to which was attached a Disposition Decision set aside
supplementary contract covering death by accident. On January 25, 1951, he died of a
gunshot wound on the occasion of a robbery committed in the house of Atty. Ojeda at BIAGTAN v. THE INSULAR LIFE ASSURANCE COMPANY, LTD.
the corner of Oroquieta and Zurbaran streets. Calanoc, the widow, was paid the sum
44 SCRA 58
of P2,000, face value of the policy, but when she demanded the payment of the
additional sum of P2,000 representing the value of the supplemental policy, the MAKALINTAL; March 29, 1972
company refused alleging, as main defense, that the deceased died because he was
murdered by a person who took part in the commission of the robbery and while NATURE
making an arrest as an officer of the law which contingencies were expressly excluded Appeal from decision of CFI Pangasinan.
in the contract and have the effect of exempting the company from liability.
- It is contended in behalf of the company that Basilio was killed which "making an FACTS
arrest as an officer of the law" or as a result of an "assault or murder" committed in - Juan Biagtan was insured with Insular for P5k and a supplementary contract
the place and therefore his death was caused by one of the risks excluded by the “Accidental Death Benefit” clause for another P5k if "the death of the Insured resulted
supplementary contract which exempts the company from liability. This contention directly from bodily injury effected solely through external and violent means
was upheld by the Court of Appeals. Hence, this petition. sustained in an accident . . . and independently of all other causes." The clause,
however, expressly provided that it would not apply where death resulted from an
ISSUE injury "intentionally inflicted by a third party."
WON the death of the victim comes within the purview of the exception clause of the - One night, a band of robbers entered their house. Juan went out of his room and he
supplementary policy and, hence, exempts the company from liability was met with 9 knife stabs. He died. The robbers were convicted of robbery with
homicide.
HELD - The family was claiming the additional P5k from Insular, under the Accidental Death
NO Benefit clause. Insular refused on the ground that the death resulted from injuries
- Basilio was a watchman of the Manila Auto Supply which was a block away from the intentionally inflicted by 3rd parties and was therefore not covered. Biagtans filed
house of Atty. Ojeda where something suspicious was happening which caused the against Insular. CFI ruled in favor of Biagtans.
latter to ask for help. While at first he declined the invitation of Atty. Ojeda to go with
him to his residence to inquire into what was going on because he was not a regular ISSUE
policeman, he later agreed to come along when prompted by the traffic policeman, WON the injuries were intentionally inflicted
and upon approaching the gate of the residence he was shot and died. The
circumstance that he was a mere watchman and had no duty to heed the call of Atty. HELD
Ojeda should not be taken as a capricious desire on his part to expose his life to YES
danger considering the fact that the place he was in duty-bound to guard was only a - Whether the robbers had the intent to kill or merely to scare the victim or to ward off
block away. In volunteering to extend help under the situation, he might have any defense he might offer, it cannot be denied that the act itself of inflicting the
thought, rightly or wrongly, that to know the truth was in the interest of his employer injuries was intentional.
it being a matter that affects the security of the neighborhood. No doubt there was
iNsuRanCe A2010 Dean Carale
pAgE 74♥

- The exception in the accidental benefit clause invoked by the appellant does not is unforeseen and unexpected by the insured. All the associated words and concepts
speak of the purpose — whether homicidal or not — of a third party in causing the in the policy plainly exclude the accidental death from the coverage of the policy only
injuries, but only of the fact that such injuries have been "intentionally" inflicted — where the injuries are self-inflicted or attended by some proscribed act of the insured
this obviously to distinguish them from injuries which, although received at the hands or are incurred in some expressly excluded calamity such as riot, war or atomic
of a third party, are purely accidental. explosion.
- Examples of unintentional: - The untenability of insurer's claim that the insured's death fell within the exception is
>> A gun which discharges while being cleaned and kills a bystander; further heightened by the stipulated fact that two other insurance companies which
>> a hunter who shoots at his prey and hits a person instead; likewise covered the insured for much larger sums under similar accidental death
>> an athlete in a competitive game involving physical effort who collides with an benefit clauses promptly paid the benefits thereof to plaintiffs beneficiaries.
opponent and fatally injures him as a result.
- In Calanoc vs. CA: Where a shot was fired and it turned out afterwards that the SUN INSURANCE v. CA (LIM)
watchman was hit in the abdomen, the wound causing his death, the Court held that it
211 SCRA 554
could not be said that the killing was intentional for there was the possibility that the
malefactor had fired the shot to scare the people around for his own protection and CRUZ; July 17, 1992
not necessarily to kill or hit the victim. A similar possibility is clearly ruled out by the
facts in this case. For while a single shot fired from a distance, and by a person who FACTS
was not even seen aiming at the victim, could indeed have been fired without intent - The petitioner issued Personal Accident Policy to Felix Lim, Jr. with a face value of
to kill or injure, nine wounds inflicted with bladed weapons at close range cannot P200,000.00. Two months later, he was dead with a bullet wound in his head. As
conceivably be considered as innocent insofar as such intent is concerned. beneficiary, his wife Nerissa Lim sought payment on the policy but her claim was
- In Hucthcraft's Ex'r vs. Travelers' Ins. Co. (US case): where the insured was waylaid rejected. The petitioner agreed that there was no suicide. It argued, however, that
and assassinated for the purpose of robbery, the court rendered judgment for the there was no accident either.
insurance company and held that while the assassination of the insured was as to him Pilar Nalagon, Lim's secretary, was the only eyewitness to his death. According to
an unforeseen event and therefore accidental, "the clause of the proviso that excludes Nalagon, Lim was in a happy mood (but not drunk) and was playing with his handgun,
the (insurer's) liability, in case death or injury is intentionally inflicted by any other from which he had previously removed the magazine. As she watched the television,
person, applies to this case." he stood in front of her and pointed the gun at her. She pushed it aside and said it
Disposition CFI decision reversed. might be loaded. He assured her it was not and then pointed it to his temple. The next
moment there was an explosion and Lim slumped to the floor. He was dead before he
SEPARATE OPINION fell.
- The term "accident" has been defined as follows:
The words "accident" and "accidental" have never acquired any technical
TEEHANKEE [dissent] signification in law, and when used in an insurance contract are to be construed and
- Calanoc v. CA is controlling in this case because the insurance company wasn’t able considered according to the ordinary understanding and common usage and speech
to prove that the killing was intentional. (Burden of proof is with the insurance of people generally. In substance, the courts are practically agreed that the words
company) "accident" and "accidental" mean that which happens by change or fortuitously,
- Insurance, being contracts of adhesion, must be construed strictly against insurance without intention or design, and which is unexpected, unusual, and unforeseen. The
company in cases of ambiguity. definition that has usually been adopted by the courts is that an accident is an
- The supplementary contract enumerated exceptions. The only exception which is not event that takes place without one's foresight or expectation — an event that
susceptible of classification is that provided in paragraph 5(e), the very exception proceeds from an unknown cause, or is an unusual effect of a known case, and
herein involved, which would also except injuries "inflicted intentionally by a third therefore not expected.
party, either with or without provocation on the part of the insured, and whether or - An accident is an event which happens without any human agency or, if happening
not the attack or the defense by the third party was caused by a violation of the law through human agency, an event which, under the circumstances, is unusual to and
by the insured." not expected by the person to whom it happens. It has also been defined as an injury
- This ambiguous clause conflicts with all the other four exceptions in the same which happens by reason of some violence or casualty to the insured without his
paragraph 5 particularly that immediately preceding it in item (d) which excepts design, consent, or voluntary co-operation.
injuries received where the insured has violated the law or provoked the injury, while
this clause, construed as the insurance company now claims, would seemingly except ISSUE
also all other injuries, intentionally inflicted by a third party, regardless of any WON what happened was an accident
violation of law or provocation by the insured, and defeat the very purpose of the
policy of giving the insured double indemnity in case of accidental death by "external HELD
and violent means" — in the very language of the policy.' YES
- It is obvious from the very classification of the exceptions and applying the rule of - The petitioner, however, cites one of the four exceptions provided for in the
noscitus a sociis, that the double-indemnity policy covers the insured against insurance contract and contends that the private petitioner's claim is barred by such
accidental death, whether caused by fault, negligence or intent of a third party which provision. It is there stated:
iNsuRanCe A2010 Dean Carale
pAgE 75♥

Exceptions —The company shall not be liable in respect of. unusual, and unforeseen. An accident is an event that takes place without one's
1. Bodily injury. foresight or expectation, an event that proceeds from an unknown cause, or is an
xxx xxx xxx unusual effect of a known cause and, therefore, not expected.
b. consequent upon. - The generally accepted rule is that, death or injury does not result from accident or
i) The insured persons attempting to commit suicide or wilfully exposing accidental means within the terms of an accident-policy if it is the natural result of the
himself to needless peril except in an attempt to save human life. insured's voluntary act, unaccompanied by anything unforeseen except the death or
- To repeat, the parties agree that Lim did not commit suicide. Nevertheless, the injury. There is no accident when a deliberate act is performed unless some
petitioner contends that the insured willfully exposed himself to needless peril and additional, unexpected, independent, and unforeseen happening occurs which
thus removed himself from the coverage of the insurance policy. That posture is produces or brings about the result of injury or death. In other words, where the death
arguable. But what is not is that, as the secretary testified, Lim had removed the or injury is not the natural or probable result of the insured's voluntary act, or if
magazine from the gun and believed it was no longer dangerous. He expressed something unforeseen occurs in the doing of the act which produces the injury, the
assured her that the gun was not loaded. It is submitted that Lim did not willfully resulting death is within the protection of policies insuring against death or injury from
expose himself to needless peril when he pointed the gun to his temple because the accident.
fact is that he thought it was not unsafe to do so. The act was precisely intended to - In the present case, while the participation of the insured in the boxing contest is
assure Nalagon that the gun was indeed harmless. voluntary, the injury was sustained when he slid, giving occasion to the infliction by
Disposition CA Affirmed his opponent of the blow that threw him to the ropes of the ring.
- The fact that boxing is attended with some risks of external injuries does not make
DE LA CRUZ v. CAPITAL INSURANCE any injuries received in the course of the game not accidental. In boxing as in other
equally physically rigorous sports, such as basketball or baseball, death is not
17 SCRA 554
ordinarily anticipated to result. If, therefore, it ever does, the injury or death can only
BARRERA; June 30, 1966 be accidental or produced by some unforeseen happening or event as what occurred
in this case.
NATURE - Furthermore, the policy involved herein specifically excluded from its coverage:
Appeal from the decision of the CFI of Pangasinan (e) Death or disablement consequent upon the Insured engaging in football, hunting,
pigsticking, steeplechasing, polo-playing, racing of any kind, mountaineering, or
FACTS motorcycling.
- Eduardo de la Cruz, employed in the Itogon-Suyoc Mines, Inc., was the holder of an - Death or disablement resulting from engagement in boxing contests was not
accident insurance policy underwritten by the Capital Insurance & Surety Co., Inc., for declared outside of the protection of the insurance contract. Failure of the defendant
the period beginning November 13, 1956 to November 12, 1957. insurance company to include death resulting from a boxing match or other sports
- On January 1, 1957, the Itogon-Suyoc Mines, Inc. sponsored a boxing contest among the prohibitive risks leads inevitably to the conclusion that it did not intend to
wherein the insured Eduardo de la Cruz participated. limit or exempt itself from liability for such death.
- In the course of his bout, Eduardo slipped and was hit by his opponent on the left Disposition The decision appealed from is affirmed
part of the back of the head, causing Eduardo to fall, with his head hitting the rope of
the ring.
- He was brought to the Baguio General Hospital, but he died as a result of FORTUNE INSURANCE v. CA (supra p.7)
hemorrhage, intracranial, left.
- Simon de la Cruz, the father and named beneficiary of the insured, filed a claim with PHIL HOME ASSURANCE CORP v. CA (EASTERN SHIPPING)
the insurance company for payment of the indemnity, but it was denied. 257 SCRA 468
- He instituted the action in the CFI of Pangasinan for specific performance.
KAPUNAN; June 20, 1996
- Defendant insurer set up the defense that the death of the insured, caused by his
participation in a boxing contest, was not accidental and, therefore, not covered by
NATURE
insurance
- Eastern Shipping Lines, Inc. loaded on board SS Eastern Explorer in Kobe, Japan, the
- The court rendered the decision in favor of the plaintiff, hence, the present appeal.
following shipment for carriage to Manila and Cebu, freight pre-paid and in good order
and condition: (a) 2 boxes internal combustion engine parts, consigned to William
ISSUE
Lines, Inc.; (b) 10 metric tons (334 bags) ammonium chloride, consigned to Orca's
WON the death of the insured was not accidental and, therefore, not covered by
Company; (c) 200 bags Glue 300, consigned to Pan Oriental Match Company; and (d)
insurance
garments, consigned to Ding Velayo. All consignations were made by virtue of a Bill of
HELD
Lading.
NO
- While the vessel was off Okinawa, a small flame was detected on the acetylene
- The terms "accident" and "accidental", as used in insurance contracts, have not
cylinder located in the accommodation area near the engine room. As the crew was
acquired any technical meaning, and are construed by the courts in their ordinary and
trying to extinguish the fire, the cylinder suddenly exploded, thus causing death and
common acceptation. Thus, the terms have been taken to mean that which happen by
severe injuries to the crew and instantly setting fire to the whole vessel.
chance or fortuitously, without intention and design, and which is unexpected,
- SS Eastern Explorer was then found to be a constructive total loss and its voyage
iNsuRanCe A2010 Dean Carale
pAgE 76♥

was declared abandoned. - There was no showing, and none was alleged by the parties, that the fire was caused
- After the fire was extinguished, the cargoes which were saved were loaded to by a natural disaster. On the contrary, there is strong evidence indicating that the
another vessel for delivery to their original ports of destination. ESLI charged the acetylene cylinder caught fire because of the fault and negligence of respondent ESLI,
consignees several amounts corresponding to additional freight and salvage charges. its captain and its crew:
- The charges were all paid by Philippine Home Assurance Corporation (PHAC) under (1) The acetylene cylinder which was fully loaded should not have been stored near
protest for and in behalf of the consignees. PHAC, as subrogee of the consignees, then the engine room where the heat generated therefrom could cause the acetylene
filed a complaint before the RTC of Manila, against ESLI to recover the sum paid under cylinder to explode by reason of spontaneous combustion. ESLI should have easily
protest on the ground that the same were actually damages directly brought about by foreseen that the acetylene cylinder, containing highly inflammable material, was in a
the fault, negligence, illegal act and/or breach of contract of ESLI. real danger of exploding.
- ESLI contended that it exercised the diligence required by law in the handling, (2) ESLI should have known that by storing the acetylene cylinder in the
custody and carriage of the shipment; that the fire was caused by an unforeseen accommodation area supposed to be reserved for passengers, it unnecessarily
event; that the additional freight charges are due and demandable pursuant to the Bill exposed its passengers to grave danger and injury.
of Lading; and that salvage charges are properly collectible under Act No. 2616, (3) The fact that the acetylene cylinder was checked, tested and examined and
known as the Salvage Law. subsequently certified as having complied with the safety measures and standards by
- RTC: dismissed PHAC's complaint and ruled in favor of ESLI. qualified experts before it was loaded in the vessel only shows to a great extent that
- The burning of the vessel was not the fault or negligence of defendant but a natural negligence was present in the handling of the acetylene cylinder after it was loaded
disaster or calamity. Salvage operations conducted by Fukuda Salvage Company was and while it was on board the ship.
perfectly a legal operation and charges made on the goods recovered were legitimate - From the foregoing premises, it indubitably follows that the cargo consignees cannot
charges. Section 19 of Act No. 2616, the Salvage Law is applicable. With respect to the be made liable to respondent carrier for additional freight and salvage charges.
additional freight charged by defendant from the consignees of the goods, the same Disposition Judgment appealed from is REVERSED and SET ASIDE. Respondent
are also validly demandable. Eastern Shipping Lines, Inc. is ORDERED to return to petitioner Philippine Home
- The burning of "EASTERN EXPLORER" while off Okinawa rendered it physically Assurance Corporation the amount it paid under protest in behalf of the consignees
impossible for defendant to comply with its obligation of delivering the goods to their herein.
port of destination pursuant to the contract of carriage. Under Article 1266 of the Civil
Code, the physical impossibility of the prestation extinguished defendant's obligation.
- Note: The goods subject of the present controversy were neither lost nor damaged
in transit by the fire that razed the carrier. In fact, these were all delivered to the
consignees, even if the transshipment took longer than necessary. What is at issue
therefore is not whether or not the carrier is liable for the loss, damage, or
deterioration of the goods transported by them but who, among the carrier, consignee
or insurer of the goods, is liable for the additional charges or expenses incurred by the
owner of the ship in the salvage operations and in the transshipment of the goods via PHILIPPINE AMERICAN LIFE INSURANCE COMPANY v. THE
a different carrier. In absolving respondent carrier of any liability, CA sustained the
AUDITOR GENERAL
trial court's finding that the fire that gutted the ship was a natural disaster or
calamity. 22 SCRA 135
SANCHEZ, JANUARY 18, 1968
ISSUE
WON the burning of the SS Eastern Explorer rendering it a constructive total loss was NATURE
a natural disaster or calamity PETITION FOR REVIEW of a ruling of the Auditor General.

HELD FACTS
NO - Philamlife, a domestic life insurance corp., and American International Reinsurance
Ratio In our jurisprudence, fire may not be considered a natural disaster or calamity Company (Airco), a corporation organized under the laws of the Republic of Panama,
since it almost always arises from some act of man or by human means. It cannot be entered into a REINSURANCE TREATY wherein Philamlife agrees to reinsure with Airco
an act of God unless caused by lightning or a natural disaster or casualty not on January 1950. Philamlife agreed to pay premiums for all reinsurances on an annual
attributable to human agency. premium basis.
Reasoning - In July 16, 1959, the Margin Law was approved and became effective, which exempts
certain “obligations from payment of margin fees, particularly contractual obligations
9 calling for payment of foreign exchange issued, approved and outstanding as of the
Section 1. When in case of shipwreck, the vessel or its cargo shall be beyond the control of the crew, or shall
date this Act takes place”.
have been abandoned by them, and picked up and conveyed to a safe place by other persons, the latter shall be
entitled to a reward for the salvage. - Central Bank of the Philippines collected P268,747.48 as foreign exchange margin
Those who, not being included in the above paragraph, assist in saving a vessel or its cargo from shipwreck, shall on Philamlife remittances to Airco purportedly totalling $610,998.63 and made
be entitled to like reward. subsequent to July 16, 1959. Philamlife filed a claim for refund on the ground that the
iNsuRanCe A2010 Dean Carale
pAgE 77♥

reinsurance premiums remitted were paid in pursuant to the January 1950 -Rationale of Margin Law: to reduce the excessive demand on and prevent further
reinsurance treaty, and therefore exempted. decline of our international reserves; to provide the Central Bank with an additional
- Monetary Board exempted Philamlife from payment of margin fee. However, Auditor instrument for effectively coping with the problem and achieving domestic and
of CB refused to pass in audit Philamlife’s claim for refund. Philamlife sought international stability of our currency; to reduce the excessive demand-for foreign
reconsideration but was denied, saying reinsurance treaty NOT EXEMPTED. exchange.
- implementation of Margin Law in accordance with police power
ISSUES 3. NO
1. WON the premia remitted were in pursuance of the reinsurance treaty between Reasoning
Philamlife and Airco of January 1959, a contract antedating the Margin Law, and - First, there is no concrete evidence that such imposition of the 25% margin fee is
therefore, Philamlife exempted from paying margin fee unreasonable, Second, if really continuance of the existing reinsurance treaty
2. WON Margin Law impairs the obligation of contract becomes unbearable, that contract itself provides that petitioner may
3. WON reinsurance contracts abroad would be made impractical by the imposition of potestatively write finis thereto on ninety days' written notice. Petitioner is
the 25% margin fee not forced to continue its reinsurance treaty indefinitely with Airco.
Disposition For the reasons given, the petition for review is hereby denied, and the
HELD ruling of the Auditor General of October 24, 1961 denying refund is hereby affirmed.
1. NO Costs against petitioner. So ordered.
- For an exemption to come into play, there must be a reinsurance policy or, as in the
reinsurance treaty provided, a "reinsurance cession" which may be automatic or FIELDMEN'S INSURANCE CO INC v. ASIAN SURETY &
facultative.
INSURANCE CO INC
Ratio A reinsurance policy is thus a contract of indemnity one insurer makes with
another to protect the first insurer from a risk it has already assumed. . . . In 34 SCRA 36
contradistinction, a reinsurance treaty is merely an agreement between two MAKALINTAL; July 31, 1970
insurance companies whereby one agrees to cede and the other to accept reinsurance
business pursuant to provisions specified in the treaty. The practice of issuing policies FACTS
by insurance companies includes, among other things, the issuance of reinsurance - On various dates between April 11, 1960 and Jan. 9, 1961 the Asian Surety &
policies on standard risks and also on substandard risks under special arrangements. Insurance Company, Inc. and the Fieldmen's Insurance Company, Inc. entered into 7
The lumping of the different agreements under a contract has resulted in the term reinsurance agreements under which the former, as the ceding company undertook to
known to the insurance world as 'treaties.' Such a treaty is, in fact, an agreement cede to the latter, as the reinsuring company, a specified portion of the amount of
between insurance companies to cover the different situations described. Reinsurance insurance underwritten by ASIAN upon payment to FIELDMEN'S of a proportionate
treaties and reinsurance policies are not synonymous. Treaties are contracts for share of the gross rate of the premium applicable with respect to each cession after
insurance; reinsurance policies or cessions are contracts of insurance. deducting a commission. Said agreements were to take effect from certain specific
Reasoning dates and were to be in force until cancelled by either party upon previous notice of at
- Even if reinsurance treaty preceded the Margin Law by over nine years, nothing in least 3 months by registered mail to the other party, the cancellation to take effect as
the treaty obligates Philamlife to remit to Airco a fixed, certain, and obligatory sum by of Dec. 31 of the year in which the notice was given.
way of reinsurance premiums. The reinsurance treaty per se cannot give rise to a - On Sep. 19, 1961 FIELDMEN'S, by means of registered mail, served notice to ASIAN
contractual obligation for the payment of foreign exchange. Philamlife’s obligation to of the former's desire to be relieved from all participation in its various agreements
remit reinsurance premiums becomes fixed and definite upon the execution of the with the latter effective Dec. 31, 1961. This communication, although admittedly
reinsurance cession. It is only after a reinsurance cession is made that payment of received by ASIAN on Sep. 25, 1961, did not elicit any reply from ASIAN.
reinsurance premium may be exacted, as it is only after Philamlife seeks to remit that - On Dec. 7, 1961 FIELDMEN'S sent another letter to ASIAN expressing regrets at
reinsurance premium that the obligation to pay the margin fee arises. alleged violations committed by the latter with respect to the various agreements
2. NO between them and reiterated its position that it would consider itself "no longer at risk
Ratio. Existing laws form part of the contract "as the measure of the obligation to for any reinsurance and/or cession" given by ASIAN which might be in force on Dec.
perform them by the one party and the right acquired by the other. If the obligation 31, 1961. Not having received any formal reply from ASIAN, FIELDMEN'S sent a new a
does not inhere and subsist in the contract itself, propio vigore, but in the letter on Feb. 17, 1962 reminding ASIAN of the cancellation of all the reinsurance
law applicable to the contract. treaties and cessions as of Dec. 31, 1961 and requested ASIAN to submit its final
accounting of all cessions made to the former for the preceding months when the
reinsurance agreements were in force.
- Meanwhile one of the risks reinsured with FIELDMEN'S issued in favor of the GSIS
Reasoning became a liability when the insured property was burned on February 16, 1962. Since
- . When petitioner entered into the reinsurance treaty of January 1, 1950 with Airco, it the policy was issued on July 1, 1961, it was supposed to expire on July 1, 1962. 2 The
did so with the understanding that the municipal laws of the Philippines at the time next day, Feb. 17, ASIAN immediately notified FIELDMEN'S of said fire loss.
said treaty was executed, became an unwritten condition thereof. Such municipal laws - FIELDMEN'S, relying on the sufficiency of its notice of termination dated September
constitute part of the obligation of contract. 19, 1961 and obviously bent on avoiding its liability under the reinsurance
iNsuRanCe A2010 Dean Carale
pAgE 78♥

agreements with ASIAN, filed a petition for declaratory relief with the CFI of Manila to made, FIELDMEN'S cannot avoid liability which arose by reason of the burning of the
seek a declaration that all the reinsurance contracts entered into between them had insured property.
terminated as of December 31, 1961 and to obtain an order directing ASIAN to render - With respect to the other 4 agreements, it would seem that the petition for
final accounting of the transactions between them with respect to said reinsurance declaratory relief is moot, and that no useful purpose would be served by defining the
treaties as of the cut-off date. respective rights and obligations of the parties thereunder. The said agreements have
- In its answer below ASIAN denied having received FIELDMEN'S letter dated Sep 19, been cancelled, and it does not appear that any claim by or liability in favor of the
1961, and argued that even assuming it did, FIELDMEN'S could not have terminated insured has actually arisen under any of the reinsurance cessions made prior to such
the reinsurance treaties as of Dec 31, 1961 because the letter was merely an cancellation. Future conflicts of the same nature as those which have motivated the
expression of FIELDMEN'S desire to cancel the treaties and not a formal notice of present action can of course be obviated by using more precise and definite
cancellation as contemplated in their reinsurance agreements. By way of special terminology in the reinsurance agreements which the parties may enter into
defense Asian contended that even if the Sep. 19 letter were considered sufficient henceforth.
notice of cancellation — thereby rendering the reinsurance agreements terminated as
of December 31, 1961 — the liability of FIELDMEN'S with respect to policies or EQUITABLE INSURANCE v. RURAL INSURANCE
cessions issued under two of the said agreements prior to their cancellation
4 SCRA 343
continued to have full force and effect until the stated expiry dates of such policies or
cessions. BARRERA; January 31, 1962
- On Dec. 4, 1962, the trial court declared 6 of the 7 reinsurance agreements in
question cancelled as of Dec 31, 1961. At the same time, it upheld ASIAN'S position FACTS
that all cessions of reinsurance made by it to FIELDMEN'S prior to the cancellation of - Plaintiff Equitable Insurance file a complaint with the CFI of Manila against defendant
the reinsurance treaties continued in full force and effect until expiry dates and Rural Insurance alleging, as first cause of action, that they entered into a reciprocal
ordered FIELDMEN'S to make an accounting of its business transactions with ASIAN facultative reinsurance agreement, wherein they agreed to cede to each other.
within 30 days. Pursuant to said agreement, plaintiff reinsured for P2k with defendant the stock
- On appeal to the CA, the decision of the trial court was substantially affirmed, with covered by fire insurance Policy No. 5880 issued by plaintiff which was later burned;
the slight modification that the order for accounting was eliminated, without prejudice the share of the loss of defendant as per insurance agreement was computed at
to the filing of a proper action between the parties for that purpose. P2,024 for which plaintiff sent to defendant a statement of account for payment by
the latter. Despite repeated demands by plaintiff, defendant refused to pay.
ISSUE - On the second cause of action, plaintiff reinsured for P2k with defendant stock
WON the cancellation as of Dec. 31, 1961 of the reinsurance treaties had the effect of covered by fire insurance Policy No. 6062 which also burned. Again, defendant refused
terminating also the liability of FIELDMEN'S as reinsurer with respect to policies or to pay its share of the loss of P1,334 hence said complaint.
cessions issued prior to the termination of the principal reinsurance contracts or - Defendant filed a motion to dismiss on the ground that it states no cause of action,
treaties as pursuant to Art VIII of the Reinsurance Agreement between the parties, before a
court action can be brought, the parties agreed to submit all disputes to a board of
HELD arbitrators. The Court denied the motion and required defendant to answer.
NO to the 2 reinsurance contracts - Defendant filed its answer, alleging that the nature of the agreement is “self-
- Of the 6 reinsurance contracts, 2 contain provisions, which clearly and expressly liquidating between the parties”, the reinsurer becoming the reinsured and vice versa;
recognize the continuing effectivity of policies ceded under them for reinsurance and that said agreement has not yet been abrogated so the liability of either to the
notwithstanding the cancellation of the contracts themselves. The said treaties other is not yet known. Defendant prayed that the complaint be dismissed and
provide "that in the event of termination of this Agreement . . ., the liability of the plaintiff filed a motion for judgment on the pleadings which the court denied.
Fieldmen's under current cessions shall continue in full force and effect until their - Instead of going into a formal hearing, the parties submitted their case for decision
natural expiry . . .;" and the 4th paragraph of Article VI of the Personal Accident stipulating the ff facts: defendant admits the allegations of the complaint and that
Reinsurance Treaty states: plaintiff admits that the issues of the complaint were not submitted to a Board of
"4. On the termination of this Agreement from any cause whatever, the liability of Arbitrators as provided in par VIII of the complaint, but instead referred it to the
the REINSURER (Fieldmen's) under any current cession including any amounts due Insurance Commissioner. The CFI rendered judgment in favor of plaintiff. Hence this
to be ceded under the terms of this Agreement and which are not cancelled in the appeal.
ordinary course of business shall continue in full force until their expiry unless the
COMPANY (Asian) shall, prior to the thirty-first December next following such notice, ISSUES
elect to withdraw the existing cessions . . ." 1. WON Equitable had no cause of action as the matter was not referred to the
- Thus, insofar as the two reinsurance agreements are concerned, there is clearly no decision of arbitrators
merit in FIELDMEN'S claim that their cancellation carried with it ipso facto the 2. WON in a facultative obligation the right to choose an alternative remedy lies only
termination of all reinsurance cessions thereunder. Such cessions continued to be in with the debtor (here the defendant) under Art 1206
force until their respective dates of expiration. Since it was under one of said
agreements that the reinsurance cession corresponding to the GSIS policy had been HELD
1. NO
iNsuRanCe A2010 Dean Carale
pAgE 79♥

- The requirement of submission for decision to 2 arbitrators or an umpire the matter WON the insured (Artex) has a cause of action against the reinsurer
of losses by fire or the liability of the parties thereto under Art VIII of the agreement
arises only if the same is disputed by one of the parties. In the instant case, there is HELD
no dispute between the parties; in the stipulation of facts defendant admitted that NO
plaintiff had paid its liability and defendant likewise admitted that it ignored plaintiff’s - Unless there is a specific grant in, or assignment of, the reinsurance contract in favor
demands for reimbursement for defendant’s failure to pay its share as reinsurer. As of the insured or a manifest intention of the contracting parties to the reinsurance
held in Maligad v United Assurance Co., if in the course of the settlement of a loss, the contract to grant such benefit or favor to the insured, the insured, not being privy to
action of the company or its agents amounts to refusal to pay, the company will be the reinsurance contract, has no cause of action against the reinsurer. It is expressly
deemed to have waived the condition precedent with reference to arbitration and a provided in Section 91 the Insurance Act 1 that "(T)he original insured has no interest
suit upon the policy will lie. in a contract of insurance."
2. NO
- There is no connection between Art 1206 NCC and the agreement of this action. The PERLA COMPANIA DE SEGUROS v. CA(LIM)
term “facultative” is used in reinsurance contracts, and it is so used in this particular
208 SCRA 487
case, merely to define the right of the reinsurer to accept or not to accept
participation in the risk insured. But once the share is accepted, as it was in the case NOCON; May 7, 1992
at bar, the obligation is absolute and the liability assumed thereunder can be
discharged by only one way—the payment of the share of the losses. NATURE
Disposition judgment appealed from the TC is affirmed Petition for certiorari by Perla Compania de Seguros and FOC Credit Corporation
seeking to annul and set aside CA decision revering the RTC decision for replevin and
damages.
ARTEX DEVELOPMENT CO INC v. WELLINGTON INSURANCE CO
INC FACTS
51 SCRA 352 - Private respondents spouses Herminio and Evelyn Lim executed a promissory note in
favor of Supercars, Inc. in the sum of P77,940.00, payable in monthly installments
TEEHANKEE; June 27, 1973
according to the schedule of payment indicated in said note, and secured by a chattel
mortgage over a brand new red Ford Laser, which is registered under the name of
FACTS
private respondent Herminio Lim and insured with the petitioner Perla Compania de
- Wellington Insurance Co. Inc. insured for P24,346,509.00 the buildings, stocks and
Seguros, Inc. (Perla for brevity) for comprehensive coverage.
machinery of plaintiff Artex Development Co. Inc. against loss or damage by fire or
- On the same date, Supercars, Inc., with notice to private respondents spouses,
lighting upon payment of the plaintiff of the corresponding premiums; that said
assigned to petitioner FCP Credit Corporation (FCP for brevity) its rights, title and
properties were insured for an additional sum of P883,034.00; that defendant insured
interest on said promissory note and chattel mortgage as shown by the Deed of
plaintiff against business interruption (use and occupancy) for P5,200,000.00;
Assignment.
Wellington entered into a contract of reinsurance with Alexander and Alexander, Inc.
- At around 2:30pm Nov9, 1982, said vehicle was carnapped while parked at the back
of New York. USA.
of Broadway Centrum. Evelyn Lim, who was driving said car before it was carnapped,
- The buildings, stocks and machineries of plaintiffs spinning department were
immediately called up the Anti-Carnapping Unit of the Philippine Constabulary to
burned.
report said incident and thereafter, went to the nearest police substation to make a
- Notice of the loss and damage was given the defendant; that as per report of the
police report regarding said incident.
adjusters, the total property loss of the plaintiff was the sum of P10,106,554.40 and
- On Nov10, 1982, Evelyn Lim reported said incident to the LTO in compliance with the
the total business interruption loss was P3,000,000.00;
insurance requirement. She also filed a complaint with the Headquarters.
- That defendant has paid to the plaintiff the sum of P6,481,870.07 of the property
Constabulary Highway Patrol Group.
loss suffered by plaintiff and P1,864,134.08 on its business interruption loss, leaving a
- On Nov11, 1982, private respondent filed a claim for loss with the petitioner Perla
balance of P3,624,683.43 and P1,748,460.00, respectively.
but said claim was denied on Nov18, 1982 on the ground that Evelyn Lim, who was
- The counsel for Artex filed a Manifestation saying that in view of the Deeds of
using the vehicle before it was carnapped, was in possession of an expired driver's
Discharge and Collateral Agreement, the only remaining liability subject of litigation
license at the time of the loss of said vehicle which is in violation of the authorized
shall be the proportion of the loss reinsured with or through Alexander and Alexander,
driver clause of the insurance policy, which states, to wit:
Inc. of New York, USA, namely, P397,813.00.
"AUTHORIZED DRIVER:
- The document recited further that Artex acknowledges receipt of the sum of P3.6M
Any of the following: (a) The Insured (b) Any person driving on the Insured's order,
paid by the insurer in full and final settlement of all or any claims of Artex against its
or with his permission. Provided that the person driving is permitted, in accordance
insurer. It discharges its insurer from all actions, proceedings, claims, demands, costs
with the licensing or other laws or regulations, to drive the Scheduled Vehicle, or
and expenses in respect thereof.
has been permitted and is not disqualified by order of a Court of Law or by reason of
- With regard the balance unpaid, Wellington contends that Artex should have been
any enactment or regulation in that behalf."
directed against the reinsurers to cover the liability and not against Wellington.
- On Nov17, 1982, private respondents requested from petitioner FCP for a suspension
of payment on the monthly amortization agreed upon due to the loss of the vehicle
ISSUE
iNsuRanCe A2010 Dean Carale
pAgE 80♥

and, since the carnapped vehicle was insured with petitioner Perla, said insurance accessory contract to the promissory note. Being the principal contract, the
company should be made to pay the remaining balance of the promissory note and promissory note is unaffected by whatever befalls the subject matter of the accessory
the chattel mortgage contract. contract.
- Perla, however, denied private respondents' claim. Consequently, petitioner FCP - The unpaid balance on the promissory note should be paid, and not just the
demanded that private respondents pay the whole balance of the promissory note or installments due and payable before the automobile was carnapped, as erronously
to return the vehicle but the latter refused. held by the CA
- On July25, 1983, petitioner FCP filed a complaint against private respondents, who in - However, this does not mean that private respondents are bound to pay the interest,
turn filed an amended third party complaint against petitioner Perla on Dec8, 1983. litigation expenses and attorney's fees stipulated in the promissory note. Because of
After trial on the merits, TC ordered sps Lim to pay jointly and severally, plaintiff the the peculiar relationship between the three contracts in this case, i. e., the promissory
sum of P55,055.93 plus interest thereon at the rate of 24% per annum from July 2, note, the chattel mortgage contract and the insurance policy, the Court is compelled
1983 until fully paid; as well as the cost of suit. It also ordered the dismissal of the to construe all three contracts as intimately interrelated to each other, despite the
Third party complaint against Third-Party Defendant. fact that at first glance there is no relationship whatsoever between the parties
- Upon appeal, CA reversed said decision thereto.
- After petitioners' separate MFRs were denied by CA, petitioners filed these separate - Under the promissory note, Lim spouses are obliged to pay Supercars, Inc. the
petitions for review on certiorari. amount stated therein in accordance with the schedule provided for. To secure said
ISSUE promissory note, private respondents constituted a chattel mortgage in favor of
1. WON there was grave abuse of discretion on the part of the appellate court in Supercars, Inc. over the automobile the former purchased from the latter. The chattel
holding that private respondents did not violate the insurance contract because the mortgage, in turn, required private respondents to insure the automobile and to make
authorized driver clause is not applicable to the "Theft" clause of said Contract the proceeds thereof payable to Supercars, Inc. The promissory note and chattel
2. WON the loss of the collateral exempted the debtor from his admitted obligations mortgage were assigned by Supercars, Inc. to petitioner FCP, with the knowledge of
under the promissory note particularly the payment of interest, litigation expenses private respondents. Private respondents were able to secure an insurance policy
and attorney's fees from petitioner Perla, and the same was made specifically payable to petitioner FCP.
- From the abovementioned provision that upon the loss of the insured vehicle, the
HELD insurance company Perla undertakes to pay directly to the mortgagor or to their
1. NO assignee, FCP, the outstanding balance of the mortgage at the time of said loss under
- The comprehensive insurance policy issued by petitioner Perla undertook to the mortgage contract. If the claim on the insurance policy had been approved by
indemnify the private respondents against loss or damages to the car (a) by petitioner Perla, it would have paid the proceeds thereof directly to petitioner FCP, and
accidental collision or overturning, or collision or overturning consequent upon this would have had the effect of extinguishing private respondents' obligation to
mechanical breakdown or consequent upon wear and tear; (b) by fire, external petitioner FCP. Therefore, private respondents were justified in asking petitioner FCP
explosion, self-ignition or lightning or burglary, housebreaking or theft; and (c) by to demand the unpaid installments from petitioner Perla.
malicious act. - Because petitioner Perla had unreasonably denied their valid claim, private
- Where a car is unlawfully and wrongfully taken without the owner's consent or respondents should not be made to pay the interest, liquidated damages and
knowledge, such taking constitutes theft, and, therefore, it is the "THEFT" clause, and attorney's fees as stipulated in the promissory note. As mentioned above, the contract
not the "AUTHORIZED DRIVER" clause, that should apply.The risk against accident is of indemnity was procured to insure the return of the money loaned from petitioner
distinct from the risk against theft. The 'authorized driver clause' in a typical insurance FCP, and the unjustified refusal of petitioner Perla to recognize the valid claim of the
policy as in contemplation or anticipation of accident in the legal sense in which it private respondents should not in any way prejudice the latter.
should be understood, and not in contemplation or anticipation of an event such as - Private respondents can not be said to have unduly enriched themselves at the
theft. The distinction often seized upon by insurance companies in resisting claims expense of FCP since they will be required to pay the latter the unpaid balance of its
from their assureds between death occurring as a result of accident and death obligation under the promissory note.
occurring as a result of intent may apply to the case at bar. - In view of the foregoing discussion, We hold that the Court of Appeals did not err in
- If the insured vehicle had figured in an accident at the time she drove it with an requiring petitioner Perla to indemnify private respondents for the loss of their insured
expired license, then, appellee Perla Compania could properly resist appellants' claim vehicle. However, the latter should be ordered to pay petitioner FCP the amount of
for indemnification for the loss or destruction of the vehicle resulting from the P55,055.93, representing the unpaid installments from December 30, 1982 up to July
accident. But in the present case, the loss of the insured vehicle did not result from an 1, 1983, as shown in the statement of account prepared by petitioner FCP, 18 plus
accident where intent was involved; the loss in the present case was caused by theft, legal interest from July 2, 1983 until fully paid.
the commission of which was attended by intent." - As to the award of moral damages, exemplary damages and attorney's fees, private
- There is no causal connection between the possession of a valid driver's license and respondents are legally entitled to the same since Perla had acted in bad faith by
the loss of a vehicle. To rule otherwise would render car insurance practically a sham unreasonably refusing to honor the insurance claim of the private respondents.
since an insurance company can easily escape liability by citing restrictions which are Besides, awards for moral and exemplary damages, as well as attorney's fees are left
not applicable or germane to the claim, thereby reducing indemnity to a shadow. to the sound discretion of the Court. Such discretion, if well exercised, will not be
2. The court agrees with FCP that Lim spouses are not relieved of their obligation to disturbed on appeal.
pay the former the installments due on the promissory note on account of the loss of Disposition the assailed decision of the CA is hereby MODIFIED to require private
the automobile. The chattel mortgage constituted over the automobile is merely an respondents to pay petitioner FCP the amount of P55,055.93, with legal interest from
iNsuRanCe A2010 Dean Carale
pAgE 81♥

July 2, 1983 until fully paid. The decision appealed from is hereby affirmed as to all said third party, arose from the offense charged in the criminal case, from which the
other respects. No pronouncement as to costs. injured (Jovencio Poblete, Sr.) has sought to recover civil damages. Hence, such claim
of petitioner against the insurance company cannot be regarded as not related to the
SHAFER v. JUDGE criminal action.
- A third party complaint is a device allowed by the rules of procedure by which the
167 SCRA 386
defendant can bring into the original suit a party against whom he will have a claim
PADILLA; November 14, 1988 for indemnity or remuneration as a result of a liability established against him in the
original suit. 13 Third party complaints are allowed to minimize the number of lawsuits
NATURE and avoid the necessity of bringing two (2) or more actions involving the same subject
Petition for review on certiorari matter. They are predicated on the need for expediency and the avoidance of
unnecessary lawsuits. If it appears probable that a second action will result if the
FACTS plaintiff prevails, and that this result can be avoided by allowing the third party
- Sherman Shafer obtained a private car policy over his Ford Laser from Makati complaint to remain, then the motion to dismiss the third party complaint should be
Insurance Company, Inc., for third party liability. During the effectivity of the policy, denied.
an information for reckless imprudence resulting in damage to property and serious - Compulsory Motor Vehicle Liability Insurance (third party liability, or TPL) is primarily
physical injuries was filed against shafer. The information said that on or about the intended to provide compensation for the death or bodily injuries suffered by innocent
17th day of May 1985, in the City of Olongapo. Shafer hit and bumped a Volkswagen third parties or passengers as a result of a negligent operation and use of motor
car owned and driven by Felino llano y Legaspi, thereby causing damage in the total vehicles. The victims and/or their dependents are assured of immediate financial
amount of P12,345.00 and as a result thereof one Jovencio Poblete, Sr. who was on assistance, regardless of the financial capacity of motor vehicle owners.
board of the said Volkswagen car sustained physical injuries which injuries causing - The liability of the insurance company under the Compulsory Motor Vehicle Liability
deformity on the face. The owner of the damaged Volkswagen car filed a separate Insurance is for loss or damage. Where an insurance policy insures directly against
civil action against petitioner for damages, while Jovencio Poblete, Sr., who was a liability, the insurer's liability accrues immediately upon the occurrence of the injury or
passenger in the Volkswagen car, did not reserve his right to file a separate civil event upon which the liability depends, and does not depend on the recovery of
action for damages. Instead, in the course of the trial in the criminal case, Poblete, Sr. judgment by the injured party against the insured.
testified on his claim for damages for the serious physical injuries which he claimed to - The injured for whom the contract of insurance is intended can sue directly the
have sustained as a result of the accident. insurer. The general purpose of statutes enabling an injured person to proceed
- The court issued an order dismissing the third party complaint on the ground that it directly against the insurer is to protect injured persons against the insolvency of the
was premature, based on the premise that unless the accused (herein petitioner) is insured who causes such injury, and to give such injured person a certain beneficial
found guilty and sentenced to pay the offended party (Poblete Sr.) indemnity or interest in the proceeds of the policy, and statutes are to be liberally construed so that
damages, the third party complaint is without cause of action. The court further stated their intended purpose may be accomplished. It has even been held that such a
that the better procedure is for the accused (petitioner) to wait for the outcome of the provision creates a contractual relation which inures to the benefit of any and every
criminal aspect of the case to determine whether or not the accused, also the third person who may be negligently injured by the named insured as if such injured person
party plaintiff, has a cause of action against the third party defendant for the were specifically named in the policy.
enforcement of its third party liability (TPL) under the insurance contract. 6 Petitioner - In the event that the injured fails or refuses to include the insurer as party defendant
moved for reconsideration of said order, but the motion was denied; hence, this in his claim for indemnity against the insured, the latter is not prevented by law to
petition. avail of the procedural rules intended to avoid multiplicity of suits. Not even a "no
action" clause under the policy-which requires that a final judgment be first obtained
ISSUE against the insured and that only thereafter can the person insured recover on the
WON the court a quo erred in dismissing petitioner's third party complaint on the policy can prevail over the Rules of Court provisions aimed at avoiding multiplicity of
ground that petitioner had no cause of action yet against the insurance company suits.
Disposition instant petition is GRANTED. The questioned order dated 24 April 1987
HELD is SET ASIDE and a new one entered admitting petitioner's third party complaint
YES against the private respondent Makati Insurance Company, Inc.
- There is no need on the part of the insured to wait for the decision of the trial court
finding him guilty of reckless imprudence. The occurrence of the injury to the third
VDA DE MAGLANA v. CONSOLACION
party immediately gave rise to the liability of the insurer under its policy. Respondent
insurance company's contention that the third party complaint involves extraneous 212 SCRA 268
matter which will only clutter, complicate and delay the criminal case is without merit. ROMERO; August 6, 1992
The civil aspect of the offense charged, i.e., serious physical injuries allegedly suffered
by Jovencio Poblete, Sr., was impliedly instituted with the criminal case. Petitioner NATURE
may thus raise all defenses available to him insofar as the criminal and civil aspects of Petition for certiorari
the case are concerned. The claim of petitioner for payment of indemnity to the
injured third party, under the insurance policy, for the alleged bodily injuries caused to FACTS
iNsuRanCe A2010 Dean Carale
pAgE 82♥

- Lope Maglana was an employee of the Bureau of Customs whose work station was at insured who causes such injury, and to give such injured person a certain beneficial
Lasa, here in Davao City. One day, when he was on his way to his work, he met an interest in the proceeds of the policy . . ." Since petitioners had received from AFISCO
accident that resulted in his death. He died on the spot. the sum of P5,000.00 under the no-fault clause, AFISCO's liability is now limited to
- The PUJ jeep that bumped the deceased was driven by Pepito Into, operated and P15,000.00.
owned by defendant Destrajo. From the investigation conducted by the traffic - However, we cannot agree that AFISCO is likewise solidarily liable with Destrajo. In
investigator, the PUJ jeep was overtaking another passenger jeep that was going Malayan Insurance Co., Inc. v. Court of Appeals, this Court had the opportunity to
towards the city poblacion. While overtaking, the PUJ jeep of defendant Destrajo resolve the issue as to the nature of the liability of the insurer and the insured vis-a-
running abreast with the overtaken jeep, bumped the motorcycle driven by the vis the third party injured in an accident. We categorically ruled thus: While it is true
deceased. The point of impact was on the lane of the motorcycle and the deceased that where the insurance contract provides for indemnity against liability to third
was thrown from the road and met his untimely death. persons, such third persons can directly sue the insurer, however, the direct liability of
- Heirs of Lope Maglana, Sr. filed an action for damages and attorney's fees against the insurer under indemnity contracts against third party liability does not mean that
operator Patricio Destrajo and the Afisco Insurance Corporation (AFISCO). An the insurer can be held solidarily liable with the insured and/or the other parties found
information for homicide thru reckless imprudence was also filed against Pepito Into. at fault. The liability of the insurer is based on contract; that of the insured is based on
- During the pendency of the civil case, Into was sentenced to suffer an indeterminate tort. In the case at bar, petitioner as insurer of Sio Choy, is liable to respondent
penalty, with all the accessory penalties provided by law, and to indemnify the heirs of Vallejos (the injured third party), but it cannot, as incorrectly held by the trial court, be
Lope Maglana, Sr. in the amount of twelve thousand pesos with subsidiary made "solidarily" liable with the two principal tortfeasors, namely respondents Sio
imprisonment in case of insolvency, plus five thousand pesos in the concept of moral Choy and San Leon Rice Mill, Inc. For if petitioner-insurer were solidarily liable with
and exemplary damages with costs. No appeal was interposed by accused who later said, two (2) respondents by reason of the indemnity contract against third party
applied for probation. liability under which an insurer can be directly sued by a third party this will result in a
- The lower court rendered a decision finding that Destrajo had not exercised sufficient violation of the principles underlying solidary obligation and insurance contracts.
diligence as the operator of the jeepney ordering him to pay plaintiffs the sum for loss - While in solidary obligations, the creditor may enforce the entire obligation against
of income; funeral and burial expenses of the deceased; moral damages, and one of the solidary debtors, in an insurance contract, the insurer undertakes for a
attorney's fees and costs of suit. The defendant insurance company is ordered to consideration to indemnify the insured against loss, damage or liability arising from an
reimburse defendant Destrajo whatever amounts the latter shall have paid only up to unknown or contingent event.
the extent of its insurance coverage. - Similarly, petitioners herein cannot validly claim that AFISCO, whose liability under
- Petitioners filed a motion for the reconsideration of the second paragraph of the the insurance policy is also P20,000.00, can be held solidarily liable with Destrajo for
decision contending that AFISCO should not merely be held secondarily liable because the total amount of P53,901.70 in accordance with the decision of the lower court.
the Insurance Code provides that the insurer's liability is "direct and primary and/or Since under both the law and the insurance policy, AFISCO's liability is only up to
jointly and severally with the operator of the vehicle, although only up to the extent of P20,000.00, the second paragraph of the dispositive portion of the decision in
the insurance coverage." Hence, they argued that the P20,000.00 coverage of the question may have unwittingly sown confusion among the petitioners and their
insurance policy issued by AFISCO, should have been awarded in their favor. counsel. What should have been clearly stressed as to leave no room for doubt was
- AFISCO argued that since the Insurance Code does not expressly provide for a the liability of AFISCO under the explicit terms of the insurance contract.
solidary obligation, the presumption is that the obligation is joint. Disposition present petition is hereby GRANTED. The award of P28,800.00
- The lower court denied the motion for reconsideration ruling that since the insurance representing loss of income is INCREASED to P192,000.00 and the death indemnity of
contract "is in the nature of suretyship, then the liability of the insurer is secondary P12,000.00 to P50,000.00.
only up to the extent of the insurance coverage."
- Petitioners filed a second motion for reconsideration reiterating that the liability of
the insurer is direct, primary and solidary with the jeepney operator because the
petitioners became direct beneficiaries under the provision of the policy which, in
effect, is a stipulation pour autrui. This motion was likewise denied for lack of merit.

ISSUE FAR EASTERN SURETY v. MISA


WON AFISCO can be held directly liable
25 SCRA 663
HELD REYES; October 26, 1968
YES
- As this Court ruled in Shafer vs. Judge, RTC of Olongapo City, Br. 75, "[w]here an NATURE
insurance policy insures directly against liability, the insurer's liability accrues Appeal by petition for review from a CA judgment
immediately upon the occurrence of the injury or even upon which the liability
depends, and does not depend on the recovery of judgment by the injured party FACTS
against the insured." - Socorro Dancel Vda.de Misa and Araceli Pinto hired a taxi cab operated by La
- The underlying reason behind the third party liability (TPL) of the Compulsory Motor Mallorca on September 3, 1957. The taxi they were riding in collided with a gravel and
Vehicle Liability Insurance is "to protect injured persons against the insolvency of the sand truck resulting to injuries to both Misa and Pinto.
iNsuRanCe A2010 Dean Carale
pAgE 83♥

- The two passengers instituted a suit for damages against La Mallorca who, while FACTS
denying responsibility, instituted a third party complaint against Far Eastern Surety to - vehicular accident with 2 children running across the path of a Chevrolet "Carry-All",
recoup from the latter any award for damages that might be recovered by the belonging to a partnership known as Diman & Company driven by its driver, Perfecto
passengers. Amar, as it was passing a national highway at barrio Makiling Calamba, Laguna. They
- It would appear from the case that a sticker was placed in all the taxis of La Mallorca were killed. It was insured with the Empire Insurance Co., Inc. under a so-called
stating that passengers of the taxis were insured against accidents. This was done to 'comprehensive coverage" policy, loss by theft excluded. The policy was in force at
entice the public into patronizing La Mallorca. the time of the accident.
- The trial court awarded to Misa and Pinto actual, moral and exemplary damages, and - Placida Peza, the managing partner of Diman & Co. filed a claim with Empire, for
attorney’s fees payable by La Mallorca and sentenced Far Eastern to pay La Mallorca payment of compensation to the family of the 2 children who died as a result of the
P10,000. on its third party liability insurance. accident. Empire refused to pay on the ground that the driver had no authority to
- On appeal, the CA, while holding that the collision was due to the fault of the driver operate the vehicle, a fact which it expressly excepted from liability under the policy.
of the gravel and sand truck, found the taxi company liable for damages to the What Peza did was to negotiate directly with the deceased children father for an out-
passengers on the strength of its representation contained in the sticker above noted of-court settlement. The father agreed to accept P6,200.00 in fun settlement of the
that the passengers were insured against accidents. In so ruling, the CA overruled the liability of the vehicles owner and driver, and Peza paid him this sum.
defense of the insurance company to the effect that it was responsible only if the - Peza thereafter sued Empire to recover this sum of P6,200.00 as actual damages, as
insured, La Mallorca, was involved in accidents caused by, or arising out of, the use of well as P20,000.00 as moral damages, P10,000.00 as exemplary damages, and
the motor vehicle. A motion for reconsideration was filed in and dismissed by the CA. P10,000.00 as attorney's fees. She amended her complaint shortly thereafter to
include Diman & Co. as alternative party plaintiff.
ISSUE - Empire's basic defense to the suit was anchored on the explicit requirement in the
WON Far Eastern Surety is liable to the insured on its insurance policy policy limiting the operation of the insured vehicle to the "authorized driver" therein
defined, namely, (a) the insured, or (b) any person driving on the insured order or with
HELD his permission, provided that-
NO ... that the person driving is permited in accordance with the licensing or other laws
- The award for damages made to the passengers was exclusively predicated on the or regulations to drive the Motor vehicle or has been so permitted and is not
representation made by La Mallorca that its passengers were insured against disqualified by order of the Court of Law of by reason of any enactment or
accidents and not because it was at fault in causing the accident. regulation in that behalf from driving such Motor Vehicle.-
Reasoning - driver Perfecto Amar, only having a temporary operator's permit (TVR) [already
- In this case, the findings of the CA and the trial court that the causative factor of the expired] his driver’s license having earlier been confiscated by an agent of the Land
mishap was the negligence of the gravel and truck driver would have been sufficient Transportation Commission for an alleged violation of Land Transportation and Traffic
to relieve the taxi company of any liability arising from the accident. However, in view Rules, was not permitted by law and was in truth disqualified to operate any motor
of the sticker in all of its taxicabs, La Mallorca has insured its passengers against vehicle; Peza attempted to neutralize that fact by(1) the issuance of the TVR by the
accidents, whether it was at fault or not. In other words, La Mallorca accepted the LTC officer to Amar; in proof of the proposition that there was no reason for
responsibility for damages or injuries to passengers even if it had no fault at all. confiscation of Amar's license (2) Amar's license had not expired, but had been
- In the case of the insurance company, the SC ruled that it neither authorized nor renewed.
consented to the representations made by the taxi company to its passengers. As - Judge Alikpala did not admit such evidence
such, the liability of the said insurance company based on its insurance contract is
limited to the recovery by the insured of all sums, cost and expenses which the ISSUES
insured shall become legally liable. The insurance company therefore cannot be held 1. WON Judge Alikapala committed grave abuse of discretion in not admitting
liable for the award. evidence
- The taxi company is adjudged to be the sole party responsible for the award. 2. WON confiscation of license and expiration of TVR of the driver would serve as bar
Disposition The decision of the CA is modified by eliminating the award against Far for Peza in recovering from Empire
Eastern.
HELD
PEZA v. ALIKPALA 1. NO
- Even positing error in the Judge's analysis of the evidence attempted to be
160 SCRA 31
introduced and his rejection thereof, it is clear that it was at most an error of
NARVASA; April 15, 1988 judgment, not such an error as may be branded a grave abuse of discretion, i.e., such
capricious and whimsical exercise of judgment as is equivalent to lack of jurisdiction,
NATURE against which the writ of certiorari will lie. In any event, the established principle is
Motion praying that Judge Alikpala be declared guilty of contempt of court for having "that ruling of the trial court on procedural questions and on admissibility of evidence
decided the case on the merits despite the pendency in this Court of the certiorari during the course of the trial are interlocutory in nature and may not be the subject of
action instituted by the plaintiffs separate appeal or review on certiorari, but are to be assigned as errors and reviewed
iNsuRanCe A2010 Dean Carale
pAgE 84♥

in the appeal properly taken from the decision rendered by the trial court on the claim to recover against the owner of the vehicle responsible for the accident shall be
merits of the case. maintained.
- In the meantime, Judge Alikpala rendered judgment on the merits, since the case Perla denied its liability under the above provision and said that the insurer of the
was then already ripe for adjudication. The judgment ordered dismissal of the case for vehicle that the respondents were riding (Malayan Insurance in this case) should be
failure on the part of the plaintiff to prove their cause of action against Empire. Notice liable. Its 2 MFRs denied, Perla filed this action
of the judgment was served on the parties in due course.
2. YES ISSUE
- It would seem fairly obvious that whether the LTC agent was correct or not in his WON Perla is the insurer liable to indemnify under Sec. 378
opinion that driver Amar had violated some traffic regulation warranting confiscation
of his license and issuance of a TVR in lieu thereof, this would not alter the undisputed HELD
fact that Amar's licence had indeed been confiscated and a TVR issued to him, and NO
the TVR had already expired at the time that the vehicle being operated by him killed Ratio The law is very clear – the claim shall lie against the insurer of the vehicle in
two children by accident. Neither would proof of the renewal of Amar's license change which the “occupant xxx is riding,” and no other. The claimant is not free to choose
the fact that it had really been earlier confiscated by the LTC agent. from which insurer he will claim the “no fault indemnity,” as the law, by using the
Disposition petition is DISMISSED for lack of merit word “shall”, makes it mandatory that the claim be made against the insurer of the
vehicle in which the occupant is riding, mounting or dismounting from.
Reasoning
- the rules on claims under the “no fault indemnity” provision, where proof of fault or
negligence is not necessary for payment of any claim for death or injury to a
passenger or to a 3rd party, are established:
1. A claim may be made against one motor vehicle only.
2. If the victim is an occupant of a vehicle, the claim shall lie against the insurer of the
vehicle in which he is riding, mounting or dismounting from.
PERLA COMPANIA DE SEGUROS v. ANCHETA 3. In any other case (i.e. if the victim is not an occupant of a vehicle), the claim shall
lie against the insurer of the directly offending vehicle.
164 SCRA 144
4. In all cases, the right of the party paying the claim to recover against the owner of
CORTES; August 8, 1988 the vehicle responsible for the accident shall be maintained.
-That the vehicle ridden might not be the one that caused the accident is of no
NATURE moment since the law itself provides that the party paying the claim under Sec. 378
Petition for certiorari and prohibition with prelim injunction to review orders of CFI may recover against the owner of the vehicle responsible for the accident. This is
Camarines Norte precisely the essence of “no fault indemnity” insurance which was introduced to and
made part of our laws in order to provide victims of vehicular accidents or their heirs
FACTS immediate compensation, although in a limited amount, pending final determination
- Perla was the insurer of a Superlines bus which figured in a collision with a III Scout of who is responsible for the accident and liable for the victims' injuries or death. In
(it’s a kind of vehicle). Injured passengers of the latter (and respondents in this case) turn, the “no fault indemnity” provision is part and parcel of the Code provisions on
filed a complaint for damages against Superlines, the bus driver, and Perla (as insurer compulsory motor vehicle liability insurance and should be read together with the
of the bus). CFI Judge Ancheta ordered that Perla should pay the respondents requirement for compulsory passenger and/or 3rd party liability insurance (Sec. 377)
immediately the P5000 under the “no fault clause” as provided in Sec. 378. which was mandated in order to ensure ready compensation for victims of vehicular
Sec. 378: Any claim for death or injury to any passenger or 3rd party pursuant to the accidents.
provisions of this chapter shall be paid without the necessity of proving fault or -Irrespective of whether or not fault or negligence lies with the driver of the Superlines
negligence of any kind. Provided, That for purposes of this section bus, as respondents were not occupants of the bus, they cannot claim the “no fault
(i) The indemnity in respect of any one person shall not exceed P5,000; indemnity” provided in Sec. 378 from Perla. The claim should be made against the
(ii) The following proofs of loss, when submitted under oath, shall be sufficient insurer of the vehicle they were riding.
evidence to substantiate the claim: Disposition Petition GRANTED. Orders of CFI ordering Perla to pay respondents
(a) Police report of accident, and immediately P5000 ANNULLED and SET ASIDE
(b) Death certificate and evidence sufficient to establish, the proper payee, or
(c) Medical report and evidence of medical or hospital disbursement in respect of
which refund is claimed; WESTERN GUARANTY CORPORATION v. CA (RODRIGUEZ & DE
(iii) Claim may be made against one motor vehicle only. In the case of an occupant of DIOS TRANS)
a vehicle, claim shall lie against the insurer of the vehicle in which the occupant is 185 SCRA 652
riding, mounting or dismounting from. In any other case, claim shall lie against the
FELICIANO; July 20, 1990
insurer of the directly offending vehicle. In all cases, the right of the party paying the
NATURE
iNsuRanCe A2010 Dean Carale
pAgE 85♥

Petition for review the decision of CA affirming in toto the damages awarded to private generate liability for petitioner Western. A car accident may, for instance, result in
respondent by the trial court. injury to internal organs of a passenger or third party, without any accompanying
amputation or loss of an external member (e.g., a foot or an arm or an eye). But such
FACTS internal injuries are surely covered by Section I of the Master Policy, since they
- Respondent Priscilla E. Rodriguez was struck by a De Dios passenger bus owned by certainly constitute bodily injuries.
respondent De Dios Transportation Co., Inc. Priscilla was thrown to the ground, hitting - The Schedule of Indemnities does not purport to restrict the kinds of damages that
her forehead. She was treated at the Protacio Emergency Hospital and later on may be awarded against Western once liability has arisen. Section 1, quoted above,
hospitalized at the San Juan De Dios Hospital. Her face was permanently disfigured, does refer to certain "Limits of Liability" which in the case of the third party liability
causing her serious anxiety and moral distress. Respondent bus company was insured section of the Master Policy, is apparently P50,000.00 per person per accident. Within
with petitioner Western Guaranty Corporation ("Western") under its Master Policy this over-all quantitative limit, all kinds of damages allowable by law" actual or
which provided, among other things, for protection against third party liability, the compensatory damages"; "moral damages'; "nominal damages"; "temperate or
relevant section reading as follows: moderate damages"; "liquidated damages"; and "exemplary damages" ? may be
Section 1. Liability to the Public ? Company will, subject to the Limits of Liability, pay awarded by a competent court against the insurer once liability is shown to have
all sums necessary to discharge liability of the insured in respect of ? arisen, and the essential requisites or conditions for grant of each species of damages
(a) death of or bodily injury to or damage to property of any passenger as defined are present. It appears to us self-evident that the Schedule of Indemnities was not
herein. intended to be an enumeration, much less a closed enumeration, of the specific kinds
(b) death of or bodily injury or damage to property of any THIRD PARTY as defined of damages which may be awarded under the Master Policy Western has issued.
herein in any accident caused by or arising out of the use of the Schedule Vehicle, - The reading urged by Western of the Schedule of Indemnities comes too close to
provided that the liability shall have first been determined. In no case, however, working fraud upon both the insured and the third party beneficiary of Section 1,
shall the Company's total payment under both Section I and Section 11 combined quoted above. For Western's reading would drastically and without warning limit the
exceed the Limits of Liability set forth herein. With respect to death of or bodily otherwise unlimited and comprehensive scope of liability assumed by the insurer
injury to any third party or passenger, the company's payment per victim in any one Western under Section 1: "all sums necessary to discharge liability of the insured in
accident shall not exceed the limits indicated in the Schedule of indemnities respect of [bodily injury to a third party]". This result- which is not essentially different
provided for in this policy excluding the cost of additional medicines, and such other from taking away with the left hand what had been given with the right hand we must
burial and funeral expenses that might have been incurred. avoid as obviously repugnant to public policy. If what Western now urges is what
- Respondent Priscilla Rodriguez filed a complaint for damages before the Regional Western intended to achieve by its Schedule of Indemnities, it was incumbent upon
Trial Court of Makati against De Dios Transportation Co. and Walter A. Saga Western to use language far more specific and precise than that used in fact by
Respondent De Dios Transportation Co., in turn, filed a third-party complaint against Western, so that the insured, and potential purchasers of its Master Policy, and the
its insurance carrier, petitioner Western. On 6 August 1985, the trial court rendered a Office of the Insurance Commissioner, may be properly informed and act accordingly.
decision in favor of respondent Priscilla E. Rodriguez, awarding moral damages, lossof - Moreover, an insurance contract is a contract of adhesion. The rule is well
earning and attorney's fees among others. entrenched in our jurisprudence that the terms of such contract are to be construed
- On appeal, the Court of Appeals affirmed in toto the decision of the trial court. strictly against the party which prepared the contract, which in this case happens to
- Petitioner contends that it cannot be held liable for loss of earnings, moral damages be petitioner Western.
and attorney's fees because these items are not among those included in the Disposition Petition denied.
Schedule of Indemnities set forth in the insurance policy.
SUMMIT GUARANTY & INSURANCE COMPANY v. ARNALDO
ISSUE
WON petitioner can be held liable for loss of earnings, moral damages and attorney's 158 SCRA 332
fees GANCAYCO; February 29, 1988
HELD NATURE
YES PETITION to review the order of the Insurance Commissioner.
- The Schedule of Indemnities does not purport to restrict the kinds of damages that
may be awarded against Western once liability has arisen. It was merely meant to set FACTS
limits to the amounts the movant would be liable for in cases of claims for death, - On Nov. 26, 1976, a Ford Pick-up truck owned by Marcos Olasco was bumped by a
bodily injuries of, professional services and hospital charges, for services rendered to cargo truck owned by Floralde.
traffic accident victims,' and not necessarily exclude claims against the insurance FGU Insurance Corporation (FG U) by reason of Motor Vehicle Insurance Policy No. IC-
policy for other kinds of damages, such as those in question. VF-07185 paid Olaso the sum of P2,817.50 as its share in the repair cost of the said
- It will be seen that the above quoted Schedule of Indemnities establishes monetary Ford Pick-up. Having been subrogated to the rights and causes of action of Olaso in
limits which Western may invoke in case of occurrence of the particular kinds of the said amount FGU formally demanded payment of said amount from Floralde and
physical injury there listed. attempted to verify Floralde's insurance carrier but failed to do so. In 1978 FGU was
- It must be stressed, however, that the Schedule of Indemnities does not purport to able to ascertain the identity of Floralde's insurance carrier to be the Summit
limit, or to enumerate exhaustively, the species of bodily injury occurrence of which Guaranty and Insurance Company, Inc. (Summit) and thus requested the insurance
iNsuRanCe A2010 Dean Carale
pAgE 86♥

commissioner for a conference with Summit and demanded from Summit through rejection of said claim was ever made even if private respondent had already
counsel on February 28, 1978 the payment of the damages sustained by the car of complied with all the requirements.
Olaso but to no avail. - In G.R. No. L-48758-petitioner company even provided legal assistance to one of the
- Hence on May 22, 1978 FGU filed a case in the Insurance Commissioner's Office private respondents in the criminal case filed against him leading Private respondents
against Summit for recovery of said amount. to believe that it was ready to pay. In the same case, petitioner company admits that
- Summit filed a motion to dismiss on the ground of prescription under Section 384 of it took no final action or adjudication of the claim. Worse still, in G.R. No. L-48679,
PD No. 612. Averring that the accident happened on November 26, 1976 while the assurances of payment were constantly given and petitioner company even said that
complaint was filed on May 22, 1978 beyond the one-year period from the time of the a check was ready for release. This Court has made the observation that some
accident provided for by the said provision. insurance companies have been inventing excuses to avoid their just obligations and
it is only the State that can give the protection which the insuring public needs from
ISSUE possible abuses of the insurers. In view of the foregoing,
WON the action must be dismissed on the ground of prescription under Section 384 of - It is not denied that an extrajudicial demand for payment was made by respondent
PD No. 612 FGU on petitioner but petitioner failed to respond to the same. Nevertheless the
complaint was filed even before a denial of the claim was made by petitioner. For all
HELD legal purposes, the one-year prescriptive period provided for in Section 384 of the
NO Insurance Code has not begun to run.The cause of action arises only and starts to run
- The case do not fall within the meaning of proper cases' as contemplated in Section upon the denial of the claim by the insurance company.The court takes note of the
384 of the Insurance Code. dilatory tactics employed by petitioner in this as in the several cases aforecited to
avoid payment of its liabilities.

Reasoning VILLACORTA v. THE INSURANCE COMMISSION


- Section 384 of PD 612 (Insurance Code)
100 SCRA 467
Any person having any claim upon the policy issued pursuant to this chapter shall,
without any unnecessary delay, present to the insurance company concerned a TEEHANKEE; October 30, 1980
written notice of claim setting forth the amount of his loss, and/or the nature, extent
and duration of the injuries sustained as certified by a duly licensed physician. FACTS
Notice of claim must be filed within six months from date of the accident, otherwise, - JEWEL VILLACORTA was the owner of a Colt Lancer, Model 1976, insured with
the claim shall be deemed waived. Action or suit for recovery of damage due to loss respondent company for P35,000.00 - Own Damage; P30,000.00 - Theft; and
or injury must be brought, in proper cases, with the Commissioner or the Courts P30,000.00 - Third Party Liability, effective May 16, 1977 to May 16, 1978.
within one year from date of accident, otherwise, the claimant's right of action shall - On May 9, 1978, the vehicle was brought to the Sunday Machine Works, Inc., for
prescribe. general check-up and repairs. On May 11, 1978, while it was in the custody of the
- It is very clear that the one-year period is only required In proper cases. Had the Sunday Machine Works, the car was allegedly taken by six (6) persons and driven out
lawmakers intended it to be the way Petitioner Company assumes it to be, then the to Montalban, Rizal. While travelling along Mabini St., Sitio Palyasan, Barrio Burgos,
phrase 'in proper cases' would not have been inserted. going North at Montalban, Rizal, the car figured in an accident, hitting and bumping a
- in Aisporna. vs. Court of Appeals: gravel and sand truck parked at the right side of the road going south. As a
'Legislative intent must be ascertained from a consideration of the statute as a consequence, the gravel and sand truck veered to the right side of the pavement
whole. The particular words, clauses and phrases should not be studied as detached going south and the car veered to the right side of the pavement going north. The
and isolated expressions, but the whole and every part of the statute must be driver, Benito Mabasa, and one of the passengers died and the other four sustained
considered in fixing the meaning of any of its parts and in order to produce a physical injuries. The car, as well, suffered extensive damage. Complainant,
harmonious whole. A statute must be so construed as to harmonize and give effect thereafter, filed a claim for total loss with the respondent company but claim was
to all its provisions whenever possible.' denied. Hence, complainant was compelled to institute the present action."
- Petitioner company is trying to use Section 384 of the Insurance Code as a cloak to - The comprehensive motor car insurance policy for P35,000.00 issued by respondent
hide itself from its liabilities. The facts of these cases evidently reflect the deliberate Empire Insurance Company admittedly undertook to indemnify the petitioner-insured
efforts of petitioner company to prevent the filing of a formal action against it. Bearing against loss or damage to the car (a) by accidental collision or overturning, or collision
in mind that if it succeeds in doing so until one year lapses from the date of the or overturning consequent upon mechanical breakdown or consequent upon wear and
accident it could set up the defense of prescription, petitioner company made private tear; (b) by fire, external explosion, self-ignition or lightning or burglary,
respondents believe that their claims would be settled in order that the latter will not housebreaking or theft; and (c) by malicious act.
find it necessary to immediately bring suit. In violation of its duties to adopt and - Respondent insurance commission, however, dismissed petitioner's complaint for
implement reasonable standards for the prompt investigation of claims and to recovery of the total loss of the vehicle against private respondent, sustaining
effectuate prompt, fair and equitable settlement of claims, and with manifest bad respondent insurer's contention that the accident did not fall within the provisions of
faith, petitioner company devised means and ways of stalling the settlement the policy either for the Own Damage or Theft coverage, invoking the policy provision
proceedings. In G.R. No. L-50997, no steps were taken to process the claim and no on "Authorized Driver" clause, which clause limits the use of the insured vehicle to two
(2) persons only, namely: the insured himself or any person on his (insured's)
iNsuRanCe A2010 Dean Carale
pAgE 87♥

permission. Apparently, the Insurance commission sees the unauthorized taking of the
vehicle for a joyride as a violation of the 'Authorized Driver' clause of the policy."
- Respondent commission likewise upheld private respondent's assertion that the car
was not stolen and therefore not covered by the Theft clause, ruling that "(T)he
element of 'taking' in Article 308 of the Revised Penal Code means that the act of
depriving another of the possession and dominion of a movable thing is coupled . . .
CHAPTER VIII. MARINE INSURANCE
with the intention, at the time of the 'taking', of withholding it with the character of
permanency MAGSAYSAY INC v. AGAN
96 PHIL 504
ISSUE
REYES; January 31, 1955
WON the Insurance commission’s findings are in accord with law

HELD FACTS
NO - The S S "San Antonio", vessel owned and operated by plaintiff, left Manila on October
- First, respondent commission's ruling that the person who drove the vehicle in the 6, 1949, bound for Basco, Batanes, vis Aparri, Cagayan, with general cargo belonging
person of Benito Mabasa, who, according to its own finding, was one of the residents to different shippers, among them the defendant. The vessel reached Aparri, but while
of the Sunday Machine Works, Inc. to whom the car had been entrusted for general still in the port, it ran aground at the mouth of the Cagayan river, and, attempts to
check-up and repairs was not an "authorized driver" of petitioner-complainant is too refloat it under its own power having failed, plaintiff had it refloated by the Luzon
restrictive and contrary to the established principle that insurance contracts, being Stevedoring Co. at an agreed compensation. Once afloat the vessel returned to Manila
contracts of adhesion where the only participation of the other party is the signing of to refuel and then proceeded to Basco, the port of destination. There the cargoes
his signature or his "adhesion" thereto, "obviously call for greater strictness and were delivered to their respective owners or consignees, who, with the exception of
vigilance on the part of courts of justice with a view of protecting the weaker party defendant, made a deposit or signed a bond to answer for their contribution to the
from abuse and imposition, and prevent their becoming traps for the unwary." average.
- The main purpose of the "authorized driver" clause, as may be seen from its text, - On the theory that the expenses incurred in floating the vessel constitute general
supra, is that a person other than the insured owner, who drives the car on the average to which both ship and cargo should contribute, plaintiff brought the present
insured's order, such as his regular driver, or with his permission, such as a friend or action in the CFI of Manila to make defendant pay his contribution, which, as
member of the family or the employees of a car service or repair shop must be duly determined by the average adjuster, amounts to P841.40.
licensed drivers and have no disqualification to drive a motor vehicle. A car owner who - Defendant denies liability to his amount, alleging, among other things, that the
entrusts his car to an established car service and repair shop necessarily entrusts his stranding of the vessel was due to the fault, negligence and lack of skill of its master,
car key to the shop owner and employees who are presumed to have the insured's that the expenses incurred in putting it afloat did not constitute general average, and
permission to drive the car for legitimate purposes of checking or road-testing the car. that the liquidation of the average was not made in accordance with law.
The mere happenstance that the employee(s) of the shop owner diverts the use of the - The lower court found for plaintiff
car to his own illicit or unauthorized purpose in violation of the trust reposed in the
shop by the insured car owner does not mean that the "authorized driver" clause has ISSUE
been violated such as to bar recovery, provided that such employee is duly qualified WON the expenses incurred in floating a vessel so stranded should be considered
to drive under a valid driver's license. general average and shared by the cargo owners
- Secondly, and independently of the foregoing (since when a car is unlawfully taken,
it is the theft clause, not the "authorized driver" clause, that applies), where a car is HELD
admittedly as in this case unlawfully and wrongfully taken by some people, be they NO
employees of the car shop or not to whom it had been entrusted, and taken on a long Ratio The law on averages is contained in the Code of Commerce. Under that law,
trip to Montalban without the owner's consent or knowledge, such taking constitutes averages are classified into simple or particular and general or gross. Generally
or partakes of the nature of theft as defined in Article 308 of the Revised Penal Code. speaking, simple or particular averages include all expenses and damages caused to
- The Court rejects respondent commission's premise that there must be an intent on the vessel or cargo which have not inured to the common benefit (Art. 809), and are,
the part of the taker of the car "permanently to deprive the insured of his car" and therefore, to be borne only by the owner of the property gave rise to same (Art. 810);
that since the taking here was for a "joy ride" and "merely temporary in nature," a while general or gross averages include "all the damages and expenses which are
"temporary taking is held not a taking insured against." deliberately caused in order to save the vessel, its cargo, or both at the same time,
- The insurer must therefore indemnify the petitioner owner for the total loss of the from a real and known risk" (Art. 811). Being for the common benefit, gross averages
insured car in the sum of P35,000.00 under the theft clause of the policy, subject to are to be borne by the owners of the articles saved (Art. 812).
the filing of such claim for reimbursement or payment as it may have as subrogee Reasoning
against the Sunday Machine Works, Inc. - the stranding of plaintiff's vessel was due to the sudden shifting of the sandbars at
the mouth of the river which the port pilot did not anticipate. The standing may,
therefore, be regarded as accidental.
iNsuRanCe A2010 Dean Carale
pAgE 88♥

- Tolentino, in his commentaries on the Code of Commerce, gives the following - The insurance contract is printed in the English common form of marine policies. One
requisites for general average: of the clauses of the document originally read as follows:
First, there must be a common danger. This means, that both the ship and the cargo, “Touching the Adventures and Perils which the said NUFIC is content to bear, and to
after has been loaded, are subject to the same danger, whether during the voyage, or take upon them in this Voyage; they are of the Seas, Men-of-War, Fire, Pirates,
in the port of loading or unloading; that the danger arises from the accidents of the Thieves, Jettison, Letters of Mart and Countermart, Surprisals, and Takings at Sea.
sea, dispositions of the authority, or faults of men, provided that the circumstances Arrests, Restraints and Detainments, of all Kings, Princes and People of what Nation,
producing the peril should be ascertained and imminent or may rationally be said to Condition or Quality soever; Barratry of the Master and Marines, and of all other
be certain and imminent. This last requirement exclude measures undertaken against Perils, Losses and Misfortunes, that have or shall come to the Hurt, Detriment, or
a distant peril. Damage of the said Vessel or any part thereof; and in case of any Loss or
Second, that for the common safety part of the vessel or of the cargo or both is Misfortunes, it shall be lawful for the Assured, his or their Factors, Servants, or
sacrificed deliberately. assigns, to sue, labour and travel for, in and about the Defence. Safeguard, and
Third, that from the expenses or damages caused follows the successful saving of the recovery of the said Vessel or any part thereof, without Prejudice to this Insurance;
vessel and cargo. to the Charges whereof the said Company, will contribute, according to the rate and
Fourth, that the expenses or damages should have been incurred or inflicted after quantity of the sum herein assured...”
taking proper legal steps and authority. - Attached to the policy over and above the said clause is a “rider” containing
- With respect to the first requisite, the evidence does not disclose that the expenses typewritten provisions, among which appears in capitalized type the following clause:
sought to be recovered from defendant were incurred to save vessel and cargo from a “AGAINST THE ABSOLUTE TOTAL LOSS OF THE VESSEL ONLY, AND TO PAY
common danger...it is the safety of the property, and not of the voyage, which PROPORTIONATE SALVAGE CHARGES OF THE DECLARED VALUE.”
constitutes the true foundation of the general average.
- As to the second requisite, we need only repeat that the expenses in question were ISSUES
not incurred for the common safety of vessel and cargo, since they, or at least the 1. WON the lower court erred in disregarding the typewritten clause endorsed upon
cargo, were not in imminent peril. the policy, expressly limiting insurer's liability thereunder of the total loss of the
- With respect to the third requisite, the salvage operation, it is true, was a success. wooden vessel Pandan and to proportionate salvage charges
But as the sacrifice was for the benefit of the vessel to enable it to proceed to 2. WON lower court erred in concluding that defendant and appellant, NUFIC is liable
destination and not for the purpose of saving the cargo, the cargo owners are not in to contribute to the general average resulting from the jettison of a part of said
law bound to contribute to the expenses. vessel's cargo
- The final requisite has not been proved, for it does not appear that the expenses
here in question were incurred after following the procedure laid down in article 813. HELD
Disposition Wherefore, the decision appealed from is reversed. 1. NO
Ratio In case repugnance exists between written and printed portions of a policy, the
written portion prevails.
JARQUE v. SMITH, BELL & CO.
Reasoning
56 PHIL 758 - Section 291 of the Code of Civil Procedure provides that “when an instrument
OSTRAND; November11, 1932 consists partly of written words and partly of a printed form and the two are
inconsistent, the former controls the latter.”
NATURE
Appeal from judgment of the lower court
2. NO
FACTS Ratio The liability for contribution in general average is not based on the express
- Plaintiff’s motorboat, “Pandan” was insured on a marine insurance policy with terms of the policy, but rests upon the theory that from the relation of the parties and
National Union Fire Insurance Company (NUFIC) for P45K. According to the provisions for their benefit, a quasi contract is implied by law.
of a “rider” attached to the policy, the insurance was against the “absolute total loss Reasoning
of the vessel only.” On Oct. 31, 1928, the ship ran into very heavy sea and it became - In the absence of positive legislation to the contrary, the liability of the defendant
necessary to jettison a portion of the cargo. As a result of the jettison, the NUFIC was insurance company on its policy would, perhaps, be limited to “absolute loss of the
assessed P2,610.86 as its contribution to the general average. vessel only, and to pay proportionate salvage of the declared value.” But the policy
- The insurance company, insisting that its obligation did not extend beyond the was executed in this jurisdiction and “warranted to trade within the waters of the
insurance of the “absolute total loss of the vessel only, and to pay proportionate Philippine Archipelago only.” Here, Art. 859 of the Code of Commerce is still in force:
salvage of the declared value,” refused to contribute to the settlement of the gen. “ART. 859. The underwriters of the vessel, of the freight, and of the cargo shall be
ave. The present action was thereupon instituted, and after trial the court below obliged to pay for the indemnity of the gross average in so far as is required of each
rendered judgment in favor of the plaintiff and ordered the defendant to pay the one of these objects respectively.”
plaintiff P2,610.86 as its part of the indemnity for the gen. ave. brought about by the - The article is mandatory in its terms, and the insurers (whether for the vessel or for
jettison of cargo. The insurance company then appealed to the SC. the freight or for the cargo) are bound to contribute to the indemnity of the general
average. The provision simply places the insurer on the same footing as other persons
iNsuRanCe A2010 Dean Carale
pAgE 89♥

who have an interest in the vessel, or the cargo therein, at the time of the occurrence that have or shall come to the hurt, detriment, or damage of the said goods and
of the general average and who are compelled to contribute (Art. 812, Code of merchandise or any part thereof."
Commerce).
- In the present case it is not disputed that the ship was in grave peril and that the
jettison of part of the cargo was necessary. If the cargo was in peril to the extent of ISSUE
call for general average, the ship must also have been in great danger, possibly WON Union Insurance is liable for the loss of the Go Tiaco Brothers
sufficient to cause its absolute loss. The jettison was therefore as much to the benefit
of the underwriter as to the owner of the cargo. The latter was compelled to HELD
contribute to the indemnity; why should not the insurer be required to do likewise? If NO
no jettison had taken place and if the ship by reason thereof had foundered, the - the words "all other perils, losses, and misfortunes" are to be interpreted as covering
underwriter's loss would have been many times as large as the contribution now risks which are of like kind (ejusdem generis) with the particular risks which are
demanded. enumerated in the preceding part of the same clause of the contract. ''According to
Disposition Appealed judgment is affirmed the ordinary rules of construction, these words must be interpreted with reference to
the words which immediately precede them. They were no doubt inserted in order to
GO TIACO v. UNION INSURANCE prevent disputes founded on nice distinctions. X x x For example, if the expression
'perils of the seas' is given its widest sense the general words have little or no effect
40 PHIL 40
as applied to that case. If on the other hand that expression is to receive a limited
STREET; September 1, 1919 construction, as apparently it did in Cullen vs. Butler (5 M. & S., 461), and loss by
perils of the seas is to be confined to loss ex marine tempestatis discrimine, the
FACTS general words become most important. X x x" (Thames and Mersey Marine Insurance
- Union Insurance Society of Canton, Ltd., issued a marine insurance policy upon a Co. vs. Hamilton, Fraser & Co.)
cargo of rice belonging to the Go Tiaoco Brothers, which was transported in the early - a loss which, in the ordinary course of events, results from the natural and inevitable
days of May, 1915, on the steamship Hondagua from the port of Saigon to Cebu. action of the sea, from the ordinary wear and tear of the ship, or from the negligent
- On discharging the rice from one of the compartments in the after hold, upon arrival failure of the ship's owner to provide the vessel with proper equipment to convey the
at Cebu, it was discovered that 1473 sacks had been damaged by sea water. The loss cargo under ordinary conditions, is not a peril of the sea. Such a loss is rather due to
was P3,875.25. what has been aptly called the "peril of the ship." The insurer undertakes to insure
- The trial court found that the inflow of the sea water during the voyage was due to a against perils of the sea and similar perils, not against perils of the ship. There must,
defect in one of the drain pipes of the ship and concluded that the loss was not in order to make the insurer liable, be "some casualty, something which could not be
covered by the policy of insurance. The trial court made the ff findings: foreseen as one of the necessary incidents of the adventure. The purpose of the policy
The drain pipe which served as a discharge from the water closet passed down is to secure an indemnity against accidents which may happen, not against events
through the compartment where the rice in question was stowed and thence which must happen." (Wilson, Sons & Co. vs. Owners of Cargo per the Xantho)
out to sea through the wall of the compartment, which was a part of the wall of - In the present case the entrance of the sea water into the ship's hold through the
the ship. The joint or elbow where the pipe changed its direction was of cast defective pipe already described was not due to any accident which happened during
iron; and in course of time it had become corroded and abraded until a the voyage, but to the failure of the ship's owner properly to repair a defect of the
longitudinal opening had appeared in the pipe about one inch in length. This existence of which he was apprised. The loss was therefore more analogous to that
hole had been in existence before the voyage was begun, and an attempt had which directly results from simple unseaworthiness than to that which results from
been made to repair it by filling with cement and bolting over it a strip of iron. perils of the sea.
The effect of loading the boat was to submerge the vent, or orifice, of the pipe - there is no room to doubt the liability of the shipowner for such a loss as occurred in
until it was about 18 inches or 2 feet below the level of the sea. As a this case. By parity of reasoning the insurer is not liable; for, generally speaking, the
consequence the sea water rose in the pipe. Navigation under these conditions shipowner excepts the perils of the sea from his engagement under the bill of lading,
resulted in the washing out of the cement-filling from the action of the sea while this is the very peril against which the insurer intends to give protection. As
water, thus permitting the continued flow of the salt water into the applied to the present case it results that the owners of the damaged rice must look to
compartment of rice. the shipowner for redress and not to the insurer.
- The court found in effect that the opening above described had resulted in course of The same conclusion must be reached if the question be discussed with reference to
time from ordinary wear and tear and not from the straining of the ship in rough the seaworthiness of the ship. It is universally accepted that in every contract of
weather on that voyage. The court also found that the repairs that had been made on insurance upon anything which is the subject of marine insurance, a warranty is
the pipe were slovenly and defective and that, by reason of the condition of this pipe, implied that the ship shall be seaworthy at the time of the inception of the voyage.
the ship was not properly equipped to receive the rice at the time the voyage was This rule is accepted in our own Insurance Law (Act No. 2427, sec. 106). It is also well
begun. For this reason the court held that the ship was unseaworthy. settled that a ship which is seaworthy for the purpose of insurance upon the ship may
- The policy purports to insure the cargo from the following among other risks: "Perils . yet be unseaworthy for the purpose of insurance upon the cargo (Act No. 2427, sec.
. . of the seas, men, of war, fire, enemies, pirates, rovers, thieves, .jettisons, . . . 106).
barratry of the master and mariners, and of all other perils, losses, and misfortunes Disposition Decision of trial court is affirmed
iNsuRanCe A2010 Dean Carale
pAgE 90♥

CATHAY INSURANCE CO. v. CA (REMINGTON INDUSTRIAL NATURE


Petition for certiorari to review the decision of the IAC
SALES CORP.)
151 SCRA 710 FACTS
PARAS; June 30 1987 - February 19, 1972 – Common carrier Manila Bay Lighterage Corp. entered into a
contract with Roque Timber Enterprises and Chiong. The contract stated that Manila
FACTS Bay would carry 422.18 cu. meters of logs on its vessel Mable 10 from Malampaya
- Remington Industrial Sales Corp insured its shipment of seamless steel pipes. It Sound, Palawan to Manila North Harbor. Roque insured the logs with Pioneer
incurred losses and damages (I gather the steel pipes rusted during the voyage from Insurance for P100,000.
Japan to the Phils. on board vessel SS "Eastern Mariner”) and filed complaint against - February 29, 1972 – 811 logs were loaded in Malampaya but en route to Manila,
Cathay Insurance Co seeking collection of the sum of P868,339.15 Mable 10 sank.
- TC decided for Remington. Cathay filed MR, which was denied. CA affirmed. - March 8,1972 – Roque and Chiong wrote a letter to Manila Bay, demanding payment
- CA said (among other things): 1. Coverage of private respondent's loss under the of P150,000.00 for the loss of the shipment plus P100,000.00 as unrealized profits but
insurance policy issued by petitioner is unmistakable; the latter ignored the demand.
2. Alleged contractual limitations contained in insurance policies are regarded with - A letter was also sent to Pioneer, claiming the full amount of P100,000.00 under the
extreme caution by courts and are to be strictly construed against the insurer; insurance policy but Pioneer refused to pay on the ground that its liability depended
obscure phrases and exceptions should not be allowed to defeat the very purpose for upon the "Total Loss by Total Loss of Vessel only".
which the policy was procured; - After hearing, the trial court favored Roque. Pioneer and Manila Bay were ordered to
3. Rust is not an inherent vice of the seamless steel pipes without interference of pay Roque P100,000. Pioneer appealed the decision.
external factors - January 30, 1984 – Pioneer was absolved from liability after finding that there was a
- Cathay contend (among other things): 1. private respondent has admitted that the breach of implied warranty of seaworthiness on the part of the petitioners and that
questioned shipment is not covered by a "square provision of the contract," but the loss of the insured cargo was caused by the "perils of the ship" and not by the
private respondent claims implied coverage from the phrase "perils of the sea" "perils of the sea". It ruled that the loss is not covered by the marine insurance policy.
mentioned in the opening sentence of the policy; 2. The insistence of private - It was alleged that Mable 10 was not seaworthy and that it developed a leak
respondent that rusting is a peril of the sea is erroneous; 3. Rusting is not a risk - The IAC found that one of the hatches was left open, causing water to enter the
insured against, since a risk to be insured against should be a casualty or some barge and because the barge was not provided with the necessary cover or tarpaulin,
casualty, something which could not be foreseen as one of the necessary incidents of the splash of sea waves brought more water inside the barge.
adventure; 4. A fact capable of unquestionable demonstration or of public knowledge - Petitioners contend that the implied warranty of seaworthiness provided for in the
needs no evidence. This fact of unquestionable demonstration or of public knowledge Insurance Code refers only to the responsibility of the shipowner who must see to it
is that heavy rusting of steel or iron pipes cannot occur within a period of a seven (7) that his ship is reasonably fit to make in safety the contemplated voyage.
day voyage. Besides, petitioner had introduced the clear cargo receipts or tally sheets - The petitioners state that a mere shipper of cargo, having no control over the ship,
indicating that there was no damage on the steel pipes during the voyage. has nothing to do with its seaworthiness. They argue that a cargo owner has no
control over the structure of the ship, its cables, anchors, fuel and provisions, the
ISSUE manner of loading his cargo and the cargo of other shippers, and the hiring of a
WON rusting is a “peril of the sea” sufficient number of competent officers and seamen.
ISSUE
HELD WON the loss should have been covered by the marine insurance policy
YES
- There is no question that the rusting of steel pipes in the course of a voyage is a HELD
"peril of the sea" in view of the toll on the cargo of wind, water, and salt conditions. At NO
any rate if the insurer cannot be held accountable therefor, We would fail to observe a Ratio It is universally accepted that in every contract of insurance upon anything
cardinal rule in the interpretation of contracts, namely, that any ambiguity therein which is the subject of marine insurance, a warranty is implied that the ship shall be
should be construed against the maker/issuer/drafter thereof, namely, the insurer. seaworthy at the time of the inception of the voyage. In marine insurance, the risks
Besides the precise purpose of insuring cargo during a voyage would be rendered insured against are classified as 'perils of the sea,’ which includes such losses that are
fruitless. of extraordinary nature, or arise from some overwhelming power, which cannot be
Disposition WHEREFORE, this petition is hereby DENIED, and the assailed decision of guarded against by the ordinary exertion of human skill and prudence.
the Court of Appeals is hereby AFFIRMED. Reasoning
- Based on Sec. 113 and Sec. 99 of the Insurance Code, the term "cargo" can be the
subject of marine insurance and that once it is so made, the implied warranty of
ROQUE v. IAC (PIONEER INSURANCE AND SURETY CORP.) seaworthiness immediately attaches to whoever is insuring the cargo whether he be
139 SCRA 596 the shipowner or not.
GUTIERREZ; November 11, 1985
iNsuRanCe A2010 Dean Carale
pAgE 91♥

- The fact that the un-seaworthiness of the ship was unknown to the insured is other perils, losses, and misfortunes," as used in the policy; the other has reference to
immaterial in ordinary marine insurance and may not be used by him as a defense in the implied warranty, on the part of the insured, as to the seaworthiness of the ship.
order to recover on the marine insurance policy. - The meaning of the expression "perils * * * of the seas * * * and all other perils,
- Since the law provides for an implied warranty of seaworthiness in every contract of losses, and misfortunes," used in describing the risks covered by policies of marine
ordinary marine insurance, it becomes the obligation of a cargo owner to look for a insurance, has been the subject of frequent discussion; and certain propositions
reliable common carrier which keeps its vessels in seaworthy condition. The shipper of relative thereto are now so generally accepted as to be considered definitely settled.
cargo my have no control over the vessel but he has full control in the choice of the - The words "all other perils, losses, and misfortunes" are to be interpreted as
common carrier that will transport his goods. covering risks which are of like kind (ejusdem generis) with the particular risks which
- In marine cases, the risks insured against are 'perils of the sea.’ The term extends are enumerated in the preceding part of the same clause of the contract.
only to losses caused by sea damage, or by the violence of the elements, and does - A loss which, in the ordinary course of events, results from the natural and inevitable
not embrace all losses happening at sea. action of the sea, from the ordinary wear and tear of the ship, or from the negligent
- It is quite unmistakable that the loss of the cargo was due to the perils of the ship failure of the ship's owner to provide the vessel with proper equipment to convey the
rather than the perils of the sea. cargo under ordinary conditions, is not a peril of the sea. Such a loss is rather due to
- Loss which, in the ordinary course of events, results from the natural and inevitable what has been aptly called the "peril of the ship." The insurer undertakes to insure
action of the sea, from the ordinary wear and tear of the ship, or from the negligent against perils of the sea and similar perils, not against perils of the ship.
failure of the ship's owner to provide the vessel with proper equipment to convey the - As was said by Lord Herschell in Wilson, Sons & Co. vs. Owners of Cargo per the
cargo under ordinary conditions, is not a ‘peril of the sea’ but is called ‘peril of the Xantho, there must, in order to make the insurer liable, be "some casualty, something
ship.’ which could not be foreseen as one of the necessary incidents of the adventure. The
Disposition Decision appealed from is affirmed. purpose of the policy is to secure an indemnity against accidents which may happen,
not against events which must happen."
- In the present case the entrance of the sea water into the ship's hold through the
LA RAZON v. UNION INSURANCE SOCIETY OF CANTON, LTD.
defective pipe already described was not due to any accident which happened during
40 PHIL 40 the voyage, but to the failure of the ship's owner properly to repair a defect of the
STREET; September 1, 1919 existence of which, he was apprised. The loss was therefore more analogous to that
which directly results from simple unseaworthiness than to that which results from
FACTS perils of the sea.
- This is an action on a policy of marine insurance issued by the Union Insurance - It is universally accepted that in every contract of insurance upon anything which is
Society of Canton, Ltd., upon a cargo of rice belonging to the plaintiffs, Go Tiaoco the subject of marine insurance, a warranty is implied that the ship shall be seaworthy
Brothers, which was transported on the steamship Hondagua from the port of Saigon at the time of the inception of the voyage. This rule is accepted in our own Insurance
to Cebu. Law (Act No. 2427, see. 106).
- On discharging the rice from one of the compartments in the after hold, upon arrival - It is also well settled that a ship which is seaworthy for the purpose of insurance
at Cebu, it was discovered that 1,473 sacks had been damaged by sea water. upon the ship may yet be unseaworthy for the purpose of insurance upon the cargo
- The loss so resulting to the owners of rice, after proper deduction had been made for (Act No. 2427, see. 106).
the portion saved, was P3,875.25. Disposition Jjudgment affirmed.
- The trial court found that the inflow of the sea water during the voyage was due to a
defect in one of the drain pipes of the ship and concluded that the loss was not MALAYAN INSURANCE v. CA (supra p.10)
covered by the policy of insurance. Judgment was accordingly entered in favor of the
defendant and the plaintiffs appealed.
- The court found in effect that the opening above described had resulted in course of FILIPINO MERCHANTS INS. CO. v. CA (supra p.19)
time from ordinary wear and tear and not from the straining of the ship in rough
weather on that voyage. The court also found that the repairs that had been made on COASTWISE LIGHTERAGE CORP v. CA (PHILIPPINE GENERAL
the pipe were slovenly and defective and that, by reason of the condition of this pipe,
INSURANCE COMPANY)
the ship was not properly equipped to receive the rice at the time the voyage was
begun. For this reason the court held that the ship was unseaworthy. 245 SCRA 796
FRANCISCO; July 12, 1995
ISSUE
WON the insurer is liable NATURE
Petition for review of CA Decision affirming decision of RTC Manila holding that
HELD Coastwise is liable to pay PhilGen Insurance the amount of P700thou plus legal
- The question whether the insurer is liable on this policy for the loss caused in the interest thereon, another sum of P100thou as attorney's fees and the cost of the suit.
manner above stated presents two phases which are in a manner involved with each
other. One has reference to the meaning of the expression "perils of the seas and all FACTS
iNsuRanCe A2010 Dean Carale
pAgE 92♥

- Pag-asa Sales, Inc. entered into a contract to transport molasses from the province - SC agrees with Coastwise's admission that the contract it entered into with the
of Negros to Manila with Coastwise, using the latter's dumb barges. The barges were consignee was one of affreightment. Pag-asa Sales, Inc. only leased three of
towed in tandem by the tugboat MT Marica, also owned by Coastwise. Upon reaching petitioner's vessels, in order to carry cargo from one point to another, but the
Manila Bay, while approaching Pier 18, one of the barges struck an unknown sunken possession, command and navigation of the vessels remained with Coastwise. As
object. The forward buoyancy compartment was damaged, and water gushed in such, Coastwise, by the contract of affreightment, was not converted into a private
through a hole "two inches wide and twenty-two inches long." carrier, but remained a common carrier and was still liable as such.
- As a consequence, the molasses at the cargo tanks were contaminated and rendered - Therefore, the mere proof of delivery of goods in good order to a carrier and the
unfit for the use it was intended. This prompted consignee Pag-asa Sales to reject the subsequent arrival of the same goods at the place of destination in bad order makes
shipment of molasses as a total loss. Thereafter, Pag-asa Sales filed a formal claim for a prima facie case against the carrier. The presumption of negligence that
with the insurer of its lost cargo (PhilGen) and against the carrier (Coastwise). attaches to common carriers, once the goods it transports are lost, destroyed or
Coastwise denied the claim and it was PhilGen which paid Pag-asa Sales the amount deteriorated, applies to Coastwise. This presumption, which is overcome only by proof
of P700k representing the value of the damaged cargo of molasses. of the exercise of extraordinary diligence, remained unrebutted in this case.
- PhilGen then filed an action against Coastwise before the RTC Manila, seeking to - The damage to the barge which carried the cargo of molasses was caused by its
recover the P700k which it paid to Pag-asa Sales for the latter's lost cargo. PhilGen hitting an unknown sunken object as it was heading for Pier 18. The object turned out
now claims to be subrogated to all the contractual rights and claims which the to be a submerged derelict vessel. The evidence on record appeared that far from
consignee may have against the carrier, which is presumed to have violated the having rendered service with the greatest skill and utmost foresight, and being free
contract of carriage. from fault, the carrier was culpably remiss in the observance of its duties.
- RTC awarded the amount prayed for by PhilGen. CA affirmed. Hence, this petition. - Jesus R. Constantino, the patron of the vessel "Coastwise 9" admitted that he was
not licensed. The Code of Commerce, which subsidiarily governs common carriers
ISSUES (which are primarily governed by the provisions of the Civil Code) provides: “Art. 609.
1. WON Coastwise Lighterage was transformed into a private carrier, by virtue of the Captains, masters, or patrons of vessels must be Filipinos, have legal capacity to
contract of affreightment which it entered into with the consignee, Pag-asa Sales, Inc. contract in accordance with this code, and prove the skill capacity and qualifications
(Corollarily, if it were in fact transformed into a private carrier, did it exercise the necessary to command and direct the vessel, as established by marine and navigation
ordinary diligence to which a private carrier is in turn bound?) laws, ordinances or regulations, and must not be disqualified according to the same
2. WON the insurer was subrogated into the rights of the consignee against the for the discharge of the duties of the position.”
carrier, upon payment by the insurer of the value of the consignee's goods lost while - Clearly, Coastwise Lighterage's embarking on a voyage with an unlicensed patron
on board one of the carrier's vessels violates this rule. It cannot safely claim to have exercised extraordinary diligence, by
placing a person whose navigational skills are questionable, at the helm of the vessel
HELD which eventually met the fateful accident. It may also logically, follow that a person
1. NO without license to navigate, lacks not just the skill to do so, but also the utmost
- The distinction between the two kinds of charter parties (i.e. bareboat or demise and familiarity with the usual and safe routes taken by seasoned and legally authorized
contract of affreightment) is more clearly set out in the case of Puromines, Inc. vs. ones. Had the patron been licensed, he could be presumed to have both the skill and
Court of Appeals, wherein SC ruled: the knowledge that would have prevented the vessel's hitting the sunken derelict ship
“Under the demise or bareboat charter of the vessel, the charterer will generally that lay on their way to Pier 18.
be regarded as the owner for the voyage or service stipulated. The charterer - As a common carrier, Coastwise is liable for breach of the contract of carriage,
mans the vessel with his own people and becomes the owner pro hac vice, having failed to overcome the presumption of negligence with the loss and destruction
subject to liability to others for damages caused by negligence. To create a of goods it transported, by proof of its exercise of extraordinary diligence.
demise, the owner of a vessel must completely and exclusively relinquish 2. YES
possession, command and navigation thereof to the charterer, anything short of - Coastwise is liable for breach of the contract of carriage it entered into with Pag-asa
such a complete transfer is a contract of affreightment (time or voyage charter Sales, Inc. However, for the damage sustained by the loss of the cargo which
party) or not a charter party at all. petitioner-carrier was transporting, it was not the carrier which paid the value thereof
On the other hand a contract of affreightment is one in which the owner of the to Pag-asa Sales, Inc. but the latter's insurer, herein private respondent PhilGen.
vessel leases part or all of its space to haul goods for others. It is a contract for - Article 2207 of the Civil Code: If the plaintiffs property has been insured, and he has
special service to be rendered by the owner of the vessel and under such contract received indemnity from the insurance company for the injury or loss arising out of
the general owner retains the possession, command and navigation of the ship, the wrong or breach of contract complained of, the insurance company shall be
the charterer or freighter merely having use of the space in the vessel in return subrogated to the rights of the insured against the wrongdoer or the person who
for his payment of the charter hire... violated the contract.”
An owner who retains possession of the ship though the hold is the property of - This legal provision is founded on the well-settled principle of subrogation. If the
the charterer, remains liable as carrier and must answer for any breach of duty as insured property is destroyed or damaged through the fault or negligence of a party
to the care, loading and unloading of the cargo.” other than the assured, then the insurer, upon payment to the assured will be
- Although a charter party may transform a common carrier into a private one, the subrogated to the rights of the assured to recover from the wrongdoer to the extent
same however is not true in a contract of affreightment on account of the that the insurer has been obligated to pay. Payment by the insurer to the assured
aforementioned distinctions between the two. operated as an equitable assignment to the former of all remedies which the latter
iNsuRanCe A2010 Dean Carale
pAgE 93♥

may have against the third party whose negligence or wrongful act caused the loss. - CA ruled that “MV Asilda” was unseaworthy for being top- heavy as 2,500 cases of
The right of subrogation is not dependent upon, nor does it grow out of, any privity of Coca-Cola softdrink bottles were improperly stowed on deck. Nonetheless, the
contract or upon written assignment of claim. It accrues simply upon payment of the appellate court denied the claim of PHILAMGEN on the ground that the assured’s
insurance claim by the insurer. implied warranty of seaworthiness was not complied with. Perfunctorily, PHILAMGEN
- Undoubtedly, upon payment by respondent insurer PhilGen of the amount of was not properly subrogated to the rights and interests of the shipper. Furthermore,
P700,000.00 to Pag-asa Sales, Inc., the consignee of the cargo of molasses totally respondent court held that the filing of notice of abandonment had absolved the
damaged while being transported by petitioner Coastwise Lighterage, the former was shipowner/agent from liability under the limited liability rule.
subrogated into all the rights which Pag-asa Sales, Inc. may have had against the
carrier, herein petitioner Coastwise Lighterage. ISSUES
Disposition Petition denied. CA affrimed. 1. WON “MV Asilda” was seaworthy when it left the port of Zamboanga
2. WON the limited liability under Art. 587 of the Code of Commerce should apply
3. WON PHILAMGEN was properly subrogated to the rights and legal actions which the
THE PHILIPPINE AMERICAN GENERAL INSURANCE COMPANY
shipper had against FELMAN, the shipowner
INC v. CA (FELMAN SHIPPING LINES)
273 SCRA 226 HELD
BELLOSILLO; June 11, 1997 1. YES
- “MV Asilda” was unseaworthy when it left the port of Zamboanga. We subscribe to
FACTS the findings of the Elite Adjusters, Inc., and the Court of Appeals that the proximate
- Coca-Cola Bottlers Philippines, Inc., loaded on board “MV Asilda,” a vessel owned and cause of the sinking of “MV Asilda” was its being top-heavy. Contrary to the ship
operated by Felman 7,500 cases of 1-liter Coca-Cola softdrink bottles to be captain’s allegations, evidence shows that approximately 2,500 cases of softdrink
transported from Zamboanga City to Cebu for consignee Coca-Cola Bottlers bottles were stowed on deck. Several days after “MV Asilda” sank, an estimated
Philippines, Inc., Cebu. The shipment was insured with petitioner Philippine American 2,500 empty Coca-Cola plastic cases were recovered near the vicinity of the sinking.
General under Marine Open Policy. Considering that the ship’s hatches were properly secured, the empty Coca-Cola cases
- The vessel sank in the waters of Zamboanga del Norte bringing down her entire recovered could have come only from the vessel’s deck cargo. It is settled that
cargo with her including the subject 7,500 cases of 1-liter Coca-Cola softdrink bottles. carrying a deck cargo raises the presumption of unseaworthiness unless it can be
- The consignee filed a claim with respondent FELMAN for recovery of damages it shown that the deck cargo will not interfere with the proper management of the ship.
sustained as a result of the loss of its softdrink bottles that sank with “MV Asilda.” However, in this case it was established that “MV Asilda” was not designed to carry
Respondent denied the claim thus prompting the consignee to file an insurance claim substantial amount of cargo on deck. The inordinate loading of cargo deck resulted in
with PHILAMGEN which paid its claim of P755,250.00. the decrease of the vessel’s metacentric height thus making it unstable. The strong
- Claiming its right of subrogation PHILAMGEN sought recourse against respondent winds and waves encountered by the vessel are but the ordinary vicissitudes of a sea
FELMAN which disclaimed any liability for the loss. Consequently, PHILAMGEN sued voyage and as such merely contributed to its already unstable and unseaworthy
the shipowner for sum of money and damages. condition.
- PHILAMGEN alleged that the sinking and total loss of “MV Asilda” and its cargo were 2. NO
due to the vessel’s unseaworthiness as she was put to sea in an unstable condition. It - The ship agent is liable for the negligent acts of the captain in the care of goods
further alleged that the vessel was improperly manned and that its officers were loaded on the vessel. This liability however can be limited through abandonment of
grossly negligent in failing to take appropriate measures to proceed to a nearby port the vessel, its equipment and freightage as provided in Art. 587. Nonetheless, there
or beach after the vessel started to list. are exceptional circumstances wherein the ship agent could still be held answerable
- FELMAN filed a motion to dismiss based on the affirmative defense that no right of despite the abandonment, as where the loss or injury was due to the fault of the
subrogation in favor of PHILAMGEN was transmitted by the shipper, and that, in any shipowner and the captain. The international rule is to the effect that the right of
event, FELMAN had abandoned all its rights, interests and ownership over “MV Asilda” abandonment of vessels, as a legal limitation of a shipowner’s liability, does not apply
together with her freight and appurtenances for the purpose of limiting and to cases where the injury or average was occasioned by the shipowner’s own fault.
extinguishing its liability under Art. 587 of the Code of Commerce. 3. YES
- Trial court dismissed the complaint of PHILAMGEN. On appeal the Court of Appeals - The doctrine of subrogation has its roots in equity. It is designed to promote and to
set aside the dismissal and remanded the case to the lower court for trial on the accomplish justice and is the mode which equity adopts to compel the ultimate
merits. FELMAN filed a petition for certiorari with this Court but it was subsequently payment of a debt by one who in justice, equity and good conscience ought to pay.
denied on 13 February 1989. Therefore, the payment made by PHILAMGEN to Coca-Cola Bottlers Philippines, Inc.,
- Trial court rendered judgment in favor of FELMAN. It ruled that “MV Asilda” was gave the former the right to bring an action as subrogee against FELMAN. Having
seaworthy when it left the port of Zamboanga as confirmed by certificates issued by failed to rebut the presumption of fault, the liability of FELMAN for the loss of the
the Philippine Coast Guard and the shipowner’s surveyor attesting to its 7,500 cases of 1-liter Coca-Cola softdrink bottles is inevitable.
seaworthiness. Thus the loss of the vessel and its entire shipment could only be - Sec. 113 of the Insurance Code provides that “(i)n every marine insurance upon a
attributed to either a fortuitous event, in which case, no liability should attach unless ship or freight, or freightage, or upon anything which is the subject of marine
there was a stipulation to the contrary, or to the negligence of the captain and his insurance, a warranty is implied that the ship is seaworthy.” Under Sec. 114, a ship is
crew, in which case, Art. 587 of the Code of Commerce should apply. “seaworthy when reasonably fit to perform the service, and to encounter the
iNsuRanCe A2010 Dean Carale
pAgE 94♥

ordinary perils of the voyage, contemplated by the parties to the policy.” Thus it would be required to make the necessary repairs and install the new machinery
becomes the obligation of the cargo owner to look for a reliable common carrier which before it could again be placed in commission. During that time the owner would be
keeps its vessels in seaworthy condition. He may have no control over the vessel deprived of the use of its vessel or the interest on its investment. When those
but he has full control in the selection of the common carrier that will transport his questions are considered the testimony is conclusive that the cost of salvage, repair
goods. He also has full discretion in the choice of assurer that will underwrite a and reconstruction was more than the original cost of the ship at the time the policy
particular venture. was issued. As found by the trial court, “t is difficult to see how there could have been
- In policies where the law will generally imply a warranty of seaworthiness, it can only a more complete loss of the vessel than that which actually occurred”. Upon the facts
be excluded by terms in writing in the policy in the clearest language. And where the shown here, any other construction would nullify the statute and as applied to the
policy stipulates that the seaworthiness of the vessel as between the assured and the conditions existing in the Manila Bay, this kind of policy would be worthless, and there
assurer is admitted, the question of seaworthiness cannot be raised by the assurer would not be any consideration for the premium.
without showing concealment or misrepresentation by the assured. 2. NO
- PHILAMGEN’s action against FELMAN is squarely sanctioned by Art. 2207 of the Civil - The defendant argues that the policy contains the provision that it “shall be of as
Code which provides: force and effect as the surest writing or policy of insurance made in London”.
Art. 2207. If the plaintiff’s property has been insured, and he has received However, for such law to apply to our courts the existence of such law must be
indemnity from the insurance company for the injury or loss arising out of the proven. It cannot apply when such proof is lacking. Nevertheless, in the English
wrong or breach of contract complained of, the insurance company shall be practice, a ship is a total loss when she has sustained such extensive damages that it
subrogated to the rights of the insured against the wrongdoer or the person would not be reasonably practical to repair her.
who has violated the contract. If the amount paid by the insurance company Disposition Decision reversed
does not fully cover the injury or loss, the aggrieved party shall be entitled to
recover the deficiency from the person causing the loss or injury.
Disposition Petition is GRANTED. Respondent FELMAN SHIPPING LINES is ordered to
pay petitioner PHILIPPINE AMERICAN GENERAL INSURANCE CO., INC., Seven Hundred
Fifty-five Thousand Two Hundred and Fifty Pesos (P755,250.00) plus legal interest
thereon counted from 29 November 1983, the date of judicial demand, pursuant to
Arts. 2212 and 2213 of the Civil Code.

PHILIPPINE MFTG. CO. v. UNION INSURANCE SOCIETY OF


CANTON CHOA TIEK SENG v. CA (FILIPINO MERCHANTS INSURANCE)
42 PHIL 378 183 SCRA 223
JOHNS; November 22, 1921 GANCAYO; March 15, 1990
FACTS
NATURE
- The plaintiff’s steel tank lighter was insured by defendant company for absolute total
Appeal from a decision of the Court of Appeals
loss. As a result of a typhoon, the lighter sunk in Manila Bay. The plaintiff demanded
payment from the defendant insurance company but the latter refused. The company
FACTS
asked the plaintiff to salvage the ship, which it was able to do so.
- Petitioner imported some lactose crystals from Holland.
- With the plaintiff able to raise the lighter, reconstruct it and placed it in commission,
- The importation involved fifteen (15) metric tons packed in 600 6-ply paper bags
the defendant insurance company claims that it was only liable for a total absolute
with polythelene inner bags, each bag at 25 kilos net. The goods were loaded at the
loss and that there was no total destruction of the lighter.
port at Rotterdam in sea vans on board the vessel "MS Benalder' as the mother
- The trial court decided in favor of the defendant, saying that the policy only covered
vessel, and thereafter aboard the feeder vessel "Wesser Broker V-25" of respondent
an actual total loss, not a constructive total loss.
Ben Lines Container, Ltd. (Ben Lines for short). The goods were insured by the
respondent Filipino Merchants' Insurance Co., Inc. (insurance company for short) for
ISSUES
the sum of P98,882.35, the equivalent of US$8,765.00 plus 50% mark-up or US
1. WON there was an absolute total loss that can be covered by the policy
$13,147.50, against all risks under the terms of the insurance cargo policy. Upon
2. WON the Marine Law of Great Britain applies
arrival at the port of Manila, the cargo was discharged into the custody of the arrastre
operator respondent E. Razon, Inc. (broker for short), prior to the delivery to petitioner
HELD
through his broker. Of the 600 bags delivered to petitioner, 403 were in bad order.
1. YES
The surveys showed that the bad order bags suffered spillage and loss later valued at
- At the time that the lighter was at the bottom of the bay, it was of no value to the
P33,117.63. Petitioner filed a claim for said loss dated February 16, 1977 against
owner, thus there was an actual total loss.
respondent insurance company in the amount of P33,117.63 as the insured value of
- The ship was sunk in July 1, 1918. After several futile attempts, it was finally raised
the loss.
on Sept. 20, 1918. It is faitr to assume that in its then condition much further time
iNsuRanCe A2010 Dean Carale
pAgE 95♥

- Respondent insurance company rejected the claim alleging that assuming that 179 SCRA 357
spillage took place while the goods were in transit, petitioner and his agent failed to
GANCAYCO; October 5, 1989
avert or minimize the loss by failing to recover spillage from the sea van, thus
violating the terms of the insurance policy sued upon; and that assuming that the
NATURE
spillage did not occur while the cargo was in transit, the said 400 bags were loaded in
Petition for review on certiorari
bad order, and that in any case, the van did not carry any evidence of spillage.
- Petitioner filed a complaint in the RTC against the insurance company seeking
FACTS
payment of the sum of P33,117.63 as damages plus attorney's fees and expenses of
- Marinduque Mining Industrial Corporation (Marinduque) shipped on board SS Arthur
litigation. Insurance company denied all the material allegations of the complaint and
Maersk from Boston, U.S.A. a shipment of 1 skid carton parts for valves. The shipment
raised several special defenses as well as a compulsory counterclaim. Insurance
was ordered from Jamesbury, Singapore PTE, LTD., which issued the cargo's packing
company filed a third-party complaint against respondents Ben Lines and broker.
list and Invoice number showing the contents of the carton. The Philippine Consulate
- RTC dismissed the complaint, the counterclaim and the third-party complaint with
in Singapore issued invoice for the shipment showing the contents and its total price
costs against the petitioner. Appealed in CA but denied. MFR was denied as well.
of $39,419.60 and the freight and other charges of $2,791.73. When the cargo arrived
in Manila, it was received and deposited in the office of Aboitiz Shipping Corporation
ISSUE
(Aboitiz) for transhipment to Nonoc Island.
WON insurance company should be held liable even if the technical meaning in
- In July 1980, Marinduque, as consignee of the cargo, made a report that said cargo
marine insurance of an “insurance against all risk" is applied
was pilfered on July 3, 1980 due to heavy rain at the Aboitiz terminal and that of the
total value of the cargo of $42,209.33, only $7,412.00 worth remains of the cargo with
HELD
the recommendation that the claim be made against Aboitiz.
YES
- The services of the Manila Adjusters and Surveyors Co. (Manila Adjusters) were
- In Gloren Inc. vs. Filipinas Cia. de Seguros, 12 it was held that an all risk insurance
engaged by the Phil-American General Insurance Co., Inc. (Phil Am) which came out
policy insures against all causes of conceivable loss or damage, except as otherwise
with the report that the cargo in question, when inspected, showed that it was
excluded in the policy or due to fraud or intentional misconduct on the part of the
pilfered. A confirmatory report was submitted by the Manila Adjusters.
insured. It covers all losses during the voyage whether arising from a marine peril or
- On August 11, 1980 Marinduque then filed a claim against Aboitiz in the amount of
not, including pilferage losses during the war.
P246,430.80 representing the value of the pilfered cargo. On the same day
- In the present case, the "all risks" clause of the policy sued upon reads as follows:
Marinduque filed a claim for the same amount against the Phil-Am on the latter's
"5. This insurance is against all risks of loss or damage to the subject matter
policy. Phil-Am paid Marinduque the sum of P246,430.80 as insurer of the cargo.
insured but shall in no case be deemed to extend to cover loss, damage, or expense
- Phil-Am then filed a complaint in RTC Manila against Aboitiz for recovery of same
proximately caused by delay or inherent vice or nature of the subject matter
amount alleging that it has been subrogated to the rights of Marinduque. Complaint
insured. Claims recoverable hereunder shall be payable irrespective of percentage."
dismissed and MFR denied. CA reversed. MFR thereof was denied. Hence, this petition.
- The terms of the policy are so clear and require no interpretation. The insurance
policy covers all loss or damage to the cargo except those caused by delay or inherent
ISSUE
vice or nature of the cargo insured. It is the duty of the respondent insurance
WON petitioner Aboitiz was properly held liable to the private respondent Phil-Am by
company to establish that said loss or damage falls within the exceptions provided for
the appellate court
by law, otherwise it is liable therefor.
- An "all risks" provision of a marine policy creates a special type of insurance which
HELD
extends coverage to risks not usually contemplated and avoids putting upon the
YES
insured the burden of establishing that the loss was due to peril falling within the
- The questioned shipment is covered by a continuing open insurance coverage (which
policy's coverage. The insurer can avoid coverage upon demonstrating that a specific
took effect after Sept. 1, 1975, as contained in Marine Open Policy No. 100184) from
provision expressly excludes the loss from coverage.
the time it was loaded aboard the SS Arthur Maersk in Boston, U.S.A. to the time it
- In this case, the damage caused to the cargo has not been attributed to any of the
was delivered to the possession of petitioner at its offices at Pier 4 in Manila until it
exceptions provided for nor is there any pretension to this effect. Thus, the liability of
was pilfered when the great majority of the cargo was lost on July 3, 1980. Hence,
respondent insurance company is clear.
petitioner Aboitiz was properly held liable to Phil-Am.
Disposition the decision appealed from is hereby REVERSED AND SET ASIDE and
Reasoning
another judgment is hereby rendered ordering the respondent Filipinas Merchants
[a] Records of the case show that Phil-Am executed a continuous and open insurance
Insurance Company, Inc. to pay the sum of P33,117.63 as damages to petitioner with
coverage covering goods of Marinduque imported into and exported from the
legal interest from the filing of the complaint, plus attorney's fees and expenses of
Philippines which took effect after Sept. 1, 1975, as contained in Marine Open Policy
litigation in the amount of P10,000.00 as well as the costs of the suit.
No. 100184. A similar insurance coverage was also executed by petitioner in favor of
Marinduque for all its goods shipped or moved within the territorial limits of the
FILIPINO MERCHANTS INS. CO. v. CA (supra p.19) Philippines also effective after Sept. 1, 1975 and contained in Marine Open Policy No.
100185.
ABOITIZ SHIPPING v. PHILAMGEN INSURANCE
iNsuRanCe A2010 Dean Carale
pAgE 96♥

[b] TC in dismissing the complaint apparently relied on Marine Risk Note No. 017545 the contract several and divisible as to the items insured. The logs on the two barges
issued by private respondent Phil-Am only on July 28, 1980 after the shipment in were not separately valued or separately insured. Only one premium was paid for the
question was already pilfered. Obviously TC mistook said Marine Risk Note as an entire shipment, making for only one cause or consideration. The insurance contract
insurance policy when it is NOT. It is only an acknowledgment or declaration of the must, therefore, be considered indivisible.
private respondent confirming the specific shipment covered by its Marine Open - More importantly, the insurer's liability was for "total loss only." A total loss may be
Policy, the evaluation of the cargo and the chargeable premium. either actual or constructive (Sec. 129, Insurance Code). An actual total loss is caused
[c] The contention of the Aboitiz that it could not be liable for the pilferage of the by:
cargo as it was stolen even before it was loaded on its vessel is untenable. Aboitiz (a) A total destruction of the thing insured;
received cargo when it arrived in Manila at its offices, and it was while in its (b) The irretrievable loss of the thing by sinking, or by being broken up;
possession and before loading it in its vessel that the cargo was pilfered. Its liability is (c) Any damage to the thing which renders it valueless to the owner for the purpose
clear. for which he held it; or
Disposition Petition DISMISSED. (d) Any other event which effectively deprives the owner of the possession, at the
port of destination, of the thing insured. (Section 130, Insurance Code).
- A constructive total loss is one which gives to a person insured a right to abandon,
ORIENTAL ASSURANCE v. CA (PANAMA SAW MILL)
under Section 139 of the Insurance Code. This provision reads:
200 SCRA 459 SECTION 139. A person insured by a contract of marine insurance may abandon the
MELENCIO-HERRERA; August 9, 1991 thing insured, or any particular portion thereof separately valued by the policy, or
otherwise separately insured, and recover for a total loss thereof, when the cause of
NATURE the loss is a peril injured against,
Petition for review on certiorari (a) If more than three-fourths thereof in value is actually lost, or would have to be
expended to recover it from the peril;
FACTS (b) If it is injured to such an extent as to reduce its value more than three-fourths;
- Sometime in January 1986, private respondent Panama Sawmill Co., Inc. (Panama) xxx xxx xxx
bought, in Palawan, 1,208 pieces of apitong logs, with a total volume of 2,000 cubic - The requirements for the application of Section 139 of the Insurance Code, quoted
meters. It hired Transpacific Towage, Inc., to transport the logs by sea to Manila and above, have not been met. The logs involved, although placed in two barges, were not
insured it against loss for P1-M with petitioner Oriental Assurance Corporation separately valued by the policy, nor separately insured. Resultantly, the logs lost in
(Oriental Assurance). the damaged barge in relation to the total number of logs loaded on the same barge
- While the logs were being transported, rough seas and strong winds caused damage cannot be made the basis for determining constructive total loss. The logs having
to one of the two barges resulting in the loss of 497 pieces of logs out of the 598 been insured as one inseparable unit, the correct basis for determining the existence
pieces loaded thereon. of constructive total loss is the totality of the shipment of logs. Of the entirety of
- Panama demanded payment for the loss but Oriental Assurance refuse on the 1,208, pieces of logs, only 497 pieces thereof were lost or 41.45% of the entire
ground that its contracted liability was for "TOTAL LOSS ONLY." shipment. Since the cost of those 497 pieces does not exceed 75% of the value of all
- Unable to convince Oriental Assurance to pay its claim, Panama filed a Complaint for 1,208 pieces of logs, the shipment cannot be said to have sustained a constructive
Damages against Oriental Assurance before the Regional Trial Court. total loss under Section 139(a) of the Insurance Code.
- RTC ordered Oriental Assurance to pay Panama with the view that the insurance Disposition judgment under review is SET ASIDE
contract should be liberally construed in order to avoid a denial of substantial justice;
and that the logs loaded in the two barges should be treated separately such that the PAN MALAYAN INSURANCE v. CA (THE FOOD AND
loss sustained by the shipment in one of them may be considered as "constructive
AGRICULTURAL ORGANIZATION OF THE UNITED NATIONS)
total loss" and correspondingly compensable. CA affirmed in toto.
201 SCRA 382
ISSUE REGALADO; September 5, 1991
WON Oriental Assurance can be held liable under its marine insurance policy based on
the theory of a divisible contract of insurance and, consequently, a constructive total FACTS
loss - The Food and Agricultural Organization of the United Nations (hereinafter referred to
as FAO), ntended and made arrangements to send to Kampuchea 1,500 metric
HELD petitions of IR-36 certified rice seeds to be distributed to the people for seedling
NO purposes
- The terms of the contract constitute the measure of the insurer liability and - LUZTEVECO was to ship the cargo amounting to US$83,325.92 in respect of one lot
compliance therewith is a condition precedent to the insured's right to recovery from of 1,500 metric petitions winch is the subject of the present action. The cargo was
the insurer. Whether a contract is entire or severable is a question of intention to be loaded on board LUZTEVECO Barge No. LC-3000 and consisted of 34,122 bags of IR-36
determined by the language employed by the parties. The policy in question shows certified rice seeds purchased by FAO from the Bureau of Plant Industry for
that the subject matter insured was the entire shipment of 2,000 cubic meters of P4,602,270.00
apitong logs. The fact that the logs were loaded on two different barges did not make
iNsuRanCe A2010 Dean Carale
pAgE 97♥

- FAO secured insurance coverage in the amount of P5,250,000.00 from petitioner, - the complete physical destruction of the subject matter is not essential to constitute
Pan Malayan Insurance Corporation an actual total loss. Such a loss may exist where the form and specie of the thing is
- On June 16, 1980, FAO gave instructions to LUZTEVECO to leave for Vaung Tau, destroyed, although the materials of which it consisted still exist (Great Western Ins.
Vietnam to deliver the cargo which, by its nature, could not withstand delay because Co. vs. Fogarty, N.Y., 19 Wall 640, 22 L. Ed. 216), as where the cargo by the process of
of the inherent risks of termination and/or spoilage. On the same date, the insurance decomposition or other chemical agency no longer remains the same kind of thing as
premiums on the shipment was paid by FAO petitioner before (Williams vs. Cole, 16 Me. 207).
- On June 26, 1980, FAO was advised of the sinking of the barge in the China Sea, - It is thus clear that FAO suffered actual total loss under Section 130 of the Insurance
hence it informed petitioner thereof and, later, formally filed its claim under the Code, specifically under paragraphs (c) and (d) thereof, recompense for which it has
marine insurance policy. On July 29, 1980, FAO was informed by LUSTEVECO of the been denied up to the present
recovery of the lost shipment, for which reason FAO formally filed its claim with -Section 135 of the Insurance Code explicitly provides that "(u)pon an actual total loss,
LUZTEVECO for compensation of damage to its cargo a person insured is entitled to payment without notice of abandonment." This is a
- LUZTEVECO failed and refused to pay. Pan Malayan likewise failed to pay for the statutory adoption of a long standing doctrine in maritime insurance law that in case
losses and damages sustained by FAO by reason of its inability to recover the value of of actual total loss, the right of the insured to claim the whole insurance is absolute,
the shipment from LUZTEVECO without need of a notice of abandonment
- Pan Malayan claims that part of the cargo was recovered and thus the claim by FAO
was unwarranted. This is evidenced by two surveys upon the cargo wherein it was
PHILIPPINE AMERICAN LIFE INSURANCE COMPANY v. CA
found that only around 78% was lost.
- FAO filed a civil case against both LUZTEVECO and Pan Malayan. Trial court found in (ELIZA PULIDO)
favor of FAO and ordered both to pay jointly and severally the full amount of the 344 SCRA 260
claim. This was affirmed by CA GONZAGA-REYES; November 15, 2000
ISSUE NATURE
1. WON respondent court committed a reversible error in holding that the trial court is This petition for review on certiorari seeks to reverse the Decision of the Special
correct in holding that there is a total loss of the shipment Second Division of the Court of Appeals

HELD FACTS
1. NO - On January 9, 1989, petitioner received from one Florence Pulido an application for
- The law classifies loss into either total or partial. Total loss may be actual or life insurance, dated December 16, 1988, in the amount of P100,000.00 which
absolute, or it may otherwise be constructive or technical. Petitioner submits that designated her sister, herein private respondent, as its principal beneficiary. Because
respondent court erred in ruling that there was total loss of the shipment despite the the insurance applied for was non-medical, petitioner did not require a medical
fact that only 27,922 bags of rice seeds out of 34,122 bags were rendered valueless to examination and issued a policy on the sole basis of the application on February 11,
FAO and the shipment sustained only a loss of 78%. - FAO, however, claims that, for 1989. On April 1992, petitioner received private respondent’s claim, which declared
all intents and purposes, it has practically lost its total or entire shipment in this case, that the insured, Florence Pulido, died of acute pneumonia on September 10, 1991.
inclusive of expenses, premium fees, and so forth, despite the alleged recovery by - Petitioner withheld payment on the ground that the policy claimed under was void
defendant LUZTEVECO. As found by the court below and reproduced with approval by from the start for having been procured in fraud. It is petitioner’s contention that
respondent court, FAO "has never been compensated for this total loss or damage, a even before they received private respondent’s claim for death benefits, their
fact which is not denied nor controverted investigation concerning the subject policy yielded the information that the insured,
- If there were some cargoes saved, by LUZTEVECO, private respondent abandoned it Florence Pulido, died in 1988, before the application for insurance on her life was
and the same was sold or used for the benefit of LUZTEVECO or Pan Malayan made. While this was communicated to private respondent in a letter dated April 29,
Corporation. Under Sections 129 and 130 of the New Insurance Code, a total loss may 1992, private respondent had already filed her claim earlier that month. In another
either be actual or constructive. In case of total loss in Marine Insurance, the assured letter dated July 27, 1992, however, petitioner confirmed to private respondent receipt
is entitled to recover from the underwriter the whole amount of his subscription of the claim papers and assured her that her case was “being given preferential
- SEC. 130. An actual total loss is caused by: (c) Any damage to the thing which attention and prompt action”.
renders it valueless to the owner for the purpose for which he held it; or - Following the filing by private respondent of her claim, petitioner caused another
(d) Any other event which effectively deprives the owner of the possession, at the port investigation respecting the subject policy. Pursuant to the findings of this second
of destination of the thing insured. investigation, petitioner stood by its initial decision to treat the policy as void and not
-as said and proven, the seeds were of fragile nature. And the wetting of said seeds to honor the claim. On November 9, 1992, private respondent enlisted the services of
affected the state of seeds. Thus rendering them useless for FAO. Although there were counsel in reiterating her claim for death benefits. Petitioner still refused to make
bags which were recovered, these were “stained” and not in the same condition it was payment and thus, this action.
brought in. in addition to this, FAO did not receive any compensation for said - Petitioner: the results of its investigations having indicated that the insured was
recovered bags as the same were distributed by LUZVETECO without authorization of already dead at the time the policy was applied for. It also counterclaimed for
FAO attorney’s fees. The first report, prepared by one Dr. Benedicto Briones, was dated
April 1, 1992, and had attached to it a questionnaire, responded to by one Ramon
iNsuRanCe A2010 Dean Carale
pAgE 98♥

Piganto, who represented to be the brother-in-law of the insured and the barangay 94 PHIL 627
chairman of Cardiz, Bagulin, La Union. To the question “Where does [Florence Pulido]
BAUTISTA ANGELO; March 29, 1954
reside now?”, Piganto had replied that Florence Pulido used to live in Cardiz, but was
dead since 1988. Piganto’s statement was signed by him, and witnessed by his wife,
Nenita Piganto. This report was petitioner’s basis for treating the disputed policy as NATURE
void since April 1992, even before receipt of private respondent’s claim. Appeal from a decision of the Court of First Instance of Manila ordering defendant to
pay to plaintiff the sum of P3,000, Philippine currency, plus legal interest thereon from
ISSUE the time of the filing of the complaint until its full payment.
WON there was fraud (whether the insured, Florence Pulido, was in fact dead before
the application for insurance on her life was made) FACTS
- On April 14, 1943, the National Life Insurance Company of the Philippines issued a
HELD policy on the life of Jose C. Londres whereby it undertook to pay its beneficiary upon
NO his death the sum of P3,000. All the premiums due under the policy were actually paid
- This the lower courts had effected ruled on, upon a preponderance of the evidence on their dates of maturity and the policy was in force when the insured died on
duly received from both parties. We see no reversible error in the finding of both February 7, 1945. Salvacion V. Londres, as beneficiary, demanded from the company
respondent court and the trial court in favor of the correctness of the entries in the payment of the proceeds of the policy, and her demand having been refused, she
Certificate of Death, duly registered with the Local Civil Registrar of Bagulin, La Union, instituted the present action against the company in the Court of First Instance of
which declared that Florence Pulido died of acute pneumonia on September 10, 1991. Manila.
Dr. Irineo Gutierrez, the Municipal Health Officer of Bagulin, La Union whose signature - Defendant in its answer denied, for lack of sufficient proof, the allegation that the
appeared in the death certificate, testified in addition that he ministered to the ailing insured died on February 7, 1945, and set up the following special defenses: (a) that
Florence Pulido for two days immediately prior to her death. This fact is likewise plaintiff's claim is covered by the Moratorium Law; (b) that the policy having been
noted in the death certificate. issued during the Japanese occupation, it is presumed that its face value should be
- Death certificates, and notes by a municipal health officer prepared in the regular paid in Japanese currency, there being no provision in the policy from which can be
performance of his duties, are prima facie evidence of facts therein stated. A duly- inferred that the parties contemplated payment in any other currency; (c) that the
registered death certificate is considered a public document and the entries found money paid by the insured as premiums, together with the money received from
therein are presumed correct, unless the party who contests its accuracy can produce other policy-holders, was all deposited by the defendant in the Philippine National
positive evidence establishing otherwise. Petitioner’s contention that the death Bank and said deposit was declared without value by Executive Order No. 49 of the
certificate is suspect because Dr. Gutierrez was not present when Florence Pulido President of the Philippines; and (d) that the policy having been issued under
died, and knew of Florence’s death only through Ramon Piganto, does not merit a abnormal circumstances, it should be considered in the light of equity which does not
conclusion of fraud. No motive was imputed to Dr. Gutierrez for seeking to perpetuate permit anyone to enrich himself at the expense of another. Defendant, however, as a
a falsity in public records. Petitioner was likewise unable to make out any clear proof of good faith, offered to pay the value of the policy in accordance with the
motive as to why Ramon Piganto would purposely lie. Mere allegations of fraud could Ballantyne scale of values, or the sum of P2,400, Philippine currency.
not substitute for the full and convincing evidence that is required to prove it. A failure - It appears that the deceased took up the policy under consideration on April 15,
to do so would leave intact the presumption of good faith and regularity in the 1943 for the sum of P3,000. All the premiums due under the policy were actually paid
performance of public duties, which was the basis of both respondent court and the on their dates of maturity and the policy was in force when the insured died on
trial court in finding the date of Florence Pulido’s death to be as plaintiff-private February 7, 1945. On said date, the battle of the liberation of the City of Manila was
respondent maintained. still raging. While the northern part may have been liberated, not so the southern
- We cannot likewise give credence to petitioner’s submission that the inconsistencies part, as shown from the very affidavits submitted by appellee wherein it was stated
in the testimonies of the witnesses for plaintiff-private respondent are in themselves that on the aforesaid date, the insured, Jose Londres, and his two sons were taken by
evidence of fraud. Such alleged inconsistencies are matters of credibility which had the Japanese soldiers from their house at Singalong Street and were massacred by
been ably passed upon by the lower court. their captors. It may therefore be said that the policy became due when the City of
- The absence of fraud, as a factual finding of the lower court adopted by the Court of Manila was still under the yoke of the enemy and became payable only after liberation
Appeals, entirely consistent with the evidence on record, will not be reversed and, which took place on March 10, 1945 when President Osmena issued Proclamation No.
hence, is final and conclusive upon this Court. 6 following the restoration of the civil government by General Douglas Mac Arthur.
Disposition The instant petition is DENIED And we say that the policy became payable only after liberation even if it matured
sometime before, because before that eventuality the insurance company, appellant
herein, was not yet in a position to pay the value of the policy for the simple reason
CHAPTER IX. CLAIMS SETTLEMENT & that it had not yet reopened.
SUBROGATION
ISSUE
WON the amount of P3,000 which appellant bound itself to pay to the insured under
LONDRES v. NATIONAL LIFE INSURANCE
the policy upon his death should be paid in accordance with the present currency or
should be adjusted under the Ballantyne scale of values
iNsuRanCe A2010 Dean Carale
pAgE 99♥

FACTS
HELD - National Life Insurance Company (NLIC) insured J. Fernandez’s life for P10,000 upon
YES, present currency. his payment of P444 from July 15, 1944 to July 14, 1945
Reasoning - The insured died on November 2, 1944, while the policy was in force
- In the case of Rutter vs. Esteban, 93 Phil., 68, the Moratorium Law was declared - After more than 7 years, in 1952, Atty de la Torre, representing the benficiaries of
invalid and unconstitutional. the policy, informed the company that Fernandez had died in 1944, and claimed the
- During those days of liberation, while the people were rejoicing because of the happy proceeds of the policy. The company said that the status of the policies issued during
event, the banks, the insurance companies, and for that matter other commercial and the Japanese occupation was still pending consideration before the courts. NLIC said
business firms, were still feeling the adverse effects of the sudden fall of values and that because the policy matured upon the insured’s death in November, 1944, they
were uncertain and apprehensive as to the manner the readjustment would be made should compute the value of their claim under the Ballantyne scale of values (which
by the new Government. It is for this reason that the beneficiary, after realizing the would amount only to P500)
truth about the death of her husband, and after gathering evidence to substantiate his - beneficiaries commenced suit, and the lower court sustained the stand of the
death, had difficulty in effecting the collection of her claim from the insurance company, dismissed the complaint.
company because at that time it had not yet reopened for business purposes. - beneficiaries maintain that the obligation of the company to pay accrued not upon
Although the record does not disclose the exact date on which the insurance company the death of Fernandez, but only upon the receipt and approval by the company, on
reopened for this purpose, this Court can take judicial notice that it only did so after proof of death of the insured, which was in 1954. The policy reads:
liberation. At that time the legal tender was already the present currency. National Life Insurance Company of the Philippine hereby agrees to pay at its Home
- As final plea, appellant invokes equity in its favor in view of the nullification of the Office, Manila, Ten Thousand Pesos to Juan D. Fernandez (hereinafter called the
deposits made by it with the Philippine National Bank of all fiat money received from insured) on the 15th day of July, 1964, if the Insured is living and this Policy is in
its policyholders, which money was declared without value by Executive Order No. 49 force, or upon receipt and approved at its Office of due proofs of the title of the
of the President of the Philippines. Appellant claims that, considering the unexpected claimant and of the prior death of the Insured while this Policy is in force to Teresa
circumstances that developed, the indemnity to be paid by it should be suffered by it Duat Vda. De Fernandez, Maria T. and Manuela Fernandez, mother and sisters
under Article 307 of the Code of Commerce which provides: "When the deposits are of respectively of the Insured (Hereinafter called the Beneficiary) subject to the right
cash, with a specification of the coins constituting them, . . . the increase or reduction of the Insured to change the beneficiary as stated on the second page of this Policy.
which their value may suffer shall be for the account of the depositor." Appellant, by - The above stipulation is apparently based on Sec. 91-A of the Insurance Law which
entering into an insurance contract, cannot claim, if it suffers loss, that the provides as follows:
beneficiary cannot enrich herself at its expense. This is a risk attendant to any The proceeds of a life insurance policy shall be paid immediately upon maturity of the
wagering contract. One who gambles and loses cannot be heard to complain of his policy, unless such proceeds are made payable in installments or a as an annuity, in
loss. To appellant, we can only repeat the following admonition: which case the installments or annuities shall be paid as they become due: Provided,
"The parties herein gambled and speculated on the date of the termination of the however, That in case of a policy maturing by the death of the insured, the proceeds
war and the liberation of the Philippines by the Americans. This can be gleaned from thereof shall be paid within sixty days after presentation of the claim and filing of the
the stipulation about redemption, particularly that portion to the effect that proof of the death of the insured. Refused to pay the claim within the time prescribed
redemption could be effected not before the expiration of one year from June 24, herein will entitle the beneficiary to collect interest on the proceeds of the policy for
1944. This kind of agreement is permitted by law. We find nothing immoral or the duration of the delay at the rate of six per centum per annum, unless such failure
unlawful in it." (Gomez vs. Tabia) or refusal to pay is based on the ground that the claim is fraudulent . . . .
Disposition Wherefore, the decision appealed from is affirmed, with costs against - Based on the foregoing provision of law and the aforequoted stipulation as well as on
appellant. the allegation that the filing of proof of death by the beneficiaries is a condition
precedent of the demandability of the obligation of the insurer to pay the proceeds,
appellants claim that they should be paid P10,000 in Philippine currency and not
under the Ballantyne scale of values.

ISSUE
VDA. DE FERNANDEZ v. NATIONAL LIFE INSURANCE CO OF WON the policy matured upon the death of the insured
THE PHILS
HELD
105 PHIL 59 YES
ENDENCIA; January 27, 1959 Ratio In life insurance, the policy matures either upon the expiration of the term set
forth therein, or upon his death occuring at any time prior to the expiration of such
NATURE stipulated term, in which case, the proceeds are payable to his beneficiaries within
Appeal from CFI decision applying the Ballantyne scale of values upon the proceeds of sixty days after their filing of proof of death.
life insurance taken and maturing during the Japanese occupation but claimed after Reasoning
liberation - The sixty day period fixed by law within which to pay the proceeds after presentation
of proof of death Is merely procedural in nature, evidently to determine the exact
iNsuRanCe A2010 Dean Carale
pAgE 100♥

amount to be paid and the interest thereon to which the beneficiaries may be entitled court judgments thereon but not to court judgments for damages arising from injury
to collect in case of unwarranted refusal of the company to pay, and also to enable to persons and loss of property which does not involve a loan. Clearly, the applicable
the insurer to verify or check on the fact of death which it may even validly waive. It is law is Article 2209 of the Civil Code.
the happening of the suspensive condition of death that renders a life policy matured, - And in the light of the fact that the contending parties did not allege the rate of
and not ht efiling of proof of death which, as above stated, is merely procedural. The interest stipulated in the insurance contract, the legal interest was properly pegged by
insured having died during the Japanese occupation, the proceeds of his policy should the Appellate Court at 6% per cent.
be adjusted accordingly, for “The rule is already settled that where a debtor could
have paid his obligation at any time during the Japanese occupation, payment after
CATHAY INSURANCE v. CA(LUGAY)
liberation must be adjusted in accordance with the Ballantyne schedule (De Asis vs.
Agdamag, among other cases). (Collaboration is defined as the acts of working 174 SCRA 11
together in a joint project. GRINO-AQUINO; June 5, 1989
Disposition Judgment affirmed
FACTS
- Petitioners are 6 insurance companies that issued fire insurance policies for the total
TIO KHE CHIO v. CA (EASTERN ASSURANCE & SURETY)
sum of P4,000,000 to the Cebu Filipina Press owned by Emilia Chan Lugay. The fire
202 SCRA 119 policies described the insured property as "stocks of Printing materials, papers and
FERNAN; September 30, 1991 general merchandise usual to the Assured's trade" stored in a one-storey building of
strong materials housing the Cebu Filipina Press located at UNNO Pres. Quirino cor.
FACTS Don V. Sotto Sts., Mabolo, Cebu City. The co-insurers were indicated in each of the
- Petitioner Tio Khe Chio imported 1,000 bags of fishmeal valued at $36,000.30 from policies. All, except one policy (Paramount's), were renewals of earlier policies issued
Agro Impex, S.A. Dallas, Texas, U.S.A. The goods were insured with respondent EASCO for the same property.
and shipped on board the M/V Peskov, a vessel owned by Far Eastern Shipping - On December 18, 1981, the Cebu Filipina Press was razed by electrical fire together
Company. When the goods reached Manila, they were found to have been damaged with all the stocks and merchandise stored in the premises. On January 15, 1982,
by sea water which rendered the fishmeal useless. Petitioner filed a claim with EASCO Lugay submitted sworn Statements of Loss and Formal Claims to the insurers, through
and Far Eastern Shipping. Both refused to pay. Whereupon, petitioner sued them their adjusters. She claimed a total loss of P4,595,000.
before the then Court of First Instance of Cebu for damages. EASCO, as the insurer, - After nearly 10 months of waiting, she sued to collect on December 15, 1982. The
filed a counterclaim against the petitioner for the recovery of the unpaid insurance insurance companies denied liability, alleging violation of certain conditions of the
premiums. policy, misdeclaration, and even arson which was not seriously pressed for, come the
- The trial court rendered judgment in favor of petitioner. The judgment became final pre-trial, the petitioners offered to pay 50% of her claim, but she insisted on full
as to EASCO but the shipping company appealed to the Court of Appeals and was recovery.
absolved from liability by the said court. - Trial court rendered judgment in her favor ordering the insurers to pay her a total of
- The trial court, upon motion by petitioner, issued a writ of execution against EASCO. P4,000,000 as indemnity, P48,000 representing expenses of the plaintiff, a separate
The sheriff enforcing the writ reportedly fixed the legal rate of interest at 12%. amount of 20% of the P4,000,000 representing fees of counsel, interests at the rate of
Respondent EASCO moved to quash the writ alleging that the legal interest to be twice the ceiling being prescribed by the Monetary Board starting from the time when
computed should be 6% per cent per annum in accordance with Article 2209 of the the case was filed, and finally, with costs. CA affirmed.
Civil Code. The trial court denied EASCO's motion. On appeal, the Court of Appeals
reversed the trial court’s denial of EASCO’s motion and ruled that the applicable ISSUES
interest is 6% per annum. Hence, this petition. 1. WON the insured's cause of action had already accrued before she filed her
complaint
ISSUE 2. WON sufficient proofs of loss had been presented by the insured
WON the applicable rate of interest is 12% per annum 3. WON the private respondents claim for loss was inflated
4. WON lower court erred in awarding damages to the private respondent in the form
HELD of interest equivalent to double the interest ceiling set by the Monetary Board
NO 5. WON attorney's fees awarded were exorbitant
- Sections 243 and 244 of the Insurance Code apply only when there is an unjustified
refusal or withholding of payment on the insured’s claim. In this case, EASCO's refusal HELD
to settle the claim to Tio Khe Chio was based on some ground which, while not 1. YES
sufficient to free it from liability under its policy, nevertheless is sufficient to negate - As the fire which destroyed the Cebu Filipina Press occurred on December 19, 1981
any assertion that in refusing to pay, it acted unjustifiably. Simply put, the said and the proofs of loss were submitted from January 15, 1982 through June 21, 1982 in
provisions of the Insurance Code are not pertinent to the instant case. They apply only compliance with the adjusters' numerous requests for various documents, payment
when the court finds an unreasonable delay or refusal in the payment of the claims. should have been made within 90 days thereafter (Sec 243), or on or before
- Circular No. 416 of the Central Bank, which raised the legal rate of interest from 6% September 21, 1982. Hence, when the assured filed her complaint on December 15,
to 12% per annum refers only to loans or forbearances of money, goods or credits and 1982, her cause of action had already accrued.
iNsuRanCe A2010 Dean Carale
pAgE 101♥

2. YES - In 1977, Noda obtained from Zenith Insurance Corporation 2 fire insurance policies:
- There is no merit in the petitioners' contention that the proofs of loss were [1] No. F-03724 with a face value of P30k covering the goods and stocks in trade in his
insufficient because Lugay failed to comply with the adjuster's request for the business establishment at the market site in Mangagoy, Bislig, Surigao del Sur and [2]
submission of her bank statements. Condition No. 13 of the policy does not require the No. F-03734 with a face value in the aggregate amount of P100k and consisting of
insured to produce her bank statements. Therefore, the insured was not obligated to Item 1 for P40k on household furniture, fixtures, fittings and other personal effects,
produce them and the insurers had no right to ask for them. Condition No. 13 was and Item 2 for P60k on stocks in trade usual to petitioner's retail business situated in a
prepared by the insurers themselves, hence, it should be taken most strongly against two-storey building at 039 Barreda St., Mangagoy, Bislig, Surigao del Sur.
them. - While both policies were in force, fire destroyed petitioner's insured properties at the
3. NO market site on September 5, 1977 and at Barreda St. on November 9, 1977.
- Both the trial court and the CA noted that the proofs were ample and more than - When petitioner failed to obtain indemnity on his claims from Zenith, he filed a
enough for defendant insurers to do a just assessment supporting the 1981 fire claim complaint with the Insurance Commission praying that Zenith be ordered to pay him
for an amount exceeding four million pesos. P130kj representing the value of the 2 policies insured by respondent with interest at
4. NO 12% per annum, plus damages, attorney's fees and other expenses of litigation. ...
- The award of double interest on the claim is lawful and justified under Sections 243 - Zenith interposed that petitioner had no cause of action; that Policy No. F-03724 was
and 244 of the Insurance Code which provide: not in full force and effect at the time of the fire because the premium on the policy
Sec. 243 Refusal or failure to pay the loss or damage within the time prescribed was not paid; that Zenith's liability under Policy No. F-03734, if any, was limited to
herein will entitle the assured to collect interest on the proceeds of the policy for P15,472.50 in view of the co-insurance; and that petitioner failed to substantiate his
the duration of the delay at the rate of twice the ceiling prescribed by the Monetary claim as to the value of the goods reputedly destroyed by fire.
Board. - While the case was pending, Zenith settled petitioner’s fire loss claim under Item 1
Sec. 244 In case of any litigation for the enforcement of any policy or contract of of Policy No. 03734 in the amount of P15,472.50.
insurance, it shall be the duty of the Commissioner or the Court, as the case may - Insurance Commissioner allowed petitioner to recover under said policy and ordered
be, to make a finding as to whether the payment of the claim of the insured has Zenith to pay him the amount of P20k with legal interest from the date the complaint
been unreasonably denied or withheld; and in the affirmative case, the insurance was filed, including P1k as attorney's fees but excluding the actual, moral and
company shall be adjudged to pay damages which shall consist of attorney's fees exemplary damages prayed for. As for petitioner's claim under Policy No. F-03734, she
and other expenses incurred by the insured person by reason of such unreasonable held that in view of the payment of P15,472.50 to petitioner, Zenith had fully
denial or withholding of payment plus interest of twice the ceiling prescribed by the discharged its liability under said policy which covered furniture, fixtures, fittings and
Monetary Board of the amount of claim due the insured. other personal belongings of petitioner.
- The petitioners' contention that the charging of double interest was improper - In allowing recovery under Policy No. F-03734, Commissioner placed much weight on
because no unreasonable delay in the processing of the fire claim was proven is the final report prepared by Dela Merced Adjustment Corporation, an independent fire,
refuted by the trial court's explicit finding that "there was a delay that was not marine and casualty adjuster contracted by Zenith to investigate the claims of its
reasonable in processing the claim and doing payments". Under Section 244, a prima various policyholders. Said report concluded that "the sound value of P26,666.67
facie evidence of unreasonable delay in payment of the claim is created by the failure represented the whole loss and damage" incurred by petitioner, but with the
of the insurer to pay the claim within the time fixed in both Sec. 242 and 243 of the IC. application of the three-fourths loss clause, Zenith's liability was reduced to P20k.
- In view of the not insubstantial value of the private respondent's claims and the
considerable time and effort expended by them and their counsel in prosecuting these ISSUES
claims for the past 8 years, attorney's fees were properly awarded to the private 1. WON Insurance Commissioner erred in denying petitioner's demand for P60k under
respondents. Item 2 of Policy No. F-03734
2. WON Insurance Commissioner erred in not awarding in favor of petitioner
5. YES exemplary damages for Zenith's unjustified and wanton refusal to pay petitioner's
- An award equivalent to 10% of the proceeds of the policies would be more claim under the said two insurance contracts
reasonable than the 20% awarded by the trial court and the CA.
Disposition Decision of the CA AFFIRMED with MODIFICATION. HELD
1. YES
NODA v. CRUZ - To prove the existence of the stocks in trade covered by Policy No. F-03734,
petitioner offered his testimony and that of his wife as well as documentary exhibits.
151 SCRA 227
The foregoing evidence for petitioner preponderantly showed the presence of some
FERNAN; June 22, 1987 P590k worth of goods in his retail store during the fire of November 9, 1977.
- While the insurer, and the Insurance Commissioner for that matter, have the right to
NATURE reject proofs of loss if they are unsatisfactory, they may not set up for themselves an
Petition to review decision of the Insurance Commissioner arbitrary standard of satisfaction. Substantial compliance with the requirements will
always be deemed sufficient.
FACTS - Zenith introduced in evidence the final report on Policy No. F-03734 submitted by its
own adjuster, Dela Merced Adjustment Corporation. Respondent Commissioner
iNsuRanCe A2010 Dean Carale
pAgE 102♥

however ignored such report, reasoning that with regard to Item 2 of Policy No. F- - RTC found that the vessel, MT Maysun, was seaworthy to undertake the voyage, and
03734 the claim for loss of the stocks in trade was not successfully proven in view of that the incident was caused by an unexpected inclement weather condition or force
petitioner's failure to present evidence; that the adjuster's report deserved scant majeure, thus exempting the common carrier from liability for the loss of its cargo.
consideration since the allegations therein were not substantiated, and that said - CA reversed RTC decision on the basis of evidence from PAG-ASA that there were no
report did not even make a recommendation for payment. 20 ft. waves in the area. CA ruled that the petitioner is liable on its obligation as
- A scrutiny of the abovementioned adjuster's report reveals that together with the common carrier to respondent insurance company as subrogee of Caltex.
formal demand for full indemnity, petitioner submitted his income tax return for 1978, Petitioner’s Claim
purchase invoices, certification from his suppliers as to his purchases, and other > In every marine insurance upon a ship or freight, or freightage, or upon any thing
supporting papers. The report even took into account the appraisals of the other which is the subject of marine insurance there is an implied warranty by the shipper
adjusters and concluded that the total loss sustained by petitioner in his household that the ship is seaworthy.10 When private respondent paid Caltex the value of its lost
effects and stocks in trade reached P379,302.12. But after apportioning said amount cargo, the act of the private respondent is equivalent to a tacit recognition that the ill-
among petitioner's six different insurers [the co-insurance being known to Zenith], the fated vessel was seaworthy.
liability of Zenith was placed at P60,592.10. It therefore recommended that Zenith pay
the petitioner the amount of P60, 592.10. Respondent’s Comment
- Said document was offered as evidence by Zenith itself and could very well be > American Home Assurance is entitled to payment by its right of subrogation.
considered as an admission of its liability up to the amount recommended. Being in
the nature of an admission against interest, it is the best evidence which affords the ISSUES
greatest certainty of the facts in dispute. Respondent Commissioner should not have 1. WON payment made by American Home to Caltex for the insured value of the lost
perfunctorily dismissed that particular evidence as a worthless piece of paper. cargo amounted to an admission that the vessel was seaworthy, thus precluding any
2. NO action for recovery against the petitioner
- There is no showing that Zenith, in contesting payment, had acted in a wanton, 2. WON MT Maysun was seaworthy at the time of the voyage (outline topic)
oppressive or malevolent manner to warrant the imposition of corrective damages. 3. WON non-presentation of the marine insurance policy bars the complaint for
Disposition Zenith Insurance Corporation ordered to pay petitioner Norman R. Noda recovery of sum of money for lack of cause of action
the sum of P60,592.10 with legal interest from the filing of the complaint until full
payment, but deducting therefrom the amount of P15,472.50 which it had earlier paid HELD
to petitioner. 1. NO
Ratio The fact of payment grants American Home the subrogatory right which
DELSAN TRANSPORT, INC. v. CA (AMERICAN HOME enables it to exercise legal remedies that would otherwise be available to Caltex as
owner of the lost cargo against the petitioner common carrier.
ASSURANCE)
Reasoning
369 SCRA 24 Art. 2207. (Civil Code)
DE LEON, JR; November 15, 2001 If the plaintiff’s property has been insured, and he has received indemnity from the
insurance company for the injury or loss arising out of the wrong or breach of
NATURE contract complained of, the insurance company shall be subrogated to the rights of
A petition for review on certiorari of the decision of CA. the insured against the wrongdoer or the person who has violated the contract. If
the amount paid by the insurance company does not fully cover the injury or loss,
FACTS the aggrieved party shall be entitled to recover the deficiency from the person
- Caltex entered into a contract of affreightment with the petitioner, Delsan Transport causing the loss or injury.
Lines, Inc. (petitioner), for a period of one year whereby the said common carrier - The right of subrogation is designed to promote and to accomplish justice and is the
agreed to transport Caltex’s industrial fuel oil from the Batangas-Bataan Refinery to mode which equity adopts to compel the ultimate payment of a debt by one who in
different parts of the country. Delsan took on board its vessel, MT Maysun, 2,277.314 justice and good conscience ought to pay. It is not dependent upon, nor does it grow
kiloliters of industrial fuel oil of Caltex to be delivered to the Caltex Oil Terminal in out of, any privity of contract or upon written assignment of claim. It accrues simply
Zamboanga City. The shipment was insured by American Home Assurance upon payment by the insurance company of the insurance claim.
Corporation (respondent). 2. NO
- August 14, 1986: MT Maysun set sail from Batangas for Zamboanga City. The vessel Ratio Seaworthiness relates to a vessel’s actual condition. Neither the granting of
sank in the early morning of August 16, 1986 near Panay Gulf in the Visayas taking classification or the issuance of certificates establishes seaworthiness.
with it the entire cargo of fuel oil. Reasoning
- Respondent paid Caltex P5,096,635.57 representing the insured value of the lost - Common carriers are bound to observe extraordinary diligence in the vigilance over
cargo. Exercising its right of subrogation under Article 2207 of the New Civil Code, the the goods and for the safety of passengers transported by them, according to all the
private respondent demanded of the petitioner the same amount it paid to Caltex. circumstances of each case. There is no liability if the loss, destruction or deterioration
Delsan refused to pay, forcing American home to file a case for collection in the RTC. is by force majeure.

10
Sec. 113 Insurance Code
iNsuRanCe A2010 Dean Carale
pAgE 103♥

- The tale of strong winds and big waves by the said officers of the petitioner however,
was effectively rebutted and belied by the weather report from PAG-ASA. MT Maysun ISSUE
sank with its entire cargo for the reason that it was not seaworthy. There was no WON Finman can be held liable for complainants’ claims against Pan Pacific
squall or bad weather or extremely poor sea condition in the vicinity when the said
vessel sank. HELD
- Petitioner may not escape liability by presenting in evidence certificates that tend to YES
show that at the time of dry-docking and inspection by the Philippine Coast Guard MT - Under Insurance Code, liability of surety in a surety bond is joint and several with
Maysun, was fit for voyage. These pieces of evidence do not necessarily take into the principal obligor.
account the actual condition of the vessel at the time of the commencement of the - Conditions of a bond specified and required in the provisions of a statute providing
voyage. At the time of dry-docking and inspection, the ship may have appeared fit. for submission of the bond, are incorporated into all bonds tendered under that
The certificates issued, however, do not negate the presumption of unseaworthiness statute even though not set out in printer’s ink.
triggered by an unexplained sinking. - POEA held and Secretary of Labor affirmed that Pan Pacific had violated Labor Code,
- Authorities are clear that diligence in securing certificates of seaworthiness does not and at least one of the conditions for the grant and continued use of the recruitment
satisfy the vessel owner’s obligation. Also securing the approval of the shipper of the license. POEA and Secretary of Labor can require Pan Pacific to refund the placement
cargo, or his surveyor, of the condition of the vessel or her stowage does not establish fees and to impose the fine.
due diligence if the vessel was in fact unseaworthy, for the cargo owner has no - If Pan Pacific is liable, and if Finman is solidarily liable with Pan Pacific, then Finman
obligation in relation to seaworthiness. is liable both to private respondents and to POEA.
3. NO - Cash and surety bonds are required from recruitment companies as means of
Ratio The presentation in evidence of the marine insurance policy is not ensuring prompt and effective recourse against such companies when held liable.
indispensable in this case before the insurer may recover from the common carrier Public policy will be effectively negated if POEA and the DoLE were held powerless to
the insured value of the lost cargo in the exercise of its subrogatory right. The compel a surety company to make good on its solidary undertaking.
subrogation receipt, by itself, is sufficient to establish not only the relationship of
respondent as insurer and Caltex, as the assured shipper of the lost cargo of industrial
fuel oil, but also the amount paid to settle the insurance claim. The right of EAGLE STAR INSURANCE CO LTD v. CHIA YU
subrogation accrues simply upon payment by the insurance company of the insurance 96 PHIL 696
claim. REYES; March 31, 1955
Disposition Petition is denied, and the decision of the CA is affirmed.
NATURE
Certiorari

FACTS
- Atkin, Kroll & Co., loaded on the S. S. Roeph Silverlight owned and operated by
FINMAN GENERAL ASSURANCE CORP v. INOCENCIO Leigh Hoegh & Co., A/S, of San Francisco California, 14 bales of assorted underwear
179 SCRA 480 valued at P8,085.23 consigned to Chia Yu in the City of Manila.
FELICIANO; November 15, 1989 - The shipment was insured against all risks by Eagle Star Ins. Co. of San Francisco,
California, under a policy issued to the shipper and by the latter assigned to the
consignee.
FACTS
- The vessel arrived in Manila but of the 14 bales (a.k.a. freights =p) consigned to Chia
- Pan Pacific is a recruitment and employment agency. It posted surety bond issued
Yu only 10 were delivered to him as the remaining 3 could not be found.3 of those
by Finman General Assurance and was granted license to operate by POEA.
delivered were also found damaged to the extent of 50 per cent.
- Inocencio, Palero, Cardones, Hernandez filed with POEA complaints against Pan
-Chia Yu claimed indemnity for the missing and damaged bales. But the claim was
Pacific for violation of Labor Code and for refund of placement fees. POEA
declined, first, by the carrier and afterward by the insurer, whereupon Chia Yu brought
Administrator motu propio impleaded Finman as surety for Pan Pacific.
the present action against both, including their respective agents in the Philippines.
- Pan Pacific moved out and no notice of transfer was furnished to POEA as required.
- An action was filed at the CFI after more than 2 years after delivery of the damaged
POEA considered that constructive service of complaints had been effected.
bales and the date when the missing bales should have been delivered, the action
- Finman denied liability and said that
was resisted by the Atkins and Eagle Star principally on the ground of prescription.
- POEA had no jurisdiction over surety bonds; jurisdiction is vested in Insurance
-TC favored Chia Yu and CA affirmed.
Commission
*** CARRIER’s defense of prescription is made to rest on the following stipulation of
- Finman had not violated Labor Code
the bill of lading:
- Complainants have no cause of action against Finman
In any event the carrier and the ship shall be discharged from all liability in respect
- Amounts claimed were paid as deposits and not as placement fees.
of loss or damage unless suit is brought within one year after the delivery of the
- POEA Administrator issued Order that respondents should pay. Finman appealed to
goods or the date when the goods should have been delivered. (This stipulation is
Secretary of Labor. Secretary upheld the POEA order.
iNsuRanCe A2010 Dean Carale
pAgE 104♥

but a repetition of a provision in the CA 65 which says that bills of lading covering
shipments from the US to the Phils should be brought w/in one year after the
delivery of the goods or the date when the goods should have been delivered to
hold the carrier liable.)
*** INSURER’s claim of prescription is founded upon the terms of the policy and not
upon the bill of lading. (But in our jurisdiction, as per A1144, prescription is 10 years ACCFA v. ALPHA INSURANCE
after action accrues.)
24 SCRA 151
No suit action on this Policy, for the recovery of any claim, shall be sustainable in
any Court of law or equity unless the insured shall have fully complied with all the REYES; July 29, 1968
terms and conditions of this Policy nor unless commenced with twelve (12) months
next after the happening of the loss . . . FACTS
- In order to guarantee the Asingan Farmers' Cooperative Marketing Association, Inc.
ISSUE (FACOMA) against loss on account of "personal dishonesty, amounting to larceny or
WON ATKIN’ s action has prescribed estafa of its Secretary-Treasurer, Ladines, the appellee, Alpha Insurance & Surety
Company had issued, on 14 February 1958, its bond, No. P-FID-15-58, for the sum of
HELD P5,000 with said Ladines as principal and the appellee as solidary surety. On the same
NO date, the Asingan FACOMA assigned its rights to the appellant, Agricultural Credit
- Being contrary to the law of the forum, the stipulation in the policy cannot be given Cooperative and Financing Administration (ACCFA for short), with approval of the
effect as it would reduce the period allowed the insured for bringing his action to less principal and the surety.
than one year (because the prescription period begins from the “happening of the - During the effectivity of the bond, Ladines converted and misappropriated, to his
loss” and that before any suit could be sustained the insured shall have to comply personal benefit, some P11,513.22 of the FACOMA funds, of which P6,307.33 belonged
with the terms and conditions of the policy first TF lessening the period to less than a to the ACCFA. Upon discovery of the loss, ACCFA immediately notified in writing the
year. ) survey company on 10 October 1958, and presented the proof of loss within the
- Insular Government vs. Frank(13 Phil. 236)~ "matters respecting a remedy, such as period fixed in the bond; but despite repeated demands the surety company refused
the bringing of suit, admissibility of evidence, and statute of limitations, depend upon and failed to pay. Whereupon, ACCFA filed suit against appellee on 30 May 1960.
the law of the place where the suit is brought" TF any policy clause repugnant to this - Defendant Alpha Insurance & Surety Co., Inc., (now appellee) moved to dismiss the
amendment to the Insurance Act cannot be given effect in an action in our courts. complaint for failure to state a cause of action, giving as reason that (1) the same was
SEC. 61-A. (Insurance Code) ~ Any condition, stipulation or agreement in any policy filed more than one year after plaintiff made claim for loss, contrary to the eighth
of insurance, limiting the time for commencing an action thereunder to a period of condition of the bond, providing as follows:
less than one year from the time when the cause of action accrues, is void. EIGHT LIMITATION OF ACTION: No action, suit or proceeding shall be had or
- The prescription clause could be harmonized with section 61-A of the Insurance Act maintained upon this Bond unless the same be commenced within one year from the
by taking it to mean that the time given the insured for bringing his suit is twelve time of making claim for the loss upon which such action, suit or proceeding, is based,
months after the cause of action accrues. in accordance with the fourth section hereof.
- If so, when did the cause of action accrue? Chia Yu’s action did not accrue until his (2) the complaint failed to show that plaintiff had filed civil or criminal action against
claim was finally rejected by the insurance company. This is because, before such Ladines, as required by conditions 4 and 11 of the bond; and (3) that Ladines was a
final rejection, there was no real necessity for bringing suit. necessary and indispensable party but had not been joined as such.
- As the policy provides that the insured should file his claim, first, with the carrier and - At first, the Court of First Instance denied dismissal; but, upon reconsideration, the
then with the insurer, he had a right to wait for his claim to be finally decided before court reversed its original stand, and dismissed the complaint on the ground that the
going to court. action was filed beyond the contractual limitation period. Hence, this appeal.
- Furthermore, there is nothing in the record to show that the claim was rejected in
the year 1947, either by the insurance company in London or its settling agents in the ISSUE
Philippines. WON the provision of a fidelity bond that no action shall be had or maintained thereon
- For the purpose of this action, Chia Yu's claim was considered to have been finally unless commenced within one year from the making of a claim for the loss upon which
rejected by the insurer on April 22, 1948. Having been filed within twelve months form the action is based, is valid, in view of Section 61-A of the Insurance Act invalidating
that date, the action cannot be deemed to have prescribed even on the supposition stipulations limiting the time for commencing an action thereon to less than one year
that the period given the insured for bringing suit under the prescriptive clause of the from the time the cause of action accrues
policy is twelve months after the accrual of the cause of action.
- Contractual limitations contained in insurance policies are regarded with extreme HELD
jealousy by courts and will be strictly construed against the insurer and should not be NO
permitted to prevent a recovery when their just and honest application would not - A fidelity bond is, in effect, in the nature of a contract of insurance against loss from
produce that result. (46 C. J. S. 273.) misconduct, and is governed by the same principles of interpretation. Consequently,
Disposition Judgment appealed from is REVERSED with respect to the carrier and its the condition of the bond in question, limiting the period for bringing action thereon, is
agents but AFFIRMED with respect to the insurance company and its agents.
iNsuRanCe A2010 Dean Carale
pAgE 105♥

subject to the provisions of Section 61-A of the Insurance Act (No. 2427), as amended WON the suit against the agent tolled the prescription period, such that the filing
by Act 4101 of the pre-Commonwealth Philippine Legislature, prescribing that: against Fulton was only 9 months after the claim was rejected
SEC. 61-A: A condition, stipulation or agreement in any policy of insurance, limiting
the time for commencing an action thereunder to a period of less than one year HELD
from the time when the cause of action accrues is void. NO
- Since a "cause of action" requires, as essential elements, not only a legal right of the - The bringing of the action against the Paramount Surety & Insurance Company, the
plaintiff and a correlative obligation of the defendant but also "an act or omission of agent of the defendant company, cannot have any legal effect except that of notifying
the defendant in violation of said legal right," the cause of action does not accrue until the agent of the claim. Beyond such notification, the filing of the action can serve no
the party obligated refuses, expressly or impliedly, to comply with its duty (in this other purpose. There is no law giving any effect to such action upon the principal.
case, to pay the amount of the bond). The year for instituting action in court must be Besides, there is no condition in the policy that the action must be filed against the
reckoned, therefore, from the time of appellee's refusal to comply with its bond; it can agent, and the Court can not by interpretation extend the clear scope of the
not be counted from the creditor's filing of the claim of loss, for that does not import agreement beyond what is agreed upon by the parties.
that the surety company will refuse to pay. In so far, therefore, as condition eight of - Their contract is the law between the parties, and their agreement that an action on
the bond requires action to be filed within one year from the filing of the claim for a claim denied by the insurer must be brought within one year from the denial,
loss, such stipulation contradicts the public policy expressed in Section 61-A of the governs, not the rules on the prescription of actions.
Philippine Insurance Act. Disposition The judgment appealed from is hereby set aside and the case dismissed,
- Condition eight of the bond, therefore, is null and void, and the appellant is not with costs against plaintiffs-appellees.
bound to comply with its provisions. The discouraging of unnecessary litigation must
be deemed a rule of public policy, considering the unrelieved congestion in the courts. TRAVELLERS INSURANCE & SURETY CORP. v. CA (MENDOZA)
As a consequence of the foregoing, action may be brought within the statutory period
272 SCRA 536
of limitation for written contracts (New Civil Code, Article 1144).
HERMOSISIMA, JR; May 22, 1997
ANG v. FULTON FIRE INSURANCE CO.
NATURE
2 SCRA 945
The petition herein seeks the review and reversal of the decision of respondent Court
LABRADOR; July 31, 1961 of Appeals affirming in toto the judgment of the Regional Trial Court in an action for
damages filed by private respondent Vicente Mendoza, Jr. as heir of his mother who
NATURE was killed in a vehicular accident.
Appeal from judgment of the CFI ordering the defendant Fulton Fire Insurance Co. to
pay the plaintiffs the sum of P10,000.00, with interest, and an additional sum of FACTS
P2,000.00 as attorney's fees, and costs. -an old lady was hit by a taxicab. The taxicab was later identified and a case was filed
against the driver and owner. Later, an amendment was filed to include the insurance
FACTS company. RTC and CA ordered that the owner, driver as well as the insurance
- The stocks of general merchandise in the store of the Ang spouses are insured with company be held solidarily liable.
Fulton. While the insurance was in force, fire destroyed the goods. The Angs filed their
first claim immediately after the fire. ISSUE
- Their claim was denied on April 6, 1956. They received notice on April 19, 1956. WON RTC and CA erred
- The Angs brought an action against the agent on May 11, 1956. The court denied the HELD
suit and the mfr on Sept. 3 and 12, 1957. YES
- The Angs filed against Fulton on May 26, 1958. - Where the contract provides for indemnity against liability to third persons, then
- There was a clause in the policy: third persons to whom the insured is liable can sue the insurer. Where the contract is
13.If the claim be in any respect fraudulent, or if any false declaration is made or for indemnity against actual loss or payment, then third persons cannot proceed
used in support thereof, or if any fraudulent means or devices are used by the against the insurer, the contract being solely to reimburse the insured for liability
Insured or any one acting on his behalf to obtain any benefit under this Policy, or, if actually discharged by him thru payment to third persons, said third persons' recourse
the loss or damage be occasioned by the wilful act or with the connivance of the being thus limited to the insured alone. But in the case at bar, there was no
Insured, or, if the claim be made and rejected and an action or suit be not contract shown. What then was the basis of the RTC and the CA to say that
commenced within twelve months after such rejection or (in case of arbitration the insurance contract was a third-party liability insurance policy?
taking place in pursuance of the 18th condition of this Policy) within twelve months Consequently, the trial court was confused as it did not distinguish between the
after the arbitrator or arbitrators or umpire shall have made their award all benefit private respondent's cause of action against the owner and the driver of the Lady
under this Policy shall be forfeited." Love taxicab and his cause of action against petitioner. The former is based on torts
and quasi-delicts while the latter is based on contract.
ISSUE - Even assuming arguendo that there was such a contract, private respondent's cause
of action can not prevail because he failed to file the written claim mandated by
iNsuRanCe A2010 Dean Carale
pAgE 106♥

the Insurance Code (before it was amended-action must be brought within > It cites Art. 220711 and Art. 130412 of the Civil Code, and claims a preferred right to
six months from date of the accident (this is what’s applicable here) ; after retain the amount coming from San Miguel, despite the subrogation in favor of
amendment- "action or suit for recovery of damage due to loss or injury must be ZENITH.
brought in proper cases, with the Commissioner or the Courts within one year from Respondent’s Arguments
denial of the claim, otherwise the claimant's right of action shall prescribe" ). He is > There was no qualification to its right of subrogation under the Release of Claim
deemed, under this legal provision, to have waived his rights as against petitioner- executed by petitioner, the contents having expressed all intents and purposes of the
insurer. parties.
Disposition petition granted
ISSUE
WON the insurer may recover the sum of P5,000
SUN INSURANCE v. CA (supra p.57)
HELD
COASTWISE v. CA (supra p.70) YES
Ratio Since the insurer can be subrogated to only such rights as the insured may
have, should the insured, after receiving payment from the insurer, release the
CEBU SHIPYARD v. WILLIAM LINES (supra p.3) wrongdoer who caused the loss, the insurer loses his rights against the latter. But in
such a case, the insurer will be entitled to recover from the insured whatever it has
MANILA MAHOGANY MANUFACTURING CORP v. CA (ZENITH paid to the latter, unless the release was made with the consent of the insurer.
INSURANCE CORP) Reasoning
- Although petitioner’s right to file a deficiency claim against San Miguel is with legal
154 SCRA 652 basis, without prejudice to the insurer's right of subrogation, nevertheless, when
PADILLA; October 12, 1987 Manila Mahogany executed another release claim discharging San Miguel from "all
actions, claims, demands and rights of action that now exist or hereafter arising out of
NATURE or as a consequence of the accident" after the insurer had paid the proceeds of the
Petition to review CA decision ordering Manila Mahogany Manufacturing Corporation policy - the compromise agreement of P5,000 being based on the insurance policy -
to pay Zenith Insurance Corporation P5,000 with 6% annual interest, attorney's fees, the insurer is entitled to recover from the insured the amount of insurance money
and costs of suit paid. Since petitioner by its own acts released San Miguel, thereby defeating private
respondent’s right of subrogation, the right of action of petitioner against the insurer
FACTS was also nullified.
- From 6 March 1970 to 6 March 1971, MLA MAHOGANY insured its Mercedes Benz 4- - As held in Phil. Air Lines v. Heald Lumber Co., under Art. 2207, the real party in
door sedan with ZENITH. interest with regard to the portion of the indemnity paid is the insurer and not the
- On 4 May 1970, the insured vehicle was bumped and damaged by a truck owned by insured.
San Miguel Corporation. For the damage caused, ZENITH paid MLA MAHOGANY P5,000 SUBROGATION: The right of subrogation can only exist after the insurer has paid the
in amicable settlement. MLA MAHOGANY's general manager executed a Release of insured, otherwise the insured will be deprived of his right to full indemnity. If the
Claim, subrogating respondent company to all its right to action against San Miguel insurance proceeds are not sufficient to cover the damages suffered by the insured,
Corporation. then he may sue the party responsible for the damage for the remainder. To the
- On 11 Dec 1972, ZENITH wrote Insurance Adjusters, Inc. to demand reimbursement extent of the amount he has already received from the insurer enjoys the right of
from San Miguel. Insurance Adjusters, Inc. refused reimbursement, alleging that San subrogation.
Miguel had already paid petitioner P4,500, as evidenced by a cash voucher and a Disposition Petition DENIED. Judgment appealed from is AFFIRMED with costs against
Release of Claim executed by the General Manager of petitioner. petitioner.
- ZENITH thus demanded from petitioner reimbursement of the sum of P4,500 paid by
San Miguel. PIONEER INSURANCE v. CA (BORDER MACHINERY & HEAVY
- City Court ordered petitioner to pay respondent P4,500.
- CFI affirmed the City Court's decision in toto. EQUIPMENT INC)
- CA affirned CFI, with the modification that petitioner was to pay the total amount of 175 SCRA 668
P5,000 it had earlier received from ZENITH.
Petitioner’s Claims 11
> It is not bound to pay P4,500, and much more, P5,000 to ZENITH as the Article 2207: “If the plaintiff's property has been insured, and he has received indemnity from the insurance
company for the injury or loss arising out of the wrong or breach of contract complained of the insurance company
subrogation in the Release of Claim it executed in favor of respondent was shall be subrogated to the rights of the insured against the wrongdoer or the person who has violated the contract.
conditioned on recovery of the total amount of damages petitioner had sustained. If the amount paid by the insurance company does not fully cover the injury or loss the aggrieved party shall be
Since total damages were valued by petitioner at P9,486.43 and only P5,000 was entitled to recover the deficiency from the person causing the loss or injury.”
12
received by petitioner, MLA MAHOGANY argues that it was entitled to go after San Article 1305: “A creditor, to whom partial payment has been made, may exercise his right for the remainder,
Miguel to claim the additional P4,500. and he shall be preferred to the person who has been subrogated in his place in virtue of the partial payment of
the same credit.”
iNsuRanCe A2010 Dean Carale
pAgE 107♥

GUTIERREZ, JR.; July 28, 1989 reinsurance in the sum of P295,000, and paid with the said amount the bulk of its
alleged liability to JDA under the said surety bond. The total amount paid by Pioneer to
NATURE JDA is P299,666.29. Since Pioneer has collected P295,000.00 from the reinsurers, the
Petitions for review on certiorari of a decision of the CA uninsured portion of what it paid to JDA is the difference between the two amounts, or
P3,666.28. This is the amount for which Pioneer may sue defendants, assuming that
FACTS the indemnity agreement is still valid and effective. But since the amount realized
- In 1965, Jacob S. Lim was engaged in the airline business as owner-operator of from the sale of the mortgaged chattels are P35,000.00 for one of the airplanes and
Southern Air Lines (SAL), a single proprietorship. P2,050.00 for a spare engine, or a total of P37,050.00, Pioneer is still overpaid by
-On May 17, 1965, at Tokyo, Japan, Japan Domestic Airlines (JDA) and Lim entered into P33,383.72. Therefore, Pioneer has no more claim against defendants.
and executed a sales contract for the sale and purchase of two (2) DC-3A Type - The payment to the petitioner made by the reinsurers was not disputed. Considering
aircrafts and one (1) set of necessary spare parts for the total agreed price of US this admitted payment, the only question was the effect of payment made by the
$109,000.00 to be paid in installments. reinsurers to the petitioner
- On May 22, 1965, Pioneer Insurance and Surety Corporation, as surety, executed and - In general a reinsurer, on payment of a loss acquires the same rights by subrogation
issued its Surety Bond No. 6639 in favor of JDA, in behalf of its principal, Lim, for the as are acquired in similar cases where the original insurer pays a loss (Universal Ins.
balance price of the aircrafts and spare parts. Co. v. Old Time Molasses Co.).
-Border Machinery and Heavy Equipment Company, Inc. (Bormaheco), Francisco and - The rules of practice in actions on original insurance policies are in general
Modesto Cervantes (Cervanteses) and Constancio Maglana contributed some funds applicable to actions or contracts of reinsurance (Delaware, Ins. Co. v. Pennsylvania
used in the purchase of the above aircrafts and spare parts. They executed two (2) Fire Ins. Co.).
separate indemnity agreements in favor of Pioneer, one signed by Maglana and the - Hence the applicable law is Article 2207 of the new Civil Code, to wit: Art. 2207. If
other jointly signed by Lim for SAL, Bormaheco and the Cervanteses. the plaintiff’s property has been insured, and he has received indemnity from the
- On June 10, 1965, Lim doing business under the name and style of SAL executed in insurance company for the injury or loss arising out of the wrong or breach of contract
favor of Pioneer as deed of chattel mortgage as security for the latter's suretyship in complained of, the insurance company shall be subrogated to the rights of the insured
favor of the former. It was stipulated therein that Lim transfer and convey to the against the wrongdoer or the person who has violated the contract. If the amount paid
surety the two aircrafts. by the insurance company does not fully cover the injury or loss, the aggrieved party
- Lim defaulted on his subsequent installment payments prompting JDA to request shall be entitled to recover the deficiency from the person causing the loss or injury
payments from the surety. - If a property is insured and the owner receives the indemnity from the insurer, it is
- Pioneer paid a total sum of P298,626.12. provided in said article that the insurer is deemed subrogated to the rights of the
- Pioneer then filed a petition for the extrajudicial foreclosure of the said chattel insured against the wrongdoer and if the amount paid by the insurer does not fully
mortgage before the Sheriff of Davao City. cover the loss, then the aggrieved party is the one entitled to recover the deficiency.
- The Cervanteses and Maglana, however, filed a third party claim alleging that they Evidently, under this legal provision, the real party in interest with regard to the
are co-owners of the aircrafts, portion of the indemnity paid is the insurer and not the insured (. Air Lines, Inc. v.
- On July 19, 1966, Pioneer filed an action for judicial foreclosure with an application Heald Lumber Co., and Manila Mahogany Manufacturing Corporation v. Court of
for a writ of preliminary attachment against Lim and respondents, the Cervanteses, Appeals)
Bormaheco and Maglana. - It is clear from the records that Pioneer sued in its own name and not as an attorney-
**Maglana, Bormaheco and the Cervanteses filed cross-claims against Lim alleging in-fact of the reinsurer. Accordingly, the appellate court did not commit a reversible
that they were not privies to the contracts signed by Lim and, by way of counterclaim, error in dismissing the petitioner's complaint as against the respondents for the
sought for damages for being exposed to litigation and for recovery of the sums of reason that the petitioner was not the real party in interest in the complaint and,
money they advanced to Lim for the purchase of the aircrafts in question. (this therefore, has no cause of action against the respondents.
constitutes the second petition but will no longer be discussed because it is not Disposition Petitions dismissed. Questioned decision of CA affirmed.
relevant to the topic)
- After trial on the merits, a decision was rendered holding Lim liable to pay
Pioneer but dismissed Pioneer's complaint against all other defendants. PAN MALAYAN INSURANCE CORPORATION v. CA (FABIE, HER
- CA modified the trial court's decision in that the plaintiff’s complaint against all UNKNOWN DRIVER)
the defendants (including Lim) was dismissed.
184 SCRA 54
ISSUE CORTES, April 3, 1990
WON the petition of Pioneer Insurance and Surety Corporation against all defendants
was rightly dismissed NATURE
PETITION to review the decision of the Court of Appeals
HELD
YES FACTS
- Both the TC and CA made the finding that Pioneer reinsured its risk of liability under - Pan Malayan Insurance Company (Panmalay) insured the Mitsubishi Colt Lancer car
the surety bond it had executed in favor of JDA, collected the proceeds of such registered in the name of Canlubang Automotive Resources Corporation (Canlubang)
iNsuRanCe A2010 Dean Carale
pAgE 108♥

under its motor vehicle insurance policy. Among the provisions of the policy was a death of or bodily injuries suffered by 3rd parties) and from “Property Damage”
“own-damage” clause whereby Panmalay agrees to indemnify Canlubang in cases of coverage (liabilities from damage caused by insured vehicle to properties of 3rd
damage caused by “accidental collision or overturning, or collision or overturning parties)
consequent upon mechanical breakdown or consequent upon wear and tear”. ON CA: the terms of a contract are to be construed according to the sense and
- On 1985, the insured car was sideswept and damaged by a car owned by Erlinda meaning of the terms which the parties thereto have used. In the case of property
Fabie, driven by an unknown driver who fled the scene. Panmalay, in accordance with insurance policies, the evident intention of the contracting parties, i.e., the insurer
the policy, defrayed the cost of repair of the insured car and was subrogated to the and the assured, determine the import of the various terms and provisions embodied
rights of Canlubang against the driver and owner of the pick-up. Panmalay then filed a in the policy. It is only when the terms of the policy are ambiguous, equivocal or
complaint for damages with RTC Makati against Erlinda Fabie and her driver on the uncertain, such that the parties themselves disagree about the meaning of particular
grounds of subrogation, with the latter failing and refusing to pay their claim. Fabie provisions, that the courts will intervene. In such an event, the policy will be
filed a Motion for Bill of Particulars. construed by the courts liberally in favor of the assured and strictly against the
- RTC: dismissed complaint for lack of cause of action (payment by PANMALAY of insurer
CANLUBANG's claim under the "own damage" clause of the insurance policy was an - Both Panmalay and Canlubang had the same interpretation regarding the coverage
admission by the insurer that the damage was caused by the assured and/or its of insured risk regarding “accidental collision or overturning…” to include damages
representatives) – Panmalay appealed caused by 3rd party to Canlubang so it was improper for CA to ascribe meaning
- CA: dismissed appeal, affirmed RTC (applying the ejusdem generis rule held that contrary to the clear intention and understanding of the parties.
Section III-1 of the policy, which was the basis for settlement of CANLUBANG's claim, - Court on several occasions defined “accident” or “accidental” as taking place
did not cover damage arising from collision or overturning due to the negligence of “without one’s foresight or expectation, an event that proceeds from an unknown
third parties as one of the insurable risk) cause, or is an unusual effect of a known cause and, therefore, not expected” [Dela
Cruz v. Capital Insurance & Surety Co.] The concept "accident" is not necessarily
ISSUE synonymous with the concept of "no fault". It may be utilized simply to distinguish
WON the insurer PANMALAY may institute an action to recover the amount it had paid intentional or malicious acts from negligent or careless acts of man.
its assured in settlement of an insurance claim against private respondents as the - damage/loss to insured vehicle due to negligence of 3rd parties not listed as
parties allegedly responsible for the damage caused to the insured vehicle, in exceptions to coverage in the insurance policy
accordance with A2207, NCC - Interpretation given by Panmalay is more in keeping with rationale behind rules on
interpretation of insurance contracts in favor of assured or beneficiary: indemnity or
HELD payment
YES - EVEN if voluntarily indemnified Canlubang, as interpreted by TC: the insurer who
Ratio Article 2207 of the Civil Code is founded on the well-settled principle of may have no rights of subrogation due to "voluntary" payment may never. theless
subrogation. If the insured property is destroyed or damaged through the fault or recover from the third party responsible for the damage to the insured property under
negligence of a party other than the assured, then the insurer, upon payment to the Article 1236 of the Civil Code. [Sveriges Angfartygs Assurans Forening v. Qua Chee
assured, will be subrogated to the rights of the assured to recover from the wrongdoer Gan]
to the extent that the insurer has been obligated to pay. Payment by the insurer to Disposition the present petition is GRANTED. Petitioner's complaint for damages
the assured operates as an equitable assignment to the former of all remedies which against private respondents is hereby REINSTATED. Let the case be remanded to the
the latter may have against the third party whose negligence or wrongful act caused lower court for trial on the merits.
the loss. The right of subrogation is not dependent upon, nor does it grow out of, any
privity of contract or upon written assignment of claim. It accrues simply upon FIREMAN'S FUND INSURANCE COMPANY v. JAMILA &
payment of the insurance claim by the insurer
COMPANY, INC.
Exceptions
(1) if the assured by his own act releases the wrongdoer or third party liable for the 70 SCRA 323
loss or damage, from liability, the insurer's right of subrogation is defeated; AQUINO; April 1976
(2) where the insurer pays the assured the value of the lost goods without notifying
the carrier who has in good faith settled the assured's claim for loss, the settlement is FACTS
binding on both the assured and the insurer, and the latter cannot bring an action - Jamila & Co., Inc. or the Veterans Philippine Scouts Security Agency contracted to
against the carrier on his right of subrogation; supply security guards to Firestone. Jamila assumed responsibility for the acts of its
(3) where the insurer pays the assured for a loss which is not a risk covered by the security guards. The First Quezon City Insurance Co., Inc. executed a bond in the sum
policy, thereby effecting "voluntary payment", the former has no right of subrogation of P20,000 to guarantee Jamila's obligations under that contract.
against the third party liable for the loss - On May 18, 1963 properties of Firestone valued at P11,925 were lost allegedly due
Reasoning to the acts of its employees who connived with Jamila's security guard. Fireman's
- Both TC and CA are incorrect. Fund, as insurer, paid to Firestone the amount of the loss. Fireman's Fund was
ON TC: “Own damage” (not found in the insurance policy) simply meant that subrogated to Firestone's right to get reimbursement from Jamila. Jamila and its
Panmalay had assumed to reimburse the cost for repairing the damage to the insured surety, First Quezon City failed to pay the amount of the loss in spite of repeated
vehicle. It’s different from “Third Party Liability” coverage (liabilities arising from the demands.
iNsuRanCe A2010 Dean Carale
pAgE 109♥

- Upon defendant's motions, the lower court dismissed the complaint as to Jamila on - Article 2207 is a restatement of a settled principle of American jurisprudence.
the ground that there was no allegation that it had consented to the subrogation and, Subrogation has been referred to as the doctrine of substitution. It is an arm of equity
therefore, Fireman's Fund had no cause of action against it. It also dismissed the that may guide or even force one to pay a debt for which an obligation was incurred
complaint as to First Quezon City on the ground of res judicata. It appears that the but which was in whole or in part paid by another.
same action was previously filed in a civil case which was dismissed because of the - Subrogation is founded on principles of justice and equity, and its operation is
failure of the same plaintiffs and their counsel to appear at the pre-trial. governed by principles of equity. It rests on the principle that substantial justice
- Upon an MR, the lower court set aside its order of dismissal and sustained plaintiff's should be attained regardless of form, that is, its basis is the doing of complete,
contention that there was no res judicata as to First Quezon City because the civil essential, and perfect justice between all the parties without regard to form.
case was dismissed without prejudice. However, due to inadvertence, the lower court - Subrogation is a normal incident of indemnity insurance. Upon payment of the loss,
did not state in its order why it set aside its prior order dismissing the complaint with the insurer is entitled to be subrogated pro tanto to any right of action which the
respect to Jamila. Jamilla had originally moved for the dismissal of the complaint on insured may have against the third person whose negligence or wrongful act caused
the ground of lack of cause of action. Its basis for its contention were: (1) that the the loss. The right of subrogation is of the highest equity. The loss in the first instance
complaint did not allege that Firestone, pursuant to the contractual stipulation quoted is that of the insured but after reimbursement or compensation, it becomes the loss of
in the complaint, had investigated the loss and that Jamila was represented in the the insurer.
investigation and (2) that Jamila did not consent to the subrogation of Fireman's Fund - Although many policies including policies in the standard form, now provide for
to Firestone's right to get reimbursement from Jamila and its surety. The lower court subrogation, and thus determine the rights of the insurer in this respect, the equitable
in its order of dismissal had sustained the second ground. right of subrogation as the legal effect of payment inures to the insurer without any
- Jamila in its MR invoked the first ground which had never been passed upon by the formal assignment or any express stipulation to that effect in the policy. Stated
lower court. But the lower court in its order granting Jamila's motion for otherwise, when the insurance company pays for the loss, such payment operates as
reconsideration, completely ignored that first ground. It reverted to the second ground an equitable assignment to the insurer of the property and all remedies which the
which was relied upon in its order previous order. The lower court reiterated its order, insured may have for the recovery thereof. That right is not dependent upon, nor does
stating that Fireman's Fund had no cause of action against Jamila because Jamila did it grow out of, any privity of contract, or upon written assignment of claim, and
not consent to the subrogation. The court did not mention Firestone, the co-plaintiff of payment to the insured makes the insurer an assignee in equity.
Fireman's Fund. - On the other hand, Firestone is really a nominal party in this case. It had already
- Firestone and Fireman's Fund filed an MR on the ground that Fireman's Fund was been indemnified for the loss which it had sustained. Obviously, it joined as a party-
suing on the basis of legal subrogation whereas the lower court erroneously plaintiff in order to help Fireman's Fund to recover the amount of the loss from Jamila
predicated its dismissal order on the theory that there was no conventional and First Quezon City. Firestone had tacitly assigned to Fireman's Fund its cause of
subrogation because the debtor's consent was lacking. action against Jamila for breach of contract. Sufficient ultimate facts are alleged in the
- The plaintiffs cited article 2207 of the Civil Code which provides that "if the complaint to sustain that cause of action.
plaintiff's property has been insured, and he has received indemnity from the
insurance company for the injury or loss arising out of the wrong or breach of contract TABACALERA v. NORTH FRONT SHIPPING
complained of, the insurance company shall be subrogated to the rights of the insured
272 SCRA 527
against the wrongdoer or the person who has violated the contract".
- The lower court denied plaintiff's motion. They filed a second MR, calling the lower BELLOSILLO; May 16, 1997
court's attention to the fact that the issue of subrogation was of no moment because
Firestone, the subrogor, is a party-plaintiff and could sue directly Jamila in its own FACTS
right. Without resolving that contention, the lower court denied plaintiffs' second MR. - 20,234 sacks of corn grains valued at P3.5M were shipped on board North Front 777,
defendant’s vessel. The cargo was consigned to Republic Flour Mills Corp. under Bill of
ISSUE Lading No. 001 and insured with Tabacalera, Prudential Guarantee & Assurance, and
WON the complaint of Firestone and Fireman's Fund states a cause of action against New Zealand Insurance.
Jamila - Republic Flour was advised of the vessel’s arrival in Manila, but did not immediately
commence the unloading operations. Unloading was sometimes stopped due to
HELD varying weather and sometimes for no apparent reason. Unloading was only
YES completed 20 days after the arrival of the barge; by then, the cargo was short 26.333
- Fireman's Fund's action against Jamila is squarely sanctioned by article 2207. As metric tons and the rest was already moldy and deteriorating.
the insurer, Fireman's Fund is entitled to go after the person or entity that violated its - Analyses showed that the deterioration was caused by moisture content from salt
contractual commitment to answer for the loss insured against. water, which could be arrested by drying. However, Republic Flour rejected the entire
- The trial court erred in applying to this case the rules on novation. The plaintiffs in cargo and demanded that defendant North Front Shipping pay the damages suffered
alleging in their complaint that Fireman's Fund "became a party in interest in this case by it. The demands were unheeded and the insurance companies were obliged to pay
by virtue of a subrogation right given in its favor by" Firestone, were not relying on the Republic Flour P2,189,433
novation by change of creditors as contemplated in articles 1291 and 1300 to 1303 of - By virtue of the insurance companies’ payment, they were subrogated to the rights
the Civil Code but rather on article 2207. of Republic Flour. Petitioners filed a complaint against North Front Shipping, claiming
the loss was exclusively attributable to the latter’s fault and negligence. Having
iNsuRanCe A2010 Dean Carale
pAgE 110♥

surveyed the vessel, it was found that the barge had cracks in its bodega. The hatches 1989. On April 1992, petitioner received private respondent’s claim, which declared
on the crates of grain were not sealed and the tarpaulins used in covering them were that the insured, Florence Pulido, died of acute pneumonia on September 10, 1991.
not new, contrary to North Front Shipping’s claims. North Front Shipping reiterated - Petitioner withheld payment on the ground that the policy claimed under was void
that the barge was inspected prior to loading and found seaworthy and were issued a from the start for having been procured in fraud. It is petitioner’s contention that
permit to sail by the Coast Guard. They further averred that the grains were farm wet even before they received private respondent’s claim for death benefits, their
and not properly dried before loading. investigation concerning the subject policy yielded the information that the insured,
- The court dismissed the complaint, ruling that the contract entered into was a Florence Pulido, died in 1988, before the application for insurance on her life was
charter-party agreement; as such, only ordinary diligence in the care of the goods was made. While this was communicated to private respondent in a letter, private
required of North Front Shipping. respondent had already filed her claim earlier that month. In another letter, however,
petitioner confirmed to private respondent receipt of the claim papers and assured
ISSUE her that her case was “being given preferential attention and prompt action”.
WON defendant is required to observe extraordinary diligence in its vigilance over the - Petitioner caused another investigation respecting the subject policy. Pursuant to
goods it transports the findings of this second investigation, petitioner stood by its initial decision to treat
the policy as void and not to honor the claim. On November 9, 1992, private
HELD respondent enlisted the services of counsel in reiterating her claim for death
YES benefitsPetitioner still refused to make payment and thus, this action.
- As a corporation engaged in the business of transporting cargo offering its services
indiscriminately to the public, it is without a doubt a common carrier. As such, it has ISSUE
the burden of proving that it observed extraordinary diligence to avoid responsibility WON lower court erred in holding that there was no fraud
for the lost cargo. The clean bill of lading it issued disprove the master of the vessel’s
claim that the grains were farm wet when loaded. If they were wet, the master of the HELD
vessel should have known that the grains would eventually deteriorate when sealed in - The records bear out that since the onset of this case, the main issue has always
hot compartments in hatches of a ship and should have undertaken precautionary been whether there was fraud in the obtainment of the disputed policy, or put
measures to avoid this. The arrival of the goods at the place of destination in bad differently, whether the insured, Florence Pulido, was in fact dead before the
order makes a prima facie case against the common carrier, which must prove its application for insurance on her life was made. This the lower courts had effected
non-liability. ruled on, upon a preponderance of the evidence duly received from both parties. We
- While petitioners presented evidence of the vessel’s bad shape and a laboratory see no reversible error in the finding of both respondent court and the trial court in
analysis revealing that the grains were contaminated with salt water, defendants favor of the correctness of the entries in Certificate of Death, duly registered with the
failed to rebut said arguments or even endeavor to establish that the loss, destruction Local Civil Registrar of Bagulin, La Union, which declared that Florence Pulido died of
or deterioration was due to a fortuitous event; an act/omission of the owner of the acute pneumonia on September 10, 1991. Dr. Irineo Gutierrez, the Municipal Health
goods; the character of the goods or defects in their packing; or an order or act of a Officer of Bagulin, La Union whose signature appeared in the death certificate,
competent public authority. testified in addition that he ministered to the ailing Florence Pulido for two days
- However, Republic Flour is also found to be guilty of contributory negligence for not immediately prior to her death. This fact is likewise noted in the death certificate.
immediately staring the unloading operations and for providing no explanation for the - Death certificates, and notes by a municipal health officer prepared in the regular
delay. As such, it should share at least 40% of the loss. performance of his duties, are prima facie evidence of facts therein stated. A duly-
Disposition The decision of the CA is REVERSED and SET ASIDE registered death certificate is considered a public document and the entries found
therein are presumed correct, unless the party who contests its accuracy can produce
positive evidence establishing otherwise. Petitioner’s contention that the death
PHILIPPINE AMERICAN LIFE INSURANCE COMPANY v. CA
certificate is suspect because Dr. Gutierrez was not present when Florence Pulido
(ELIZA PULIDO) died, and knew of Florence’s death only through Ramon Piganto, does not merit a
344 SCRA 360 conclusion of fraud. No motive was imputed to Dr. Gutierrez for seeking to perpetuate
GONZAGA-REYES; November 15, 2000 a falsity in public records. Petitioner was likewise unable to make out any clear
motive as to why Ramon Piganto would purposely lie. Mere allegations of fraud could
NATURE not substitute for the full and convincing evidence that is required to prove it. A failure
This petition for review on certiorari seeks to reverse the Decision of the Special to do so would leave intact the presumption of good faith and regularity in the
Second Division of the Court of Appeals performance of public duties, which was the basis of both respondent court and the
trial court in finding the date of Florence Pulido’s death to be as plaintiff-private
FACTS respondent maintained.
- On January 9, 1989, petitioner received from one Florence Pulido an application for - We cannot likewise give credence to petitioner’s submission that the inconsistencies
life insurance, dated December 16, 1988, in the amount of P100,000.00 which in the testimonies of the witnesses for plaintiff-private respondent are in themselves
designated her sister, herein private respondent, as its principal beneficiary. Because evidence of fraud. Such alleged inconsistencies are matters of credibility which had
the insurance applied for was non-medical, petitioner did not require a medical been ably passed upon by the lower court.
examination and issued a policy on the sole basis of the application on February 11, Disposition the instant petition is DENIED
iNsuRanCe A2010 Dean Carale
pAgE 111♥

limitation of the carrier's liability is sanctioned by the freedom of the contracting


parties to establish such stipulations, clauses, terms, or conditions as they may deem
ST.PAUL FIRE & MARINE INSURANCE CO v. MACONDRAY & CO
convenient, provided they are not contrary to law, morals, good customs and public
INC policy. A stipulation fixing or limiting the sum that may be recovered from the carrier
70 SCRA 122 on the loss or deterioration of the goods is valid, provided it is (a) reasonable and just
ANTONIO; March 25, 1976 under the circumstances, and (b) has been fairly and freely agreed upon. In the case
at bar, the liabilities of the defendants-appellees with respect to the lost or damaged
FACTS shipments are expressly limited to the C.I.F. value of the goods as per contract of sea
- Winthrop Products, Inc. shipped aboard the SS "Tai Ping", owned and operated by carriage embodied in the bill of lading.
Wilhelm Wilhelmsen, 218 cartons and drums of drugs and medicine, with the freight - The plaintiff-appellant, as insurer, after paying the claim of the insured for damages
prepaid, which were consigned to Winthrop-Stearns, Inc. Barber Steamship Lines, Inc., under the insurance, is subrogated merely to the rights of the assured. As subrogee, it
agent of Wilhelm Wilhelmsen issued Bill of Lading No. 34, in the name of Winthrop can recover only the amount that is recoverable by the latter. Since the right of the
Products, Inc. as shipper, with arrival notice in-Manila to consignee Winthrop-Stearns, assured, in case of loss or damage to the goods, is limited or restricted by the
Inc. The shipment was insured by the shipper against loss and/or damage with the St. provisions in the bill of lading, a suit by the insurer as subrogee necessarily is subject
Paul Fire & Marine Insurance Company. to like limitations and restrictions.
- The SS "Tai Ping" arrived at the Port of Manila and discharged its aforesaid shipment 2. On the date of the discharge of the cargo. The peso equivalent was based by the
into the custody of Manila Port Service, the arrastre contractor for the Port of Manila. consignee on the exchange rate of P2.015 to $1.00 which was the rate existing at that
The said shipment was discharged complete and in good order with the exception of time.
one (1) drum and several cartons which were in bad order condition. Because
consignee failed to receive the whole shipment and as several cartons of medicine PHILAM v. AUDITOR (supra p.59)
were received in bad order condition, the consignee filed the corresponding claim in
the amount of P1,109.67 representing the C.I.F. value of the damaged drum and
FIELDMEN’S v. ASIAN SURETY (supra p.60)
cartons of medicine with the carrier and the Manila Port Service. However, both
refused to pay such claim. Consequently, the consignee filed its claim with the insurer,
St. Paul Fire & Marine Insurance Co., the insurance company, on the basis of such EQUITABLE v. RURAL INSURANCE (supra p.60)
claim, paid to the consignee the insured value of the lost and damagcd goods,
including other expenses in connection therewith, in the total amount of $1,134.46
COQUIA v. FIELDMEN'S INSURANCE CO. INC.
U.S. currency.
- As subrogee of the rights of' the shipper and/or consignee, the insurer, St. Paul Fire 26 SCRA 178
& Marine Insurance Co., instituted an action against the defendants for the recovery of CONCEPCION; November 29, 1968
said amount of $1,134.46, plus costs.
- The defendants resisted the action. However, for the purpose only of avoiding NATURE
litigation without admitting liability to the consignee, the defendants offered to settle Appeal from the decision of the CFI certified by CA
the latter’s claim in full by paying the C.I.F. value of the damaged cargo, but this offer
was declined by the plaintiff. FACTS
- The LC rendered judgment ordering the defendants to pay the plaintiff the sum of - December 1, 1961, appellant Fieldmen's Insurance Company, Inc. issued, in favor of
P300.00. The plaintiff filed a MFR contending that it should recover the amount of the Manila Yellow Taxicab Co., Inc. a common carrier accident insurance policy,
$1,134.46 or its equivalent in pesos at the rate of P3.90, instead of P2.00, but this was covering the period from December 1, 1961 to December ,1962. It was stipulated in
denied. Hence, this appeal. said policy that:
"The Company will, subject to the Limits of Liability and under the Terms of this Policy,
ISSUES indemnify the Insured in the event of accident caused by or arising out of the use of
1. WON in case of loss or damage, the liability of the carrier to the consignee is limited Motor Vehicle against all sums which the Insured will become legally liable to pay in
to the C.I.F. value of the goods which were lost or damaged respect of: Death or bodily injury to any fare-paying passenger including the Driver,
2. WON the insurer who has paid the claim in dollars to the consignee should be Conductor and/or Inspector who is riding in the Motor Vehicle insured at the time of
reimbursed in its peso equivalent on the date of discharge of the cargo or on the date accident or injury."
of the decision - While the policy was in force, or on February 10, 1962, a taxicab of the Insured,
driven by Carlito Coquia, met a vehicular accident to which he died. The Insured filed
HELD therefor a claim for P5,000.00 to which the Company replied with an offer to pay
1. YES P2,000.00, by way of compromise. The Insured rejected it and made a counter-offer
Ratio The purpose of the bill of lading is to provide for the rights and liabilities of the for P4,000.00, but the Company did not accept it.
parties in reference to the contract to carry. The stipulation in the bill of lading - On September 18, 1962, the Insured and Carlito's parents filed a complaint against
limiting the common carrier's liability to the value of the goods appearing in the bill, the Company to collect the proceeds of the policy. In its answer, the Company
unless the shipper or owner declares a greater value, is valid and binding. This
iNsuRanCe A2010 Dean Carale
pAgE 112♥

admitted the existence thereof, but pleaded lack of cause of action on the part of the "If any difference or dispute shall arise with respect to the amount of the Company's
plaintiffs. liability under this Policy, the same shall be referred to the decision of a single
- TC rendered a decision sentencing the Company to pay to the plaintiffs the sum of arbitrator to be agreed upon by both parties or failing such agreement of a single
P4,000.00 and the costs. Hence, this appeal by the Company, which contends that arbitrator, to the decision of two arbitrators, one to be appointed in writing by each
plaintiffs have no cause of action because: 1) the Coquias have no contractual relation of the parties within one calendar month after having been required in writing so to
with the Company; and 2) the Insured has not complied with the provisions of the do by either of the parties and in case of disagreement between the arbitrators, to
policy concerning arbitration. the decision of an umpire who shall have been appointed in writing by the
arbitrators before entering on the reference and the costs of and incidental to the
ISSUES reference shall be dealt with in the Award. And it is hereby expressly stipulated and
1. WON there was contractual relations between the Coquias and the Company declared that it shall be a condition precedent to any right of action or suit upon this
2. WON the insured has not complied with the provisions of the policy concerning Policy that the award by such arbitrator, arbitrators or umpire of the amount of the
arbitration Company's liability hereunder if disputed shall be first obtained."
- The record shows that none of the parties to the contract invoked this section, or
HELD made any reference to arbitration, during the negotiations preceding the institution of
1. Although, in general, only parties to a contract may bring an action based thereon, the present case. In fact, counsel for both parties stipulated, in the trial court, that
this rule is subject to exceptions, one of which is found in the Art 1311 CC, reading: none of them had, at any time during said negotiations, even suggested the
"If a contract should contain some stipulation in favor of a third person, he may settlement of the issue between them by arbitration, as provided in said section. Their
demand its fulfillment provided he communicated his acceptance of the obligor before aforementioned acts or omissions had the effect of a waiver of their respective right
its revocation. A mere incidental benefit or interest of a person is not sufficient. The to demand an arbitration.
contracting parties must have clearly and deliberately conferred a favor upon a third Disposition The decision appealed from should be as it is hereby affirmed in toto,
person." with costs against the herein defendant-appellant, Fieldmen's Insurance Co., Inc.
- Does the policy in question belong to such class of contracts pour autrui?
In this connection, said policy provides, inter alia: COUNTRY BANKERS INSURANCE CORP v. LIANGA BAY
"Section I — Liability to Passengers. 1. The Company will, subject to the Limits of
DE LEON; January 25, 2002
Liability and under the Terms of this Policy, indemnify the Insured in the event of
accident caused by or arising out of the use of Motor Vehicle against all sums which
the Insured will become legally liable to pay in respect of: Death or bodily injury to any NATURE
fare-paying passenger including the Driver. . . who is riding in the Motor Vehicle Petition for review on certiorari
insured at the time of accident or injury.
"Section II. — Liability to the Public FACTS
"3. In terms of and subject to the limitations of and for the purposes of this - Lianga Bay is a duly registered cooperative judicially declared insolvent and is here
Section, the Company will indemnify any authorized Driver who is driving the Motor represented by, Cornelio Jamero. Country Bankers Insurance and Lianga Bay entered
Vehicle . . . " into a contract of fire insurance. Country Bankers insured the respondent’s stocks-in-
"Conditions trade against fire loss, damage or liability during the period starting from June 20,
"7. In the event of death of any person entitled to indemnify under this Policy, 1989 at 4:00 p.m. to June 20, 1990 at 4:00 p.m., for the sum of P200,000.00.
the Company will, in respect of the liability incurred by such person, indemnify his - On July 1, 1989, at or about 12:40 a.m., the respondent’s building at Barangay
personal representatives in terms of and subject to the limitations of this Policy, Diatagon, Lianga, Surigao del Sur was gutted by fire, resulting in the total loss of the
provided, that such representatives shall, as though they were the Insured, observe, respondent’s stocks-in-trade, pieces of furnitures and fixtures, equipments and
fulfill and be subject to the Terms of this Policy insofar as they can apply. records.
"8. The Company may, at its option, make indemnity payable directly to the - Due to the loss, the respondent filed an insurance claim with the petitioner under its
claimants or heirs of claimants, with or without securing the consent of or prior Fire Insurance Policy, submitting: (a) the Spot Report of Pfc. Arturo V. Juarbal, INP
notification to the Insured, it being the true intention of this Policy to protect, to the Investigator, dated July 1, 1989; (b) the Sworn Statement of Jose Lomocso; and (c) the
extent herein specified and subject always to the Terms of this Policy, the liabilities of Sworn Statement of Ernesto Urbiztondo.
the Insured towards the passengers of the Motor Vehicle and the Public." - The petitioner, however, denied the insurance claim on the ground that, based on
- Thus, the policy under consideration is typical of contracts pour autrui, this character the submitted documents, the building was set on fire by 2 NPA rebels who wanted to
being made more manifest by the fact that the deceased driver paid fifty percent obtain canned goods, rice and medicines as provisions for their comrades in the
(50%) of the corresponding premiums, which were deducted from his weekly forest, and that such loss was an excepted risk under paragraph No. 6 of the policy
commissions. Under these conditions, it is clear that the Coquias — who, admittedly, conditions of Fire Insurance Policy No. F-1397, which provides:
are the sole heirs of the deceased — have a direct cause of action against the This insurance does not cover any loss or damage occasioned by or through or in
Company, and, since they could have maintained this action by themselves, without consequence, directly or indirectly, of any of the following occurrences, namely:
the assistance of the Insured, it goes without saying that they could and did properly (d) Mutiny, riot, military or popular uprising, insurrection, rebellion, revolution,
join the latter in filing the complaint herein. military or usurped power.
2. Based upon Section 17 of the policy:
iNsuRanCe A2010 Dean Carale
pAgE 113♥

Any loss or damage happening during the existence of abnormal conditions - In the evening of July 27, 1988, the radio station of Radio Mindanao Network located
(whether physical or otherwise) which are occasioned by or through or in at the SSS Building in Bacolod City was burned down causing damage in the amount
consequence, directly or indirectly, of any of said occurrences shall be deemed to of over one million pesos. Respondent sought to recover under two insurance policies
be loss or damage which is not covered by this insurance, except to the extent that but the claims were denied on the basis that the case of the loss was an excepted risk
the Insured shall prove that such loss or damage happened independently of the under condition no. 6 (c) and (d), to wit:
existence of such abnormal conditions. 6. This insurance does not cover any loss or damage occasioned by or through or in
- Finding the denial of its claim unacceptable, Lianga Bay then instituted in the trial consequence, directly or indirectly, of any of the following consequences, namely:
court the complaint for recovery of "loss, damage or liability" against Country (c) War, invasion, act of foreign enemies, hostilities, or warlike operations (whether
Bankers. war be declared or not), civic war.
- RTC ruled in favor of the cooperative. CA affirmed. (d) Mutiny, riot, military or popular uprising, insurrection, rebellion, revolution, military
or usurped power.
ISSUE - The insurers maintained that based on witnesses and evidence gathered at the site,
WON the cause of the loss was an excepted risk under the terms of the fire insurance the fire was caused by the members of the Communist Party of the Philippines/New
policy People’s Army. Hence the refusal to honor their obligations.
- The trial court and the CA found in favor of the respondent. In its findings, both
HELD courts mentioned the fact that there was no credible evidence presented that the
- Where a risk is excepted by the terms of a policy which insures against other perils CCP/NPA did in fact cause the fire that gutted the radio station in Bacolod.
or hazards, loss from such a risk constitutes a defense which the insurer may urge,
since it has not assumed that risk, and from this it follows that an insurer seeking to ISSUE
defeat a claim because of an exception or limitation in the policy has the burden of WON the insurance companies are liable to pay Radio Mindanao Network under the
proving that the loss comes within the purview of the exception or limitation set up. If insurance policies
a proof is made of a loss apparently within a contract of insurance, the burden is upon
the insurer to prove that the loss arose from a cause of loss which is excepted or for HELD
which it is not liable, or from a cause which limits its liability. Stated else wise, since YES
Country bank here is defending on the ground of non-coverage and relying upon an - The Court will not disturb the factual findings of the appellant and trial courts absent
exemption or exception clause in the fire insurance policy it has the burden of compelling reason. Under this mode of review, the jurisdiction of the court is limited to
proving the facts upon which such excepted risk is based, by a preponderance of reviewing only errors of law.
evidence. But petitioner failed to do so. - Particularly in cases of insurance disputes with regard to excepted risks, it is the
- The petitioner relies on the Sworn Statements of Jose Lomocso and Ernesto insurance companies which have the burden to prove that the loss comes within the
Urbiztondo as well as on the Spot Report of Pfc. Arturo V. Juarbal. A witness can purview of the exception or limitation set up. It is sufficient for the insured to prove
testify only to those facts which he knows of his personal knowledge, which means the fact of damage or loss. Once the insured makes out a prima facie case in its favor,
those facts which are derived from his perception. Consequently, a witness may not the duty or burden of evidence shifts to the insurer to controvert said prima facie
testify as to what he merely learned from others either because he was told or read or case.
heard the same. Such testimony is considered hearsay and may not be received as Disposition Petition dismissed. Decision of the CA is affirmed.
proof of the truth of what he has learned.
Disposition the appealed Decision is MODIFIED. The rate of interest on the
LEA MER INDUSTRIES v. MALAYAN INSURANCE
adjudged principal amount of Two Hundred Thousand Pesos (P200,000.00) shall be six
percent (6%) per annum computed from the date of filing of the Complaint in the trial 471 SCRA 698
court. The awards in the amounts of Fifty Thousand Pesos (P50,000.00) as actual PANGANIBAN; September 30, 2005
damages, Fifty Thousand Pesos (P50,000.00) as exemplary damages, Five Thousand
Pesos (P5,000.00) as litigation expenses, and Ten Thousand Pesos (P10,000.00) as NATURE
attorney?s fees are hereby DELETED. Petition for Review

DBP POOL OF ACCREDITED INSURANCE v. RADIO MINDANAO FACTS


- Ilian Silica Mining entered into a contract of carriage with Lea Mer Industries for the
NETWORK
shipment of 900 metric tons of silica sand valued at P565,000. Consigned to Vulcan
480 SCRA 314 Industrial and Mining Corporation, the cargo was to be transported from Palawan to
MARTINEZ; January 27, 2006 Manila. The silica sand was placed on board Judy VII, a barge leased by Lea Mer, the
vessel sank, resulting in the loss of the cargo.
NATURE
Petition for certiorari ISSUE
WON Lea Mer is liable for the loss of the cargo
FACTS
iNsuRanCe A2010 Dean Carale
pAgE 114♥

HELD - June 25, 1984 - Captain Montera of M/V Weasel ordered the vessel to be forced
YES aground which rendered the entire shipment of cement as good as gone due to
- Common carriers are bound to observe extraordinary diligence in their vigilance over exposure to sea water. LOASTAR thus failed to deliver the goods to MARKET in
the goods and the safety of the passengers they transport, as required by the nature Manila.
of their business and for reasons of public policy. Extraordinary diligence requires - MARKET demanded from LOADSTAR full reimbursement of the cost of the lost
rendering service with the greatest skill and foresight to avoid damage and shipment. LOADSTAR refused to reimburse the MARKET despite repeated demands.
destruction to the goods entrusted for carriage and delivery. - March 11, 1985 – PIONEER paid the MARKET P1,400,000 plus an additional amount of
- Common carriers are presumed to have been at fault or to have acted negligently for P500,000, the value of the lost shipment of cement. In return, the MARKET executed
loss or damage to the goods that they have transported. This presumption can be a Loss and Subrogation Receipt in favor of PIONEER concerning the latter’s
rebutted only by proof that they observed extraordinary diligence, or that the loss or subrogation rights against LOADSTAR.
damage was occasioned by any of the following causes: - October 15, 1986 – PIONEER filed a complaint against LOADSTAR with the RTC
“(1) Flood, storm, earthquake, lightning, or other natural disaster or calamity; Manila alleging that: (1) the M/V Weasel was not seaworthy at the commencement of
“(2) Act of the public enemy in war, whether international or civil; the voyage; (2) the weather and sea conditions then prevailing were usual and
“(3) Act or omission of the shipper or owner of the goods; expected for that time of the year and as such, was an ordinary peril of the voyage for
“(4) The character of the goods or defects in the packing or in the containers; which the M/V Weasel should have been normally able to cope with; and (3)
“(5) Order or act of competent public authority.” LOADSTAR was negligent in the selection and supervision of its agents and employees
- To excuse the common carrier fully of any liability, the fortuitous event must have then manning the M/V Weasel.
been the proximate and only cause of the loss. It should have exercised due diligence - LOADSTAR alleged that no fault nor negligence could be attributed to it because it
to prevent or minimize the loss before, during and after the occurrence of the event. exercised due diligence to make the ship seaworthy, as well as properly manned and
- Petitioner bore the burden of proving that it had exercised extraordinary diligence to equipped and failure to deliver was due to force majeure.
avoid the loss, or that the loss had been occasioned by a fortuitous event -- an - February 15, 1993 - RTC decided in favor of PIONEER and that LOADSTAR , as a
exempting circumstance. common carrier, bears the burden of proving that it exercised extraordinary diligence
- The evidence presented by petitioner in support of its defense of fortuitous event in its vigilance over the goods it transported. The trial court explained that in case of
was sorely insufficient. It was not enough for the common carrier to show that there loss or destruction of the goods, a statutory presumption arises that the common
was an unforeseen or unexpected occurrence. carrier was negligent unless it could prove that it had observed extraordinary
Disposition Petition is DENIED and the assailed Decision and Resolution are diligence. LOADSTAR’S defense of force majeure was found bereft of factual basis as
a PAG-ASA report that at the time of the incident, tropical storm “Asiang” had moved
AFFIRMED. Costs against petitioner. away from the Philippines was presented.
- October 15, 2002 – CA affirmed RTC Decision with modification
LOADSTAR SHIPPING CO INC v. PIONEER ASIA INSURANCE
ISSUES
CORP
1. WON LOADSTAR is a common carrier under Article 1732 CC
GR No. 157481 2. Assuming it is a common carrier, WON proximate cause of the loss of cargo was
QUISUMBING; January 24, 2006 not a fortuitous event but was allegedly due to the failure of petitioner to exercise
extraordinary diligence
NATURE
Review on certiorari (1) the Decision dated October 15, 2002 and (2) the Resolution HELD
dated February 27, 2003 of CA 1. YES
- A1732 CC defines a “common carrier” as follows:
FACTS Common carriers are persons, corporations, firms or associations engaged in the
- June 6, 1984 - Petitioner Loadstar Shipping Co., Inc. (LOADSTAR), registered owner business of carrying or transporting passengers or goods or both, by land, water, or
and operator of the vessel M/V Weasel, entered into a voyage-charter with Northern air, for compensation, offering their services to the public.
Mindanao Transport Company, Inc. for the carriage of 65,000 bags of cement from - LOADSTAR is a corporation engaged in the business of transporting cargo by water
Iligan City to Manila. The shipper was Iligan Cement Corporation, while the consignee and for compensation, offering its services indiscriminately to the public. Thus,
in Manila was Market Developers, Inc. (MARKET) without doubt, it is a common carrier. Even if it entered into a voyage-charter
- June 24, 1984 - 67,500 bags of cement were loaded on board M/V Weasel and agreement with Northern Mindanao Transport Company, Inc, it did not in any way
stowed in the cargo holds for delivery to the consignee. The shipment was covered by convert the common carrier into a private carrier.
petitioner’s Bill of Lading dated June 23, 1984. > Planters Products, Inc. v. CA - public carrier shall remain as such,
- Prior to the voyage, the consignee insured the shipment of cement with respondent notwithstanding the charter of the whole or portion of a vessel by one or more
Pioneer Asia Insurance Corporation (PIONEER) for P1,400,000, for which it issued persons, provided the charter is limited to the ship only, as in the case of a time-
Marine Open Policy No. MOP-006 dated September 17, 1980, covering all shipments charter or voyage-charter. It is only when the charter includes both the vessel and its
made on or after September 30, 1980 crew, as in a bareboat or demise that a common carrier becomes private, at least
insofar as the particular voyage covering the charter-party is concerned.
iNsuRanCe A2010 Dean Carale
pAgE 115♥

2. YES
- As a common carrier, LOADSTAR is required to observe extraordinary diligence in the
vigilance over the goods it transports. When the goods placed in its care are lost,
LOADSTAR is presumed to have been at fault or to have acted negligently. LOADSTAR
has the burden of proving that it observed extraordinary diligence in order to avoid
responsibility for the lost cargo.
- Compania Maritima V CA - It requires common carriers to render service with the
greatest skill and foresight and “to use all reasonable means to ascertain the nature
and characteristics of goods tendered for shipment, and to exercise due care in the
handling and stowage, including such methods as their nature requires.
- A1734 CC enumerates the instances when a carrier might be exempt from liability
for the loss of the goods.
(1) Flood, storm, earthquake, lightning, or other natural disaster or
calamity;
(2) Act of the public enemy in war, whether international or civil;
(3) Act or omission of the shipper or owner of the goods;
(4) The character of the goods or defects in the packing or in the
containers; and
(5) Order or act of competent public authority
- LOADSTAR claims that the loss of the goods was due to a fortuitous event under
paragraph 1. Yet, its claim is not substantiated. It is supported by evidence that the
loss of the entire shipment of cement was due to the gross negligence of LOADSTAR
- Records show that in the evening of June 24, 1984, the sea and weather conditions
in the vicinity of Negros Occidental were calm. The records reveal that LOADSTAR
took a shortcut route, instead of the usual route, which exposed the voyage to
unexpected hazard. LOADSTAR has only itself to blame for its misjudgment.
Disposition petition is DENIED
i

You might also like